Nelson Pediatrics Review(MCQs)17ed



Comments



Description

Question . 1. Clinical findings are influenced by a child's developmental stage.Which of the following features of the physical examination is most strongly influenced by developmental stage? Skin turgor Respiratory rate Explanation: Respiratory rate is highest in the neonate and gradually becomes slower approaching adult rates in the preadolescent-adolescent age period. Respiratory rate may be influenced by serious pulmonary or airway infection or inflammation in addition to non-respiratory causes, such as fever, anxiety, acidosis, pain, heart failure, central nervous system disease, drugs (stimulants, depressants, aspirin), and toxins (ammonia). (See Chapter 49 in Nelson Textbook of Pediatrics, 17th edition.) Lung aeration Mitral valve closure Fontanel tension Question . 2. The vulnerable child syndrome most influences which of the following components of the well child assessment? Physical examination Observation Developmental assessment History Explanation: Parents perceive vulnerability due to many real or exaggerated risks: premature birth, difficult pregnancy, previous death of a child, disease in other siblings, or minor low-risk conditions in this child. The history is usually exaggerated, embellished, and quite detailed. (See Chapter 49 in Nelson Textbook of Pediatrics, 17th edition.) Growth Question . 3. During examination of an 8-mo-old child, the difficulty of optimal chest auscultation is due primarily to: Recent meal with gastric distention Rapid respiratory rate Stranger anxiety Explanation: Crying makes the chest physical examination almost impossible, except for detection of the grossest abnormalities,. If at all possible, the chest auscultation should be preceded by observation and then performed in the sleeping, calm, nursing, or feeding infant. (See Chapter 49 in Nelson Textbook of Pediatrics, 17th edition.) Transmitted nasal sounds Pliable chest wall Question . 4. Assessment of the state of well-being by observation relies mostly on the child's: Nutritional status Motor ability Visual behavior Interaction with parents Explanation: The interaction with the parents is most critical, as this is the child's "normal" environment and is free of additional factors such as stranger anxiety. As children come with all varieties of behaviors, it is essential to ask the parents if the observed behavior is "normal" for the child and, if not, how it is different. (See Chapter 49 in Nelson Textbook of Pediatrics, 17th edition.) State variation Question . 5. Based on an orientation to child development, when would you tell parents the highest risk of poisoning in children is present? 6 mo 1 yr 2 yr Explanation: Self-poisoning in toddlers occurs once they become ambulatory and are able to walk and climb. Consider no place safe, especially if there are older siblings. Also consider that child-proof medicine containers will rarely be a deterrent to a motivated toddler. Remember that plants, berries, and liquids also present risks for ingestion in this age group. (See Chapter 49 in Nelson Textbook of Pediatrics, 17th edition.) 4 yr 6 yr Question . 6. Injury control is a more appropriate term than accident prevention because accidents are: Not predictable Not preventable Random Not due to chance Explanation: Injuries have definable risks, are not random, and are both predictable and preventable. The term "accident" confuses our meaning and should no longer be used. (See Chapter 50 in Nelson Textbook of Pediatrics, 17th edition.) Not common Question . 7. Motor vehicle injuries lead the list of injury deaths for all ages and are most often associated with: Alcohol use Occupant injury Driver age younger than 17 yr Presence of more than two occupants in the car Night driving All of the above Explanation: All are risk factors that can be modified by behaviors and laws, such as a graduated driver's licensing program. (See Chapter 50 in Nelson Textbook of Pediatrics, 17th edition.) Question . 8. Risk factors for injury include all of the following except: Toddler age Female gender Explanation: After 1-2 yr of age, males have a much higher injury rate than females. This risk lasts until the 7th decade. Adolescent risk-taking behavior accounts for some of the difference during the teen years. (See Chapter 50 in Nelson Textbook of Pediatrics, 17th edition.) Poverty Front-seating in an automobile Chaotic family stress Question . 9. The proper age at which a child may cross a busy street alone after careful instruction is: 4-5 yr 6-7 yr 8-10 yr 10-12 yr Developmentally, children below this age are unable to understand the risk and judge such factors as car speed. Chronological age does not determine readiness, which requires instruction, trial runs with a parent, and parental judgment of the child's ability. (See Chapter 50 in Nelson Textbook of Pediatrics, 17th edition.) None of the above Question . 10. The majority of children with medical emergencies present to all of the following for care except: Pediatrician's office Children's hospital emergency department Explanation: Most children receive all or initial emergency care at sites other than a specialized pediatric emergency room in a children's hospital. (See Chapter 51 in Nelson Textbook of Pediatrics, 17th edition.) Community hospital emergency department Clinic Urgent care facility Question . 11. A general pediatric office should be prepared for emergencies. This preparedness includes all of the following except: Training in advanced trauma care Explanation: Although advanced life support is ideal, it is unrealistic for a general office to be prepared for advanced trauma care. The patient should be stabilized as best as possible, with placement of an airway and intravascular lines, and transported to a facility capable of caring for pediatric trauma patients as soon as possible. (See Chapter 51 in Nelson Textbook of Pediatrics, 17th edition.) Education of staff in CPR Policies for triage Procedure for resuscitation Transport protocols Question . 12. A 15-mo-old child is in your office with stridor at rest and cyanosis with the presumptive diagnosis of viral croup. You should do all of the following except: Transport immediately in the parent's car to the nearest emergency department Explanation: A cyanotic child in respiratory distress is at high risk for a respiratory arrest. No acutely ill child should be transported by the parents no matter how short the distance or how long the delay for EMS to arrive. (See Chapter 51 in Nelson Textbook of Pediatrics, 17th edition.) Administer oxygen Administer dexamethasone Administer racemic epinephrine Transport after EMS personnel arrive 20 min later Question . 13. Enhanced 911: Is one-touch dialing Identifies patient location Explanation: The location of the caller is automatically identified. This is especially important if the caller is unable to communicate their location (young age, coma, seizure, drug overdose, disability). (See Chapter 51 in Nelson Textbook of th Pediatrics, 17 edition.) Triages based on severity of illness Responds within 5 min Provides MD presence with EMS Question . 14. Emergency Medical Treatment and Active Labor Act (EMTALA) is a set of federal regulations that addresses the transfer of patients. Which of the following statements regarding EMTALA is not true? The transferring hospital must provide an appropriate medical screening to assess if the patient has an emergency condition If an emergency condition exists, the patient's condition must be stabilized, or if stabilization measures exceed that hospital's expertise, the patient must be transferred to a hospital capable of such measures Transfer of unstable patients is permitted under limited circumstances The law does not apply to pediatricians who are on call for consultation to the emergency department Explanation: This law is meant to benefit patients and includes patients of all ages and their doctors. All patients must be stabilized to the best of the ability of the emergency room staff. All pertinent data must also be transferred with the patient to the most appropriate hospital capable of caring for the patient's condition. (See Chapter 51 in Nelson Textbook of Pediatrics, th 17 edition.) Question . 15. There are several requirements in preparing a child for transfer to a higher level of care (e.g., from the office to the emergency department). In preparing for transfer of a child, which of the following is not recommended? Obtaining written consent for transfer from the patient's parent or guardian Copying diagnostic tests, radiographs and the child's medical record Calling and giving report to the appropriate transport agency Instructing transport agency to call receiving physician to secure acceptance for transfer Explanation: Doctor-to-doctor communication is essential when transferring a critically ill patient. This is not the responsibility of an ambulance company. (See Chapter 51 in Nelson Textbook of Pediatrics, 17th edition.) Documenting name of transport agency and the time that the transport occurred Question . 16. Emergencies involving children are stressful for the child, parent, and EMS-C providers. All of the following are useful in decreasing stress to children and their families in emergency settings except: Keeping the parents away during procedures or resuscitation Explanation: Indeed, most parents can provide additional calming and distraction during procedures, and their presence should be encouraged. The question of parents being present during resuscitation is controversial, but most physicians find that it usually does no harm and may be of value later to grieving parents. (See Chapter 51 in Nelson Textbook of th Pediatrics, 17 edition.) Training staff in calming and distraction techniques Separating the child from other frightening sights and sounds in the treatment area Communicating clearly, with written instructions accompanying verbal information whenever possible Screening for mental health needs Question . 17. The safest and quickest manner to transport a critically ill child from a community hospital to the regional pediatric center is: Have the parents drive the child from their local hospital Request that the local paramedics transport the child Accompany the child in the ambulance with the local paramedics Request that the tertiary pediatric facility assist and transport the patient Explanation: The care and transport of a critically ill child requires staff with specific experience and knowledge of the pediatric population and the illnesses necessitating transportation. In addition, the equipment, medications, and means to monitor children require pediatric-specific expertise. Coordinated efforts with a pediatric transport program yield the safest methods of transport. (See Chapter 53 in Nelson Textbook of Pediatrics, 17th edition.) Question . 18. The transport team from the tertiary hospital is composed of all of the following except: A parent who can assist in the care of the child Explanation: Parents are not expected to provide care during pediatric transports. Nonetheless, if room is available in the transport vehicle, a parent may accompany the child. Usually this is not possible, and the parent follows the transport van in another vehicle. (See Chapter 53 in Nelson Textbook of Pediatrics, 17th edition.) Team members skilled in various aspects of pediatric critical care A dispatch service that facilitates communication with the referring hospitals A medical control physician who is available for telephone consultation Question . 19. Appropriately trained and equipped pediatric transport teams should be able to: Perform major surgical procedures at the referring hospitals Provide appropriate medical care during the transport Explanation: Appropriately trained and prepared (based on information from the referring hospital) transport staff should be able to care for the patient en route to the PICU. This does not mean that a patient's condition cannot deteriorate during transport as part of the natural history of the disease. Special transports (ECMO) are not common and are used for only unusual circumstances. (See Chapter 53 in Nelson Textbook of Pediatrics, 17th edition.) Place a patient in extremis on ECMO Transport a patient without incident Question . 20. The mother of a 5-yr-old near-drowning victim arrives at the pediatric intensive care unit (PICU). She is highly upset and emotional, and forcefully demands to see her child. The best response of the PICU staff to the mother would be: To sit with her and explain the procedures of the PICU, including times permitted for visitation, the number of visitors permitted To direct her to a social worker who would provide a description of the rules for visitation in the PICU To require that she speak with the child's physician before being permitted to visit the bedside To direct her to the parents' waiting area, and inform her that she will be summoned when the time is right To take her as soon as possible to the bedside, after having provided a brief description of what the room might look like, what medical devices will be present, and what level of response she might expect from her child Explanation: It is not always possible for a parent to immediately be brought into a child's PICU room. A health care provider should be there for the parent to explain the patient's condition and facilitate ongoing communication. Nonetheless, this process should be brief, as any delay increases anxiety and possibly mistrust. (See Chapter 54 in Nelson Textbook of Pediatrics, 17th edition.) Question . 21. A child has been in the PICU for 10 days and still faces at least a week of further treatment. Various family members have consulted with a variety of treating medical staff about prognosis. As a result, the family has heard several contradictory versions of what the treatment plan will be. You should now: Advise the family to speak only to you in the future Explanation: All of these answers have been suggested (except the letter to the administrator) and depend on the circumstances in the PICU and the patient. Although "B" has value, once confusion has taken over it is important for one person to communicate with the family. Practically, this is not always possible. (See Chapter 54 in Nelson Textbook of Pediatrics, 17th edition.) Schedule regular meetings where representatives of the different services are present and consensus can be reached Advise the family to write a letter to the hospital administrator Advise the family not to speak to consulting physicians Advise the family that this degree of ambiguity is unavoidable in this setting Question . 22. A 2-yr-old child arrives in the PICU in respiratory distress and soon requires intubation and mechanical support. Because of a heart murmur detected 2 days later, a cardiology consultation is requested, and the fellow performing the consultation speaks with the family, indicating the need for immediate heart surgery. The family is distressed at this news, and wonders why you have not mentioned the possibility of surgery. The most appropriate next step in management is to: Ask the family to discuss the matter further with the cardiology service Contact the chief of cardiology and lodge a complaint about the actions of the fellow Convene a meeting with representatives from your service and the cardiology service, develop a plan, and then meet with the family to present recommendations Explanation: When such communication catches you off guard, regroup the team and family and discuss the events that led to the diagnosis and surgery. Never forget the best interest of the patient despite less-than-optimal communication. (See Chapter 54 in Nelson Textbook of Pediatrics, 17th edition.) Advise the family not to give much credence to the fellow's opinions Summon the fellow to your office and instruct him/her never to speak to the family about treatment plans unless you are present Question . 23. The mother of one of your PICU patients regularly looks through the bedside medical chart of her child. Bedside nurses report this to you and express their discomfort with the practice. Your most appropriate response would be to: Advise the nurses that it is the mother's right to view the chart and nothing should be done Report the matter to the hospital authorities Suggest to the mother that you or your representative would like to go through the chart with her on a regular basis to clarify the jargon and explain the content more fully Explanation: Charts should not be read in isolation. Notes or laboratory data are easily misinterpreted and require a health care worker to help communicate their meaning and significance. (See Chapter 54 in Nelson Textbook of Pediatrics, 17th edition.) Instruct the mother that she may not view the chart since it contains the writings of several different health care providers who have not consented to her viewing it Enlist the help of a social worker to persuade her that viewing the chart is not appropriate Question . 24. You inform the family of a gravely ill child in the PICU that she is very likely to die soon. The family, consistent with their faith, wishes to apply oils to her body and place various amulets on the bed. Your reaction should be to: Refer the matter to the hospital attorney Refer the matter to the chaplain Persuade them that the application of oils and the presence of amulets cannot possibly influence the child's health status Inform them that so long as what they wish to do does not pose immediate threat to their child's health, you support their wishes Explanation: This is a most important example of understanding cultural issues in health, life, and dying. To this family, not performing the ritual may prevent the child from dying peacefully. A chaplain familiar with the family's faith is also useful, whether employed by the hospital or present as the family's personal spiritual advisor. (See Chapter 54 in Nelson Textbook of Pediatrics, 17th edition.) Inform them that hospital policies forbid applying the oil because it is an unauthorized form of medical treatment Question . 25. Proper use of the PRISM scoring system would include: Decision-making in end-of-life issues for a chronically ill child Withdrawal of support decisions for a child with multiple organ failure Comparison of level of disease severity between treatment and control groups Explanation: The Pediatric Risk of Mortality score is based on 17 physiologic variables (vital and neurologic signs, acid-base, blood chemistries, hematologic parameters) subdivided into 26 ranges and taking into consideration age (neonate, infant, child, adolescent). It is best in predicting mortality for populations of patients and not for an individual PICU patient. Decisionmaking at the end of life should never be based on an acute PRISM score, especially in a chronically ill child. It has no relevance or reliability in non-PICU patients, such as those receiving chemotherapy. (See Chapter 56 in Nelson Textbook of Pediatrics, 17th edition.) Assessment of performance of a chemotherapy regimen Question . 26. Which of the following scoring systems is useful for triage decisions? PRISM (Pediatric RISK of Mortality) Pediatric Trauma Score Explanation: The pediatric trauma score is made specifically for triage to a higher-level unit, such as a level I trauma center. The other scores are most useful in assessing physiologic instability resource utilization in an acute PICU setting. (See Chapter 56 in Nelson Textbook of Pediatrics, 17th edition.) APACHE (Acute Physiology and Chronic Health Evaluation) TISS (Therapeutic Intervention Scoring System) Question . 27. Regarding resuscitative efforts, the most important goal is: Restoration of age-appropriate heart rate Appropriate movement of the chest wall Auscultation of equal breath sounds in both lung fields Adequate oxygen delivery and utilization for the body tissues Explanation: Although all of these goals are important, they all reflect the rescuer's ability to restore perfusion and oxygen delivery to vital tissues. The effectiveness of resuscitation can be assessed by visualizing good chest rise and palpating good pulses during rescue breathing and chest compressions, respectively. (See Chapter 57.1 in Nelson Textbook of Pediatrics, 17th edition.) Palpation of equal pulses in all four extremities Question . 28. A 9-mo-old boy is brought to the emergency room in a limp and unresponsive state. Initial examination shows a pulse rate of 35/min and occasional irregular breaths. After initiation of CPR (including tracheal intubation), delivery of oxygen via positive-pressure breaths, and chest compressions, multiple attempts to insert an IV line fail. The most appropriate next step in management should be to: Obtain an arterial blood gas sample Place an intraosseous needle and administer fluids and inotropic agents Explanation: Intraosseous (IO) lines should be placed if venous access is not obtained within 1-2 min of an arrest. The anterior tibia is the most common site utilized. Chemistries and other laboratory analyses can be obtained, including a blood culture, while intravenous fluids and medications can be delivered through an IO line. The risk of infection is very low. Once perfusion improves, venous access is usually attainable, and the IO line can be removed. (See Chapter 57.1 in Nelson Textbook of Pediatrics, 17th edition.) Obtain a "stat" head CT study to evaluate reasons for unresponsiveness Place a transthoracic cardiac pacemaker Place a thoracostomy tube to evacuate a possible pneumothorax Question . 29. A 9-yr-old boy suddenly experiences respiratory distress while dining in a restaurant, and despite obvious great effort to breathe, he seemingly cannot move air. He points to his neck and appears progressively cyanotic. The most appropriate next step should be to: Ask the headwaiter to call 911 immediately Give the child a glass of water and urge him to drink it quickly Carefully review what the child may have eaten last Deliver a series of up to five abdominal thrusts (Heimlich maneuver) Explanation: The child demonstrates the universal sign of airway obstruction from a foreign body. If he is awake, finger sweeps should not be done; the Heimlich maneuver is the treatment of choice. If airway obstruction continues and the patient becomes unresponsive, EMS should be activated. (See Chapter 57.1 in Nelson Textbook of Pediatrics, 17th edition.) Hyperextend the neck in an effort to open the airway Question . 30. Which of the following is true regarding provision of assisted ventilation in the early moments of an emergency? Any technique that helps clear the upper airway and safely produces air movement in both lung fields is acceptable No one should attempt to deliver positive-pressure ventilation until tracheal intubation can be accomplished Mouth-to-mouth ventilation is preferred strongly over bag-mask ventilation In both infants and older children, rescue breathing should involve the rescuer forming a seal over the patient's mouth, with the patient's nose being occluded by the rescuer's thumb and forefinger In proper assisted ventilation, there should be no movement of the abdominal wall during inspiration Explanation: With mouth-to-mouth or with endotracheal tube ventilation, chest rise is paramount. One should avoid air entry to the esophagus by proper placement of the head and neck. Techniques to open the airway differ, especially if there is head and neck trauma. Mouth-to-mouth-and-nose ventilation is appropriate in infants. (See Chapter 57.1 in Nelson Textbook of Pediatrics, 17th edition.) Question . 31. A 2-yr-old patient, previously admitted to the hospital for respiratory distress, is found apneic and pulseless in the early morning hours. She is intubated and ventilated adequately, and an intra-osseous needle is placed, but she remains pulseless. The first medications you should employ are: Lidocaine and epinephrine Epinephrine and norepinephrine Norepinephrine and atropine Lidocaine and atropine Epinephrine and atropine Explanation: By this time in an arrest, an ECG should be available to identify the rhythm. These are the drugs recommended for asystole, but pulseless electrical activity (electrical mechanical dissociation) should lead one to search for a treatable cause of poor pulses in the presence of a cardiac rhythm capable of effective mechanical cardiac activity which should be able to support the cardiac output. (See th Chapter 57.1 in Nelson Textbook of Pediatrics, 17 edition.) Question . 32. Causes of pulseless electrical activity include all of the following except: Supraventricular tachycardia is characterized by all of the following except: Rate >220 beats/minute in infants Response to vagal stimuli Rate >180 beats/minute in children Response to adenosine Gradual onset and gradual termination Explanation: Supraventricular tachycardia (SVT) classically has a sudden onset and sudden termination if it occurs spontaneously.1 in Nelson Textbook of Pediatrics.1 in Nelson Textbook of Pediatrics. adenosine is the treatment of choice. previously known as electrical-mechanical dissociation. Lidocaine is a second but effective choice.) Pericardial tamponade Pneumothorax Question . The drug of choice for shock refractory ventricular fibrillation is: Magnesium Bretylium Aminophylline Amiodarone Explanation: Amiodarone is now the treatment of choice for shock-resistant ventricular tachycardia or ventricular fibrillation. 35.Toxins Pulmonary embolism Intracranial hemorrhage Explanation: Agents that work directly on the heart (toxins. (See Chapter 57. If this is present. In adults. the cause must be searched for and treated to achieve successful resuscitation. Therefore in all ages.) Digoxin Question . (See Chapter 57. 17th edition.) Question . 17th edition. pneumothorax) may all produce pulseless electrical activity. verapamil can cause cardiac arrest. verapamil was once a choice as treatment for SVT. 17th edition. A 9-yr-old with vomiting and diarrhea has a systolic blood pressure of . In children with SVT and poor cardiac output. 34.1 in Nelson Textbook of Pediatrics. hypoxia) or interfere with cardiac output (tamponade. 33. (See Chapter 57. the most accurate method is measuring end-tidal CO2.75 mm Hg. or 88 mm Hg. Normal saline or lactated Ringer solution without glucose is the initial treatment of choice.) Administer 20 mL/kg of lactated Ringer solution in 5% dextrose in water Obtain upper and lower limb blood pressure readings Begin administration of epinephrine or atropine Question . one should do all of the following except: Visualize the vocal cords Monitor end-tidal CO2 Listen for equal breath sounds Listen over the stomach Obtain a lateral chest x-ray Explanation: Most would obtain an anterior-posterior chest xray. To check the proper placement of an endotracheal tube. Most agree . Nonetheless. Controversy exists about the use of bicarbonate therapy to treat this lactic acidosis. Normal blood pressure between 1 yr to 10 yr should be 70 + 2 x age (yr). (See Chapter 57.1 in Nelson Textbook of Pediatrics. You should: Check the fundi for papilledema Administer 20 mL/kg of normal saline Explanation: This is an abnormally low systolic blood pressure. 36. 17th edition. This is less accurate in a patient in asystole. All of the following are anticipated reactions of tissues to the shock state except: Increased capillary filling time due to diminished perfusion of tissues Stage of "warm shock" reflecting initial vasodilation Later stage of "cool shock" due to preservation of blood flow to vital internal organs Increased production of organic acids due to switch to anaerobic metabolism Increased production of bicarbonate due to anaerobic metabolism Explanation: Indeed. anaerobic metabolism produces a severe lactic acidosis. in a patient with a perfusing rhythm. 37. Looking for mist humidity in the ETT is not 100% accurate and may be misleading Question . the injury many be reversible. (See Chapter Question . The factor most clearly predicting mortality in shock is: Cardiac failure Renal failure Hepatic failure Metabolic acidosis Multiple organ system failure Explanation: Multiple organ system failure. also known as multiple organ dysfunction syndrome (MODS). The condition of a patient in the PICU appears to be worsening.2 in Nelson Textbook of Pediatrics. which reflects oxygen delivery and oxygen consumption. The drug pair that meets the dual goals of stimulating the heart and relaxing peripheral vasculature is: Isoproterenol and epinephrine Dopamine and dobutamine Explanation: Dopamine acts on dopaminergic and adrenergic receptors of the heart.) Question . nonetheless. You suggest: A mixed venous O2 saturation measurement Explanation: The mixed venous saturation and possibly a serum lactate level measurement are excellent tests. if possible. mortality increases substantially with each dysfunctioning organ system. If tissue oxygen consumption is greater than oxygen delivery. In many patients.2 in Nelson Textbook of Pediatrics. the mixed venous saturation declines. 17th edition. The mixed venous saturation reflects tissue oxygen extraction. while dobutamine acts on - . because the CO2 generated from the bicarbonate may exacerbate hypercarbia and produce intracellular acidosis. 39. 17th edition. Bicarbonate should not be used if the patient has poor ventilation. (See Chapter 57. 40.) An arterial blood gas analysis A venous blood gas A hematocrit A metabolic panel including assessment of liver and kidney function Question . (See Chapter 57. the underlying etiology (such as hypovolemia or infection) must be treated first. Bicarbonate therapy may increase morbidity and possibly mortality (in experimental animals). 38. and the supervising physician asks for the one best test to determine if shock is present.that. is a serious consequence of shock. low-grade fever.receptors of the heart and the peripheral arteries. Intravenous fluid (crystalloid) resuscitation is urgently needed and should be given as soon as possible. (See Chapter 57. (See Chapter 57.2 in Nelson Textbook of Pediatrics. CBC. A patient in respiratory distress presents to your office. dopamine has -adrenergic effects. 41.) 50 mm Hg >100 mm Hg Question . 43. and lethargy. A 5-yr-old boy presents with petechiae.) Administration of 1-2 mg/kg of furosemide (Lasix) Question . It may need to be repeated often. intravenous antibiotics must be given rapidly. You admit him to the hospital and start an IV infusion.) Dobutamine and amrinone Epinephrine and norepinephrine Amrinone and norepinephrine Question .2 in Nelson Textbook of Pediatrics. 42. or rarely an alveolar diffusion defect. The most appropriate next step in management would be: Administration of high-dose Solu-Medrol or Decadron A CT scan of the head to rule out meningococcal meningitis Collection of blood for a culture. Steroids may be needed later if adrenal insufficiency is demonstrated in a patient unresponsive to fluid boluses and inotropic agents. 17th edition. cardiac right-to-left shunt. 17th edition. intrapulmonary shunt. fresh bruises. In addition. and platelet count Administration of 20 mL/kg of normal saline Explanation: The dizziness and lethargy suggest poor central nervous system perfusion and hypotension. The normal alveolar-arterial (A-a) oxygen gradient is: >300 mm Hg <10 mm Hg Explanation: The alveolar-arterial oxygen gradient in normal children from about 1 month of age onward is usually <10 mm Hg. The most appropriate immediate response is to: Attempt a blood gas determination Immediately intubate the airway and begin positive-pressure .3 in Nelson Textbook of Pediatrics. (See Chapter 57. dizziness. because early therapy of meningococcal sepsis with antibiotics has proven to reduce mortality. Any deviation suggests a ventilation/perfusion defect. In high doses. 17th edition. Nosocomial acquisition of the highly resistant bacterial flora of the PICU.ventilation Call 911 Place the child in a comfortable position. reassess the airway. (See Chapter 57.3 in Nelson Textbook of Pediatrics. (See Chapter 57. but nasal cannulas are usually well tolerated. poor mucociliary transport. The initial ventilator settings are determined by: The patient's underlying disease Explanation: The initial ventilator settings are determined by the patient's condition. (See Chapter 57. or a patient with increased . acute airway compromise and respiratory distress are the most common and potentially predictable causes of "cardiac" arrest in children. and it may help.3 in Nelson Textbook of Pediatrics. oxygen will not harm a patient in respiratory distress.) Question . Nonetheless. These include a patient with normal lungs requiring ventilation for surgery or neurologic problems. Respiratory failure accounts for PICU admissions in what percentage of patients? 100% 80% 50% Explanation: The percentage of children admitted with respiratory failure varies. and mucosal barrier breakdown all increase in the risk of infection. Oxygen should be used-albeit cautiously-in children with chronic hypercarbia to avoid respiratory depression if the oxygen chemoreceptors become inhibited. and provide oxygen and other supportive measures as necessary Explanation: In almost all patients. suppressive broadspectrum antibiotics (which select for superinfection).) <25% Question . Complications of mechanical ventilation include all of the following except: Air leak Obstructed endotracheal tubes Alterations of cardiac output Reduction in nosocomial infections Explanation: Indeed. 17th edition. 17th edition. A mask may frighten some infants. with any indwelling device. endotracheal tube placement increases the risk of infection. 46.4 in Question . who actually have respiratory arrest. 45. atelectasis. 44. a patient with decreased compliance. and high tidal volumes (>15 mL/kg) Question . 49.4 in Nelson Textbook of Pediatrics. 17th edition. (See Chapter 57.g. and low tidal volumes (<6 mL/kg) Low rates. acute tubular or (if even more severe) cortical necrosis may produce intrinsic renal failure. which will further exacerbate the ongoing pulmonary pathology. If hypotension is severe and prolonged. 48. prolonged inspiratory/expiratory times. The most important maneuver in preserving renal function in septic shock is: Intravenous infusion of furosemide Infusion of dopamine at a rate of 1-3 g/kg/min Rapid restoration of the circulating volume Explanation: Renal dysfunction in shock of any type may be due to prerenal or renal etiologies. Prerenal renal dysfunction is due to poor perfusion of the kidney secondary to hypotension. asthma) who require mechanical ventilation may benefit from which of the following initial ventilator parameters? Rapid rates. Acute tubular necrosis is often reversible. prolonged inspiratory/expiratory times. Which of the following statements regarding continuous venovenous hemofiltration (CVVH) is true? . such as adjusting PEEP. If no intrinsic renal injury occurs. FiO2. renal function will improve with restoration of renal blood flow. prolonged inspiratory/expiratory times. 17th edition. or tidal volume for CO2 elimination. Hypoxia and hypercarbia will require different strategies. 17th edition. (See Chapter 57. rate.) Proper antibiosis against the offending organism Question . but cortical necrosis results in chronic renal insufficiency.) Low rates.airway resistance. and tidal volumes <6 mL/kg Low rates. and moderate tidal volumes (8-10 mL/kg) Explanation: Because of high airway resistance. (See Chapter 57. 47.5 in Nelson Textbook of Pediatrics.) The patient's preferences Standard order sets Attempts to normalize the blood gases Question . one must avoid air trapping. short inspiratory times. Patients with severe forms of reactive airways disease (e. particularly during expiration.4 in Nelson Textbook of Pediatrics.. 50. 17th edition. (See Chapter 57. In the severely ill child.000 daltons in size are removed from the bloodstream Explanation: Removal of the molecules helps treat azotemia but is of value in removing inflammatory cytokines in patients with the systemic inflammatory response syndrome.5 in Nelson Textbook of Pediatrics. and giving age-appropriate formulas. This later metabolic problem is often a risk factor for increased morbidity and mortality in an ICU. The differential diagnosis for the afebrile child with nausea and vomiting should include: Intracranial tumor Explanation: Nausea and vomiting are common complaints in a pediatric practice. using the stomach. 52. pharyngitis). Appropriate nutritional support for the child receiving mechanical ventilation for bacterial pneumonia should include: Amino acids. 17 edition.) An elemental formula via nasojejunal tube Question . a catabolic state ensues. it is best to provide nutrition by the enteral route.Circuit patency is dependent on systemic blood pressure Water and molecules less than 17. the practitioner must . Although most are due to non-specific (presumed viral) infections or other identifiable infections (gastroenteritis. This improves nitrogen balance but also reduces the risk of sepsis by lowering the incidence of transmucosal migration (bacterial translocation) of enteric bacteria. (See Chapter 57 Question . 51. otitis media. Which of the following statements is true? Administration of growth hormone diminishes insulin resistance Hyperglycemia is a beneficial state in the catabolic child Branch-chain amino acids are beneficial to the previously normal child Intensive insulin therapy has reduced mortality in adult ICU patients Explanation: Insulin may reverse the catabolic state and prevent hyperglycemia. given intravenously Full maintenance solution containing 35% dextrose An age-appropriate enteral formula via nasogastric tube Explanation: If gastrointestinal motility is normal. (See Chapter 57. 2 g/kg/day.6 in Nelson Textbook of th Pediatrics.) CVVH membranes are poorly biocompatible Hemofiltration can induce hypoalbuminemia Question . On arrival in the ED. Which mode of support might best prevent the progression from grade II to grade III+ hepatic encephalopathy? Fluid restriction and furosemide infusion Porcine hepatocyte column filtration Continuous hemofiltration plus plasma exchange Explanation: Various modalities have been proposed to treat hepatic encephalopathy. head tilt. or cranial nerve abnormalities should suggest an intracranial cause of vomiting. A 15-yr-old boy is struck by a car while walking. 17th edition.7 in Nelson Textbook of Pediatrics. but breath sounds are decreased in the left hemithorax. The controversy has not been resolved. cerebral edema. and brain death within several days. Cortical brain death proponents want to recognize cortical death alone as the criterion for legal death. Pulse is 140/min. 53. respiratory rate 40 breaths/min. which includes the brainstem and obviously precludes effective spontaneous ventilation. which of the following is true? Liver and kidney damage are irreversible Isolated brainstem function might be preserved Explanation: The brainstem may be preserved in the presence of cortical cell death.7 in Nelson Textbook of Pediatrics. (See Chapter 57. Fulminant hepatic failure will lead to encephalopathy. and lowering ammonia levels. Heart sounds are distinct. What is the most appropriate next step in patient management? . 54. In the child who has suffered a severe hypoxic ischemic injury. The nature of the nausea. (See Chapter 57. (See Chapter 57. preventing bleeding.) CT scans do not reveal abnormalities until after 1 mo following injury Intracranial pressure monitoring improves outcome Question . Today we agree on the legal definition of whole brain death. he is alert and has no signs of upper airway obstruction.7 in Nelson Textbook of Pediatrics. associated headache or vision problems. Always be cautious and.) Viral gastroenteritis Salmonella infection Type A influenza Question .) Hemodialysis Question . when possible. 55. and blood pressure 70/50 mm Hg. 17th edition. 17th edition. many have proposed emergency liver transplant. This is a controversial point in the discussion of brain death.always be cautious of an intracranial cause. In addition to controlling intracranial pressure. check the fundi for papilledema. she is being ventilated at a rate of 20 breaths/min.) Intravenous bolus of 3% saline Question .Needle thoracentesis of the left hemithorax Explanation: Although an x-ray may be helpful for diagnosis and fluids may improve venous return. 57. blood pressure is 100/70 mm Hg. On arrival in the ED. (See Chapter 57. His abdomen is diffusely tender. 70/30 mm Hg. What is the most appropriate next step in patient management? Immediate head CT Hyperventilation to attain a PCO2 of 25 mm Hg Gentle hyperventilation and an intravenous bolus of mannitol Evaluation of level of alertness and pupil size and reactivity Explanation: At this point. and peripheral perfusion is good. Pulse is 190/min. A 3-yr-old boy is brought to the ED after falling two stories from an open window. Pulse is 100/min. and blood pressure. 17th edition. 17th . 56. It is clear that he has a femur fracture. (See Chapter 57. the patient has no signs of increased intracranial pressure and thus doesn't need mannitol or hyperventilation. and her chest wall rises adequately. Further clinical assessment in a stable patient before a head CT is quite appropriate. After a 20 mL/kg bolus of Ringer's lactate. A 4-yr-old girl sustains a head injury after pulling a television set onto her. 28 breaths/min. respiratory rate. prompt evacuation of a symptomatic hemothorax or pneumothorax is the most appropriate step. (See Chapter 57.8 in Nelson Textbook of Pediatrics. The most appropriate next step in patient management would be: Insertion of a central venous catheter Emergent abdominal laparotomy 20 mL/kg bolus of Ringer's lactate Explanation: Repeated boluses of Ringer lactate or normal saline solution have been life-saving if administered early and aggressively in pediatric patients with shock. 17th edition. his pulse is 180/min and blood pressure is 72/35 mm Hg. He arrives appropriately immobilized and is alert. The paramedics intubate her at the scene because of inadequate respiratory effort. With time and signs of blood loss.) Upright radiograph of the chest Pericardiocentesis Rapid intravenous bolus of Ringer's lactate Arterial blood gas analysis Question . red cell transfusions become indicated.8 in Nelson Textbook of Pediatrics.8 in Nelson Textbook of Pediatrics. 17th edition. kidneys) but will also demonstrate pancreatic injury.) Plain abdominal radiographs Question . PaO2 90 mm Hg. respiratory rate.edition.50. PaO2 100 mm Hg. It is most useful for solid organs (liver. appropriate goals for pH. (See Chapter 57. (See Chapter 58 in Nelson Textbook of Pediatrics. and intestinal injury. PCO2 60 mm Hg Explanation: This reflects the lung protection strategy with permissive hypercarbia. PCO2 25 mm Hg pH 7. 58. There is no need for hyperoxia. The most appropriate diagnostic study for this patient would be: Abdominal ultrasound study MRI of the abdomen Diagnostic peritoneal lavage Abdominal CT study with intravenous contrast Explanation: Abdominal CT is now the standard for blunt pediatric abdominal trauma. Pulse is 80/min. and PCO2 include: pH 7. oxygenation. 18 breaths/min. 59. packed red blood cells Question .35. In the support of the child with severe ARDS. 110/70 mm Hg. His abdomen is diffusely tender.40. hemorrhage (free fluid). 60. . packed red blood cells 10 mL/kg infusion of O-negative. and PaO2 >90 mm Hg is a sign to reduce the FiO2 and thus potentially decrease the risk of oxygen toxicity. PaO2 125 mm Hg.) 10 mL/kg infusion of cross-matched. For the child with ARDS receiving mechanical ventilation the prone position is alternated with the supine position. spleen. it has replaced peritoneal lavage.25. PCO2 40 mm Hg pH 7.8 in Nelson Textbook of Pediatrics. This measure is used to: Redistribute total body edema Equalize pulmonary blood flow to dependent and independent lung segments Prevent atelectasis and increased consolidation in dependent lung segments Explanation: Body position changes help to improve respiratory function in the previously dependent lung segments. PaO2 60 mm Hg. PCO2 45 mm Hg pH 7. and blood pressure. A 10-yr-old boy is brought to the ED after being kicked in the abdomen by a horse. When available. 17th edition Question . ) Heart or kidney Question . and neutropenia and their complications remain the predominant risks until engraftment occurs. (See Chapter 60 in Nelson Textbook of Pediatrics. In the management of a child with ARDS and respiratory failure. (See Chapter 58 in Nelson Textbook of th Pediatrics. excessive PEEP and PIP. 17 edition. (See Chapter 58 in Nelson Textbook of Pediatrics. Complications of transplantation particularly likely to be seen with bone marrow transplantationinclude: Graft-versus-host disease (GVHD) and veno-occlusive disease (VOD) Explanation: GVHD and VOD may occur after bone marrow transplantation. Split or partial liver transplantation from a living related donor is also successful. all of the following measures are important except: Normalize blood gases Explanation: To avoid oxygen toxicity. This principle of respiratory management has been the most significant improvement in the care of ARDS in the last 10 years. 61. thrombocytopenia.) Decrease the risk of pressure sores Question .) Use permissive hypercapnia Accept PaO2 measurements of 60-80 mm Hg Reduce airway pressures to prevent barotrauma and volutrauma Question . no study has demonstrated a survival advantage to this procedure.) . 62. Early in the posttransplantation period.Although PaO2 may improve with position changes. 17th edition. complications due to pancytopenia occur. 17th edition. carry risks for the donor. but particularly liver transplantation. 17th edition. Anemia. Both procedures. 63. Transplantation of part of an organ from a donor who will survive the donation process is possible with transplantation of: Heart or lung Heart or liver Cornea or heart Kidney or liver Explanation: Kidney transplantation is quite successful from a related living donor. and volutrauma or barotrauma. including death. blood gases should not be normalized. (See Chapter 60 in Nelson Textbook of Pediatrics. particularly when T lymphocyte function or number is reduced. 66. Factors promoting successful acceptance of a transplanted kidney by the recipient include: Maintenance of high urine output and maintenance of lowerthan-normal blood pressure Maintenance of high urine output and use of angiotensinconverting enzyme (ACE) inhibitors Maintenance of high urine output and maintenance of higher-than-normal blood pressure Explanation: It is essential to maintain renal perfusion and renal blood flow. 64. and two doses of epinephrine are administered.) Question . A 2-yr-old boy is pulseless and apneic after extrication from a swimming pool. On arrival. his heart . intubate him. In addition. and administer 100% oxygen while providing bag-endotracheal tube breaths. 65. He is not breathing spontaneously.) Maintenance of low urine output and maintenance of lowerthan-normal blood pressure Maintenance of low urine output and maintenance of higherthan-normal blood pressure Question . One of the more common pathogens responsible for pulmonary infections in immunocompromised patients following transplantation is: Staphylococcus epidermidis Haemophilus influenzae Clostridium botulinum Streptococcus pneumoniae Pneumocystis carinii Explanation: Pneumocystis carinii pneumonia (PCP) is seen in any immunosuppressed patient. An intraosseous line is placed. 17th edition. it may reduce the risk of anastomotic vascular thrombosis. A pulse is found after 10 min of resuscitative measures.GVHD and hyperthermia GVHD and thrombocytosis VOD and hypothermia VOD and thrombocytosis Question . Paramedics start CPR. 17th edition. (See Chapter 60 in Nelson Textbook of Pediatrics. Forced diuresis is critical to achievement of this goal. Prophylaxis against PCP has greatly reduced this potentially lethal complication. (See Chapter 60 in Nelson Textbook of Pediatrics. The child is transferred to your ED. he is deeply comatose with a temperature of 34°C. (See Chapter th 61 in Nelson Textbook of Pediatrics. not proven. The other interventions are dangerous (A). Oxygen saturation is 85%. 67. Which of the following interventions is most likely to improve the patient's neurologic outcome? Administration of high-dose dextrose to improve cerebral metabolism Fluid restriction to minimize potential cerebral edema The addition of positive end-expiratory pressure. Three months later. and he has cold extremities. respectively. His vital signs are normal and he is comatose.5-inch mesh) isolation fence D A 5-ft-tall ornamental iron isolation fence (vertical bars 3 inches apart. Smith install a new swimming pool in their backyard. In counseling the family regarding the child's prognosis. he is intubated and mechanically ventilated. Children under the age of 3-4 yr cannot "swim" safely. Monitor blood pressure to improve cerebral perfusion pressure and possibly to avoid secondary ischemic injury from raised intracranial pressure. and Mrs. which of the following results is most likely to represent a favorable outcome? . 17 edition. none has been proven beneficial. Which of the following safety measures would have been most likely to prevent this tragedy? A lightweight plastic swimming pool cover A pool alarm that sounds when water movement is detected A 5-ft-tall chain link (2. their 2-yr-old son is found pulseless and asystolic in the pool. 68. 17th edition. and PaO 2 145 mm Hg. PaCO2 35 mm Hg. A 4-yr-old boy is admitted to the PICU after resuscitation from a swimming pool submersion. (See Chapter 61 in Nelson Textbook of Pediatrics.12. Mr. horizontal crossbars 45 inches apart) Explanation: Fencing is the best preventive measure.rate is 120 beats/min.) Keeping the patient hypothermic for the first 12-24 hr of his PICU hospitalization Hyperventilation to a PaCO2 of 25 mm Hg Question . On presentation to the PICU. His initial arterial blood gas is pH 7. or provide a false sense of security (E). with a Glasgow Coma Score of 4. Finger stick glucose is 200 mg/dL. administration of an isotonic fluid bolus.) Swimming lessons Question . The best approach is to support normal oxygenation and normal blood pressure to avoid continued cerebral hypoxia and ischemia. and an epinephrine infusion to improve oxygenation and circulation Explanation: Despite many hopeful new interventions for hypoxic-ischemic neurologic injury. He was pulseless and apneic at the scene. blood pressure is 60/25 mm Hg. A 5-yr-old girl who accidentally spilled hot water on her face and trunk and is brought to the emergency room. 17th edition. Unfortunately. 70. (See Chapter 62 in Nelson Textbook of Pediatrics. Hyperbaric oxygenation is of value in severe carbon monoxide poisoning. no laboratory or radiologic test has had any valuable predictive power. while ECMO is of value if the patient doesn't respond to inhaled NO. His condition continues to deteriorate. the presence of clinical improvement during 6-12 or 12-24 hr is the best predictor of intact outcome.) Hyperbaric oxygen Question . (See Chapter 62 in Nelson Textbook of Pediatrics. necessitating high FiO2 and PEEP of at least 12. Therapy is usually started at 5 ppm and titrated to 30 ppm. Of the following. abnormalities (such as those seen on CT scans) are usually self-evident from severity of the abnormalities on clinical examination. Indeed. the most therapeutic approach is: Continued positive-pressure ventilation ECMO Nitric oxide therapy Explanation: Inhaled nitric oxide (NO) has shown promise in managing respiratory (hypoxic) failure after burns. A 16-yr-old boy with 50% body surface area burns from a house fire had been on controlled positive-pressure ventilation.A normal head CT scan Normal intracranial pressure after placement of a Camino monitor A serum glucose of 180 mg/dL on admission to the PICU Resolution of metabolic acidosis on arterial blood gas measurement Spontaneous purposeful movement 12 hr after admission Explanation: Although his initial clinical appearance sounds ominous. (See Cha Question .5 cm H2O. Which of the following is the best method for estimating body surface area burn? Rule of 9s Rule of palm The Lund and Browder chart Explanation: Body surface area (BSA) of various anatomic sites changes with growth and development. Serial examination is the best method to follow the course and predict the outcome. High-frequency ventilation should also be used before ECMO. 69.) A growth chart . 17th edition. The head has a greater body surface area in the youngest children. 73. and there is controversy about whether it should be used at all. Of the following.Question .) Use of topical analgesics Question . the most important treatment is: 7-day course of penicillin 5-day course of penicillin 10-day course of penicillin None of the above Explanation: Initial treatment is to remove the clothing and place warm saline dressings over the wound. the most therapeutic approach is: Aggressive use of topical antibacterial agents with frequent dressing changes Use of intravenous appropriate antibiotics Excision of the burn wounds and grafting Explanation: To prevent infection and to facilitate healing. 17 edition. All of the following statements about brain death are true except: Brain death alone can be used as a justification for withdrawing all life support . 71. (See Chapter 62 in Nelson Textbook of th Pediatrics.) Question . A 4-yr-old girl sustained a 40% second. Which of the following regimens will provide the best pain management? A. C. 72. Penicillin is not an immediate therapy. Morphine bolus Morphine continuous infusion Morphine and Versed bolus Oral morphine and Ativan Explanation: Preemptive narcotics before the procedure and an anxiolytic are the best ways to avoid future behavioral problems and to provide appropriate pain relief. A 10-yr-old boy sustained 30% body surface area burns and had been requiring dressing changes for physical therapy. 17th edition. Of the following. Grafting follows. excision of the wound and removal of all devitalized and dead tissue must be performed rapidly. His pants became ignited and he suffered 20% body surface area burns.and third-degree total body surface area burn from scalding hot water. B. 74. A 10-yr-old boy spilled gasoline on his legs. Boluses may be added to preemptive therapy Question . (See Chapter 62 in Nelson Textbook of Pediatrics. ) None of the above . such as presence of blood and edema. Ancillary tests to confirm brain death include all of the following except: Electroencephalogram Head ultrasound study Explanation: Head ultrasonography demonstrates structural problems. for the PCO2 to rise to the acceptable level of 60 mm Hg. It may take some time. 76. the legal and medically acceptable definition of brain death is whole brain death. separated by at least 48 hr Declaration of brain death in a child 5 yr of age can be made on clinical criteria alone Brain death can be declared even if certain brainstem reflexes (e. such as electrical activity or blood flow. (See Chapter 64 in Nelson Textbook of Pediatrics.g. Some suggest that neocortical brain death should be used. With further experience. Correct performance of an apnea test requires: Pre-oxygenation with 100% O2 for several minutes B Significant elevation of the CO2 level in the blood without resultant respiratory efforts Maintenance of acceptable O2 arterial saturation throughout the test (usually 2-4 min) DAll of the above Explanation: The apnea test helps define absent brain stem reflexes related to respirations.Declaration of brain death in children younger than 2 mo of age requires two assessments. but does not help define function.. papillary response) are still present Explanation: Currently. usually >10 min.) Radionuclide angiogram Apnea test Cerebral angiogram Question . Doppler sonography may be of value.) Acceptance of brain death is sufficient to declare full legal death and should facilitate more organ donation opportunities Question . which includes the brainstem and cortex. (See Chapter 64 in Nelson Textbook of Pediatrics. 75. This remains objectionable because of the rare patients who wake up from PVS. (See Chapter 64 in Nelson Textbook of Pediatrics. four-vessel angiography remains the gold standard. 17th edition. as noted in patients in a persistent vegetative state (PVS). 17th edition. With whole brain death. 17th edition. 17th edition. (See Chapter 65 in Question . Use before. (See Chapter 65 in Nelson Textbook of Pediatrics.Question . Risks associated with the use of succinylcholine include all of the following except: . 77. (See Chapter 65 in Nelson Textbook of Pediatrics. Fentanyl anesthesia in neonates does all of the following except: Avoid postoperative hypotension after PDA ligation Provide analgesia Cause hyperglycemia Explanation: Fentanyl is an effective and safe analgesic anesthetic in newborns and has improved the outcome of complex surgeries in the neonatal period. Sedation and analgesia must also be used when a patient receives neuromuscular blocking agents. Deep sedation is associated with all of the following except: Loss of airway protective reflexes Loss of airway patency Loss of cardiovascular stability Apnea Seizures Explanation: Most agents producing deep sedation will actually suppress physical and electrical seizure activity. It simply means no movement. 80. Akinesia is: Analgesia Sedation Absence of movement Explanation: Indeed. 78. 17th edition.) Avoid hypoglycemia Prevent acidosis Question . Deep sedation is a risk factor for a cardiopulmonary arrest and should only be performed in a setting equipped for response to apnea and bradycardia. and after the procedure has reduced morbidity and probably mortality. akinesia induced by neuromuscular blocking agents (muscle relaxants) has no effect on reducing pain or producing amnesia.) Amnesia Nystagmus Question . 79. during. The most appropriate response to her stated fear would be: Remind her that her daughter is terminally ill. hyperpyrexia.) Hyperkalemia Malignant hyperpyrexia Myoglobinuria Elevated intracranial pressure Question . so it doesn't matter if she becomes addicted Tell her that the oral morphine will be maintained for now.Seizures Explanation: With much better and safer neuromuscular blocking muscle relaxants. succinylcholine is rarely used in the operating room. metabolic acidosis. 82. 17th edition. 17th edition. She is on oral morphine and is experiencing increased frequency of uncontrollable pain. but make no promises about the future Educate her about addiction. . (See Chapter 65 in Nelson Textbook of Pediatrics. There is a great difference between addiction and tolerance to the effects of an opiate.) A positive family history for the disorder Question . and help her make a decision based on the priorities of her child and the family Explanation: Many people (lay and medical) believe in myths and don't understand opiate drugs. tolerance and dependence. Her mother is hesitant to change to intravenous morphine because of fear that her daughter will become addicted to the medication. and cardiovascular collapse. Malignant hyperthermia is associated with all of the following except: Hypercarbia Masseter spasm Metabolic acidosis Muscular dystrophy Morphine Explanation: Narcotic agents are not associated with the development of this potentially serious and lethal familial condition. The spectrum may range from masseter spasms or a mild increase in intraoperative temperature and CO2 elimination to severe muscle rigidity. A 10-yr-old girl with metastatic primitive neuroectodermal tumor (PNET) of the bone is a hospice patient. 81. (See Chapter 65 in Nelson Textbook of Pediatrics. describe the potential risks and benefits of intravenous morphine. Other approaches to pain management that might be considered for the patient described in Question 82 include: Adjuvant analgesics Complementary interventions. such as changing from an oral medication to a parenteral one (fear of needles) or loss of approval by a parent.(See Chapter 66 in Nelson Textbook of Pediatrics. Always be cautious about what children with chronic pain say. The type of pain the patient is experiencing is most likely to be: Neuropathic Explanation: Neuropathic pain may be difficult to diagnose. When the patient described in Question 82 learns that the pain medication might be changed. (See Chapter 66 in Nelson Textbook of Pediatrics. and burning. such as massage or hypnotherapy Spiritual care All of the above Explanation: Adding other non-opiate analgesic medications and using other non-drug related therapies are especially useful in children with poorly controlled chronic pain.) Question . she tells the hospice nurse that she is no longer hurting. This patient has classic symptoms of what used to be called reflex sympathetic dystrophy but is now called complex regional pain syndrome. He describes the pain as sharp. (See Chapter 66 in Nelson Textbook of Pediatrics. He is able to walk on the leg and is attending school. 83. shooting. while behavioral approaches help with anticipation. 84. A bright 14-yr-old boy has had persistent right leg pain for 6 mo after a minor soccer injury has healed. and articulation of pain. type I. 17th edition. because what they say is often mediated by many important related factors Question . coping.) Somatic . 17th edition. What is the likely reason she might have misled the nurse? Because she knows her mother is against the idea. Combination NSAID and opiates may improve control and reduce opiate dose. Results of all imaging studies are normal.) Ask her to trust your judgment that this is the best route to take Question . 17th edition. 85. Because she is afraid of needles Because she doesn't want to bother the nurse All of the above Explanation: The statement by a child who should be in pain that he or she no longer hurts often indicates a fear of something else. Unless the parents will create a stressful situation. 87. Other types of treatment that may be considered for the child described in Question 85 include: Physical therapy Explanation: Physical therapy is very useful in rehabilitation. (See Chapter 66 in Nelson Textbook of Pediatrics. (See Chapter 66 in Nelson Textbook of Pediatrics. 17th edition. Which of the following combination of interventions is most likely to be helpful to her? Education abut the procedure and hypnotherapy Oral sucrose and a pacifier Distraction and a local anesthetic Explanation: Local anesthesia plus distraction is a helpful combination for bone marrow aspiration. they should be present to help their child and even create a helpful distraction. 88. a fear many patients have with chronic neuropathic extremity pain.) Crutches to eliminate weight bearing Home schooling to reduce stress All of the above Question . 89.) Separation from her parents and a benzodiazepine Question . Low-dose tricyclic antidepressants. 17th edition.Visceral Peripheral Question . Indications for admission to the hospital after a burn injury may . Which type of medication should first be considered to treat the pain in the child described in Question 85? Opioids Tricyclic antidepressants and/or anticonvulsants Explanation: Neuropathic pain is quite resistant to opiates. A 3-yr-old girl is scheduled to have a bone marrow aspiration for the first time.) NSAIDs Corticosteroids Question . with or without various anticonvulsant medications. (See Chapter 66 in Nelson Textbook of Pediatrics. 86. Some would also premedicate with a benzodiazepine. It also teaches the patient to use the extremity without an increase in disability. are helpful. 17th edition. The other choices indicate that the patient is at high risk and requires hospitalization. Former premature infants up to perhaps 50-60 wk postconceptual age should be monitored for at least 12-18 hr after anesthesia. 17th edition. The infant had received mechanical ventilation for the first 6 days of life and had apnea of prematurity that resolved 5 wk previously. current practice is still to monitor in hospital for apnea for at least 12 hr after a .) Inhalation injury Question . but preoperative transfusion is not recommended for this hematocrit finding. A 3-mo-old. even after apnea of prematurity has resolved or has never been present. with complications that may include bowel obstruction and infarction of testes or ovaries After spinal anesthetic. If apnea does not occur in the first 12 hr.include all of the following except: Suspected child abuse Electric burns through an extremity Perineal burns Poor follow-up No tetanus immunization Explanation: Lack of immunization against tetanus may be managed with tetanus toxoid and (if a wound is large or dirty) with tetanus immune globulin. The risk of postoperative apnea decreases with age. has been scheduled for repair of bilateral inguinal hernias. The HMO clerk approves the surgery on an outpatient basis. (See Chapter 62 in Nelson Textbook of Pediatrics. The infant is feeding well and gaining weight and has no requirement for supplemental oxygen. All of the following statements are true except: Risk of postoperative apnea is increased by anemia Accepted standard of care includes overnight inpatient apnea monitoring for this infant after general anesthesia because of a significant risk of postoperative apnea Postponing the surgery incurs a small but real risk of incarceration. it is unlikely to occur thereafter. These risks must be balanced against findings of other studies that suggest that delay in repairing inguinal hernia may result in incarceration that cannot be reduced and in additional complications associated with more emergent surgery. Retrospective analysis of anesthetic risk supports delaying elective surgery in the first 1-2 mo of life. Although apnea is reported to be much less common after spinal anesthesia for inguinal surgery in formerly premature infants. and the infant can be sent home from the postanesthetic care unit on the day of surgery Explanation: Postoperative severe apnea can occur in former premature infants up to roughly 50-55 wk postconception. The hematocrit is 28%. Anemia is an independent risk factor for apnea. 90. formerly a 29-wk premature infant. despite premature birth. monitoring for apnea is not required. rhonchi. In response to social overtures (being held. talked to). Auscultation of the chest would most probably show rales. a heart rate of 140 beats/min. but rarely to this degree Question . Anesthesia was maintained with halothane in a mixture of nitrous oxide and oxygen. Malignant hyperthermia is an inherited muscle disorder that produces acute hypermetabolism. obtain a blood culture and have the parents return to the office if the patient remains febrile . and "off" color are typical for the diagnosis of atelectasis Explanation: Atelectasis is the most common cause of fever immediately after surgery. The anesthesia record notes stable vital signs during surgery. tachypnea. and is not alert to stimuli. as often on emergence as on induction. a febrile infant does not smile. 92. which lasted 30 minutes. expressionless face. (See Chapter 65 in Nelson Textbook of Pediatrics.) Untreated pain increases respiratory rates. a respiratory rate of 60 breaths/ min. hypoxemia. hugged. All of the following are true except: Prior to your examining the patient. Although a rapid-sequence induction may reduce the likelihood of aspiration. Clinically significant aspiration pneumonitis is comparatively uncommon in children after anesthesia and surgery but should be suspected if tachypnea.) Question . (See Chapter 65 in Nelson Textbook of Pediatrics. before awakening. and fever are present postoperatively. The nurse notes from the record that anesthetic induction was performed with thiopental sodium and succinylcholine. but the severity of the fever. You are called because the child has a fever of 40. or wheezes. this child's severity of fever. and its likelihood among the differential diagnostic posibilities is very low Atelectasis is a more common cause of postoperative fever than either malignant hyperthermia or aspiration pneumonitis. and the "color isn't too good" description (whether reflecting cyanosis or impaired circulation) are atypical for ordinary postoperative atelectasis. it does not completely prevent aspiration of gastric contents. kissed. and skin color described as "a bit off. aspiration pneumonitis was prevented. previously healthy child is now on the pediatric ward 2 hr after repair of a forearm fracture under general anesthesia. 17th edition. the nurse reviews the anesthetic record and reports that general anesthesia was administered after a "rapid-sequence induction" because the child had eaten a large meal just before the injury. rhabdomyolysis. and the child received morphine. the degree of tachypnea. 17th edition. has a dull. your two primary considerations in the differential diagnosis are aspiration pneumonitis and malignant hyperthermia Because a rapid-sequence induction was successful and the intraoperative vital signs were stable.6°C. 3 mg IV. A 12-yr-old. and fever.spinal anesthesia. and chest radiograph would (eventually) show infiltrates. The most appropriate approach to management is to: Administer ceftriaxone IM after a blood culture and have the parent and child return to the office in the morning If the child is older than 6 mo. 45-kg. increased CO2 production." Over the phone. based on this history. Aspiration may occur. 91. After the lumbar puncture.) Administer a normal saline bolus of 20 mL/kg and reevaluate in 1 hr Question . the patient in Question 97 remains unconscious. and a blood pressure of 65/20 mm Hg. After receiving normal saline pushes. This 3-mo-old had pneumococcal meningitis. and a glucose level of 75 mg/dL. The most appropriate initial therapy is: Application of cooling blankets Administration of aspirin (100 mg/kg) Administration of ceftriaxone (150 mg/kg) Administration of dantrolene (10 mg/kg) Administration of normal saline (20-40 mL/kg) Explanation: Despite an uncertain etiology. A lumbar puncture reveals 3 WBCs/cu mm. the physiologic condition is that of shock. or a bulging fontanel. tone. abdomen. he is noted to be bleeding at venipuncture sites. (See Chapter 49 in Nelson Textbook of Pediatrics. grunting. and CSF specimens for culture and admit the child to the hospital Explanation: Clinical observation of young patients is critical in helping you to evaluate and distinguish the degree of risk of infection and physiologic impairment. 94. Paradoxical irritability may be present with all of the following except: Osteomyelitis Appendicitis Extremity cellulitis Meningitis Pneumonia Explanation: Paradoxic irritability is present when a child becomes anxious and cries during attempts to cuddle and hold the patient. a respiratory rate of 70 breaths/min. urine. 10 RBCs/cu mm. or neck may elicit this response Question . Question . He has a 1-day history of diarrhea (five stools in 24 hr) and is now unresponsive to verbal commands or painful stimuli. 17th edition. a protein level of 30 mg/dL. A previously healthy 7-mo-old white baby boy presents one summer day with a temperature of 41. In addition to observing color. the response to social stimuli is valuable. The circulation needs to be reestablished to perfuse vital organs.Administer acetaminophen and reassess after the infant is no longer febrile Administer ceftriaxone after obtaining a blood. The most likely diagnosis . a pulse of 190/min. 93. Movement of a painful extremity.1°C. 95. (See Chapter 57 in Nelson Textbook of Pediatrics. 17th edition. He has been otherwise well prior to this hospital admission. but it is a distinct disorder characterized by encephalopathy. A 12-yr-old boy with spina bifida experiences respiratory distress during induction of anesthesia for an orthopedic procedure. and other organ failure (heart. He is on ampicillin prophylaxis for recurrent urinary tract infections since birth and has to be catheterized for urination. The most likely diagnosis is: Ampicillin hypersensitivity Urosepsis Reactive airway disease Status epilepticus Latex anaphylaxis Explanation: Latex allergy is common in children with multiple surgical procedures and those who have required catheterization for urinary retention. The presentation includes urticaria.) Malignant hyperthermia Question . 97. and hypotension. (See Chapter Question . or capillary refilling is best classified as: First degree Moderate to severe Second degree Midlevel Full thickness Explanation: This defines a full-thickness burn (also known as a third-degree burn). Past medical history reveals surgery for closure of the spina bifida at age 3 days. fever. 96. liver). wheezing.is: Herpes simplex encephalitis Meningococcemia Salicylate poisoning Hemorrhagic shock encephalopathy syndrome Explanation: Hemorrhagic shock encephalopathy syndrome may look like heat stroke. It has a high mortality rate and morbidity. Bananas may cross- . shock. bleeding. placement of a ventricular peritoneal shunt at 1 mo of life. A burn wound characterized by the absence of painful sensation. disseminated intravascular coagulopathy. and release of contractures at 6 yr of life. (See Chapter 65 in Nelson Textbook .react with latex. Question . 1. Autosomal dominance inheritance is characterized by all of the following except: It affects individuals in every generation It has a high spontaneous mutation rate It affects males more commonly than females Explanation: All others are very typical of autosomal dominant inheritance. The sex ratio should be equal. Often the parents are not affected because the disease is a spontaneous mutation in the child. Neurofibromatosis is an example of an autosomal dominant disease. (See Chapter 69 in Nelson Textbook of Pediatrics, 17th edition.) Phenotypically normal parents do not transmit the disease to the child There is a 50% chance of transmission Question . 2. Autosomal recessive disorders are characterized by all of the following except: Equal sex ratio Consanguinity Greater incidence than that of autosomal dominant disorders Explanation: Autosomal dominant disorders are more common because only one gene is needed for the disease to be manifested. In autosomal recessive disorders, two genes are needed. (See Chapter 69 in Nelson Textbook of Pediatrics, 17th edition.) Asymptomatic carrier state in parents Recurrence risk in sibling of 25% Question . 3. The risk of an autosomal recessive disease in the offspring of a consanguineous mating between first cousins is: 2-4% 6-8% Explanation: This risk is twice the risk of an autosomal recessive trait being manifest in non-consanguineous mating. (See Chapter 69 in Nelson Textbook of Pediatrics, 17th edition.) 10-12% 50% Zero Question . 4. A disease affecting all of the daughters but none of the sons of an affected father, and 50% of the sons or daughters of an affected mother, is most likely: X-linked recessive X-linked dominant Explanation: This is the classic pattern of an X-linked dominant condition. These are rare conditions, which include hypophosphatemic rickets and incontinentia pigmenti (IP). IP is lethal to the male fetus and thus no males are born alive with this disorder. (See Chapter 69 in Nelson Textbook of Pediatrics, 17th ed. Mitochondrial deletion syndrome Autosomal dominant Autosomal recessive Question . 5. A disease passed on only through the mother and affecting both sons and daughters, with variable manifestations in the affected siblings, is most likely: Mitochondrial inheritance Explanation: Because mitochondria in the fetus are usually derived only from the ovum, mitochondrial inheritance typically passes disorders through the mother but to both her sons and daughters. (See Chapter 69 in Nelson Textbook of Pediatrics, 17th edition.) Multifactorial inheritance X-linked recessive inheritance X-linked dominant inheritance Environmentally induced Question . 6. What does a patient with 45 XX (t13q2.1-14q1.3) have? Turner syndrome Female with Robertsonian translocation of chromosomes 13 and 14 Female with reciprocal translocation of chromosomes 13 and 14 Explanation: There are only 45 chromosomes and two sex chromosomes. Therefore, there is one autosomal chromosome missing. The nomenclature states it is a translocation with chromosomal fusion but no net loss of chromosome DNA. Carriers of translocations usually have a normal phenotype. (See Chapter 70 in Nelson Textbook of Pediatrics, 17th edition.) Male with reciprocal translocation of chromosomes 13 and 14 Female with pericentric inversion involving 13q and 14q Question . 7. Uniparental disomy: Covers/uncovers imprinting Can lead to autosomal recessive disorders Is associated with advanced maternal age Is associated with mosaicism for trisomy All of the above Explanation: This is a fascinating area of genetics whereby two chromosomes come to the embryo from the same parent. If three chromosomes are present in the embryo (a trisomy), one may drop out, producing the uniparental disomy. (See Chapter 70 in Nelson Textbook of Question . 8. A baby is born with classic features of Down syndrome. Indications for chromosome studies include: Maternal age older than 35 yr Family history of Down syndrome Prospective adoption of the child Maternal age younger than 35 yr All of the above?such studies are indicated in all children so affected Explanation: Chromosome studies should always be performed in a child with features suggestive of a trisomy. In the case of trisomy 21, if the neonate has a translocation, chromosome studies should also be done on both parents to determine if either is a translocation carrier. If one is a carrier, the risk of an affected sibling is about 30%. (See Chapter Question . 9. Prior to genetic counseling, all of the following should be available or performed except: Physical examination Family history Screening laboratory studies Explanation: Laboratory tests are usually chosen on the basis of the history and physical examination. Counseling will help reveal the need for laboratory studies. (See Chapter 72 in Nelson Textbook of Pediatrics, 17th edition.) Pregnancy and delivery history Review of medical records Question . 10. A mentally retarded 15-yr-old boy is found to have macro-orchidism and large, prominent ears. The most likely diagnosis is: Cerebral giantism Acromegaly Hypothyroidism Trisomy 21 Fragile X syndrome Explanation: Fragile X syndrome is a common chromosomal cause of mental retardation in boys. Affected boys have allelic expansion of trinucleotide repeats to more than 200 (normal is 6-54 repeats). (See Chapter 70 in Nelson Textbook of Pediatrics, 17th edition.) Question . 11. Patients with Turner syndrome should undergo careful analysis of their chromosomes for Y chromosome material because they may: Become masculinized Grow tall Become pregnant Experience gonadoblastoma Explanation: Y chromosome material is present in 5-10% of girls with Turner syndrome. Gonadoblastoma may develop in the ovary, thus necessitating bilateral oophorectomy as a preventive measure. (See Chapter 70 in Nelson Textbook of Pediatrics, 17th edition.) None of the above Question . 12. Kearns-Sayre syndrome and Leber hereditary optic neuropathy are noted in both males and females but are inherited only through the mother. These conditions are examples of: Uniparental disomy Mitochondrial inheritance Explanation: Mitochondrial inheritance of the diseases listed in the question involves mutation of the mitochondrial genome, which originated solely from the ovum. (See Chapter 77 in Nelson Textbook of Pediatrics, 17th edition.) Anticipation X-linked recessive inheritance X-linked dominant inheritance Question . 13. Tay-Sachs disease is best described as: Only affecting Ashkenazi Jews Having a single genetic defect Producing disease in only males Having genetic heterogenicity Explanation: This autosomal recessive disorder, due to a defect in the protein hexosaminidase A, has a degree of genetic heterogenicity in that the defect in Ashkenazi Jews is a frameshift mutation, whereas that in French Canadians is due to a missing gene segment. (See Chapter 69 in Nelson Textbook of Pediatrics, 17th edition.) Having an adult-onset variety Question . 14. FISH in genetic testing is best described as: A way to prepare RNA An enzyme assay to detect mutations Scraping of the buccal cells A test to identify the chromosomal location of an affected gene Explanation: Fluorescence in situ hybridization to a chromosome region with a deletion or mutated gene is a powerful new tool in genetic diagnosis. (See Chapter 68 in Nelson Textbook of Pediatrics, 17th edition.) Fibroblast inhibition selective histology Question . 15. The polymerase chain reaction is best described as: A method to produce many antigenic epitopes A method to amplify small quantities of DNA or RNA Explanation: This powerful tool is helpful in genetic research and diagnosis. It also has been valuable in the rapid diagnosis of infectious diseases such as herpes simplex virus and tuberculosis. (See Chapter 68 in Nelson Textbook of Pediatrics, 17th edition.) A Southern blot A Northern blot A Western blot Question . 16. Trinucleotide repeats are implicated in the etiology of all of the following except: Fragile X syndrome Neurofibromatosis Explanation: Multiple repeats of trinucleotides in the coding region and the untranslated or translated region of these genes produce significant disease in the all of the other named choices. (See Chapter 68 in Nelson Textbook of Pediatrics, 17th edition.) Friedreich ataxia Spinocerebellar ataxia type I Myotonic dystrophy Question . 17. A 1-yr-old presents with a disease that is classically an autosomal recessive trait (such as cystic fibrosis). The father is tested and, with 99% confidence, he is demonstrated to be negative for the carrier state. The most likely explanation is: Mutation to an autosomal dominant trait Uniparental isodisomy transmission from the mother Explanation: Inheritance of two copies of the same affected chromosome from the mother is the best explanation. Although the mother is an obligate carrier, two copies of the affected chromosome are transmitted to the offspring; the normal chromosome was not transmitted. (See Chapter 70 in Nelson Textbook of Pediatrics, 17th edition.) Uniparental isodisomy transmission from the father Imprinting of the missing recessive gene Mitochondrial recombination Question . 18. A newborn infant is noted to have dysmorphic features. The pregnancy was complicated by breech presentation, decreased fetal movements, and polyhydramnios. The child demonstrates hypotonia, a flat face, flattened occiput, epicanthal folds, and abdominal distention. The most likely cause of this child's dysmorphology is: Trisomy 13 Trisomy 18 Edwards syndrome Trisomy 8 Trisomy 21 Explanation: Trisomy 21, or Down syndrome, is also associated with other cardiac, gastrointestinal, and skeletal problems. (See Chapter 70 in Nelson Textbook of Pediatrics, 17th Question . 19. To evaluate the abdominal distention in the patient described in Question 18, an x-ray study of the kidneys, ureters, and bladder (KUB) is performed and reveals a "double-bubble" sign. The best explanation for the neonate's abdominal distention is: Hirschsprung disease Meconium ileus Meconium plug Duodenal atresia Explanation: Duodenal atresia is common in neonates with trisomy 21 and may produce polyhydramnios. After birth, intestinal obstruction requires that the child be NPO and undergo intestinal decompression prior to surgery. (See Chapter 70 in Nelson Textbook of Pediatrics, 17th edition.) Pyloric atresia ALLERGY Question . 1. Which of the following are characteristic of allergens? Proteins of molecular weight <10 kd Proteins of molecular weight 10-70 kd Explanation: Most allergens are proteins that have molecular weights of 10-70 kd. Molecules smaller than 10 kd would not bridge adjacent IgE antibody molecules on the surface of mast cells or basophils. Most molecules larger than 70 kd would not pass through mucosal surfaces needed to reach antigen-presenting cells for stimulation of the immune system. Allergens frequently function in their natural state as proteolytic enzymes, which may contribute to increased mucosal permeability and sensitization. (See Chapter 130 in Nelson Textbook of Pediatrics, 17th ed.) Proteins of molecular weight >70 kd Lipopolysaccharides Carbohydrates Question . 2. Which of the following factors is characteristic of an atopic response? Th1 release of cytokines promoting phagocytosis Th1 release of cytokines promoting synthesis of opsonizing antibodies Th1 and Th2 release of cytokines promoting synthesis of complement-fixing antibodies Th2 release of cytokines promoting phagocytosis Th2 release of cytokines promoting synthesis of IgE antibodies Explanation: Nonatopic subjects respond with the proliferation of T helper type 1 (Th1) cells, which secrete Th1 type cytokines (e.g., IFN- ) involved in the elicitation of allergen-specific IgG antibodies. Th1 cells are generally involved in the eradication of intracellular organisms such as mycobacteria, because of the ability of Th1 cytokines to activate phagocytes and promote the production of opsonizing and complement-fixing antibodies. However, genetically predisposed atopic individuals respond with a brisk expansion of T helper type 2 (Th2) cells that secrete cytokines favoring IgE synthesis. (See Chapter 130 in Nelson Textbook of Pediatrics, 17th ed.) Question . 3. Which of the following types of cells are distributed throughout connective tissues, often adjacent to blood vessels and below epithelial surfaces that are exposed to the external environment, and release a diverse array of mediators of allergic inflammation? Eosinophils Basophils Mast cells Explanation: Mast cells contain or produce a diverse array of mediators of allergic inflammation. (See Chapter 130 in Nelson Textbook of Pediatrics, 17th ed.) Th2 cells Dendritic cells Question . 4. Which of the following antigen-presenting cells are actively phagocytic and reside in peripheral sites such as the skin, intestinal lamina propria, and lungs? Eosinophils Basophils Mast cells Th2 cells Dendritic cells Explanation: Antigen-presenting cells (APCs) are a heterogeneous group of cells that present antigens in the context of the major histocompatibility complex (MHC). Dendritic cells are actively phagocytic cells that reside in peripheral sites such as the skin, intestinal lamina propria, and lungs. (See Chapter 130 in Nelson Textbook of Pediatrics, 17th ed.) Question . 5. Which of the following statements best describes the relationship between allergic disorders and a possible genetic basis? Allergic disorders are a response to only environmental factors Allergic disorders are a response to only environmental factors and infectious agents Asthma and allergic rhinitis are the only allergic disorders with a familial predisposition Any familial predisposition is related to polymorphisms of a single gene located on chromosome 10 Explanation: Both environmental and genetic factors are important in allergic diseases. The clinical expression of these diseases is a complex interaction of many genetic loci and polymorphisms in each of these genes. (See Chapter 130 in Nelson Textbook of Pediatrics, 17th ed.) Any familial predisposition is related to many genetic loci and also many polymorphisms Question . 6. Which of the following factors may contribute to the worldwide rise in prevalence of allergic diseases, particularly in Westernized metropolitan areas? Increasing genetic polymorphisms of CD14 Increased numbers of children in group daycare Excessive use of antibiotics in first 2 yr of life Explanation: Widespread antibiotic use, particularly in young children, alters the microbial flora in the gastrointestinal tract and may produce an environment that is less effective in driving a Th1 response. (See Chapter 130 in Nelson Textbook of Pediatrics, 17th ed.) Reduced exposure to pollutants in Westernized metropolitan areas since 1980 Reduced exposure to indoor allergens Question . 7. All of the following may be signs of moderate to severe airway obstruction resulting from allergic response except: Dennie lines (Dennie-Morgan folds) Explanation: Dennie lines (Dennie-Morgan folds) are prominent symmetric skinfolds that extend in an arc from the inner canthus beneath and parallel to the lower lid margin. Like allergic shiners and the allergic salute, they are signs of persistent rhinorrhea associated with allergic rhinitis. A "silent chest' in a patient with asthma (answer E) is a severe sign suggesting inspiratory and expiratory obstruction. Cyanosis is always present in such severe cases. (See Chapter 131 in Nelson Textbook of Pediatrics, 17th ed.) Supraclavicular and intercostal retractions Cyanosis Pulsus paradoxus Respiratory distress with minimal wheezing and a few crackles Question . 8. A 7-yr-old boy with asthma has roughness over the extensor surfaces of the upper arms and thighs, which is caused by keratin plugs lodged in the openings of hair follicles. This physical finding is termed: Keratosis pilaris Explanation: Xerosis, or dry skin, is the most common skin abnormality of allergic children. Keratosis pilaris, often found on the extensor surfaces of the upper arms and thighs, is characterized by roughness of the skin caused by discrete follicular papules. These are the result of hyperkeratosis with keratin plugs lodged in the openings of hair follicles, and re-form after removal. (See Chapter 131 in Nelson Textbook of Pediatrics, 17th ed.) Fibroepitheliosis Hidradenitis Xerosis Acrochordon Question . 9. The radioallergosorbent test (RAST) determines: Bronchial reactivity to subcutaneous serotonin Bronchial reactivity after inhalation bronchial provocation test The proportion of total allergic immunoglobulin Antigen-specific serum IgE concentrations Explanation: The RAST (radioallergosorbent test) determines the serum IgE concentrations against specific antigens. The RAST correlates well with medical history and allergy skin testing but is somewhat less sensitive than skin testing. (See Chapter 131 in Nelson Textbook of Pediatrics, 17th ed.) The overall allergic risk profile based on absolute eosinophil count, total IgE, and skin test results Question . 10. All of the following statements regarding skin testing for allergic reactivity are true except: Antihistamines given prior to testing may inhibit the reaction Intradermal tests are more sensitive than puncture tests Positive skin test results by intradermal testing correlate better than results by puncture tests with clinical symptoms Explanation: Positive skin test results obtained by the puncture technique correlate better than the more sensitive, less specific intradermal tests with measurements of specific IgE antibody and with the appearance of clinical symptoms on exposure to the allergen. (See Chapter 131 in Nelson Textbook of Pediatrics, 17th ed.) The reaction peaks within approximately 20 min and usually resolves over 20-30 min Larger reactions have greater clinical relevance (See Chapter 131 in Nelson Textbook of Pediatrics. Which of the following is an advantage of skin testing over RAST to determine specific IgE? Skin testing is not affected by administration of antihistamines Skin testing has greater sensitivity than RAST Explanation: Because skin tests are more sensitive than RAST. 17th ed. 12. 11.Question . they are more reliable than RAST in confirming risk of life-threatening anaphylactic conditions. Which of the following physical findings would be least likely on examination of a child with moderate to severe asthma? Tachypnea Wheezing Clubbing Explanation: Digital clubbing (hypertrophic pulmonary osteoarthropathy) is rarely observed in children with uncomplicated asthma and should prompt evaluation to exclude other potential diagnoses. (See Chapter 131 in Nelson Textbook of Pediatrics. All of the other responses are incorrect.) Skin testing is semiquantitative Skin testing is associated with less risk of allergic reaction Skin testing is not confounded by dermographism Question .) Decreased air exchange over the right middle lobe An increased anterior-posterior diameter of the chest . 17th ed. Clothes and bedding should be washed in hot water (>130°F) to kill dust mites. Carpet and upholstered furniture. (See Chapter 132 in Nelson Textbook of Pediatrics. Regular vacuuming with a HEPA-filtered and double-thickness-bag vacuum cleaner is encouraged. (See Chapter 132 in Nelson Textbook of Pediatrics. 14. 17th ed.) Removing carpet decreases cat allergen exposure .Question . cat allergen is light and remains suspended in the air for long periods of time.) Place the mattress and pillow in allergen-proof encasements Wash bed linens in hot water weekly Remove the old carpet from the bedroom Question . Dehumidifiers may be necessary in damp basements. The air conditioning should be set at the lowest level during the warmer months. if retained. 13. should be vacuumed weekly using a vacuum with a HEPA filter. Carpeted flooring is not recommended. All of the following statements regarding decreasing exposure to cat allergens are true except: Removing the cat from the home is the most effective means of reducing exposure to cat allergen Keeping the cat out of the child's bedroom and other rooms where the sensitized child spends large amounts of time reduces cat allergen exposure Washing the cat regularly reduces cat allergen exposure Using HEPA-filtered air cleaners does not reduce cat allergen exposure Explanation: Advice to remove a pet cat from the home or keep it outdoors is often ignored. Use of vaporizers should be avoided. 17th ed. Recommendations to the parents of a child with dust mite allergy to help reduce dust mite exposure should include all of the following except: Use a humidifier regularly Explanation: Household humidity should be kept at less than 50% to inhibit survival of mites. In contrast to dust mite allergens. ) . such as sedation. Secondgeneration antihistamines have fewer sedative adverse effects. 17th ed. should be informed not to expect immediate results. but to retain the present carpeting and upholstered furniture. 17th ed. Her family elects to remove the pet cat from the house. It may take 6 months to 1 year for the levels of cat allergen to drop to a level found in homes without a cat. Which of the following statements regarding antihistamines is true? Classification of antihistamines from type I to type VI is based on increasing antihistamine activity Second-generation antihistamines are distinguished by greater effectiveness than first-generation antihistamines Antihistamines should not be administered in combination with decongestants Antihistamines are more effective in treating than preventing the action of histamine The choice of antihistamines should be based on associated adverse effects and cost Explanation: There is little reason to choose one antihistamine over another except for avoidance of adverse effects.Question .) Question . impairment of function. A 12-yr-old girl with moderate to severe asthma is sensitive to cat dander. 15. (See Chapter 132 in Nelson Textbook of Pediatrics. and thoroughly wiping down all walls and hard surfaces. (See Chapter 132 in Nelson Textbook of Pediatrics. The chemical classification of antihistamines (type I to type VI) does not have functional significance. 16. and cost. What is the length of time required before the levels of cat allergen drop to levels found in homes without a cat? Immediately 2 days 2 wk 2 mo 6 mo Explanation: Cat owners who remove the cat from the home without also removing carpeting and upholstered furniture. 17. It has no bronchodilator properties and is useful only if given prophylactically. 18.Question . frigid air. Which of the following is an advantage of second-generation antihistamines over first-generation antihistamines? Second-generation antihistamines are often less expensive Second-generation antihistamines are more frequently available in oral preparations Second-generation antihistamines have less of a sedative effect and produce less cognitive impairment Explanation: One of the primary advantages of secondgeneration antihistamines is that they are nonsedating or much less so than first-generation antihistamines. Which of the following statements regarding the use of cromolyn in the management of asthma is true? Cromolyn prevents antibody-mediated mast cell degranulation and mediator release Cromolyn prevents non-antibody-mediated mast cell degranulation Cromolyn has no bronchodilator properties The incidence of adverse effects is low All of the above Explanation: Cromolyn prevents bronchoconstriction caused by immunologic as well as nonimmunologic stimuli (e. (See Chapter 132 in Nelson Textbook of Pediatrics.) .g. exercise). 17th ed. (See Chapter 132 in Nelson Textbook of Pediatrics. 17th ed.) Many more second-generation antihistamines are available as over-the-counter medications Second-generation antihistamines are generally more effective than first-generation antihistamines Question .. He has perennial allergic rhinitis. increase in heart rate) than may occur with agents with both 1 and 2 activities. and weed pollens Dust mites. Which of the following groups of allergens would be the most likely to contribute to his symptoms? Dust mites. animal danders. nasal and ocular pruritus. nasal congestion.) Tree. The type of adrenergic activity of drugs most desirable in treatment of asthma is: 1 2 1 2 Explanation: Agents with greater 2-selective activity provide effective bronchodilation with less cardiac stimulation (e. (See Chapter 133 in Nelson Textbook of Pediatrics. 17th ed. grass pollen. and molds Explanation: Perennial allergic rhinitis is most often associated with indoor allergens: house dust mites.Question . 20. He keeps twenty stuffed animals on his bed and sleeps with a feather pillow on an old mattress. weed. He lives in a warm climate.) 3 Question . (See Chapter 132 in Nelson Textbook of Pediatrics. and grass pollens Tree pollen. allergic shiners. 17th ed. conjunctival injection. and milk protein . and occasional fits of sneezing. animal danders. An environmental history is significant for two cats in the home and flooding of the basement when it rains. Seasonal worsening of his symptoms has not been observed. and molds. tree pollens.. 19.g. A 4-yr-old boy experiences perennial clear rhinorrhea. Eosinophils . A teenage boy presents in April with symptoms consistent with seasonal allergic rhinitis. grasses in the early summer. Which of the following allergens is the most likely cause of her symptoms? Milk protein Tree pollen Grass pollen Weed pollen Explanation: In temperate climates. On examination of his nose. and nasal itching.Question .) Question . which of the following findings suggest the need for further evaluation to exclude another diagnosis? Nasal polyps Explanation: Nasal polyps and nasal septal deviation are structural disorders that can mimic allergic rhinitis. clear nasal secretions A transverse nasal crease Continuous open-mouth breathing Question . Physical examination reveals boggy. and weeds in the late summer. A 12-yr-old presents with sneezing.) Pale-to-purple nasal mucosa Thin. pale nasal edema with a clear discharge. 17th ed. clear rhinorrhea. 17th ed. (See Chapter 133 in Nelson Textbook of Pediatrics. A 7-yr-old girl presents with allergic nasal symptoms that are prominent from the middle of August through the first frost. (See Chapter 133 in Nelson Textbook of Pediatrics. The most likely diagnosis is: Foreign body Vasomotor rhinitis Neutrophilic rhinitis Nasal mastocytosis Allergic rhinitis Explanation: Allergic rhinitis is often seasonal and associated with allergic conjunctivitis. 21. 23. airborne pollen responsible for SAR appears in distinct phases: trees pollinate in the spring. 22. and nasal obstruction. 17th ed. Begin seasonal use of oral sympathomimetic drugs. fluticasone. Explanation: Topical intranasal corticosteroids (e. The serum IgE level is normal. 17th ed. (See Chapter 133 in Nelson Textbook of Pediatrics. Begin seasonal use of topical intranasal corticosteroids. (See Chapter 133 in Nelson Textbook of Pediatrics. fever. The physical examination is remarkable only for swollen turbinates and clear nasal secretions. A consultation with an allergist is recommended for patients with allergic rhinitis that does not respond to intranasal corticosteroids. and fever may all herald the onset of sinusitis. facial pain. the patient described in Question 23 complains of headache. 25. 24. A 12-yr-old child presents with watery rhinorrhea.g.to 14-day course of cefpodoxime . A change in the nature of the nasal discharge. paroxysmal sneezing.) Foreign body Rhinitis medicamentosa Choanal stenosis Ciliary dyskinesia Question . Two weeks later. A trial of antihistamine-decongestant therapy for 3 wk has not relieved symptoms. The most likely diagnosis is: Sinusitis Explanation: Sinusitis is a possible complication of allergic rhinitis. and a change in the nature of the nasal discharge to mucopurulent discharge. poor nasal airflow requiring mouth breathing.Chapter 133 Question . and skin test results are negative.. Which of the following is the recommended management? Institute strict measures to avoid outdoor allergen exposure. budesonide) should be used in children with allergic rhinitis that is resistant to antihistaminedecongestant therapy.predominate in the nasal secretions.) Give a 10-day course of amoxicillin Give a 10. Chapter 134 Question .Chapter 133 Elevated serum IgE level Positive result on skin testing for the house dust mite allergen Nasal eosinophils Question . Which of the following is a strong risk factor for persistent asthma in toddlers with recurrent wheezing? Eczema Explanation: Only a minority of young children who experience recurrent wheezing will go on to have persistent asthma in later childhood. The parents of a 3-yr-old girl with a history of several previous coughing and wheezing exacerbations are wondering if their toddler is likely to develop persistent asthma. Which of the following is most useful in establishing the diagnosis of seasonal allergic rhinitis? History of good clinical response to an intranasal corticosteroid preparation History of exacerbation of symptoms in the spring Explanation: Seasonal allergic rhinitis follows a welldefined course of cyclical exacerbation.Question . Chapter 134 Colic Living on a farm Female gender . 26. whereas perennial allergic rhinitis causes year-round symptoms. Several risk factors have been identified. 27. Common triggers of asthma in children include all of the following except: Secondary tobacco smoke Ozone Cold air Exercise Gelatin Explanation: Asthma symptoms may be provoked by numerous events or exposures. 28. ) Daily inhaled corticosteroid .) Albuterol toxicity Pediazole intolerance Question . 29. 17th ed. 30. thus potentially producing theophylline toxicity. He previously experienced coughing and wheezing at least three times each week. For the past 4 days. (A peak serum theophylline concentration 5 mo ago was 16 g/mL). an emergency department physician began treatment with erythromycinsulfisoxazole (Pediazole) for otitis media. The intensity of treatment depends on the severity of exacerbations. (See Chapter 134 in Nelson Textbook of Pediatrics. recommended treatment is with a short-acting inhaled 2agonist as needed for symptoms. This morning the youngster began vomiting. (See Chapter 134 in Nelson Textbook of Pediatrics. The most appropriate treatment option is: Environmental control and patient education only?no medication is indicated Oral theophylline Cromolyn Inhaled 2-agonist as needed for symptoms Explanation: For mild intermittent symptoms of asthma.Otitis media with effusion Question . Two days ago. The need for short-acting inhaled 2-agonist use more than two times a week may indicate the need to initiate long-term-control therapy. he has again experienced mild coughing and wheezing responsive to inhaled albuterol. 17th ed. The likely cause of the vomiting is: Provocation by coughing (post-tussive emesis) Sequelae of otitis media Theophylline toxicity Explanation: The erythromycin (a macrolide antibiotic) component of Pediazole inhibits hepatic theophylline metabolism. A 10-yr-old child has intermittent symptoms of mild asthma. A 4-yr-old boy with asthma has had mild wheezing only four times since you began treating him 6 mo ago with theophylline (Slo-bid Gyrocaps) twice each day. recommended treatment is with a daily-inhaled corticosteroid and a long-acting inhaled 2-agonist. ipratropium. He comes to the emergency department with chest tightness. inhaled -agonist every 20 min for 1 hr.Chapter 134 . a short-acting 2-agonist is also used as needed for quick relief of symptoms. management should include all of the following except: Close monitoring Supplemental oxygen Inhaled albuterol Theophylline Explanation: Initial emergency department management of an asthma exacerbation includes close monitoring of clinical status. 31. In addition.Chapter 134 Daily inhaled corticosteroid. and wheezing. The child described in Question 30 experiences worsening of symptoms. then adding intravenous theophylline could be considered. Alternatives to the inhaled 2-agonist are sustained-release theophylline and a leukotriene receptor antagonist. Asthma symptoms have continued to progress despite frequent albuterol use at home. and if necessary. A 12-yr-old asthmatic boy has developed an asthma exacerbation in the past few days. The most appropriate treatment option is: Oral theophylline Inhaled 2-agonist as needed for symptoms Daily inhaled corticosteroid and oral theophylline Daily inhaled corticosteroid and a long-acting inhaled 2-agonist Explanation: For moderate persistent symptoms of asthma. and oral theophylline 2- Question . which are now persistent and of moderate severity. a long-acting inhaled agonist. dyspnea. Inhaled ipratropium may be added to the agonist treatment if no significant response is seen with the first inhaled -agonist treatment. treatment with supplemental oxygen. and parenteral glucocorticoids. and in moderate respiratory distress.Question . for moderate persistent symptoms of asthma. 32. If a child responds poorly to intensive therapy with nebulized albuterol. In this setting. systemic glucocorticoids (2 mg/kg/day) given either orally or intravenously. and she has used a total of 5 albuterol metered-dose inhalers. with concurrent daily inhaled glucocorticoid Begin use of a long-acting inhaled -agonist each morning Question . sustained-release theophylline is an alternative. The most appropriate management for this asthmatic girl is: Continue albuterol as needed and before physical exercise activities Begin daily controller medication with an inhaled glucocorticoid. In the past year. and cromolyn/nedocromil are the recommended controllers for mild persistent asthmatics.S.Systemic glucocorticoids Question . which happens on most school days except when she uses her albuterol inhaler before going to recess and physical education classes. especially the use of antiinflammatory controller medications Genetic profiling Explanation: The NAEPP guidelines were recently adapted for childhood asthma in a joint-effort publication of the American Academy of Allergy. Lung and Blood Institute and the American Academy of Pediatrics entitled Pediatric Asthma: Promoting Best Practice. National Asthma Education & Prevention Program (NAEPP) guidelines include all of the following except: Regular assessment and monitoring Control of factors contributing to asthma severity Asthma pharmacotherapy. National Institutes of Health's National Heart.S. A 7-yr-old girl has had intermittent asthma symptoms over the past 5 yr. Asthma & Immunology with the U. increasing the dose monthly until good control is obtained Administer daily oral glucocorticoid treatment for one week. initially used more frequently to gain control.Chapter 134 . leukotriene pathway modifiers. Chapter 134 Begin daily inhaled glucocorticoid in a low dose. She mostly has exercise-induced asthma symptoms. 33. 34. she has had two asthma exacerbations with viral upper respiratory tract infections. Components of the U. then a reduced amount in a few months to maintain control Explanation: Low-dose inhaled glucocorticoids. Her asthma symptoms have been treated with inhaled albuterol as needed. Patient education Question . Soaps should have minimal defatting activity and a neutral pHChapter 135 Use of soaps that are especially effective in removing fatty substances . and adding a second rinse cycle Explanation: Using a liquid rather than a powder laundry detergent and adding a second rinse cycle will facilitate removal of the detergent. Features characteristically associated with atopic dermatitis include all of the following except: Allergic rhinitis or asthma Elevated serum IgE level Peripheral blood eosinophilia Lymphopenia Explanation: Most patients with atopic dermatitis have peripheral blood eosinophilia and elevated serum IgE level. 36. Which of the following environmental modifications is recommended? A bland diet. 35. Urticaria and angioedema are not characteristic features of atopic dermatitis Chronic or relapsing course Personal or family history of atopic disease Question . Major features of atopic dermatitis in children include all of the following except: Pruritus Facial and extensor eczema Angioedema Explanation: Angioedema is similar to urticaria but has deeper tissue involvement. 37. A 2-yr-old is diagnosed with atopic dermatitis. Nearly 80% of patients with atopic dermatitis develop allergic rhinitis and/or asthma. especially minimizing meats Installation of wool carpeting instead of synthetic carpeting Use of a liquid rather than powder laundry detergent. Question . 17th ed. (See Chapter 135 in Nelson Textbook of Pediatrics. reasonable but not complete resolution of symptoms is usually possible. Recent studies have reported that atopic dermatitis disappears in approximately 20% of children followed from infancy until adolescence.) Symptoms will resolve completely at puberty Question . 17th ed. The most appropriate prognosis to convey to the parents of the 2-yr-old with atopic dermatitis described in Question 37 is: The child will be asymptomatic with environmental modifications Symptoms will gradually worsen during childhood and persist stably through adulthood Symptoms will exhibit a remittent but progressively worsening course through adulthood Symptoms will gradually decrease over the next several years with an approximately 50% chance of spontaneous improvement Explanation: Atopic dermatitis generally tends to be more severe and persistent in young children. With control of trigger factors and appropriate local treatment. elevated IgE levels. Periods of remission appear more frequently as the child grows older. Which of the following is the major feature of atopic dermatitis? Onset shortly before or during puberty Pruritus Explanation: All patients with atopic dermatitis have pruritus. Spontaneous resolution of atopic dermatitis has been reported to occur after age 5 yr in 4060% of patients affected during infancy. 38. but it had become less severe in 65%. aureus skin infections. Staphylococcus aureus cutaneous infections Elevated serum IgE Immediate skin test reactivity to allergens .Bathing less often than daily Question . (See Chapter 135 in Nelson Textbook of Pediatrics. particularly if their disease is mild. or S. not all patients with atopic dermatitis have other allergic symptoms.) C. However. 39. especially when supported by historical evidence. Drugs and foods are the most common causes of acute urticaria. A 5-yr-old boy with severe atopic dermatitis develops illness with dozens of vesicles primarily covering areas of skin previously affected by atopic dermatitis.) Eczema vaccinatum Coxsackievirus infection Question . Additionally. or eczema herpeticum. (See Chapter 135 in Nelson Textbook of Pediatrics. Kaposi varicelliform eruption is clinically distinguished from zoster by its random distribution. 41. Findings on physical examination are normal except for urticaria.Question . The distribution crosses many dermatomes. which may involve many dermatomes. Similar eruptions have been described in association with vaccinia virus (smallpox vaccination) and coxsackievirus infections. results from herpes simplex virus infection of skin with altered immunity. usually from atopic dermatitis.) Serum IgE and RAST Skin biopsy None of the above . The most likely diagnosis is: Chickenpox Zoster Kaposi varicelliform eruption Explanation: Kaposi varicelliform eruption. Detailed history reveals no clues to the possible etiology. (See Chapter 136 in Nelson Textbook of Pediatrics. A skin biopsy is indicated only if urticarial vasculitis is suspected. 40. A 14-yr-old presents with acute-onset urticaria that has gradually worsened over the past 10 days. Allergy skin testing can be helpful in sorting out causes of acute urticaria. 17th ed. 17th ed. Which of the following diagnostic options is recommended? Systematic elimination diets to determine a possible ingestant cause Allergy skin testing Explanation: No laboratory test confirms or excludes the diagnosis of urticaria. as are the vesicles of zoster. lesions of eczema herpeticum are often isolated and are not grouped. Findings include fever and lymphadenopathy. g. or a microsomal-derived antigen (peroxidate) even if they are euthyroid. Loratadine) is often the preferred therapy for urticaria for school-aged children to minimize the effect on learning and school performance. 17th ed. The incidence of abnormal thyroid function (either increased or decreased T4 and/or increased or decreased TSH) is approximately 20%. 42. Such patients generally have antibodies to thyroglobulin.) Heterophile antibody testing . Which of the following treatment options is recommended for the patient described in Question 42? A bland diet Wearing cotton garments Oral antihistamine Explanation: Antihistamines are usually effective for treatment of urticaria. (See Chapter 136 in Nelson Textbook of Pediatrics. Which of the following laboratory tests is most likely to give abnormal results in a patient with chronic urticaria? Serum IgE level determination Skin prick testing for egg sensitivity C4 level assay Assay for antibodies to thyroglobulin Explanation: There is an increased association of chronic urticaria with Hashimoto thyroiditis. (See Chapter 136 in Nelson Textbook of Pediatrics.. A nonsedating antihistamine (e. 43. 17th ed. Patients with chronic urticaria usually have normal IgE levels.) Oral prednisone Topical corticosteroid Question .Question . Diphenhydramine and hydroxyzine are effective but also cause sedation. 17th ed. respiratory rate 32/min. If the blood pressure does not respond. The most likely diagnosis for the patient described in Question 45 is: Streptococcal toxic shock Scarlet fever Stevens-Johnson syndrome Reye syndrome Anaphylaxis Explanation: Anaphylaxis to penicillin usually occurs within 30-90 min of administration of this drug. A 12-yr-old girl with repeated episodes of streptococcal pharyngitis experiences another episode of sore throat. (See Chapter 137 in Nelson Textbook of Pediatrics. and prednisone are second-line therapeutic agents to be administered after epinephrine and fluids. Next she becomes apprehensive.) . and temperature 37. she has giant urticaria and the following vital signs: pulse 130.Question . 45.2°C. has difficulty swallowing. lactated Ringer's solution should be administered. Benadryl. blood pressure 70/30 mm Hg. One hour later. (See Chapter 137 in Nelson Textbook of Pediatrics. with the first dose given in the office. Anaphylactic shock is often missed as a diagnosis unless a complete history is obtained and there is a high index of suspicion. 17th ed. and oral amoxicillin is started. and develops a hoarse voice. cimetidine. The most appropriate therapy is administration of: Epinephrine Explanation: Intramuscular epinephrine is the treatment of choice.) Prednisone Diphenhydramine Albuterol Lactated Ringer's solution Question . she experiences a "funny feeling" and a tingling sensation around her mouth. The rapid strep test result is positive. 44. On arrival at the emergency department. the penicillin V should be stopped and a substitute nonpenicillin antibiotic chosen.Question . 47. with instructions to return to your office or the nearest emergency department if he becomes short of breath or loses consciousness Schedule a visit for a laboratory test to determine serum trypticase level Substitution of erythromycin for penicillin Question . and Hymenoptera venom Oral drugs carry a higher risk of anaphylaxis than that associated with injected drugs Explanation: Reactions to medications can be reduced and minimized by using oral medications in preference to injected forms. In addition. 17th ed. Which of the following should you recommend? A dose of oral Benadryl. 17th ed. The mother of an 8-yr-old boy with acute streptococcal tonsillitis calls to report that now. he is complaining of itching and has developed hives. foods. latex. with instructions to call again if he has not improved within 30 min Immediate return to your office or the nearest emergency department Explanation: The urticarial reaction described in the question may develop into anaphylaxis.) Anaphylactic reactions to foods usually begin within minutes to 2 hr of exposure Exercise alone can elicit an anaphylactoid reaction . (See Chapter 137 in Nelson Textbook of Pediatrics. within 15 min after the first dose of oral penicillin V that you prescribed.) Careful monitoring at home. 46. All of the following statements regarding anaphylaxis are true except: Virtually any foreign substance can elicit an anaphylactic reaction Most anaphylactic reactions are due to drugs. the latter requires emergency treatment. (See Chapter 137 in Nelson Textbook of Pediatrics. 000. All of the following measures for management of this sting are appropriate except: Infiltration of one half of the epinephrine dose subcutaneously around the site of the sting Repeat doses of aqueous epinephrine at 15-min intervals if necessary Placement of a tourniquet above the site of the sting Incision of and suction of venom from the site of the sting Explanation: With anaphylaxis due to injection of allergen extract or to a Hymenoptera sting on an extremity. 0. which can achieve more rapid effective concentrations than obtainable with subcutaneous injection.000) by subcutaneous injection Aqueous epinephrine (1:1. 48. The first-aid kit that is available includes aqueous epinephrine 1:1. 49. A 16-yr-old with history of anaphylaxis to Hymenoptera suffers a sting on an extremity.000) by intravenous infusion Question . inhaled -agonists. intravenous fluids.5 mL for an adult) by intramuscular injection. The tourniquet can be loosened after improvement or briefly at intervals of 3 min. Aqueous epinephrine (1:1. one half of the dose of epinephrine may be diluted in 2 mL of normal saline and infiltrated subcutaneously at the site of the sting to slow absorption.000 and other necessary medical supplies.01 mL/kg (maximum 0. oxygen. A tourniquet above the site can also slow systemic distribution.3 mL for a child or 0.Question . Immediate transport to an appropriate medical facility should be arranged . Administration of which of the following drugs is the treatment of choice for anaphylaxis? Diphenhydramine orally Diphenhydramine by intravenous infusion Aqueous epinephrine (1:1. Intravenous epinephrine may be added as a continuous drip for persistent shock. Doses can be repeated at 15-min intervals if necessary. Intramuscular or intravenous H1 and H2 antagonist antihistamines. and corticosteroids may also be required. 1:1.000) by intramuscular injection Explanation: The principal treatment of choice for anaphylaxis is aqueous epinephrine. Transport to an emergency department Question . A 12-yr-old child with a history of allergy to yellow jackets is stung and immediately begins experiencing tightness in the chest and wheezing. 50.3 mL for a child or 0. 1:1. accounting for about one half of the anaphylactic reactions reported in pediatric surveys.5 mL for an adult) by intramuscular injection. The most common single cause of anaphylaxis outside of the hospital is: Insect sting allergy Drug allergy Food allergy Explanation: Food allergy is the most common cause of anaphylaxis occurring outside of the hospital.) Oral corticosteroids . 0. (See Chapter 137 in Nelson Textbook of Pediatrics. (See Chapter 137 in Nelson Textbook of Pediatrics. 17th ed. The drug of first choice for management of this child is: Inhaled albuterol Subcutaneous epinephrine Intramuscular diphenhydramine Intramuscular epinephrine Explanation: The principal treatment of choice of anaphylaxis is aqueous epinephrine.) Latex allergy Food-associated exercise-induced anaphylaxis Question .01 mL/kg (maximum 0.000. 17th ed. which can achieve more rapid effective concentrations than obtainable with subcutaneous injection. 51. 17th ed. The symptoms develop as antibodies appear against the antigen at a time when the antigen is still present.) Question . the skin. Which of the following would be the optimal long-term management of the child described in Question 51? Daily oral non-sedating antihistamine Daily low-dose oral corticosteroid Daily inhaled corticosteroid Inhaled corticosteroid immediately upon insect sting Immunotherapy Explanation: Children experiencing systemic anaphylactic reactions to an insect sting should be evaluated and treated with immunotherapy. 17th ed. A 2-yr-old child who has completed 8 days of a 10-day course of cefaclor presents with low-grade fever. irritability. or immune complex disease.) . and a generalized erythematous rash that is mildly pruritic. 52. Immune complexes may stimulate complement and deposit in joints.Question . The most likely diagnosis is: Partially treated meningitis Infectious mononucleosis Kawasaki disease Type I hypersensitivity reaction Type III hypersensitivity reaction Explanation: Serum sickness is a classic example of a type III hypersensitivity reaction. malaise. and the renal glomeruli. 53. lymphadenopathy. (See Chapter 137 in Nelson Textbook of Pediatrics. which is >90% protective. (See Chapter 138 in Nelson Textbook of Pediatrics. 17th ed. intermittent dosing frequency (compared with prolonged. continual administration. 54. 55. then there is no contraindication to receive the product more than once.) No previous exposure (compared with previous administration) All of the above . A 14-yr-old child received equine-derived antivenom for a snake bite 5 yr ago and now requires it again.. previous reaction. (See Chapter 139 in Nelson Textbook of Pediatrics. age (20-49 yr). 17th ed. If there is no alternative treatment. continuous dosing) Explanation: Risk factors for adverse drug reactions include previous exposure. (See Chapter 138 in Nelson Textbook of Pediatrics. immune complex-mediated hypersensitivity reaction). Which of the following statements is true? Premedication with corticosteroids is warranted to prevent serum sickness Negative skin tests indicate that it is highly unlikely that he will develop serum sickness He should not receive this product more than once Serum sickness may begin within a few days of administration of the antivenom Explanation: Because he received the preparation previously. Risk factors for adverse drug reactions include: Topical administration (compared with parenteral administration) Low dose (compared with high dose) Frequent.Question . dose (high). in slow acetylators). Results of skin testing to the product are negative.) Question . he may experience an accelerated form of serum sickness starting before the usual time course of 712 days following injection. Skin testing helps to identify the potential for immediate-type hypersensitivity (IgE antibody-mediated) to the serum components but does not predict serum sickness (a type III. Premedication with corticosteroids does not prevent serum sickness. route of administration (parenteral). Frequent. as well as genetic predisposition (e. intermittent administration is more likely to elicit sensitization than prolonged.g. and dosing schedule (intermittent). Question . he broke out in an itchy rash during treatment with amoxicillin. A 7-yr-old boy presents with fever and otalgia. she informs you that when the child was 4 yr old. it is highly unlikely that the child is still allergic and he can now take the amoxicillin safely Explain to the mother that most adverse drug reactions to amoxicillin are not IgE mediated and that amoxicillin can be safely given. but both contribute to risk for an adverse reaction. Adverse drug reactions are immune complex reactions (Gell and Coombs type III) Approximately 80% of patients with a history of penicillin allergy will have evidence of penicillin-specific IgE antibodies on testing Epidermal detachment of >30% suggests Stevens-Johnson syndrome Question . Prescribe a cephalosporin and explain to the mother that there is no cross-reaction between penicillins and cephalosporins Prescribe a macrolide antibiotic and explain to the mother that there is no cross-reaction between penicillins and macrolides Explanation: Risk factors for adverse drug reactions include previous exposure and previous reaction. On examination. The most appropriate approach to management of this patient would be: Reassure the mother that since more than 2 yr have passed. As you hand his mother a prescription for amoxicillin. 57. 56. he has a bulging right tympanic membrane. A macrolide is recommended for otitis media in penicillinallergic patients Give the child a prescription for amoxicillin. Which of the following statements concerning adverse drug reactions is true? Adverse drug reactions are primarily IgE mediated Drug-induced thrombocytopenia results from circulating immune complexes Both parental and topical exposures to a drug increase the risk for an adverse reaction Explanation: Parenteral administration poses greater risk than topical administration. and instruct the mother to pre-treat him with diphenhydramine (which is . ) There is a 9% risk of anaphylaxis to a first-generation cephalosporin but an almost 0% risk of anaphylaxis to a fourth generation cephalosporin There is a 9% risk of anaphylaxis to a cephalosporin There is a 50% risk of anaphylaxis to a cephalosporin . skin testing to major and minor determinants of penicillin should preferably be done to determine if the patient has penicillin-specific IgE antibodies. anaphylactic reactions after administration of a cephalosporin have occurred in patients with a positive history of penicillin anaphylaxis. and desensitization to the required cephalosporin. If results of skin tests are negative. the patient can receive a cephalosporin with no greater risk than in the general population. in view of the 2% risk of an anaphylactic reaction. (See Chapter 139 in Nelson Textbook of Pediatrics. If a patient has a history of penicillin allergy and requires a cephalosporin. 58. If skin tests are positive to penicillin. recommendations may include administration of an alternative antibiotic. Skin testing to major and minor determinants of penicillin is positive.available without a prescription) Question . cautious graded challenge with appropriate monitoring. 17th ed. The parents of a 6-yr-old girl relate a history of urticarial reaction and vomiting following administration of amoxicillin in the past. Which of the following statements regarding administration of a cephalosporin constitutes appropriate advice for the parents? The child can receive a cephalosporin with no greater risk of anaphylaxis than in the general population There is a 2% risk of anaphylaxis to a cephalosporin Explanation: Although the risk of allergic reactions to cephalosporins in patients with positive skin tests to penicillin appears to be low (less than 2%). 17th ed. conjunctivitis.) . Venom from stinging insects causes IgE-mediated sensitivity that may lead to urticaria and anaphylaxis. A skin biopsy reveals 8% epidermal detachment. asthma). Which of the following best describes this disorder? Toxic shock syndrome Anticonvulsant hypersensitivity syndrome Allergy to sulfamethoxazole Stevens-Johnson syndrome Explanation: Stevens-Johnson syndrome is a blistering mucocutaneous disorder induced by drugs. After 9 days of antibiotic treatment she has recurrence of fever and develops confluent purpuric macules on her face and trunk with erosive mucosal lesions of her mouth and conjunctivae. Epidermal detachment of less than 10% suggests Stevens-Johnson syndrome.) Toxic epidermal necrolysis Question . A 14-yr-old girl.Question .g.. Biting insects may cause local reactions that do not involve IgE. All of the following may be manifestations of insect allergy except: Rhinitis and conjunctivitis Asthma Wheal and flare Anaphylaxis Uveitis Explanation: Clinical findings in allergy caused by insects are similar to those occurring with usual inhalant allergens (e. rhinitis. 17th ed. 59. (See Chapter 139 in Nelson Textbook of Pediatrics. develops fever and a urinary tract infection and is prescribed trimethoprim-sulfamethoxazole. (See Chapter 140 in Nelson Textbook of Pediatrics. classically sulfonamides. 60. who has a long-standing seizure disorder for which she takes phenytoin. (See Chapter 140 in Nelson Textbook of Pediatrics. 61.) Question . (See Chapter 140 in Nelson Textbook of Pediatrics.) Tree pollen allergy House dust mite allergy Anaphylaxis to a wasp sting . 17th ed. 62. 17th ed. Immunotherapy provides symptomatic improvement in all of the following except: Ragweed allergy Local reaction to bee sting Explanation: Local reactions to Hymenoptera venom in children are not managed by immunotherapy.Question . All of the following statements concerning allergic reactions to stinging insects are true except: The majority are due to Hymenoptera There is substantial cross-reactivity among vespid venoms Systemic reactions can occur after the first sting Most reactions are IgE mediated Negative results on skin testing and RAST reliably exclude the likelihood of anaphylaxis Explanation: There are patients with convincing histories of sting anaphylaxis with negative skin test results and RAST results. Question . and consider antihistamines and pain medication Explanation: The child has experienced a large local reaction to the sting. Supportive care directed at the reaction is appropriate. 17th ed. She is experiencing pain. 17th ed. so testing for allergy and providing emergency medications are not warranted. A 10-yr-old girl was stung on her left cheek by a yellow jacket.) Perform or refer her for skin testing to Hymenoptera venom Prescribe self-injectable epinephrine and provide instructions to school/camp All of the above . or children younger than 17 yr who have experienced systemic reactions confined to the skin (generalized urticaria). By 4 hr following the sting the left side of her face is so swollen that her left eye is virtually closed. There are no other complaints.) An epinephrine auto-injector (EpiPen) for administration after a subsequent sting Wearing shoes when outdoors A Medic-Alert bracelet Wearing long pants Question . An 8-yr-old boy experienced immediate urticaria surrounding a large local reaction to a honeybee sting 2 mo ago. 63. He had no other symptoms. Skin testing with honeybee venom has been strongly positive at a weak concentration. The best course of action would be: Apply cold compresses. (See Chapter 140 in Nelson Textbook of Pediatrics. Individuals with local reactions are not at increased risk for severe systemic reactions on a subsequent sting and are not candidates for Hymenoptera venom immunotherapy. are not at significantly increased risk for a severe systemic reaction upon subsequent stings. (See Chapter 140 in Nelson Textbook of Pediatrics. Individuals who have experienced only large local reactions. Appropriate recommendations include all of the following except: Hymenoptera venom immunotherapy Explanation: Immunotherapy is indicated only for systemic reactions. 64. a repetitive cough. he does not need to have self-administered epinephrine readily available . Test results may be negative during a refractory period in the weeks following the reaction. after 4-6 wk if they are negative initially. (See Chapter 140 in Nelson Textbook of Pediatrics.Question .) If results of venom skin tests are negative. It is not necessary to know exactly which insect caused the sting before proceeding with testing and treatment. A 7-yr-old boy was stung by an unidentified insect and within minutes developed generalized urticaria. so they should be repeated. along with RAST. Those at high risk include any individual with positive results on skin tests/RAST who experienced a systemic reaction to a sting with symptoms beyond generalized skin rashes (e. in cases of anaphylaxis proximate to a sting. the benefits outweigh the risks for those at high risk for anaphylaxis from a subsequent sting. epinephrine.g. Which of the following statements is accurate? If skin tests to Hymenoptera venom are performed 1 wk later and results are negative. 65. Although venom immunotherapy may not be indicated for patients without identifiable IgE to the venom. While venom immunotherapy carries some risks for local and systemic adverse effects. He was treated in the emergency department with antihistamines. and corticosteroids. respiratory. patients should be equipped with self-administered epinephrine because the risk for a subsequent anaphylactic reaction is increased. and extreme dizziness. he is not a candidate for venom immunotherapy Testing and venom immunotherapy cannot be undertaken until the insect is identified Venom immunotherapy could reduce the risk for a severe anaphylaxis on a subsequent sting from more than 50% to less than 3% Explanation: Venom immunotherapy is highly effective in reducing the risk of anaphylaxis. cardiovascular reactions) or those 17 yr of age and older with systemic reactions confined to the skin (generalized urticaria). 17th ed. difficulty breathing.. Question . and reddened and edematous conjunctivae. Children often complain of stinging or burning with use of topical ophthalmic preparations and usually prefer oral antihistamines for allergic conjunctivitis. viral infections. corticosteroids. A treatment option effective for the ocular symptoms would be: Topical antihistamines Topical decongestants Topical mast cell stabilizers Topical nonsteroidal anti-inflammatory drugs All of the above?each is an effective secondary treatment regimen for ocular allergies Explanation: Allergic conjunctivitis in the patient with hay fever generally responds well to treatment regimens including topical application of antihistamines. or rarely oral. (See Chapter 141 in Nelson Textbook of Pediatrics. Local administration of topical corticosteroids may be associated with increased intraocular pressure. 67. topical mast cell stabilizers. 66. and topical nonsteroidal anti-inflammatory drugs.) Question .) . A 15-yr-old with a history of seasonal hay fever now also has itchy eyes. 17th ed. (See Chapter 141 in Nelson Textbook of Pediatrics. topical decongestants. Allergen immunotherapy can be very effective in seasonal and perennial allergic conjunctivitis. especially when associated with rhinitis. and cataract formation. The patient described in Question 66 continues to have symptoms. 17th ed. The most appropriate next step in management would be: Combination therapy such as with an antihistamine and a vasoconstrictive agent Immunotherapy Topical corticosteroids Oral corticosteroids All of the above?each is an effective tertiary treatment regimen for ocular allergies Explanation: Tertiary treatment of ocular allergy includes topical. It can decrease the need for oral or topical medications to control allergy symptoms. profuse tearing. (See Chapter 142 in Nelson Textbook of Pediatrics.) The majority of children with positive results on prick skin tests to a food will not react when the food is ingested Elimination diets are the only means to establish the diagnosis of food allergies Question . 69. bronchospasm (wheezing). vomiting. 17th ed. Which of the following is an uncommon clinical manifestation of food allergies? Acute urticaria Angioedema Wheezing Diarrhea Chronic fatigue Explanation: Chronic fatigue is not recognized to be caused by food allergies. Acute urticaria and angioedema (but not chronic urticaria and angioedema). All of the following statements concerning allergic reactions to foods are true except: Skin tests are of little diagnostic value for cell-mediated gastrointestinal hypersensitivity Cow's milk sensitivity is the most common cause of proteininduced enteropathy Gastrointestinal anaphylaxis is mediated by IgA Explanation: Gastrointestinal anaphylaxis generally presents as acute abdominal pain and vomiting that accompanies other IgE-mediated allergic symptoms. (See Box 142-1 and Chapter 142 in Nelson Textbook of Pediatrics. 17th ed.) . acute rhinoconjunctivitis. 68.Question . and protracted diarrhea are all manifestations of food allergies. ) Use only creamy peanut butter and not chunky peanut butter or whole peanuts in the child's diet (after 1 yr of age) . 17th ed. All of the following foods are characteristically associated with allergy except: Peanuts Tree nuts Legumes Explanation: Peanuts. and peanuts. 70. Which of the following approaches is recommended? Begin and extend breast-feeding until age 2 yr. 71. several authorities recommend delaying introduction of major food allergens to infants from atopic families. However. Because of a strong family history on both sides.Question . tree nuts. 17th ed. excluding peanuts from the mother's diet while breast-feeding and from the child's diet until age 3 yr Explanation: There is no consensus on whether food allergies can be prevented. eggs.) Eggs Seafood Question . (See Chapter 142 in Nelson Textbook of Pediatrics. the parents of a newborn baby ask for guidance about preventing their child from developing an allergy to peanuts. (See Chapter 142 in Nelson Textbook of Pediatrics. and seafood all are characteristically associated with food allergies. with exclusion of peanuts from the mother's diet while breastfeeding Begin and extend breast-feeding until age 2 yr. egg until 18-24 mo of age. Recommendations include promotion of breast-feeding with maternal exclusion of peanut and nut products from the mother's diet and delay in introducing major allergenic foods: cow's milk until 1 yr of age. tree nuts. with the mother ingesting gradually increasing amounts of creamy peanut butter from 18-24 mo of age Begin and continue breast-feeding as routinely recommended. with the mother regularly ingesting small amounts of peanuts but not introducing peanuts in the child's diet until age 1 yr Begin and continue breast-feeding as routinely recommended. and seafood until 3 yr of age. (See Chapter 142 in Nelson Textbook of Pediatrics.Question . Which of the following is the most definitive test for diagnosing a food protein-induced enterocolitis? Positive clinical history Positive food challenge Explanation: Unfortunately there are no laboratory studies that help identify foods responsible for cell-mediated reactions. 17th ed. 73. The most likely diagnosis is: Generalized anaphylaxis Milk-induced enterocolitis syndrome Explanation: Food protein-induced enterocolitis syndrome typically manifests in the first several months of life with irritability. Vomiting generally occurs 1-3 hr following feeding. anemia. and failure to thrive. 72. Consequently.or soy protein-based formulas but occasionally result from food proteins passed in maternal breast milk. not infrequently resulting in dehydration. elimination diets followed by food challenges are the only way to establish the diagnosis. 17th ed. A 6-mo-old infant develops protracted projectile vomiting. and lethargy about 2 hr after ingesting a milk formula. Symptoms are most commonly provoked by cow's milk. and continued exposure may result in bloody diarrhea. abdominal distention. (See Chapter 142 in Nelson Textbook of Pediatrics. protracted vomiting and diarrhea.) Positive result on skin prick test Positive RAST result Quantitative IgE level .) Gastrointestinal anaphylaxis Allergic eosinophilic esophagitis Allergic eosinophilic gastroenteritis Question . Physical examination reveals a lethargic child with vital signs of respiratory rate 70/min.g. the abdomen is soft and reveals persistence of the umbilical cord. which predisposes to sepsis. previously healthy. heart rate 185/min. (4) infections with unusual organisms. 10 yr prior to the birth of this child. The most likely diagnosis is: Chronic granulomatous disease Congenital leukemia Kostmann syndrome Leukocyte adhesion deficiency Explanation: This infant has the clinical picture of sepsis with additional findings of delayed separation of the umbilical cord. (3) infections at unusual sites. Peripheral perfusion is poor. A 40-day-old. and the extremities are cool.IMMUNOLOGY Question . extreme leukocytosis. such as (1) two or more systemic or serious bacterial infections.g. sepsis.. meningitis) A systemic fungal infection (e. mean blood pressure 25 mm Hg. chronic. Laboratory studies reveal a white blood cell count of 67. lethargy. or recurrent infections. coccidioidomycosis) Eight or more upper respiratory tract infections within 12 mo Infection with unusual organisms (e.000/mm3. Nocardia) Explanation: Immune evaluations should be initiated for children with unusual. full-term female infant manifests fever. and poor feeding for 12 hr. and (5) infections with common childhood pathogens but of unusual severity. and a family history of early childhood death. Family history reveals that a male sibling died suddenly at the age of 2 mo. and temperature 39. Neutrophil myeloperoxidase deficiency Question ..5°C. This pattern is suggestive of a leukocyte adhesion deficiency. (2) three or more serious respiratory or documented bacterial soft tissue infections within 12 mo.. All of the above . The chest examination reveals retractions. Evaluation of immune function should be initiated for otherwise healthy children with which of the following infections? A life-threatening bacterial infection (e.800/mm3 and a platelet count of 105. 2.g. 1. platelet count. Which of the following is an effective screening test for T-cell function? Absolute lymphocyte count Flow cytometry for CD4 (helper) and CD8 (cytotoxic) T cells Respiratory burst assay Candida skin test Explanation: The Candida skin test is a cost-effective test of T-cell function. (See Chapter 112 in Nelson Textbook of Pediatrics. and immunoglobulin levels assay are cost-effective screening tests. 17th ed. 3.Question .) Mumps antibody titer after mumps vaccination . 4. serious skin and soft tissue infections caused by Staphylococcus aureus and group A streptococcus. 17th ed. The initial screening laboratory evaluation for possible immunodeficiency includes all of the following except: Complete blood count and manual differential Absolute lymphocyte count Platelet count CH50 Explanation: The CH50 assay is a screening test for complement defects and is not recommended for the initial evaluation of immunodeficiency unless there is history of infection with encapsulated organisms.) Immunoglobulin levels Question . The CBC. The absolute lymphocyte count and flow cytometry measure T-cell numbers and subsets but not function. (See Chapter 112 in Nelson Textbook of Pediatrics. A 3-yr-old girl has a history of recurrent. The other studies all provide useful information on B-cell function and antibody production. marcescens abscess suggests chronic granulomatous disease (CGD).) Strongyloides stercoralis hyperinfection syndrome Question . 5. Useful test for evaluation of possible B-cell (antibody) deficiency include all of the following except: Isohemagglutinins Antibodies to tetanus Flow cytometry for CD3 cells Explanation: CD3 flow cytometry detects all T cells. 17th ed. (See Chapter 112 in Nelson Textbook of Pediatrics.) Serum IgA level Total IgG level . which normally constitute about 70% of peripheral circulating lymphocytes. 6. (See Chapter 112 in Nelson Textbook of Pediatrics.Question . S. Infections characteristic of T-cell immunodeficiency include all of the following except: Pneumocystis carinii pneumonia Cryptosporidium diarrhea Severe disseminated chickenpox (varicella) Serratia marcescens abscess Explanation: Abscess formation is not characteristic of Tcell immune deficiency. even though Serratia marcescens is an unusual cause of infections in children. 17th ed. and the specific genes for many of these have been identified. The family history will often be positive in X-linked immunodeficiency syndromes. Which of the following constitutes appropriate advice for the parents regarding possible future pregnancies? Immune deficiencies primarily occur as spontaneous mutations and therefore no genetic counseling is necessary. (See Chapter 112 in Nelson Textbook of Pediatrics. 17th ed. 17th ed. Explanation: Many of the recognized immunodeficiencies are inherited. preparative methods inactivate these viruses. (See Chapter 114 in Nelson Textbook of Pediatrics. The absence of other immunodeficiencies in this family indicates that this immunodeficiency most likely does not have a genetic component. hepatitis B virus.) Systemic reactions . and therefore genetic counseling is not accurate. Nonetheless. Complications of intravenous immunoglobulin (IVIG) therapy include all of the following except: Anaphylaxis Fluid overload Transmission of HIV Aseptic meningitis Explanation: AIDS has not been reported as a complication of the use of intravenous immunoglobulin (IVIG) prepared from human donors. IVIG has been a remarkable aid in the treatment of patients with congenital antibody deficiency states. There is no other family history suggestive of immunodeficiency. Specific genetic tests are available for many of these immunodeficiencies. A 2-yr-old boy is diagnosed with an immune deficiency. Inherited immunodeficiencies reflect the complex interactions of multiple genes with variable expressivity. This immune deficiency is almost certainly X-linked. 7. and hepatitis C virus. Donors are screened for human immunodeficiency virus. furthermore.) Question . 8. None of the above. but not all immune deficiencies are X-linked.Question . The remaining choices are rare but reported complications of IVIG therapy. but this deficiency is occasionally associated with illness. Affected patients are also unable to respond to immunizations. IgA. IgM.33%.) Question . (See Chapter 114 in Nelson Textbook of Pediatrics. 17th ed. is suggested by low concentrations of all Ig classes. or Bruton agammaglobulinemia.Question .) X-linked agammaglobulinemia X-linked lymphoproliferative syndrome Ataxia-telangiectasia . and no palpable lymph nodes. unlike infants with transient agammaglobulinemia of infancy. and IgE are below the 95% lower limits for age. 17th ed. A 1-yr-old child is found to have a normal WBC count but no circulating B cells. small tonsils. 9. 10. The most likely diagnosis is: Ataxia-telangiectasia X-linked lymphoproliferative syndrome DiGeorge syndrome Common variable immunodeficiency X-linked agammaglobulinemia Explanation: X-linked agammaglobulinemia (XLA). (See Chapter 114 in Nelson Textbook of Pediatrics. Serum concentrations of IgG. Most affected persons are healthy. with a frequency of 0. The most common defined immunodeficiency disorder is: Common variable immunodeficiency Selective IgA deficiency Explanation: Selective IgA deficiency is the most common well-defined immunodeficiency disorder. Many patients have partial or incomplete DiGeorge syndrome.) Enteroviruses Catalase-positive bacteria Neisseria meningitidis Pneumocystis carinii Question . these patients may have sufficient lymphocyte activity to avoid serious infections. (See Chapter 115 in Nelson Textbook of Pediatrics. The X-linked lymphoproliferative (XLP) syndrome is classically associated with overwhelming infection by which of the following agents? Epstein-Barr virus Explanation: The genetic defect for X-linked lymphoproliferative (XLP) syndrome also predisposes to severe primary EBV infection.Question .) . most patients with complete (severe) DiGeorge syndrome manifest serious infections before 1 yr of age. 17th ed. 17th ed. which is associated with 50% mortality in affected patients. Features of the complete DiGeorge syndrome include susceptibility to infection and: Neonatal hypocalcemia Anomalies of the great vessels Graft versus host disease after blood transfusion with nonirradiated blood Micrognathia Onset of infections after age 12 mo Explanation: Because T-lymphocyte deficiency is present at birth. 11. 12. (See Chapter 114 in Nelson Textbook of Pediatrics. Children with partial DiGeorge syndrome may have little trouble with infections.) More frequent occurrence in males . All of the following statements regarding DiGeorge syndrome are true except: It occurs approximately equally in both males and females. Variable hypoplasia of the thymus and parathyroids is more common than complete aplasia. which is an X-linked recessive syndrome. All of the following are prominent features of Wiskott-Aldrich syndrome except: Atopic dermatitis Thrombocytopenia Recurrent infections with encapsulated bacteria Autosomal dominant inheritance Explanation: The combination of atopic dermatitis. 17th ed. reduced. 17th ed. depending on the degree of thymic deficiency. 14. (See Chapter 116 in Nelson Textbook of Pediatrics. Question . the absolute lymphocyte count is usually only moderately low for age. Explanation: In persons with DiGeorge syndrome. Lymphocyte responses to mitogen stimulation are absent. Absolute lymphocyte counts are usually markedly low. and susceptibility to infection in males is the classic presentation of Wiskott-Aldrich syndrome. (See Chapter 115 in Nelson Textbook of Pediatrics. or normal. thrombocytopenic purpura. 13.) Concentrations of serum immunoglobulins are usually normal.Question . bones. 17th ed. Abnormalities in dentition.) .Question 15. 16. The recommended treatment for severe combined immunodeficiency is: Gene therapy Monthly IVIG Monthly IVIG and IFNMonthly IVIG and IL-8 monoclonal antibody Stem cell transplantation Explanation: Stem cell transplantation remains the most important and effective therapy for severe combined immunodeficiency. manifestations of hyper IgE syndrome frequently include: Recurrent pneumonia Pneumatoceles Recurrent fractures Hyperextensible joints All of the above Explanation: The hyper IgE syndrome is inherited as an autosomal dominant trait with variable expressivity that is associated with recurrent pneumonias and pneumatoceles. and connective tissue are common. (See Chapter 116 in Nelson Textbook of Pediatrics. These patients do not require pretransplantation conditioning or prophylaxis of graft versus host disease.) Question . (See Chapter 116 in Nelson Textbook of Pediatrics. 17th ed. 17th ed. Monocytes persist for 26-104 hr in the circulation and can persist in tissues as macrophages for months. Only neutrophils kill organisms by ingestion. have an unlimited capacity to divide.Question . Explanation: Neutrophils and monocytes share many primary functions. Monocytes in tissues (macrophages) can persist for months.) There are many varieties of mononuclear phagocytes. (See Chapters 118 in Nelson Textbook of Pediatrics. 17. All of the following statements regarding monocytes and neutrophils are true except: Monocytes. In contrast. unlike neutrophils. when mature. All of the following cell types are monocyte-derived except: Dendritic cells Microglial cells Kupffer cells Osteoblasts Explanation: Tissue macrophages derived from monocytes include osteoclasts. Neutrophils persist for 6 hr in the circulation. 17th ed. but only one type of neutrophil Question . which are large multinucleated cells associated with the absorption and removal of bone. Monocytes remain longer in the circulation than do neutrophils. including the ability to ingest organisms. Monocytes leave the bloodstream and differentiate in the tissues into macrophages with variable morphology and function. 18. osteoblasts arise from fibroblasts and.) Multinucleated giant cells . are associated with the production of bone. (See Chapters 117 and 118 in Nelson Textbook of Pediatrics. 17th ed. (See Chapters 118 in Nelson Textbook of Pediatrics. such as giardiasis (Giardia lamblia) or pinworm infestation (Enterobius vermicularis).or lymphocyte receptors for IL-12. suffer a severe. Physical examination reveals generalized lymphadenopathy. Past medical history is negative for infection except for Salmonella paratyphi septicemia and liver abscess at 3 yr of age. and selective susceptibility to infection by nontuberculous mycobacteria such as Mycobacterium avium or bacille Calmette-Guérin (BCG). the acid-fast stain is positive. or in IL-12 itself. About half of these patients have had disseminated Salmonella infection. Cervical lymph node biopsy shows marked histiocytic infiltration but no granulomas or giant cells.) Severe combined immunodeficiency disease Normal child Langerhans cell histiocytosis Question . successfully treated with antibiotics. A 6-yr-old girl has had intermittent fever.Question . These abnormalities are now grouped under the term leukocyte mycobactericidal defects. and a chest film shows prominent paraaortic nodes. and weight loss since starting first grade 4 mo ago. profound.) Wiskott-Aldrich syndrome . decreased appetite. All of the following are typically associated with an eosinophilic response and eosinophilia except: Allergic rhinitis Hypersensitivity drug reactions Trichinosis (Trichinella spiralis) Pinworms (Enterobius vermicularis) Explanation: Eosinophilia is associated with tissueinvasive helminthic parasites but not parasitic infections that are solely intraluminal. 20. The most likely diagnosis is: Congenital hypogammaglobulinemia Leukocyte mycobactericidal defect Explanation: Individuals with inherited deficiency in macrophage receptors for IFN. 19. (See Chapter 119 in Nelson Textbook of Pediatrics. 17th ed. ) Myeloperoxidase deficiency Excessive formation of H2O2 . 17th ed.Question . Neutrophils and monocytes have the ability to ingest but not the ability to kill catalase-positive organisms. frequently also associated with infection of the cord stump. (See Chapter 120 in Nelson Textbook of Pediatrics. The phagocytic defect of chronic granulomatous disease is: Defect of transendothelial migration Inability to ingest microorganisms Inability to kill some microorganisms Explanation: Chronic granulomatous disease is characterized a defect in the generation of microbial oxygen metabolites. Delayed separation of the umbilical cord after birth suggests which of the following types of immune dysfunction? B-cell defect T-cell defect Combined B. 17th ed.and T-cell defect Phagocyte function defect Explanation: Children with phagocyte dysfunction may have a history of delayed separation of the umbilical cord. 21. (See Chapter 120 in Nelson Textbook of Pediatrics. 22.) Complement component deficiency Question . A 5-yr-old boy presents with his third episode of painful cervical lymphadenitis. 17th ed. All of the following are associated with chronic granulomatous disease (CGD) except: X-linked and autosomal recessive inheritance Pyloric outlet obstruction Aspergillus pneumonia Perianal abscess Hypogammaglobulinemia Explanation: Because of chronic or recurrent infections. Each was treated with incision and drainage. The most important laboratory test is: PCR assay for ADA deficiency Fluorescence assay using dihydrorhodamine 123 Explanation: This boy has a history suggestive of chronic granulomatous disease. most patients with CGD demonstrate hypergammaglobulinemia. (See Chapter 120 in Nelson Textbook of Pediatrics. and cultures grew Staphylococcus aureus. It is being rapidly replaced by a more accurate flow cytometry fluorescence assay using dihydrorhodamine 123 (DHR). At the age of 2 yr. 24.Question .) Question . The NBT test measures the neutrophils' ability to generate superoxide anion and thereby kill ingested bacteria. (See Chapter 120 in Nelson Textbook of Pediatrics. The nitroblue tetrazolium (NBT) is the classic test for chronic granulomatous disease.) MAC-1 assay Neutrophil count Bone marrow aspiration . 23. 17th ed. he required surgical aspiration of a liver abscess. Long-term effective therapy for the patient described in Question 24 is best accomplished with: Intravenous immunoglobulin IFNExplanation: IFN. 17th ed. 25. (See Chapter 120 in Nelson Textbook of Pediatrics. The most likely diagnosis for the patient described in Question 24 is: Bruton agammaglobulinemia AIDS Chronic granulomatous disease Explanation: Chronic granulomatous disease (CGD) in the patient described in the question is most commonly inherited as an X-linked disorder (in approximately two thirds of cases) and is associated with an absence of cytochrome b.increases superoxide anion generation in vitro and reduces the incidence of new infections.Question . Longterm use of trimethoprim-sulfamethoxazole may also be effective in reducing infections. (See Chapter 120 in Nelson Textbook of Pediatrics.) Kostmann disease Cyclic neutropenia Question . 17th ed.) IL-2 Bone marrow transplantation Granulocyte transfusion . 26. NBT testing reveals failure to generate intracellular superoxide anion. 17th ed.) Shwachman-Diamond syndrome Question . benign neutropenia associated with various non-life-threatening viral infections is the most common cause of neutropenia in previously healthy children. 27.Question . Which of the following is the most likely diagnosis? Cystic fibrosis Shwachman-Diamond syndrome Explanation: Schwachman-Diamond syndrome is an autosomal recessive disorder characterized by digestive abnormalities and leukopenia. A 5-mo-old girl presents with diarrhea and malabsorption. 28.) Cyclic neutropenia Chronic granulomatous disease Severe combined immunodeficiency . (See Chapter 121 in Nelson Textbook of Pediatrics. and on initial laboratory testing has a WBC count of 900/mm3. or preeclamptic processes is often asymptomatic and transient. 17th ed. (See Chapter 121 in Nelson Textbook of Pediatrics. autoimmune. Neonatal neutropenia due to alloimmune. Neutropenia is noted in children in all of the following conditions except: Kostmann disease Viral infection Maternal preeclampsia Hunter syndrome Explanation: Transient. All of the following may be associated with neutropenia except: Leukocyte adhesion deficiency Explanation: Persons with leukocyte adhesion deficiency have impaired transendothelial migration.Question . may occur. with a paucity of neutrophils in the infected tissues. The circulating neutrophil count with infection is typically above 30. (See Chapter 121 in Nelson Textbook of Pediatrics. notably with Clostridium perfringens.) . (See Chapter 121 in Nelson Textbook of Pediatrics. notably with Clostridium perfringens All of the above Explanation: Severe infections including pneumonia and life-threatening sepsis. 17th ed.) Shwachman-Diamond syndrome Cartilage-hair hypoplasia Chédiak-Higashi syndrome Glycogen storage disease type Ib Question .000/mm3 and can surpass 100. 29.000 per mm3. 17th ed. 30. Features associated with cyclic neutropenia include: An oscillatory period of 21 ± 3 days Oral ulcerations and stomatitis Serious infections including pneumonia Septicemia. 000/mm3. 10% eosinophils. 17th ed. The platelet count is 650. A brother and a female cousin died at the ages of 18 mo and 2 yr. 17th ed. and at times lethal (by age 3 yr) infection.Question . 31. Long-term treatment of the disease described in Question 31 is best accomplished with: Prophylactic antibiotics Intravenous immunoglobulin monthly IFNRecombinant human G-CSF Explanation: Recombinant human granulocyte colonystimulating factor (rhG-CSF) is the treatment of choice and has dramatically improved the neutrophil count while reducing the incidence and severity of infection.) Cyclic neutropenia Chronic granulomatous disease Question . an autosomal recessive severe infantile form of agranulocytosis. respectively.) Stem cell transplantation . The most likely diagnosis is: AIDS Severe combined immunodeficiency Kostmann disease Explanation: Kostmann disease. 35% monocytes. A 6-mo-old child presents with recurrent cellulitis and bacteremia due to Staphylococcus aureus. and 50% lymphocytes. 32. (See Chapter 121 in Nelson Textbook of Pediatrics. manifests with persistently low absolute neutrophil counts (<200/mm3) and severe. The white blood cell count is 2500/mm3 with 5% neutrophils. (See Chapter 121 in Nelson Textbook of Pediatrics. recurrent. A 12-yr-old boy has a total white blood cell count of 13. 33.000/mm3. and lymphopenia Question . 25% lymphocytes. An increase in band forms is termed a "shift to the left. 34. shift to the left. and the absolute lymphocyte count is 2. C6.and T-cell defect Phagocyte function defect Complement component deficiency Explanation: Congenital deficiencies of the terminal components of complement (especially C5.Question . (See Chapter 124. 14% bands. 17th ed. Repeated meningococcal infections suggest which of the following types of immune disorder? B-cell defect T-cell defect Combined B.) Leukocytosis and shift to the left Neutrophilia.800/mm3.2 in Nelson Textbook of Pediatrics. The percentage of neutrophils is within normal limits. or C8) have been associated with repeated meningococcal and extragenital gonococcal infections. and therefore there is not a lymphopenia. and therefore there is not a leukocytosis.) . and 1% monocytes. C7. The percentage of lymphocytes is within normal limits." The total white count is normal. Which of the following describes this result? Neutrophilia Leukocytosis Shift to the left Explanation: Band forms normally constitute 1-5% of the circulating neutrophils. and therefore there is not a neutrophilia. (See Chapter 122 in Nelson Textbook of Pediatrics. 17th ed. with 60% neutrophils. Cells for allogeneic transplantation are obtained from a nonidentical person.) . She has an HLA-matched family donor. 17th ed. 35.Question .000/mm3). thrombocytopenia (platelet count <20. and monthly IVIG Recombinant human G-CSF. A 4-mo-old girl has severe persistent neutropenia (absolute neutrophil count 200 cells/mm3). (See Chapter 125 in Nelson Textbook of Pediatrics. and reticulocytopenia (reticulocyte count <1% when anemia is present). and monthly (as needed) red blood cell transfusions Stem cell transplantation Explanation: Stem cell transplantation is the treatment of choice for patients with severe aplastic anemia who have an HLA-matched family donor.and IL-8 IFN. 17th ed. A 10-yr-old boy undergoes stem cell transplantation with stem cells harvested from his 14-yr-old sister. platelet-derived growth factor. Which of the following terms describes this type of transplantation? Autologous Syngeneic Familiogeneic Allogeneic Explanation: Cells for autologous transplantation (after removal of cancer cells) are obtained from the patient. (See Chapter 125 in Nelson Textbook of Pediatrics. 36. platelet-derived growth factor.(. Cells for syngeneic transplantation are obtained from an identical twin.) Xenogeneic Question . The treatment of choice is: IFNIFN. 000 mL of feces per day. A 9-yr-old boy undergoes stem cell transplantation for acute myelogenous leukemia in remission. hyperbilirubinemia. 38.) Acute cytomegalovirus infection Reactivated cytomegalovirus infection . generalized erythroderma. He complains of xerostomia. and diarrhea. bilirubin 8. 37. (See Chapter 126 in Nelson Textbook of Pediatrics. The most likely diagnosis is: Post-transplantation Sj gren syndrome Acute graft versus host disease Chronic graft versus host disease Explanation: Chronic graft versus host disease resembles a multisystem autoimmune process with maculopapular rash. (See Chapter 127 in Nelson Textbook of Pediatrics. and diarrhea with >1. 17th ed. usually after 100 days after transplantation. The most important factor contributing to graft failure and graft rejection is: HLA disparity Explanation: HLA disparity is the most important variable influencing graft failure and graft rejection. 17th ed.) Pretransplantation alloimmunization by transfusions The conditioning regimen Transplanted stem cell dose Post-transplantation viral infections Question . Approximately 4 mo after transplantation. he has persistent maculopapular rash over 25% of his body.4 mg/dL.Question . (See Chapter 129 in Nelson Textbook of Pediatrics.Question . Explanation: The renal toxicity associated with cyclosporine can be reduced by adjusting dosing based on blood cyclosporine levels. The only significant drug interaction requiring dosage adjustment is with ketoconazole.) Question . are uncommon. 39.) Growth depression Cataracts . The dosage should be adjusted regularly based on drug levels. other than immunosuppression. may result in hypothyroidism. Which of the following statements regarding cyclosporine is true? Adverse effects. Cyclosporine is as effective as methotrexate for post-stem cell transplantation immunosuppression. 17th ed. 17th ed. 40. Well-recognized late effects of total body irradiation as part of the preparative regimen for stem cell transplantation include all of the following except: Leukoencephalopathy Secondary malignancies Hyperthyroidism Explanation: The use of total body irradiation (TBI) with or without additional conventional irradiation involving the thyroid gland. (See Chapter 128 in Nelson Textbook of Pediatrics. Cyclosporine should generally be used with tacrolimus for synergy. not hyperthyroidism. 1. 17th ed. Urine obtained by suprapubic puncture should normally be sterile. (See Chapter 160 in Nelson Textbook of Pediatrics.Question . A 14-yr-old boy in Connecticut has an illness characterized by fever and an erythematous rash. any bacterial growth is considered significant. The most likely interpretation of these serologic tests is: Acute Lyme disease Atypical Lyme disease Relapsing disease . and of 105 organisms/mL or greater in urine collected by clean-catch (midstream) void. An initial Borrelia burgdorferi IgG antibody titer was 1:32. What is the minimum concentration of bacterial growth in the culture that would generally be considered diagnostic of a urinary tract infection in this young girl? 1 bacterium/mL 10 bacteria/mL 103 bacteria/mL Explanation: Urine colony counts of 103 organisms/mL or greater are generally considered significant in urine collected by catheterization. 2. Question 3. A 3-yr-old girl has fever. 3.) 105 bacteria/mL None of the above: urine must be obtained by suprapubic aspiration to be considered diagnostic in children of this age Question 2. Which of the following defines the concept of bacterial tolerance? MIC in vivo >4 times the MIC in vitro MBC in vivo greater than the MIC in vivo MIC in vivo >2 times the MIC in vitro MIC >4 times the MBC MBC >4 times the MIC Explanation: A mean bactericidal concentration (MBC) > 4 times the minimum inhibitory concentration (MIC) defines bacterial tolerance. Urine is collected by catheterization for culture. Repeat antibody titer 5 wk later is 1:64. not acute Lyme disease. 5. 17th ed. Amoxicillin was begun 4 days ago for treatment of otitis media in a 4-yr-old boy. To confirm acute infection using IgG testing.Syphilis None of the above Explanation: These two antibody titers are not different and reflect past Lyme disease. inexpensive method for demonstrating the presence of bacteria and fungi. Question . He now presents with signs of meningitis and is found to have a pleocytosis with126 leukocytes/mm3. Serologic tests are performed on serial twofold dilutions of serum. it is necessary to demonstrate either seroconversion (i. A 12-yr-old girl has a positive throat culture for group A streptococci. It is a rapid. The actual titer is probably between 1:32 and 1:64 in both sera.e. Which of the following antimicrobial susceptibility tests should be performed? Bauer-Kirby (agar disk diffusion) E-test Minimum inhibitory concentration (MIC) Minimum inhibitory concentration (MIC) and minimum bactericidal concentration (MBC) None of the above Explanation: Group A streptococci have a predictable antimicrobial susceptibility pattern and are universally susceptible to penicillin. a fourfold increase in a convalescent titer obtained 2-3 wk after the acute titer indicates a significant change in titer and is considered diagnostic in most situations. from seronegative to seropositive) or a significant rise in IgG titer. Bacitracin susceptibility is generally used to identify the organisms as . 4. The diagnostic stain of the cerebrospinal fluid that would be most useful is: Gram stain Explanation: The Gram stain remains the most useful diagnostic stain for bacterial meningitis. (See Chapter 160 in Nelson Textbook of Pediatrics..) Methenamine silver stain Kinyoun stain Trichrome stain Wright stain Question 5. 6. metabolic disorders. A 17-yr-old boy is suspected of having an acute infection. 17th ed. or a 2tube (fourfold) difference in antibody titers supports recent infection. (See Chapter 161 in Nelson Textbook of Pediatrics. Patients with which of the following underlying conditions should be treated with antipyretics for fever as an essential part of treatment? Osteomyelitis Diabetes mellitus Inflammatory bowel disease Chronic lung disease Explanation: Antipyretic therapy is directly beneficial in high-risk patients who have chronic cardiopulmonary diseases. (See Chapter 160 in Nelson Textbook of Pediatrics. The titers of 1:64 and 1:128 are only 1 tube (twofold) different. Because this difference is only 1 tube (twofold). is performed at a reference laboratory for antibodies against the suspected organism. All of the following results support recent infection except: Positive IgM antibody titer and negative IgG antibody titer Positive IgM antibody titer and positive IgG antibody titer A fall in IgG titer from 1:64 to 1:8 A rise in IgG titer from 1:64 to 1:128 Explanation: A positive IgM antibody titer. or a fourfold decrease if taken later. Most evidence suggests that fever is an adaptive response.group A streptococci. Depending on the timing of samples relative to the onset of infection. 17th ed. Susceptibility testing specifically to penicillin is not necessary.) Absence of IgG antibody titer in the acute serum and IgG antibody titer of 1:16 in the convalescent serum Question 7. the IgG antibody titers may demonstrate seroconversion (from seronegative to seropositive). a fourfold increase. Serologic testing of acute and convalescent sera. 7. taken 4 wk apart. seroconversion. this difference does not necessarily indicate a change. this may reflect the interpretation of a titer that is between the two dilutions tested.) . Question 6. and that antipyretics provide symptomatic relief but do not change the course of diseases except in selected circumstances. or neurologic diseases. Meningitis Question 8. and adenopathy (PFAPA). drug fever.A 4-yr-old girl has a temperature of 41. Which of the following is a periodic fever syndrome? Rat bite fever Colorado tick fever Cyclic neutropenia Explanation: Periodic fever is used to narrowly describe fever syndromes with a regular periodicity. such as cyclic neutropenia. Causes of very high temperatures (>41°C) include central fever (resulting from central nervous system dysfunction involving the hypothalamus). aphthous stomatitis. 8. (See Chapter 161 in Nelson Textbook of Pediatrics. 17th ed. A 16-yr-old boy who has recently traveled to sub-Saharan Africa is diagnosed with Plasmodium vivax malaria.8°C.9. or heatstroke. The term that best characterizes this fever pattern is: Hectic fever Remittent fever .) Excessive sweating Ventricular shunt malfunction Most bacterial infections 10. and the syndrome of periodic fever. pharyngitis. Which of the following is the most likely cause? Cerebellar disorders Hypothalamic disorders Explanation: Temperatures in excess of 41°C are most often associated with a noninfectious cause. The fever pattern is one of febrile periods every other day. (See Chapter 161 in Nelson Textbook of Pediatrics.) Hectic fever Marburg fever Ques. malignant neuroleptic syndrome. malignant hyperthermia. 17th ed. g. An 8-yr-old boy with sickle cell disease presents with fever and symptoms of an acute illness. (See Chapter 161 in Nelson Textbook of Pediatrics.) Infants with vesicoureteral reflux . 17th ed. malaria caused by Plasmodium malariae).. 13. The combination of splenic hypofunction and a deficiency of the properdin system of complement activation places patients with sickle cell disease at even greater risk for bacteremia from encapsulated organisms. Any of the manifestations listed requires inpatient management for presumed bacterial sepsis.000 cells/µL WBC count > 30.) Periodic fever 12. You consider treating him as an outpatient with intramuscular ceftriaxone. and quartan fever occurs on the 1st and 4th days (e. All of the following patient groups have an increased risk of urinary tract infections except: Infant girls Uncircumcised infant boys Infants with urinary tract anomalies Infants with Mediterranean ancestors Explanation: Ancestry rarely predisposes to urinary tract infections. Which of the following findings should lead to hospitalization? Temperature > 40°C WBC count < 5.Biphasic fever Tertian fever Explanation: Tertian fever occurs on the 1st and 3rd days (e.000>cells/µL Pulmonary infiltrates Any of the above Explanation: Febrile children with sickle cell disease are at an increased risk for overwhelming sepsis. whereas the other choices are all significant risk factors..g. 17th ed. Pyelonephritis is a frequent cause of fever in young infants. malaria caused by Plasmodium vivax). (See Chapter 162 in Nelson Textbook of Pediatrics. ) The absolute band count is elevated.) Neisseria meningitidis Salmonella Streptococcus pneumoniae 15. The physical examination is normal. There is a urinary tract infection. and results of a Gram stain of the urine are negative. went home with his mother at 24 hr of life. . joint.000/µL. 17th ed.000-15. who have been previously healthy. and who have a total white blood cell (WBC) count of 5. bone. Explanation: Infants younger than 3 mo with fever who appear generally well. S. 17th ed. who have no evidence of skin. and normal urinalysis results are unlikely to have a serious bacterial infection. pneumoniae accounts for 90% of cases of occult bacteremia. an absolute band count of <1. influenzae type b. This infant fails to meet criteria for low risk of serious bacterial infection because: He was born prematurely. H.14. (See Chapter 162 in Nelson Textbook of Pediatrics. and has done well since. or nontyphoidal Salmonella occurs in approximately 4% of relatively well-appearing children between 3 and 36 mo of age with fever (rectal temperature >38. soft tissue. There are 4 WBCs/mm3 in an unspun urine sample. The infant's WBC count is 19. A 22-day-old infant is noted by his mother to have a rectal temperature of 38. She reports that he has been acting normal and appears generally well. N. He was born at 37 wk of gestation.500/mm3 and the absolute band count is 850/mm3.0°C). The WBC count is elevated.3°C. The negative predictive value with 95% confidence of these criteria for any serious bacterial infection is >98%. There is no known underlying illness. and for bacteremia. (See Chapter 162 in Nelson Textbook of Pediatrics. All of the following organisms are recognized as an important cause of occult bacteremia of infants and children except: Haemophilus influenzae type b Moraxella catarrhalis Explanation: Occult bacteremia (bacteremia without an obvious focus of infection) due to Streptococcus pneumoniae. or ear infection. >99%. meningitidis.500/µL. Neonates are at higher risk of sepsis and meningitis caused by group B streptococci and other organisms. Of the following immunizations. to inform you that a blood culture obtained in the emergency department yesterday from a 10-mo-old febrile girl who is followed in your practice is growing Streptococcus pneumoniae. race. and age do not appear to affect the risk for occult bacteremia among children 3-36 mo of age. and they report that she is "doing well. pneumoniae. 17th ed. (See Chapter 162 in Nelson Textbook of Pediatrics.M. 16." The most appropriate next step in management would be to: Instruct the family that you will telephone a prescription for amoxicillin to the pharmacy. A 10-mo-old black male infant from an urban area presents to the emergency department with a temperature of 39. gender. The triage history of a 9-mo-old child presenting with a temperature of 39. which is the most important to document before determining any management decisions? Hepatitis B DTaP IPV Pneumococcal conjugate vaccine Explanation: "Up-to-date" is not a satisfactory immunization history for a 6-mo-old infant presenting with a temperature of 39°C. The laboratory calls at 8:45 A. You contact her parents.3°C and an unremarkable physical examination. Conjugate pneumococcal vaccination is recommended for all children beginning at 2 mo of age and decreases the risk for occult bacteremia caused by S. Which of the following is a risk factor for occult bacteremia in this child? Absence of signs of pneumonia on physical examination Race Socioeconomic status Sex Fever Explanation: Socioeconomic status." Further review of the medical record shows that the patient has had three doses of Hib conjugate vaccine.) Meningococcal conjugate vaccine 18. and that they should pick up the prescription and . 17.He is less than 1 mo of age.2°C shows the immunization history as "up-to-date. pneumoniae and has received the three doses recommended by this age. Instruct the parents that they should have the child return to your office if new symptoms develop. The patient's extremities are warm. This muscle develops embryologically when the hypothalamus is also undergoing differentiation.start treating their child this morning. The nurse tells you that the patient's heart rate is still 120/min. since the medical record shows that she has been immunized against S. Instruct the parents to bring the child to your office as soon as possible to be re-examined. (See Chapter 162 in Nelson Textbook of Pediatrics. 20. This finding is associated with: Toxoplasmosis Ectodermal dysplasia Juvenile rheumatoid arthritis Hypothalamic dysfunction Explanation: Fever of unknown origin is sometimes due to hypothalamic dysfunction. with bounding pulses. (See Chapter 162 in Nelson Textbook of Pediatrics. pneumoniae is found in the initial blood culture. Thirty minutes after admission. 17th ed. A 1-yr-old child presents with a history of recurrent fevers for several weeks. you should report the vaccine failure to the state health department. 17th ed. the child should return for re-evaluation as soon as possible once the culture results are known. that her blood pressure is 70/30 mm Hg. A clue to this disorder is failure of pupillary constriction due to absence of the sphincter constrictor muscle of the eye. Explanation: If S.) Thyrotoxicosis 22. Inform the parents that the blood culture was positive and that they should have the child return to your office if new symptoms develop. in addition. an 18-yr-old girl with meningococcemia has just finished receiving the last of her 60 mL/kg infusion of normal saline. frothy material is being suctioned from the patient's endotracheal tube. The next step in the resuscitation is to: Administer 20 mL/kg of lactated Ringer's solution . On physical examination you find that the pupils fail to constrict. and that pink.) Instruct the parents to call 911 and have their child taken to the emergency department immediately. AST 550 U/L.5 mg/dL. a pressor is indicated. In this age group S. pneumoniae should be considered as a cause. Because the patient's pulses are good and she is well perfused. However. platelets 100.Administer 10 mL/kg of 25% albumin Administer NaHCO3 Administer nitroprusside Administer norepinephrine Explanation: This patient is exhibiting signs of fluid overloadpulmonary edema-and therefore should be given pharmacologic therapy to restore her blood pressure. The most appropriate next step in the management would be to: Obtain a stat head CT study and ophthalmology consultation. and the vaccine does not provide protection against all serotypes. and therefore vancomycin in addition to ceftriaxone is indicated until antimicrobial susceptibilities are known. However. The laboratory results for the patient described in Question 25 are significant for BUN 20 mg/dL. WBC count 4. The vaccination history for S. pneumoniae infection. In the case described in Question 23. which provides some indication of the probability of S. and ALT 1. HCO3 16 mmol/L. 26. . hematocrit 30%. 17th ed.200 U/L. (See Chapter 163 in Nelson Textbook of Pediatrics. which resolves spontaneously.000/mm3. the most appropriate antibacterial regimen to use until culture results are known would be: Ceftriaxone Ceftriaxone and clindamycin Piperacillin and gentamicin Ampicillin and gentamicin Ceftriaxone and vancomycin Explanation: The cloudy cerebrospinal fluid indicates meningitis. Depressed mental status. Intubate the patient and add acyclovir to the antibacterial regimen.) 24. even completed immunization does not provide absolute protection to the vaccine serotypes. pneumoniae should be elicited. As you are studying these results the baby has a focal seizure. creatinine 1. Explanation: Focal seizures in a febrile neonate with evidence of systemic shock and hepatic dysfunction suggest the possibility of systemic herpes simplex virus infection.500/mm3 with differential count pending. the nurse reports that the patient is intermittently apneic. prothrombin time (PT) 20 sec. which conserves blood flow to central organs in addition to maintaining blood pressure. Almost any organism can cause severe and even life-threatening infection. 17th ed. 27. There may also be a component of myocardial dysfunction due to cytokines released during the SIRS response. Consult nephrology and measure serum phosphate and ionized calcium. (See Chapter 163 in Nelson Textbook of Pediatrics. Explanation: In immunocompromised persons. Which of the following statements concerning infections in immunocompromised persons is true? Fever is an insensitive sign of infection. and receiving the last part of her third 20 mL/kg normal saline bolus. These patients are best served by an agent that has inotropic as well as afterload-reducing properties. with delayed capillary refill.airway compromise due to seizures. Despite some improvement in peripheral pulses. Normal skin flora can cause life-threatening infection. She is intubated. (See Chapter 163 in Nelson Textbook of Pediatrics.) Consult gastroenterology for possible Reye syndrome and obtain a serum ammonia level. 17th ed. fever is a sensitive and specific sign of infection. and the need for preservation of cardiorespiratory status in shock dictate intubation in this patient. and tachycardic. Perform a lumbar puncture prior to administering antibacterial agents. Her blood pressure is 120/70 mm Hg." Compensatory mechanisms include peripheral vasoconstriction. and her heart rate is 120/min. sedated. hypotensive. her peripheral extremities remain cool. Absolute neutrophil counts of .) Administration of 20 mL/kg normal saline and institution of dopamine Administration of 20 mL/kg normal saline and institution of labetalol 28. A 6-mo-old girl with streptococcal bacteremia admitted earlier to the PICU is now unresponsive. The recommended next step in management of this girl is: Administration of 20 mL/kg of 5% albumin Administration of 20 mL/kg hetastarch Administration of 20 mL/kg normal saline and institution of milrinone Explanation: This patient exhibits what used to be called "cold shock. These infections usually respond well to intravenous antimicrobial therapy.) Absolute neutrophil counts of <5000 cells/mm3 are predictive of infection. aureus is the most common organism causing infections. aureus and also: Granulocyte colony-stimulating factor Interferon gamma and trimethoprim-sulfamethoxazole Explanation: The management of chronic granulomatous disease includes lifelong treatment with interferon. 29. Multiple infections.<500/mm3 are predictive of infection. Of the following.) None of the above 30. are common. which is the most likely diagnosis? Congenital neutropenia Cyclic neutropenia Severe combined immunodeficiency syndrome Chronic granulomatous disease Explanation: S. 17th ed. (See Chapter 164 in Nelson Textbook of Pediatrics. (See Chapter 164 in Nelson Textbook of Pediatrics. 17th ed. aureus. (See Chapter 164 in Nelson Textbook of Pediatrics.and also trimethoprimsulfamethoxazole. with the risk directly proportional to the duration and depth of neutropenia. that are associated with chronic granulomatous disease. Multiple infections are uncommon. The risk of serious infection is highest in the first 48 hr of neutropenia. including liver abscess.) Intramuscular immunoglobulin . The boy described in Question 29 should receive specific antibiotic therapy directed against S. His maternal uncle died during childhood from "an infection" that was also thought to be caused by S. 17th ed. either concomitant or sequential. A 4-yr-old boy who was previously well is found to have a Staphylococcus aureus hepatic abscess. as well as additional antimicrobial therapy as needed for acute infections. Results of bacterial cultures of the blood. She had chickenpox at age 11 mo from which she had an uneventful recovery. and a polymerase chain reaction assay of the CSF for herpes simplex virus is negative. (See Chapter 164 in Nelson Textbook of Pediatrics.Which of the following immunologic laboratory abnormalities would be most consistent with this child's history? Very low to undetectable IgG levels Explanation: This boy is most likely to have X-linked agammaglobulinemia. The CSF glucose is 60 mg/dL (blood glucose is 100 mg/dL). Which of the following tests often yields an abnormal result in a child with this condition? Antibody level against varicella zoster virus Antibody level against H. aureus Very low to undetectable levels of CD4 cells . and CSF protein is 55 mg/dL. platelet count of 323. 56% lymphocytes. with low to undetectable levels of IgG.Cyclosporine Amphotericin B 31.200/mm3 with a differential of 32% granulocytes.) Abnormal result on NBT test Abnormal neutrophil killing of S. 17th ed. A 2-yr-old boy presents in late summer with fever. urine. lethargy. After 4 days the boy's clinical status remains unchanged. (See Chapter 164 in Nelson Textbook of Pediatrics.000/mm3. and CSF are negative. A lumbar puncture CSF specimen contains 230 leukocytes/mm3 with 80% lymphocytes and 20% granulocytes.2 g/dL. influenza type b Neutrophil count Delayed type hypersensitivity testing against Candida Explanation: This girl has mucocutaneous candidiasis. Gram stain shows moderate white blood cells and no organisms. He was hospitalized once for pneumonia but responded well to antibiotics. and has received her other routine childhood vaccinations without problems. She has never required hospitalization or had life-threatening infection. His current illness is probably an enterovirus meningoencephalitis.) Total IgG level 32. A 13-yr-old girl with hyperthyroidism has chronic infection of her skin and nails with Candida albicans. and white blood cell count of 14. despite chronic infection. 3% bands. This is often associated with an absent delayed-type hypersensitivity response to skin testing with Candida. 17th ed. and 9% mononuclear cells. CBC shows a hemoglobin of 13. and a stiff neck. The patient's past medical history reveals that he has had many ear infections and required bilateral myringotomy tubes at age 13 mo. A child with fever is found to have a white blood cell count of 1. 37% lymphocytes. The child has monthly recurrence of aphthous ulcers.400/mm3 with a differential count showing 56% neutrophils. with only one episode of acute cellular rejection at 1 mo after transplantation. 3% band forms. with 19% neutrophils. Explanation: The child with neutropenia whose counts are on the way down is at higher risk for bacterial complications of chemotherapy-associated neutropenia than a child whose counts are recovering or one who is neutropenic because of bone marrow suppression from viruses or medications or cyclic neutropenia. She is currently on cyclosporine and prednisone given orally. tachypnea. and cough. 85% lymphocytes. 34. 8% band forms.Elevated levels of IgE 33. A number of classmates have likewise been ill. pneumoniae (pneumococcus) . with 1 % neutrophils.) The child was recently diagnosed with a seizure disorder and was started on carbamazepine (Tegretol) 2 weeks ago. congestion.400/mm3. Which of the following is the least likely pathogen in this patient? Cytomegalovirus Pneumocystis carinii S. She takes no other medications. 2% band forms. and a diagnosis is established. Chest radiograph shows a diffuse interstitial infiltrate White blood cell count is 5. The child has leukemia and received chemotherapy 20 days ago. and 3% mononuclear cells. 65% lymphocytes. Bronchoalveolar lavage is performed. The child has leukemia and received chemotherapy 7 days ago. A 4-yr-old girl underwent liver transplantation 4 mo ago after liver failure from biliary atresia. She has done relatively well. and 4% eosinophils. The lungs are clear on auscultation. and maculopapular rash. His WBC count 2 days ago was 800/mm3. His WBC count 2 days ago was 3. He had a 5-day history of rhinorrhea. 65% lymphocytes. 0% band forms. 17th ed. 4% mononuclear cells. She presents to your office with fever of 3 days' duration. and 4% atypical lymphocytes. (See Chapter 164 in Nelson Textbook of Pediatrics. and 44% mononuclear cells. The clinical examination reveals a moderately ill child with increased respiratory effort and perioral cyanosis.400/mm3 with a differential count of 10% neutrophils. Which of the following scenarios indicate the greatest risk for this child to have a new serious bacterial complication? The child is a previously healthy 3-yr-old who attends out-of-home daycare. He presents with fever and erythema over the catheter tunnel. Until identification and susceptibility testing are available. (See Chapter 165 in Nelson Textbook of Pediatrics. A 9-yr-old boy with cancer had an indwelling Broviac catheter placed 8 mo ago for cancer chemotherapy. Antibiotic treatment is successful for most systemic bacterial infections without removal of the device. with follow-up in the clinic the next day and subsequent treatment based on culture results Administering urokinase through the Hickman catheter Admitting the patient to the hospital for observation. 17th ed. unusual pathogens are more likely in a post-transplantation patient receiving immunosuppressive therapy. A 6-yr-old boy with acute lymphocytic leukemia has had a central venous catheter (Hickman catheter) in place for 1 yr to facilitate administration of chemotherapy. empirical therapy with a third-generation cephalosporin or aminoglycoside plus vancomycin is indicated. He is not neutropenic. Which of the following antibiotics is recommended? . He was brought into the clinic with a temperature of 39°C.) Adenovirus Parainfluenza virus 35. Antibiotics should be administered after blood cultures are obtained. with no antibiotic therapy as long as the cardiovascular status is stable 36. 17th ed. (See Chapter 164 in Nelson Textbook of Pediatrics. when he developed the fever and had rigors. Blood culture specimens are taken. The most appropriate approach to management of this child is: Immediately removing the Hickman catheter and beginning therapy with vancomycin and ceftazidime Obtaining blood cultures via the catheter lumen and a peripheral vein followed by initiation of antibiotic therapy with vancomycin and ceftazidime Explanation: The risk of catheter-associated infection is significant. without other localizing symptoms. His mother reports that he was well until an hour after she routinely flushed his line with heparin.Explanation: Although pneumococcal pneumonia can occur in both immunocompetent and immunocompromised persons. his vital signs and findings on physical examination are normal. Antibiotic therapy should be directed to the isolated pathogen and given for a total of 10-14 days.) Obtaining blood cultures via the catheter lumen and a peripheral vein. Aside from increased heart rate. 17th ed. discharge the child with instructions to take acetaminophen every 4 hr for the fever. The most likely diagnosis is: Staphylococcal scalded skin syndrome Kawasaki disease Toxic shock syndrome . (See Chapter 165 in Nelson Textbook of Pediatrics. Begin oral antibiotic therapy with amoxicillin/clavulanate. strawberry tongue. Most coagulase-negative staphylococci are resistant to nafcillin.) 37. 38. Explanation: Culture of CSF obtained from the shunt is the most useful test to diagnose a CSF shunt infection. An 18-mo-old infant has a CSF shunt in place because of congenital hydrocephalus. He presents to the emergency department with increased irritability and decreased appetite of 3 days' duration. Obtain a blood culture specimen and a CSF culture specimen via a tap of the CNS shunt. 17th ed.) Perform a radionuclide dye study to determine if the shunt is functioning properly. myalgias. watery diarrhea.Nafcillin Cephalexin Ceftriaxone Clindamycin Vancomycin Explanation: Vancomycin is the choice for empirical treatment of indwelling line and prosthesis infections caused by coagulasenegative staphylococci. A 9-yr-old boy presents with fever >39°C for 4 days.7°C. conjunctival infection. There are no focal signs of infection. He underwent a surgical revision of the shunt 4 mo ago because of malfunction. and moderately elevated hepatic transaminases. blood pressure of 105/45 mm Hg. (See Chapter 165 in Nelson Textbook of Pediatrics. Obtain a blood culture specimen and a CSF culture specimen via a lumbar puncture. The most appropriate management of this child would be: If the WBC count is normal. diffuse erythroderma. The rectal temperature is 38. Explanation: Toxic shock syndrome and Kawasaki disease share many features. (See Chapter 166. Kawasaki disease is uncommon after age 5 yr. The most likely diagnosis is: Hemolytic-uremic syndrome Kawasaki disease Rocky Mountain spotted fever Toxic shock syndrome Explanation: The diagnosis of toxic shock syndrome is based on the clinical manifestations.) Stevens-Johnson syndrome Toxic epidermal necrolysis 39. In severe cases. aureus produce toxins. hyperemia of the pharyngeal and conjunctival membranes. Toxic shock syndrome can complicate focal infections caused by TSST-1-producing strains of Staphylococcus aureus. C2. scalded skin syndrome) eruptions.g.) . oliguria. the hypotension may progress to shock.2 in Nelson Textbook of Pediatrics. 17th ed. Staphylococcal enterotoxins A. TSST-1 is the toxin of toxic shock syndrome.. Strains of Staphylococcus aureus can produce which of the following toxins? Exfoliatin A and B Hemolysins Enterotoxins A through E Toxic shock syndrome toxin-1 (TSST-1) All of the above Explanation: Many strains of S. and E are associated with food poisoning. D. and is not associated with hypotension or diarrhea. (See Chapter 166.. Exfoliatins A and B are associated with localized (e. Kawasaki disease also typically occurs in children younger than 5 yr.g. but Kawasaki disease is not accompanied by hypotension and shock. 17th ed. 40. bullous impetigo) or generalized (e. vomiting. C1. B. A 16-yr-old girl experiences abrupt onset of high fever. and postural hypotension.2 in Nelson Textbook of Pediatrics. is not as severe or as progressive. and diarrhea. Hemolysins act on cell membranes. with a diffuse sunburn-like rash. infections of central venous catheters and cerebrospinal fluid shunts. Which of the following organisms characteristically produce(s) an exopolysaccharide protective biofilm (slime layer)? Proteus mirabilis Coagulase-negative staphylococci Explanation: Slime produced by coagulase-negative staphylococci surrounds the organism. Toxic shock syndrome is caused by TSST-producing Staphylococcus aureus.3 in Nelson Textbook of Pediatrics. including indwelling catheters and prostheses. resists phagocytosis. Corticosteroids and intravenous immunoglobulin are reserved for severe cases. 17th ed. 17th ed. and enhances adhesion to foreign surfaces.) Neonatal bacteremia . Coagulase-negative staphylococci are commonly associated with all of the following except: Toxic shock syndrome Explanation: Coagulase-negative staphylococci are associated with nosocomial neonatal infections. (See Chapter 166.Stevens-Johnson syndrome 41. (See Chapter 166. urinary tract infections associated with urinary catheters or following urinary tract surgery.3 in Nelson Textbook of Pediatrics. Recommended therapy for the patient described in Question 40 is: Renal dialysis IVIG Doxycycline An antistaphylococcal antibiotic Explanation: Treatment of toxic shock includes an antistaphylococcal antibiotic and aggressive intravenous fluid therapy to prevent or treat hypotension.) Corticosteroids 42.2 in Nelson Textbook of Pediatrics. (See Chapter 166. 17th ed.) Group A streptococci Haemophilus influenzae Moraxella catarrhalis 43. patients who are strongly suspected of having bacterial meningitis should receive which of the following antimicrobial agents added to ceftriaxone or cefotaxime as empirical antimicrobial therapy? Clindamycin Azithromycin Vancomycin Explanation: Vancomycin is the current treatment of choice for .Urinary tract infection Intravascular catheter-associated infection 44. The recommended course of management is: Intravenous nafcillin Vancomycin mouthwashes Intravenous vancomycin Removal of the Broviac catheter and administration of intravenous vancomycin None of the above: no treatment is indicated Explanation: Coagulase-negative staphylococci are part of the normal flora of the skin. throat. cultures of specimens taken 4 days after presentation remain positive. A 12-yr-old child is being treated with nafcillin for Staphylococcus aureus bacteremia. or focal infection that requires drainage. 46. vagina. mouth. 45. In regions where penicillin-resistant pneumococci are prevalent. an endovascular focus such as endocarditis. In the absence of specific symptoms. A throat culture from a 16-yr-old boy with osteosarcoma and a Broviac catheter grows coagulase-negative staphylococci. However. no treatment is indicated. and urethra. Which of the following is the least likely cause of the positive cultures? Antimicrobial resistance to nafcillin Endocarditis Septic thrombophlebitis An undrained abscess Penicillin allergy Explanation: Persistent Staphylococcus aureus bacteremia suggests the possibility of antimicrobial resistance. Vancomycin is started empirically because of the possibility of penicillin/cephalosporin-resistant pneumococci. (See Chapter 167 in Nelson Textbook of Pediatrics. some types of immune deficiency (e. but he has not received either the conjugate or polysaccharide pneumococcal vaccine. such as those with asplenia. Which of the following is the optimal means for protecting this child from developing another episode of invasive pneumococcal disease? Give one dose of the pneumococcal conjugate vaccine and continue penicillin prophylaxis. High-risk children >2 yr of age. lethargy. Which of the following is the most appropriate next step in the management of this infant? MRI scan Intravenous penicillin Intravenous ceftriaxone Intravenous vancomycin and ceftriaxone Explanation: Intravenous vancomycin and ceftriaxone are recommended. A 7-mo-old infant presents to your office with fever. heart. 17th ed. (See Chapter 167 in Nelson Textbook of Pediatrics.g. Blood and CSF specimens are sent for culture. Give one dose of the pneumococcal polysaccharide vaccine and continue penicillin prophylaxis. A 26-mo-old child with sickle cell disease appears in your office for the first time. or chronic lung.. sickle cell disease. antibody deficiencies). (See Chapter 167 in Nelson Textbook of Pediatrics. and a stiff neck. 17th ed.) Streptomycin Any of the above 47. 17th ed.) . may benefit also from the 23-valent pneumococcal polysaccharide vaccine.) Withholding of antibiotics until culture results are available 48. Currently the child is on penicillin prophylaxis.penicillin/cephalosporin-resistant pneumococcal infection. and continue penicillin prophylaxis. He has a history of a prior hospitalization for pneumococcal bacteremia. HIV infection. Give one dose of the pneumococcal conjugate vaccine followed one month later by one dose of the pneumococcal polysaccharide vaccine. Explanation: Immunization with the conjugate polysaccharide vaccine is recommended for all infants. or kidney disease (including nephrotic syndrome). A lumbar puncture reveals cloudy cerebrospinal fluid. ) Failure of the vaccine to elicit a protective immune response against a pneumococcal serotype contained in the vaccine An anatomic defect with communication between the middle ear and the subarachnoid space Cardiac defect with right-to-left shunting 50.5°C. Have a surgeon perform a pleural tap followed by placement of a chest tube. 49. and start intravenous penicillin. An 8-mo-old infant who has received all recommended vaccines at the appropriate age presents to your office with a temperature of 39. . 9V. Have a surgeon perform a pleural tap followed by placement of a chest tube. and crackles in the left lung. 19F. A chest radiograph shows a left lower lobe infiltrate with a large pleural effusion. A previously healthy 2-yr-old child in your practice who has been fully immunized with the pneumococcal conjugate vaccine has now developed pneumococcal meningitis.Give one dose of the pneumococcal conjugate vaccine and switch from penicillin prophylaxis to Augmentin prophylaxis.000/mm3. The peripheral white blood cell count is 22. These serotypes cause >80% of pneumococcal infections among children. The child is admitted to the hospital but is not thought to require intensive care or intubation. and 18C. 17th ed. with 60% neutrophils and 15% bands. Give one dose of pneumococcal conjugate vaccine and switch to intramuscular benzathine penicillin prophylaxis. 6B. (See Chapter 167 in Nelson Textbook of Pediatrics. Start intravenous vancomycin and ceftriaxone and follow closely. Start intravenous cefuroxime and follow closely. and start intravenous ceftriaxone.The most appropriate next step in management would be to: Start intravenous penicillin and follow closely. increased respiratory rate. The most likely reason for development of pneumococcal meningitis after receipt of the vaccine in this child is: An underlying primary immunodeficiency A pneumococcal serotype that the vaccine did not protect against Explanation: The currently available heptavalent vaccine contains conjugated capsular polysaccharides of serotypes 4. 23F. 14. For invasive pneumococcal infections outside the central nervous system (e. high-dose cefotaxime or ceftriaxone is usually effective even for those infections caused by cephalosporinintermediate or-resistant strains (MIC 1. 17th ed. 52. lobar pneumonia with or without bacteremia). Scarlet fever is caused by: Staphylococcus aureus strains that produce exfoliatins Staphylococcus aureus strains that produce enterotoxins Group A streptococci strains that produce pyrogenic exotoxins Explanation: Scarlet fever is the result of infection with a strain of group A streptococci that produces one of three pyrogenic (erythrogenic) exotoxins (A. A 4-yr-old boy presents with well-demarcated. 51. The most likely etiology is: Lupus erythematosus Crohn disease Clostridium difficile infection Staphylococcus aureus infection Group A streptococcal infection Explanation: Perianal streptococcal disease is a distinct clinical entity caused by group A streptococci. 53.0+ mg/L).0-2.g. (See Chapter 168 in Nelson Textbook of Pediatrics. All of the following are features of scarlet fever except: Pastia lines Desquamation White strawberry tongue Red strawberry tongue . B. perianal erythema associated with anal pruritus and blood-streaked stools.Explanation: The likely organisms causing this infection are either pneumococci or group A streptococci. or C).) Human herpesvirus type 6 Parvovirus B19 Question . 56. (See Chapter 169 in Nelson Textbook of Pediatrics. The pathogen that most commonly causes systemic and focal infections in the newborn is: Staphylococcus aureus Group A streptococci Group B streptococci Explanation: Group B streptococcal organisms are the major cause of severe systemic and focal infections in the newborn. 55. 54. The recommended regimen for selective intrapartum prophylaxis for group . (See Chapter 169 in Nelson Textbook of Pediatrics.Preauricular lymphadenopathy Explanation: The classic rash of scarlet fever has a texture of gooseflesh or coarse sandpaper.) Pregnant women who are colonized are usually asymptomatic. Preauricular lymphadenopathy is not typical Question . in African Americans.) Escherichia coli Herpes simplex virus Question . White strawberry tongue is characteristic of the early illness.5-2% of colonized infants become infected. as the white coat desquamates. Coagulasenegative staphylococcal infections are the most common nosocomial infections in the neonatal intensive care unit. All of the following statements concerning perinatal group B streptococcal neonatal infections are true except: Approximately 20-40% of pregnant women are colonized. Approximately 40-70% of infants born to colonized women become colonized. Pastia lines are areas of hyperpigmentation that do not blanch with pressure that may appear in creases. Colonization rates are increased in women older than 40 yr. the red strawberry tongue persists. Explanation: Group B streptococcal colonization rates are increased in women younger than 20 years of age. and in higher socioeconomic groups. A newborn develops sepsis and shock. Skin desquamation begins toward the end of the first week of illness. 17th ed. particularly in the antecubital fossae. 17th ed. Approximately 0. and among lower socioeconomic groups. in whites. Question . widespread implementation of maternal chemoprophylaxis led to a striking 65% decrease in the incidence of early-onset neonatal group B streptococcal disease in the United States.000. For penicillin-allergic women at high risk for anaphylaxis. while the incidence of late-onset disease remained essentially stable at approximately 0.000.4 per 1.6 per 1.500.500. 57. (See Chapter 169 in Nelson Textbook of Pediatrics. followed by 2. the AAP and CDC recommend administration of penicillin G. Regarding early-onset versus late-onset neonatal infection.000. 5.) Penicillin G 5. Maternal chemoprophylaxis has led to striking decreases in both early-onset and late-onset disease. (See Chapter 169 and Figure 169-1 in Nelson Textbook of Pediatrics. cefazolin may be used.B streptococcal infection is: Penicillin G 5.7 per 1. whereas late-onset disease usually presents as bacteremia and meningitis. to the mother at the onset of labor. Early-onset disease usually presents as sepsis and pneumonia. Explanation: In the 1990s.000 units IV to the mother at the onset of labor.000 units IV to the mother at the onset of labor Penicillin G 5.000 units IV every 4 hr until delivery. 17th ed. Ampicillin is an alternative.000 live births to 0. 17th ed. Early-onset disease usually presents within the first 7 days of life. all of the following are true except: Group B streptococci are the most common cause of early-onset infection.) Early-onset disease is often associated with maternal obstetric complications. followed by continuous infusion of 500.000 units IV to the mother at the onset of labor.000. . For penicillin-intolerant women.000 units IV every 4 hr until delivery Explanation: For group B streptococcal prophylaxis. whereas late-onset disease presents after 7 days. clindamycin or erythromycin should be used.000 units IM to the newborn Question . from 1. whereas late-onset disease is not.000 live births.000.000 units IM to the newborn Procaine penicillin G 150.000 units IV. followed by 2.000 units per hour until delivery Crystalline penicillin G 50. 59. 60. and blood culture) except: An infant born at 36 wk whose mother had not been tested for .Question . (See Chapter 169 in Nelson Textbook of Pediatrics. and previous delivery of an infant who developed group B streptococcal disease. 17th ed. (See Chapter 169 in Nelson Textbook of Pediatrics. All of the following infants routinely require a sepsis evaluation (at least a complete blood count with differential. The risk of early-onset group B streptococcal infection in an infant of a 34yr-old mother is increased with all of the following except: Heavy maternal vaginal or rectal colonization with group B streptococci Prolonged rupture of membranes (greater than 18 hr) Maternal fever with temperature greater than 38°C Maternal bacteriuria with group B streptococci during her last pregnancy Explanation: Risk factors for early-onset disease include heavy maternal vaginal or rectal colonization with group B streptococci. group B streptococcal (GBS) bacteriuria during pregnancy. or a previous infant with invasive GBS disease should receive intrapartum antibiotics. prematurity. 17th ed. Which of the following pregnant women should receive prophylaxis against group B streptococci? A woman who tested positive for group B streptococci at 37 wk of gestation and is having a planned cesarean section (without rupture of membranes) A woman who tested negative for group B streptococci at 35 wk of gestation and goes into spontaneous labor at 36 wk A woman whose group B streptococcal status is unknown and who had a previous infant at home with invasive group B streptococcal infection Explanation: Any woman with a positive prenatal screening culture.) A woman who tested negative for group B streptococci at 37 wk and has had prolonged rupture of membranes (>20 hr) when she delivers at 39 wk of gestation Question . maternal bacteriuria during pregnancy.) Previous infant with invasive group B streptococcal infection Question . 58. prolonged rupture of membranes. intrapartum fever. In addition to removing the catheter. His throat culture is positive for group C streptococci.group B streptococci Explanation: Guidelines from the Centers for Disease Control and Prevention (CDC) provide an algorithm for the empirical management of a newborn whose mother received intrapartum antimicrobial agents for prevention of early-onset group B streptococcal disease or suspected chorioamnionitis. (See Chapter 170 in Nelson Textbook of Pediatrics. She has a Foley catheter.) Oral clindamycin Oral erythromycin Intramuscular ceftriaxone None of the above: no treatment is necessary with a positive culture for group C streptococci Question . which antibiotic is the recommended agent for treatment of Enterococcus infections? Penicillin Ampicillin . 17th ed. 62. The recommended management is: Oral penicillin as for group A streptococcal infection Explanation: Penicillin is the drug of choice for treating infections due to either group C or group G streptococci. A 19-yr-old freshman in college presents with signs of pharyngitis. (See Chapter 169 and Figure 169-2 in Nelson Textbook of Pediatrics. 61.) An infant born at 38 wk whose mother had a culture that was positive for group B streptococci and who was delivered 2 hr after the mother received her first dose of antibiotics An infant born at 40 wk whose mother had a culture that was negative for group B streptococci but was treated for suspected chorioamnionitis An infant born at 38 wk whose mother had a culture that was positive for group B streptococci and had spontaneous rupture of membranes but then delivered by cesarean section for failure to progress An infant born at 39 wk whose mother had a culture that was negative for group B streptococci Question . and a urine culture is positive for Enterococcus. A 12-yr-old girl is hospitalized in intensive care. 17th ed. ) Clindamycin Erythromycin Vancomycin Question . and dysphagia help differentiate diphtheria from exudative pharyngitis due to group A streptococci and Epstein-Barr virus. minor localized infections due to Enterococcus can be treated with ampicillin alone. and spread in a family's refrigerator. and lethargy. The most likely diagnosis is: Epiglottitis Bacterial tracheitis Group A streptococcal infection Vincent angina Diphtheria Explanation: The leather-like adherent membrane. replicate. dysphagia. in the immunocompetent person. Which of the following routes of transmission has been associated with several large outbreaks of human listeriosis? Aerosol transmission Person-to-person spread Zoonotic transmission Drinking contaminated water Food-borne transmission Explanation: Listeria monocytogenes may contaminate dairy products (milk. (See Chapter 171 in Nelson Textbook of Pediatrics. 63. (See Chapter 173 in Nelson Textbook of . 17th Question . extension beyond the faucial area. As a visiting physician in a developing country. (See Chapter 172 in Nelson Textbook of Pediatrics.Explanation: In general. you are asked to see a 2-yrold child who presents with a history of fever. Physical examination shows a gray-brown leather-like adherent membrane over the posterior oropharynx and hypopharynx. 64. 17th ed. cheese) and may survive. relative lack of fever. g.Question . 66. and tetracycline but are not susceptible to the cephalosporins. gentamicin) lowers the minimum bactericidal concentration. especially to immunocompromised persons and via vertical transmission to newborns. (See Chapter 173 in Nelson Textbook of Pediatrics. zoonotic spread.. Which of the following is the recommended treatment for neonatal listeriosis? Ceftriaxone Ampicillin with or without an aminoglycoside Explanation: Listeria isolates are usually sensitive to penicillin. All of the following represent risk factors for infection with Listeria monocytogenes except: Age younger than 1 mo Cancer chemotherapy Previous treatment with broad-spectrum antibiotics Explanation: Likely routes of spread of Listeria include food-borne infection. The addition of an aminoglycoside (e. 65. Ingestion of unpasteurized cheese Occupational exposure as a veterinarian Question . All of the following are effective in the prevention of infection due to Listeria monocytogenes except: Use of the approved Listeria vaccine for high-risk individuals Explanation: There is no Listeria vaccine available. and person-to-person transmission. Avoidance of unpasteurized dairy food Careful handwashing to prevent nosocomial spread in the hospital Prophylaxis with trimethoprim/sulfamethoxazole for immunocompromised patients . erythromycin. 17th ed.) Cefotaxime with or without an aminoglycoside Erythromycin Vancomycin Question . including the third-generation cephalosporins. ampicillin. 67. She undergoes an appendectomy. The hallmark of Nocardia infection is: A tendency for remissions and exacerbations Explanation: Nocardia infection is characterized by remissions and exacerbations. Amebiasis Question . she has local pain and has an irregular. (See Chapter 175 in Nelson Textbook of Pediatrics. 17th ed. Her past medical history is significant only for sexual activity and placement of an IUD 1 yr previously. All of the following serogroups are included in the quadrivalent meningococcus vaccine except: A B Explanation: Meningococcal group B polysaccharide is poorly immunogenic. in which her appendix is found to be normal. Nocardia infection is an acute.) Involvement of bone A self-limited disease with scar formation Association in the pelvis with IUD placement An association with sickle cell disease Question . The most likely diagnosis is: Yersinia pseudoappendicitis Lymphoma Inflammatory bowel disease Pelvic actinomycosis Explanation: Pelvic actinomycosis is an unusual complication of IUD use. subacute.) C . 69. A 16-yr-old girl presents with signs and symptoms of appendicitis.Thorough cooking of meats Question . hard mass in her ileocecal area. 70. 17th ed. One month postoperatively. 68. or chronic suppurative infection that primarily causes pulmonary disease in immunocompromised patients. (See Chapter 176 in Nelson Textbook of Pediatrics. There is no vaccine available for this serogroup. platelet count of . hyperpyrexia. (See Chapter 176 in Nelson Textbook of Pediatrics. 73. The index patient should also receive rifampin prophylaxis if penicillin was used for treatment. Which of the following contacts should receive rifampin chemoprophylaxis after diagnosis of invasive Neisseria meningitidis in a child? Unimmunized or partially immunized children younger than 4 yr of age in the same household All unimmunized or partially immunized children in the same household All children and adults in the same household if there is an unimmunized or partially immunized child younger than 48 mo Unimmunized or partially immunized children in the same household and in the same daycare facility All children and adults in the same household or daycare facility regardless of immunization history Explanation: Chemoprophylaxis for contacts of a person with proven or suspected N. and nursery care contacts. A 7-mo-old girl presents with a temperature of 103. purpura fulminans. Which of the following is associated with a poorer prognosis for persons presenting with meningococcal disease? Presence of petechiae for <12 hr Explanation: Poor prognostic signs for meningococcal disease include hypotension and shock. (See Chapter 176 in Nelson Textbook of Pediatrics.W135 Y Question . 72. meningitidis infection is indicated for all household.) Meningitis Thrombocytosis Leukocytosis Low circulating levels of tumor necrosis factor Question . 71. The presence of petechiae for less than 12 hr before admission. leukopenia. daycare.) Question .8°F. 17th ed. seizures. and the absence of meningitis reflect rapid clinical progression and poorer prognosis. diffuse petechiae first noted 4 hr before presentation. blood pressure of 70/30 mm Hg. 17th ed. thrombocytopenia. and high circulating levels of endotoxin and tumor necrosis factor. petechiae.) Haemophilus influenzae type b Escherichia coli O157:H7 Question . Which of the following agents should be used for prophylaxis for the child care classmates of this infant? Chloramphenicol Cefixime Rifampin Explanation: Rifampin or ceftriaxone is recommended for prophylaxis of close contacts of patients with meningococcal disease.88. 17th ed. and hypotension suggest meningococcemia. . (See Chapter 176 in Nelson Textbook of Pediatrics.) Ciprofloxacin Trimethoprim-sulfamethoxazole Question . Parents of a college-bound high school senior ask whether their child should receive a meningococcal vaccine. Vaccine recipients may develop scattered petechiae 5 to 7 days after vaccination. The highest risk of meningococcal disease among college students occurs near the end of each semester. The infant has received all recommended vaccinations. (See Chapter 176 in Nelson Textbook of Pediatrics. Which of the following is the most likely bacterial etiology of this presentation? Staphylococcus aureus Streptococcus pneumoniae Neisseria meningitidis Explanation: The presence of fever. and white blood cell count of 4300/mm3. 74. 17th ed.000/mm3. with 23% neutrophils and 42% bands. A 15-mo-old child who attends a child care center 5 days per week is diagnosed with meningococcal infection. Which of the following statements should comprise part of the information that is provided to them? The quadrivalent polysaccharide vaccine will prevent over 90% of the cases of meningococcal infection among college students. 75. 76. involves large joints. and is accompanied by sterile effusions that respond to nonsteroidal antiinflammatory agents. Question .or oligoarticular. Which of the following is most likely to reveal an abnormality that may have predisposed him to develop meningococcal disease? Quantitative immunoglobulins (IgM.) Secondary bacterial infection from the immunosuppressive effects of meningococcal infection Question . The patient improves greatly after 7 days of antibiotic therapy and is ready for discharge. IgG. are at about 2. 17th ed. (See Chapter 176 in Nelson Textbook of Pediatrics. especially freshmen living in dormitories.The risk for college students living on campus is severalfold higher than for their non-collegiate peers. 77.) Freshmen living off-campus have the highest risk of meningococcal infection. A blood culture yields Neisseria meningitidis. Long-term sequelae are uncommon. Explanation: College students. Mild hypotension responds to intravenous fluids without the need for pressor support. and IgA) Serial complete blood counts to assess for cyclic neutropenia Genotype analysis of the TNF-alpha promoter region .to 8-fold higher risk than non-collegiate peers for meningococcemia. Which of the following is the most likely etiology of this finding? Hemorrhage into the joint occurring as a result of disseminated intravascular coagulation Progression of septic arthritis that was unrecognized at the onset of illness Nonspecific edema from progressive sepsis-related capillary leak Immune complex deposition resulting from production of antimeningococcal antibodies Explanation: Nonsuppurative complications of meningococcal disease appear to be immune complex-mediated and become apparent 4-9 days after the onset of illness. The arthritis usually is mono. Arthritis and cutaneous vasculitis (erythema nodosum) are most common. A 13-yr-old boy develops fever with petechiae and a few purpura. 17th ed. (See Chapter 176 in Nelson Textbook of Pediatrics. A 6-yr-old child with meningococcal meningitis develops a swollen left knee on the fifth day of antibiotic treatment. C. such that serogroups B. Historically. Elementary school-aged children have higher rates of disease than those in 15. Most genital tract infections in children are symptomatic. (See Chapter 176 in Nelson Textbook of Pediatrics.) The frequency of serogroup Y disease has decreased since 1990. 79.) Total lymphocyte subsets (B cells. culture-negative tenosynovitis. CD8+ T cells) Question . or terminalcomponent deficiencies. B. W135.Screening assay for deficiencies of complement factors (CH50) Explanation: Persons with primary complement deficiency have an increased risk of developing meningococcal disease. C. Question . serogroups B and C each accounted for nearly half of cases among young children in developed countries. Cases caused by serogroup Y strains increased during the 1990s. predominantly among adolescents and adults. but as many as 80% of sexually active mature females . 17th ed. and disseminated gonococcal infection with bacteremia. factor D. meningitidis. and Y. suppurative arthritis. Of individuals with properdin. underscoring the important role of complement in host defense against meningococci. 50-60% will develop serious bacterial infections. CD4+ T cells. (See Chapter 176 in Nelson Textbook of Pediatrics.to 19-yr-olds. 17th ed. In addition to genital tract infections. Purpura fulminans is the most common clinical presentation during childhood. Which of the following statements about the epidemiology of meningococcal infections is true? Serogroup A strains are common in developed countries. caused almost solely by N. Serogroup B strains continue to cause about half of the cases among children in developed countries. well-recognized forms of Neisseria gonorrhoeae infection include all of the following except: Asymptomatic infection Lymphadenitis Explanation: Gonococcal infections range from asymptomatic carriage to localized urogenital infections. and Y now each account for about one third of cases in these age groups. 78. Explanation: The vast majority of meningococcal disease worldwide is caused by serogroups A. 17th ed. 17th ed. The Fitz-Hugh-Curtis syndrome is characterized by: Right upper quadrant pain caused by gonococcal perihepatitis Explanation: In addition to gonococcal disease.with urogenital gonorrhea infections are asymptomatic. arthritis. 82. Which of the following drugs is recommended for initial therapy for nondisseminated gonococcal disease in children and adults? Penicillin Tetracycline Ceftriaxone Explanation: Even with disseminated disease (bacteremia. Fitz-Hugh-Curtis syndrome may also be due to Chlamydia.) Conjunctivitis Suppurative arthritis Disseminated disease with bacteremia Question . (See Chapter 177 in Nelson Textbook of Pediatrics. Conjunctivitis occurs in neonates born to mothers with genital tract gonorrhea. Which of the following contacts should receive rifampin chemoprophylaxis after diagnosis of invasive Haemophilus influenzae type b infection in a child? . meningitis). 81. ceftriaxone is the drug of choice because of the high rate of penicillin resistance. gonorrhoeae Question .) Cefazolin Erythromycin Question .) Polyarticular arthritis and rash of disseminated gonococcal infection Lower quadrant pain caused by gonococcal endometritis Gonococcal meningitis Monoarticular arthritis and urethral exudate caused by N. (See Chapter 177 in Nelson Textbook of Pediatrics. 80. (See Chapter 177 in Nelson Textbook of Pediatrics. 17th ed. Unimmunized or partially immunized children younger than 4 yr in the same household All children and adults in the same household if there is an unimmunized or partially immunized child younger than 48 mo Explanation: Chemoprophylaxis for contacts of a person with proven H.) Question . (See Chapter 180 in Nelson Textbook of Pediatrics. 17th ed. erythromycin should be given to the patient and to which family members? Only those with a cough Only those younger than 7 yr Only those who are incompletely immunized Only those with compromised immunity All family members regardless of age. 17th ed. symptoms. including the index patient. or immunization status Explanation: Pertussis is a "family" disease with various degrees of symptoms and colonization. Which of the following contacts should receive erythromycin after diagnosis of pertussis in a child? Unimmunized children younger than 4 years in the same household All unimmunized children in the same household . under these circumstances. Question . influenzae type b infection is indicated if the close contact group includes one or more children younger than 48 mo of age who are not fully immunized. 83. rifampin prophylaxis is indicated for all members of the close contact group. 84. After diagnosis of pertussis in a toddler. (See Chapter 178 in Nelson Textbook of Pediatrics.) Unimmunized or partially immunized children younger than 48 mo in the same household and in the same daycare facility All children and adults in the same household or daycare facility regardless of immunization history All immunocompormised children and adults in the same household or daycare facility regardless of immunization history. 86. with mild cough and rhinorrhea. The infant is afebrile and appears well. inconsolable crying lasting longer than 3 hr within 2 days of pertussis vaccination An anaphylactic reaction to a previous dose of whole-cell pertussis vaccine Explanation: The more serious adverse events associated with DTP vaccine.) Question . and findings on lung auscultation are normal. A 4-wk-old infant has the history of gagging and gasping episodes of acute onset.000-100. occur significantly less frequently with DTaP vaccine and are not considered contraindications to further doses. 3% monocytes. or symptoms. WBC count 42. regardless of age. (See Chapter 180 in Nelson Textbook of Pediatrics. The most likely etiologic agent of this illness is: Chlamydia trachomatis Rhinovirus Bordetella pertussis Explanation: Leukocytosis (15.000/mm3 (10% neutrophils.) .0 gm/dL. The only absolute contraindication to subsequent administrations of acellular pertussis vaccine is: Collapse or shocklike state within 2 days of pertussis vaccination Persistent.000 cells/mm3) due to absolute lymphocytosis is characteristic in the catarrhal stage of pertussis. 17th ed. such as those in daycare. 85.Symptomatic children and adults in the same household Unimmunized children in the same household and in the same daycare. and platelet count 650. 17th ed. 17th ed. such as high fever. and symptomatic adults in the same household All children and adults in the same household or daycare regardless of immunization history or symptoms Explanation: Antimicrobial prophylaxis for pertussis is always given to all household contacts and other close contacts. (See Chapter 180 in Nelson Textbook of Pediatrics. history of immunization. and hyperresponsive episodes. (See Chapter 180 in Nelson Textbook of Pediatrics. persistent crying. 87% lymphocytes).000/mm3.) Convulsions with or without fever within 3 days of pertussis vaccination History of sudden infant death syndrome (SIDS) in the family Question . Complete blood count reveals hemoglobin 12. There are also three siblings (ages 20 mo. She has read about pertussis vaccine and questions you on the acellular pertussis vaccine you are recommending.Group B streptococci Respiratory syncytial virus Question . maximum 2 g/24 hr) for 14 days should be administered promptly to . which of the following household contacts should also be treated? Mother Mother. and the 8-yr-old sibling Mother and the 20-mo-old sibling All three of the siblings All of the household contacts Explanation: Erythromycin (40-50 mg/kg/24 hr divided qid PO. 87. and his father is well. 88. A 6-wk-old infant is proved to have pertussis. persistent crying for longer than 3 hr. father. In addition to the infant. You should advise her that compared with whole-cell pertussis vaccine.) Is less likely to cause encephalopathy Is less likely to cause anaphylaxis Question . hypotonic hyporesponsive episodes. You are advising the mother of a 2-mo-old infant about recommended immunizations. His mother has had a bad cough for 2 wk. for whom erythromycin is prescribed. (See Chapter 180 in Nelson Textbook of Pediatrics. and 8 yr) living in the household. 17th ed. and seizures) occur significantly less frequently among infants who receive DTaP than among those who receive DTP vaccine. all are appropriately immunized for age and are asymptomatic. 6 yr. the acellular vaccine: Is more effective in preventing pertussis Has a longer protective effect Is less likely to cause seizures Explanation: Mild local and systemic adverse events as well as more serious events (including high fever. or symptoms. 17th ed. The highest risk appears to be in the first 2 wk of life in term infants. regardless of age. and with courses of 14 days or longer. (See Chapter 181. Yersinia is rare. Clarithromycin and azithromycin are potential but not proven alternative agents for those who cannot tolerate erythromycin. Shigella is rare in infancy but common in children 1-3 yr of age. (See Chapter 180 in Nelson Textbook of Pediatrics. The most likely agent causing this illness is: Nontyphoidal Salmonella Explanation: Salmonella is by far the most common cause of febrile gastroenteritis in early infancy. The risk of IHPS after treatment with azithromycin or clarithromycin is unknown.all household contacts and other close contacts of the index patient with pertussis.) Question .1 in Nelson Textbook of Pediatrics. such as those in daycare.) Any macrolide administered during the first 2 wk of life Any macrolide administered during pregnancy or during the first 2 wk of life Question .to 10-fold relative risk for infantile hypertrophic pyloric stenosis (IHPS) has been reported in infants younger than 6 wk of age treated with orally administered erythromycin. Campylobacter is second in frequency to Salmonella as a bacterial cause of enteritis in infancy. Rotavirus rarely causes bloody diarrhea. 17th ed. 90. A 1-mo-old infant develops bloody diarrhea associated with fever. history of immunization.) Shigella Campylobacter Yersinia Rotavirus . (See Chapter 180 in Nelson Textbook of Pediatrics. Which of the following is recognized to be associated with infantile hypertrophic pyloric stenosis? Maternal azithromycin administered during pregnancy Any maternal macrolide administered during pregnancy Erythromycin administered during the first 2 wk of life Explanation: A 7. 89. 17th ed. California. particularly in young infants. (See Chapter 182 in Nelson Textbook of Pediatrics. (See Chapter 181. 17th ed. A 10-mo-old child presents to the emergency department with a 4-day history of fever. Explanation: Salmonella ser. 92. 17th ed. The most likely cause of this syndrome is: Salmonella gastroenteritis Aeromonas gastroenteritis Shigella gastroenteritis Explanation: Shigella usually causes diarrhea and fever and sometimes. The child has been exposed to raw seafood. A child who has recently returned from a visit to a developing country experiences a 2-wk illness characterized by gradually increasing fever with temperature that eventually reaches 104°F. and has just experienced a generalized seizure. Question .Question . and abdominal pain. Isolation of Salmonella serotype marina from an infant suggests that: The child is in contact with an iguana. seizures. malaise.1 in Nelson Textbook of Pediatrics. serious illness. 17th ed. and watery diarrhea.2 in Nelson Textbook of Pediatrics. Nothing: the isolation of this serotype has no special epidemiologic significance. associated with headache. The most likely diagnosis is: Cholera Diphtheria Shigellosis Typhoid fever Explanation: Typhoid fever may or may not be associated with diarrhea or constipation and is a prolonged.) Tetanus Question .) The child lives near a marina. (See Chapter 181. 93.) . 91. cough. The child lives in Marina. with temperatures to 105°. marina is usually acquired by contact with an iguana. The symptoms and fever in the child described in Question 94 resolve over the ensuing 3 days. (See Chapter 182 in Nelson Textbook of Pediatrics. Stool culture specimens are obtained. fever with temperatures to 105°F. and diarrhea that initially is watery but becomes grossly bloody.) Continue the antibiotic for 14 days Continue the antibiotic for 14 days and also obtain specimens for culture from all other family members for Shigella . (See Chapter 182 in Nelson Textbook of Pediatrics. 95. The recommended management is to: Continue loperamide Discontinue all antibiotics Continue the antibiotic for 5 days Explanation: A child who has typical dysentery and who responds to initial empirical antibiotic treatment should be continued on that drug for a full 5-day course even if the stool culture is negative. and oral rehydration is initiated. 17th ed. The next step in management should be to administer: Loperamide Ampicillin intravenously Ceftriaxone parenterally Explanation: Shigella infection is the most likely treatable etiology for this acute dysenteric syndrome.) Trimethoprim-sulfamethoxazole None of the above: no antibiotics should be administered until culture results are known Question . The boy's stool culture is reported as negative for enteric pathogens. A 3-yr-old boy acutely develops severe abdominal pain. 94.Rotavirus gastroenteritis Drug ingestion Question . His 8yr-old sister had a similar presentation 9 days ago. including a stool culture that was positive for Shigella flexneri. Stool cultures have low sensitivity for culturing Shigella. vomiting. Ampicillin and TMP-SMZ are poor choices for empirical management because of the frequency of resistant Shigella. 17th ed. Loperamide may prolong illness. A stool culture on MacConkey sorbitol media shows growth of bacteria that suggests E. (See Chapters 182 and 183 in Nelson Textbook of Pediatrics. anuria. Hemolytic-uremic syndrome is typically a complication of infection with E. coli O157:H7 is an enterohemorrhagic pathogen and is responsible for most episodes of hemolytic-uremic syndrome. coli (STEC) commonly produce significant amounts of Shiga toxin and cause hemolytic-uremic syndrome. The most appropriate next step in the management of this girl is: Administration of loperamide . A 2-yr-old girl has an acute afebrile diarrheal syndrome characterized by abdominal pain. (See Chapters 182 in Nelson Textbook of Pediatrics. vomiting.) Any species of Shigella Question . azotemia. and thrombocytopenia after a bout of febrile bloody diarrhea. 17th ed. The same process. and grossly bloody stools. 98. Shigella dysenteriae is occasionally responsible for the hemolytic-uremic syndrome. coli O157:H7. 96. The most likely etiologic agent of this illness is: Campylobacter jejuni Salmonella typhi Enterohemorrhagic Escherichia coli Explanation: Shiga toxin-producing E.Question .) Aeromonas Non-typhi Salmonella Question . can also follow infection with: Salmonella typhi or Campylobacter jejuni Shigella flexneri Shigella sonnei Shigella dysenteriae serotype 1 Explanation: Only Shigella dysenteriae serotype 1 and certain Shiga toxin-producing E. microangiopathic hemolytic anemia with renal failure. A 20-mo-old child develops hemolytic anemia. 97. A fecal toxin electroimmunoassay suggests that a Shiga toxin is present. coli O157:H7. 17th ed. jejuni is a commonly associated pathogen in Guillain-Barré syndrome (autoimmune polyneuropathy).Oral antibiotic therapy. 17th ed.) . and/or renal failure Question . Which of the following indicates a poor prognosis for cholera in children? Illness complicated by hypokalemia Acidosis Acute tubular necrosis Tachycardia and tachypnea Hypoglycemia and seizures Explanation: Coma and death are more common with cholera that is complicated by seizures associated with hypoglycemia. (See Chapter 184 in Nelson Textbook of Pediatrics. Which of the following is the likely infectious agent precipitating this syndrome? Corynebacterium diphtheriae Clostridium botulinum S. 100. In one study.7% of children without hypoglycemia. with the choice based on susceptibility of the Shiga toxin-producing E. 14. hypoglycemia is the most common life-threatening consequence of cholera in children. 17th ed. coli A parenteral antibiotic.3% of children with cholera complicated by hypoglycemia died. The cerebrospinal fluid is normal except for an elevated protein level. The cranial nerves are intact. 101. with the choice based on susceptibility of the Shiga toxin-producing E.) Question . A 1-yr-old child experiences ascending paralysis with peripheral neuropathy. (See Chapter 185 in Nelson Textbook of Pediatrics. coli Hospitalization in intensive care and treatment with intravenous immune globulin (IVIG) Careful follow-up evaluation to monitor for development of thrombocytopenia. compared with 0. dysenteriae serotype 1 Campylobacter jejuni Explanation: C. After dehydration. anemia. Transfusion-related disease remains a risk but is uncommon.) Mesenteric lymph node histology Abdominal ultrasonography Mesenteric lymph node culture Endoscopy Question . the stool culture results may be negative. 103. (See Chapter 186. from contact with animals (dogs). He appears toxic. 17th ed. milk. Of the following diagnostic tests. enterocolitica infection most often develops from ingestion of contaminated food (animal products. If the extent of infection is limited to the mesenteric lymph nodes. enterocolitica? Albumin Fresh frozen plasma Platelet concentrates Intravenous immune globulin Packed RBCs after 2-wk storage Explanation: Y.) Question . and thus a stool culture is not even considered as part of the diagnostic evaluation. 102. or from human-to-human contact. Which of the following blood products has been shown to be associated with the greatest risk for transfusion-associated disease due to Y. (See Chapter 186.2 in Nelson Textbook of Pediatrics.1 in Nelson Textbook of Pediatrics. An adolescent in rural New Mexico living in a household with domestic cats that roam freely presents with fever and chills of sudden onset and extraordinarily painful lymphadenopathy. water). The most likely etiology is: Bartonella henselae infection . 104. which is least useful in establishing a diagnosis of mesenteric lymphadenitis due to Yersinia pseudotuberculosis? Stool culture Explanation: Many patients affected by Y. pseudotuberculosis do not have diarrhea. 17th ed.Clostridium tetani Question . but are rarely encountered in other persons. . (See Chapter 186. 17 Rickettsia rickettsii infection Hantavirus pulmonary syndrome Question . Her symptoms are resolved by the time the culture result is known. See Chapter 187 in Nelson Textbook of Pediatrics. especially in the presence of systemic toxicity. Which of the following organisms is the most likely cause of her exacerbation? Aeromonas Plesiomonas Pseudomonas aeruginosa (nonmucoid strains) Stenotrophomonas maltophilia Burkholderia cepacia Explanation: Mucoid strains of P. 106.3 in Nelson Textbook of Pediatrics. distinguishes bubonic plague caused by Y. aeruginosa are common in persons with cystic fibrosis.Francisella tularensis infection Yersinia pestis infection Explanation: The unusually severe lymph node tenderness. 17th ed. 105. A stool culture identifies the presumed etiology. causing insidious but progressive respiratory deterioration. Which of the following organisms is least likely to be the cause of her illness? Salmonella Campylobacter jejuni Aeromonas hydrophila Plesiomonas shigelloides Pseudomonas aeruginosa (nonmucoid strains) Explanation: Aeromonas and Plesiomonas are common causes of self-limited diarrheal illness.) Question . A 13-yr-old girl with cystic fibrosis experiences an acute exacerbation requiring hospitalization for antibiotic therapy. A 3-yr-old girl experiences mild enteritis. pestis from the typical lymphadenopathy associated with cat-scratch disease or tularemia. 1 in Nelson Textbook of Pediatrics.) A previously well child presenting with a temperature of 40°C.1 and 188. malnutrition (especially in impoverished populations). 107. (See Chapter 188. Sputum culture reveals Pseudomonas aeruginosa. cystic fibrosis. malignancies. leukocytosis. In which of the following patients. extensive burns. and progressive respiratory failure. A 12-yr-old boy with cystic fibrosis experiences an acute exacerbation and is admitted for intravenous antibiotic therapy. and primary immunodeficiencies as well as those receiving immunosuppressive therapy.1 in Nelson Textbook of Pediatrics. ticarcillin-clavulanate. cefoperazone. 17th ed. Gentamicin or another aminoglycoside may be used concomitantly for synergistic effect.) Question .B. and piperacillin-tazobactam. and a white blood cell count of 20. 17th ed. 17th ed. would isolation of Pseudomonas aeruginosa from the sputum be likely? A child with tetralogy of Fallot undergoing cardiac repair A child with cystic fibrosis who has moderate to severe lung disease Explanation: Most human infections due to Pseudomonas and Burkholderia are opportunistic and occur among low birthweight infants and in older infants and children with impaired host defenses. all age 10 yr.) Cefotetan Ceftriaxone Ciprofloxacin Question . (See Chapters 188. 108. The antibiotic recommended for treatment is: Ampicillin-sulbactam Ceftazidime Explanation: Appropriate single agents for treatment of Pseudomonas aeruginosa infection include ceftazidime. such as those with traumatic wounds. (See Chapter 188. cepacia is an opportunist that rarely infects immunocompetent persons but is common in persons with cystic fibrosis and causes an acute respiratory syndrome with fever. a left lower lobe infiltrate.000/mm3 .2 in Nelson Textbook of Pediatrics. ) Question . can be transmitted by many different modes. All of the following are means of transmission of Francisella tularensis except: Tick bite Contact with infected animals Consumption of contaminated foods Inhalation Person-to-person spread Explanation: F. the cause of tularemia. but person-to-person transmission has not occurred. 111. tularensis. 17th ed. 17th ed. All of the following have been shown to be routes of transmission for tularemia except: The bite of an infected tick The bite of a mosquito . (See Chapter 188. 109.) Piperacillin/tazobactam Ciprofloxacin Ticarcillin/clavulanate Question . (See Chapter 189 in Nelson Textbook of Pediatrics. An 18-yr-old adolescent hospitalized in intensive care develops symptomatic nosocomial bacteremia due to Pseudomonas aeruginosa. All of the following are appropriate agents for treatment of this infection except: Ceftazidime Cefotaxime Explanation: Cefotaxime does not provide coverage for Pseudomonas aeruginosa. 110.A child with asthma previously under good control who presents with wheezing unresponsive to bronchodilators An adolescent who has recently been using a hot tub Question .1 in Nelson Textbook of Pediatrics. arthralgias. 17th ed.) Francisella tularensis . 112. headache. an important zoonotic infection. 113.) Ceftriaxone or cefotaxime Trimethoprim-sulfamethoxazole Chloramphenicol Vancomycin Question . (See Chapter 190 in Nelson Textbook of Pediatrics. fleas. 17th ed. Physical examination shows hepatosplenomegaly. The recommended treatment for tularemia is administration of: Gentamicin or streptomycin Explanation: Gentamicin or streptomycin is the agent of choice for treatment of tularemia.) Question . The most likely etiologic agent of this illness is: Actinomyces Bartonella henselae Brucella Explanation: A history of exposure to animals or ingestion of unpasteurized dairy produces is helpful in the diagnosis of brucellosis. (See Chapter 189 in Nelson Textbook of Pediatrics. lassitude. birds) are additional vectors. A 12-yr-old boy develops progressive symptoms of fever. beavers. lice. and abdominal pain. (See Chapter 189 in Nelson Textbook of Pediatrics. Flies. 17th ed. and many animals (squirrels. Chloramphenicol and third-generation cephalosporins are associated with a high clinical failure rate. mosquitos. is most often acquired from a tick bite or directly from rabbits. Further questioning discovers that he and his family live in a rural area and include unpasteurized dairy products in their diet.Aerosol transmission Drinking contaminated water Person-to-person transmission Explanation: Tularemia. (See Chapter 191 in Nelson Textbook of Pediatrics. has been established empirically as effective therapy for legionellosis.) Vancomycin Question . sometimes accompanied by fever. Which of the following is the most useful test for prompt diagnosis of Legionella pneumonia? Culture of respiratory secretions Detection of antigen in respiratory secretions Detection of antigen in urine Explanation: Because of the difficulty in culture techniques and the inconsistency of seroconversion. The most common clinical manifestation of atypical cat-scratch disease is: Seizures and coma Systemic disease Erythema nodosum .) Serologic testing for IgM antibodies Serologic testing for total (IgM and IgG) antibodies Question .Yersinia enterocolitica Question . Cat-scratch disease classically presents as lymphadenopathy. For the patient described in Question 114. the urinary antigen test for Legionella is a useful method in the prompt diagnosis of Legionella infection. 116. the diagnostic test confirms legionellosis. The newer macrolides (azithromycin and clarithromycin) and the quinolones also have excellent activity in vitro. with or without rifampin. 17th ed. (See Chapter 191 in Nelson Textbook of Pediatrics. 17th ed. 114. 115. A 16-yr-old boy is suspected of having legionellosis. The recommended treatment for Legionella infection is drug therapy with: Gentamicin or streptomycin Ceftriaxone or cefotaxime Trimethoprim-sulfamethoxazole Erythromycin Explanation: Erythromycin. 2 in Nelson Textbook of Pediatrics.Oculoglandular syndrome of Parinaud Explanation: The oculoglandular syndrome (conjunctivitis. 17th ed. if any. All of the following statements regarding cat-scratch disease are true except: The causative organism is Bartonella henselae. with an average incubation of 7-12 days.2 in Nelson Textbook of Pediatrics.) Neuroretinitis Question . Explanation: The diagnosis of cat-scratch disease. 17th ed. 118. All of the following statements concerning the symptomatology of catscratch disease are true except: Lymph nodes are tender in 80% of patients Lymph nodes vary in size from 1 to 12 cm . 117. (See Chapter 192.) 6-8 mo (range: 4-12 mo) Question . benefit. The prognosis is generally excellent. Question .) Treatment affords minimal. 17th ed. (See Chapter 192. henselae. 119. is usually confirmed by serologic testing but can also be confirmed by culture. lymph node) is the most frequent atypical manifestation of cat-scratch disease. Which of the following is the characteristic incubation period for development of cutaneous papules from Bartonella henselae after a cat bite or scratch? <24 hr 2-4 days (range: 1-7 days) 7-12 days (range: 3-30 days) Explanation: The lag phase after a cat scratch may be 3-30 days. caused by B.2 in Nelson Textbook of Pediatrics. The diagnosis can be confirmed only by culture of the organism. (See Chapter 192. Recovery usually occurs slowly over many months. 17th ed.) . 2 dogs. 121. 17th ed.2 in Nelson Textbook of Pediatrics. (See Chapter 192. The most reasonable therapeutic approach would be: Reassurance and observation Explanation: This is a typical presentation of cat-scratch disease caused by Bartonella henselae.1°C) occurs in 90% of patients Explanation: Fever occurs in only 30% of cases of cat-scratch disease. which has resolved. You palpate an enlarged preauricular node on the same side. He initially had a low-grade fever and fatigue. 3cm left axillary node that is tender. is the presumed mode of spread. Unless there are complications. Physical examination shows a large.2 in Nelson Textbook of Pediatrics. 17th ed.) Malaise and fatigue occur in 30% of patients Splenomegaly occurs in 9% of patients Question . conservative care and observation constitute appropriate management. but with no change in the node.Fever (>38.2 in Nelson Textbook of Pediatrics. His family reports that he took most of the prescribed doses. (See Chapter 192. (See Chapter 192. A 5-yr-boy presents to your office with an enlarged axillary node that has been present for 3 wk. There is no other lymphadenopathy or abnormalities except for healing scratches on both arms.) Staphylococcus aureus Toxoplasma gondii Question . A 3-yr-old presents to your office with a unilateral conjunctivitis that has been present for 5 days. Direct inoculation of the eye. The most likely etiologic agent responsible for her condition is: Borrelia burgdorferi Francisella tularensis Bartonella henselae Explanation: Bartonella henselae is the cause of Parinaud oculoglandular syndrome (unilateral conjunctivitis accompanied by preauricular lymphadenopathy). He had visited the emergency department of a local hospital for the same problem 2 wk ago and was prescribed a course of cephalexin. 120. He has no known exposure to infection although the family has 3 birds. There is no treatment that is clearly beneficial. perhaps by contaminated hands after contact with a cat. She has no history of recent travel. and 2 cats. but her family did receive a kitten as a pet approximately 2 mo ago. Repeating the course of cephalexin Hospital admission for intravenous oxacillin therpay Referral for immediate node removal Azithromycin therapy for 14 days Question . pale color. 17th ed. Their 12-yr-old boy has become ill with fever.) Supportive treatment only. The preferred treatment for infant botulism is: Penicillin G Clindamycin Hydration and cathartics Botulism immune globulin (BIG) Explanation: Botulism immune globulin can be obtained from the California Department of Health Services. A family returns to your travel clinic after a three-week vacation to the Peruvian Andes. is found only in the valleys of the Andes Mountains in Peru. 17th ed. caused by Bartonella bacilliformis. (See Chapter 192. only supportive care can be provided.1 in Nelson Textbook of Pediatrics. Ecuador. (See Chapter 193 in Nelson Textbook of Pediatrics. because BIG is not universally available. tachycardic with mild icterus. he is pale. Colombia. and has generalized lymphadenopathy. 122. and Bolivia. malaise. In many countries. The most likely etiologic agent is: Yersinia pestis Rickettsia rickettsii Bartonella bacilliformis Explanation: Bartonellosis. 123. and a yellowish. Use of BIG significantly reduces both mean hospital stay and hospital costs. Chile.) Staphylococcus aureus Salmonella Question . On physical examination. including intensive care if necessary . ) Soft feces Normal deep tendon reflexes Metabolic acidosis Question . 17th ed. and even obstructive apnea. feeble cry. weak suck.) Question . 125. 4 children who attend the same elementary school present to the emergency department with vomiting. bulbar palsies are evident as poor feeding. Laboratory results include normal results on complete blood count and spinal fluid analysis. health care jurisdictions. 17th ed. and weakness of the neck and arms. from either natural occurrence or bioterrorism. but no fever. (See Chapter 193 in Nelson Textbook of Pediatrics. Obtain chest and abdominal MRI scans on the most severely affected patients. fatigued appearance. A 2-mo-old infant is suspected of having infant botulism. drooling. You suspect that these illnesses represent an outbreak. and swallow Explanation: In infants. 126. Over a 6-hr period. double vision. slurred speech. Their parents report that other classmates are having similar symptoms. Which is the best means to confirm the diagnosis? Lumbar puncture . Which of the following steps should be taken immediately? Administer parenteral broad-spectrum antibiotics. (See Chapter 193 in Nelson Textbook of Pediatrics. Which of the following findings on the history or physical examination excludes a diagnosis of infant botulism? No ingestion of honey or corn syrup Normal latch. as are a parent and a teacher who chaperoned the class on its field trip yesterday to the State Capitol building. 124. suck. Perform an edrophonium (Tensilon) test on the most severely affected patients. Suspected botulism represents a medical and public health emergency that is immediately reportable by telephone in most U.Question . Telephone the local or state health department to report a possible outbreak of botulism. Administer gastric lavage and catharsis.S. Explanation: Outbreaks of food-borne botulism have been reported in North America associated with many different types of foods. His last tetanus immunization was at age 12 yr. (See Chapter 193 in Nelson Textbook of Pediatrics. 17th ed. avoidable source of botulinum spores for infants is honey. 128. Honey is an unsafe food for any child younger than 1 yr of age.) Question . The injury occurred 1 hr ago in a school play yard.CT scan Muscle biopsy Electromyography Fecal specimen Explanation: The diagnosis of botulism is unequivocally established by demonstrating the presence of botulinum toxin in serum or of C. 17th ed. An 18-yr-old adolescent presents to the emergency department with a nail puncture wound of his foot.) All of the above Question . The child was born at home. A 2-mo-old child is seen in your clinic for the first time. botulinum toxin or organisms in wound material or feces. His wound is cleansed and debrided. 127. and this is the first well-child visit. Risk factors for infant botulism that should be communicated to the parents include: Gardening Home construction Frozen vegetables Honey Explanation: The one identified. (See Chapter 193 in Nelson Textbook of Pediatrics. Which of the following measures is recommended? Tetanus vaccination alone TD Td DTaP None of the above: no immunization is necessary Explanation: A tetanus toxoid booster (preferably Td) is given to all persons with any wound if their tetanus immunization status is . as confirmed in his medical records. (See Chapter 194 and Table 194-1 in Nelson Textbook of Pediatrics. With delayed wound care. or (2) the wound is more serious and more than 5 yr have passed since the last booster.) Question . (See Chapter 194 and Table 194-1 in Nelson Textbook of Pediatrics. or (2) the wound is more serious and more than 5 yr have passed since the last booster. A 12-yr-old boy presents to the emergency department with a nail puncture wound of his foot. 129. His parents came to the United States 4 yr ago from Mexico. as confirmed in his medical records. active immunization should be started at once. An 12-yr-old boy presents to the emergency department with a nail puncture wound of his foot. His wound is cleansed and debrided. His wound is cleansed and debrided. His last tetanus immunization was at age 4 yr. The injury occurred 1 hr ago in a school play yard. The injury occurred 1 hr ago in a school play yard." Which of the following is recommended? Tetanus vaccination alone Tetanus vaccination plus tetanus immune globulin Td Explanation: A tetanus toxoid booster (preferably Td) is given to all persons with any wound if their tetanus immunization status is unknown or incomplete. and his vaccination records from childhood are not available.) DTaP None of the above: no immunization is necessary Question . Which of the following is recommended? Tetanus vaccination alone Tetanus vaccination plus tetanus immune globulin Td Explanation: A tetanus toxoid booster (preferably Td) is given to all persons with any wound if their tetanus immunization status is unknown or incomplete. active immunization should be started at once. A booster is given to injured persons who have completed their primary immunization series if (1) the wound is . A booster is given to injured persons who have completed their primary immunization series if (1) the wound is clean and minor but more than 10 yr have passed since the last booster. He has not had medical evaluation in the United States since. although his parents report that "all his childhood immunizations were completed. A booster is given to injured persons who have completed their primary immunization series if (1) the wound is clean and minor but more than 10 yr have passed since the last booster. 17th ed. 17th ed. 130.unknown or incomplete. With delayed wound care. difficile colitis is gentamicin . . 17th ed. broad-spectrum cephalosporins. and clindamycin are the most frequent offenders. difficile. 132. (See Chapter 194 and Table 194-1 in Nelson Textbook of Pediatrics. The antibiotic that most commonly causes C. In most instances this course combined with appropriate fluid and electrolyte replacement is sufficient. if at all possible. (See Chapter 195 in Nelson Textbook of Pediatrics. Explanation: Virtually all known antibiotics have been implicated. active immunization should be started at once. In fact. 131. difficile. Her fever and tachypnea have resolved. penicillins.) DTaP None of the above?no immunization is necessary Question . She has been receiving intravenous ampicillin.) Obtain ventilation-perfusion scan Obtain CT scan of the abdomen Add erythromycin or doxycycline Question . With delayed wound care. A 15-yr-old white girl develops bloody diarrhea on her sixth day of hospitalization for therapy of presumed bacterial pneumonia.) Most children with antibiotic-associated diarrhea will improve without specific antibiotic treatment. All of the following statements regarding Clostridium difficile-associated diarrhea are true except: Antibiotic-associated diarrhea is often related to production of a toxin by C. Newborn and young infants are commonly colonized by C. or (2) the wound is more serious and more than 5 yr have passed since the last booster. 17th ed. and her oxygenation is now normal.clean and minor but more than 10 yr have passed since the last booster. she was scheduled for discharge. (See Chapter 195 in Nelson Textbook of Pediatrics. Which of the following actions would be most appropriate? Switch the antibiotic to a third-generation cephalosporin Discontinue ampicillin Explanation: The first and essential step in treatment of Clostridium difficile-associated diarrhea is the discontinuation of the current antibiotics. 17th ed. difficile is the most common risk factor for C. A previously healthy 6-mo-old child who just completed antibiotic treatment for acute otitis media and now is healthy and asymptomatic undergoes a stool culture to check for antibiotic-associated diarrhea. 134.Treatment of C. The preferred antibiotic treatment is: Oral clindamycin Oral vancomycin Intravenous vancomycin Oral metronidazole None of the above: antibiotic treatment is not indicated Explanation: difficile is frequently isolated from the stool of healthy infants. All of the following are risk factors for Clostridium difficile-associated diarrhea except: Antibiotics that alter normal gut flora but do not affect growth of C.. difficile.) Bowel stasis Question . Question . difficile Chemotherapy Bowel surgery Oral ingestion of Lactobacillus (e. difficile . difficile colitis should include stopping antibiotics whenever feasible.) Question .g. The interpretation of a positive stool culture or toxin requires clinical correlation. Treatment would not be indicated for an asymptomatic infant. 133. 17th ed. (See Chapter 195 in Nelson Textbook of Pediatrics. 17th ed. The stool culture grows C. (See Chapter 195 in Nelson Textbook of Pediatrics. Which of the following best describes the pathogenesis of Clostridium difficile-associated diarrhea? Invasion of the small bowel wall by C. 135. difficileassociated diarrhea. yogurt) Explanation: Use of antibiotics that impair growth of normal gut flora but not C. difficile Production of toxins Explanation: Clostridium difficile strains may produce two toxins.) . 136. carbapenems (imipenem and meropenem). clindamycin. (See Chapter 195 in Nelson Textbook of Pediatrics. Both toxins are internalized and act within cells to modify proteins. 17th ed. Penicillin and vancomycin are active against the gram-positive anaerobes.) Gas formation in tissues Tissue necrosis Question . ticarcillin-clavulanate. A and B. and chloramphenicol. All of the following may be clues to anaerobic infection except: Pus that shows no growth on routine laboratory testing Highly variable morphology of organisms seen on Gram stain Sweet-smelling odor Explanation: Anaerobic infections should be suspected with foulsmelling pus.Invasion of the large bowel wall by C. (See Chapter 196 in Nelson Textbook of Pediatrics. difficile None of the above: the pathogenesis is unknown Question . resulting in cell death. In addition.) Enteroaggregative strains of C. All of the following antimicrobials can provide good anti-anaerobic coverage except: Clindamycin Cefoxitin Meropenem Gentamicin Explanation: Antibiotics that are active against anaerobic organisms include metronidazole. cefoxitin. 137. the culture may be negative if the specimen is not handled properly under anaerobic conditions. and piperacillin-tazobactam). which is present in half of anaerobic infections. penicillins combined with -lactamase inhibitors (ampicillin-sulbactam. 17th ed. (See Chapter 196 in Nelson Textbook of Pediatrics. 17th ed. 138. (See Chapter 196 in Nelson Textbook of Pediatrics. Which of the following infections is most likely to involve anaerobes? Neonatal meningitis Tubo-ovarian abscess Explanation: Pelvic inflammatory disease and tubo-ovarian abscesses are frequently due to mixed aerobes and anaerobes.)i Staphylococcus aureus and Prevotella melaninogenica Klebsiella pneumoniae and Enterococcus faecalis Escherichia coli and Klebsiella pneumoniae Question .Metronidazole Question . (See Chapter 196 in Nelson Textbook of Pediatrics. which of the following specimens is most likely to yield the responsible pathogen? Blood Stool Throat swab Peritoneal fluid Explanation: Rupture of the gut leads to spillage of gut flora into the peritoneal cavity. 17th ed.) Urine Question . 17th ed. 17th ed) Pharyngitis . and thus the peritoneal fluid is the best source for a culture specimen. (See Chapter 196 in Nelson Textbook of Pediatrics. 139. Which of the following organisms are most likely to cause postoperative infection in such cases? Peptostreptococcus and Escherichia coli Bacteroides fragilis and Escherichia col Explanation: Bacteroides fragilis and other Bacteroides species are the most common anaerobes causing infection. A 10-yr-old boy who presents with right lower quadrant pain is found to have a ruptured appendix. Anaerobic bacteremia is uncommon but may occur. which is usually mixed. In the patient described in Question 138. 140. including chest film. Which of the following is the best interpretation of this finding? Mycobacterium tuberculosis infection Explanation: Tuberculosis infection typically occurs after inhalation of M. 143.) 50% 90-95% Almost 100% Question . The most common means of transmission of Mycobacterium tuberculosis is: Inhalation of organisms originating from soil or other environmental sources Ingestion of organisms originating from soil or other environmental sources . A reactive skin test and the absence of clinical and radiographic manifestations of disease are the hallmark of this stage. (See Chapter 197 in Nelson Textbook of Pediatrics. A 9-yr-old child is found to have a positive PPD test result on routine screening. Laboratory tests. 5-10% of asymptomatic persons with M. 141. 17th ed. (See Chapter 197 in Nelson Textbook of Pediatrics. In the child described in Question 141. 142. tuberculosis infection will experience disease in the future. There are no symptoms. 17th ed. tuberculosis. are normal.Otitis media Paronychia Question . the risk of developing clinical and radiographic evidence of tuberculosis without institution of therapy is: <1% 5-10% Explanation: Without appropriate treatment.) Mycobacterium tuberculosis disease Mycobacterium tuberculosis or atypical Mycobacterium infection Mycobacterium tuberculosis or atypical Mycobacterium disease Recent exposure to Mycobacterium tuberculosis but neither infection nor disease Question . but this is variable. 17th ed. 144. The best culture specimen in young children is three consecutive early-morning gastric aspirates obtained before the child has arisen and before peristalsis has emptied the stomach contents. Reaction is measured by the amount of erythema and induration. transmission of M. A 3-yr-old child in whose mother tuberculosis was just diagnosed has a positive PPD skin test result. (See Chapter 197 in Nelson Textbook of Pediatrics. tuberculosis. Corticosteroids may decrease the reaction. Up to 50% of persons with tuberculous meningitis or disseminated disease do not react initially to PPD. 17th ed. (See Chapter 197 in Nelson Textbook of . Question . A chest film shows a localized.) Corticosteroid therapy may increase the amount of reaction. 145. tuberculosis is from person-to-person spread by airborne mucus droplet nuclei. most become reactive after several months of therapy. Negative cultures never exclude the diagnosis of tuberculosis.Person-to-person spread by direct contact with infected discharge or contaminated fomite Person-to-person spread by infected airborne droplets Explanation: In almost all cases. nonspecific infiltrate in the peripheral segments of the right lower lobe. The yield is approximately 50%. Persons with tuberculosis meningitis often do not react to the PPD skin test. The reaction is measured 24 to 48 hr after administration. (See Chapter 197 in Nelson Textbook of Pediatrics. Explanation: The 5-TU PPD skin test is administered by intradermal injection and is read by measuring the amount of induration (not erythema) 48-72 hr after administration. usually from an actively infected person with cavitary tuberculosis and coughing. A true statement concerning the 5-unit PPD skin test for tuberculosis is: It is administered by subcutaneous injection.) Question . The most appropriate course of action to confirm the diagnosis of tuberculosis disease in this child is: Culture of sputum Culture of pulmonary secretions obtained by bronchoscopy Culture of gastric contents obtained by gastric aspiration Explanation: Pulmonary tuberculosis is best confirmed by culture and isolation of M. but the yield from bronchoscopy is even lower. tuberculosis drug resistance is chromosomally mediated and is not transferable. 146. (See Chapter 197 in Nelson Textbook of Pediatrics. The estimated frequency of naturally drug-resistant organisms varies from 105 to 108. Drug resistance is transferable between organisms. 17th ed. Drug resistance to any one drug is independent of resistance to other drugs. 147. Question .) Peripheral neuritis is rare in children. and pyrazinamide are bacteriostatic. None of the above. 17th ed. Explanation: M. Hepatotoxicity is rare in children.Pediatrics. A true statement concerning drug resistance of Mycobacterium tuberculosis is: Naturally resistant organisms occur at a frequency of about 102 to 103 .) Isoniazid. It can be given orally or intramuscularly. 17th ed. Drug resistance to each drug is independent of resistance to other drugs. The recommended treatment for active pulmonary tuberculosis in children is: . (See Chapter 197 in Nelson Textbook of Pediatrics.) Percutaneous lung and pleural biopsies for culture and histopathology Segmental lobectomy for culture and histopathology Question . rifampin. Rapid acetylation is associated with lower rates of peripheral neuritis and hepatotoxicity. Explanation: Rapid acetylation of isoniazid is more frequent in African-Americans and Asians than among whites. There is no correlation between acetylation rate and either efficacy of treatment or adverse events in children. Question . 148. All of the following statements concerning isoniazid are true except: It penetrates all body tissues and fluids. Treat the mother and isolate her from the newborn until she has three consecutive negative sputum smears and cultures. and congenital tuberculosis is suspected. The mother has no other symptoms and is ready to be discharged from the hospital and is willing to comply with her recommended treatment. rifampin. 149. 17th ed. The most important clue to diagnosis is a family history of tuberculosis. (See Chapter 197 in Nelson Textbook of Pediatrics. The recommended management strategy for the newborn is to: Treat the mother and isolate her from the newborn until she has been treated for 2 wk.) Positive PPD skin test results in family members Acid-fast organisms on gastric aspirate Meningitis Hepatitis Question . 150. nodular infiltrate throughout both lungs. with ethambutol during the first 2 mo Question . All of the following are expected additional findings in this newborn except: Positive PPD skin test result Explanation: The PPD test usually does not yield positive results initially in children with congenital tuberculosis. severe respiratory distress. The mother of a newborn is found to have an abnormal-appearing admission chest film and acid-fast bacilli on sputum smear. . with pyrazinamide during the first 2 mo Explanation: The American Academy of Pediatrics and the Centers for Disease Control and Prevention recommend that children with pulmonary tuberculosis be treated with 6 mo of isoniazid and rifampin supplemented with pyrazinamide for the first 2 mo. (See Chapter 197 in Nelson Textbook of Pediatrics. and hepatomegaly. and pyrazinamide for 6-9 mo Isoniazid and rifampin for 6 mo. A 2-wk-old neonate experiences high fever. 17th ed. The chest film shows a fine.) Isoniazid and rifampin for 6 mo. but results become positive in 1-3 mo.Isoniazid for 9 mo Isoniazid and rifampin for 6 mo Isoniazid. 17th ed. 151.) Borderline lepromatous leprosy Borderline leprosy Borderline tuberculoid leprosy Tuberculoid leprosy Question . The form of leprosy characterized by a single large lesion that slowly enlarges. (See Chapter 199 in Nelson Textbook of Pediatrics. The disease associated most frequently with atypical mycobacteria in children is: Cellulitis Lymphadenitis Explanation: Lymphadenitis of the superior or anterior cervical or submandibular areas is the most frequent manifestation of atypical mycobacterial infections in children.Treat the mother. a vigorous and specific cell-mediated immune response. but complete resolution may take 8-12 mo. Response of lepromatous leprosy to therapy may take 2-5 yr. destruction of the cutaneous nerve fibers. no isolation is necessary. no isolation is necessary. 152. Treat the mother and treat the infant with isoniazid and rifampin for 6 mo. 17th ed. but few or absent bacilli. is: Lepromatous leprosy Explanation: Tuberculoid and lepromatous leprosy constitute the ends of the spectrum of leprosy. Treat the mother and treat the infant with isoniazid until the mother is sputum culture-negative for 3 mo. (See Chapter 198 in Nelson Textbook of Pediatrics.) Question . Tuberculoid leprosy responds well to treatment. 17th ed. and granulomas composed of epithelioid cells and lymphocytes. with pyrazinamide during the first 2 mo. no isolation is necessary. Explanation: Isoniazid therapy for newborns has been so effective that separation of the mother and infant is no longer mandatory unless the mother is ill enough to require hospitalization or is expected not to adhere to her treatment regimen. (See Chapter 197 in Nelson Textbook of Pediatrics. heavy cellular infiltration in the dermis.) Skeletal infections . Fine-needle aspiration may help with the diagnosis. with pyrazinamide during the first 2 mo Clarithromycin and rifampin for 6 mo Complete surgical excision followed by clarithromycin and rifabutin for 6 mo Complete surgical excision alone Explanation: Complete surgical excision is the preferred treatment of lymphadenitis caused by atypical mycobacteria. (See Chapter 199 in Nelson Textbook of Pediatrics. 155. but excisional biopsy is the cure. 17th ed. It began at the site of an abrasion that he incurred while cleaning his fish aquarium. The recommended treatment for cervical lymphadenitis caused by atypical mycobacteria is: Isoniazid and rifampin for 6 mo Isoniazid and rifampin for 6 mo. begin antituberculosis therapy Oral clarithromycin therapy continued until the swelling resolves Question .) Question . 17th ed. Antimycobacterial therapy is necessary only if there is concern for M.Pneumonia Urinary tract infections Question . (See Chapter 199 in Nelson Textbook of Pediatrics. Which of the following therapeutic options is the optimal method to manage symptomatic nontuberculous mycobacterial lymphadenitis? Complete surgical excision Explanation: The treatment of choice for symptomatic nontuberculous mycobacterial lymphadenitis is complete excision. 154. The most likely infecting organism in this case is: Sporothrix schenckii . solitary nodule that slowly enlarged over several weeks. 153.) Administration of isoniazid and rifampin for 6 mo Await suppuration. A 16-yr-old adolescent developed a nontender. then incise and drain Perform partial biopsy. tuberculosis infection or if chronic drainage develops. FTA-ABS) are not quantified and usually remain positive for life. Which of the following serologic patterns is most consistent with successful treatment of primary syphilis in an adolescent 2 yr previously? Negative VDRL. positive MHA-TP Question . and serial maternal VDRL titers that do not . (See Chapter 199 in Nelson Textbook of Pediatrics. all of the following are risk factors for congenital syphilis except: Treatment of the mother with erythromycin Treatment of the mother with doxycycline Change in maternal VDRL titer from 1:32 at treatment to 1:16 at delivery Treatment of the mother more than 30 days before delivery Explanation: Risk factors for congenital syphilis are maternal treatment that was inadequate. unknown. RPR) can be quantified and usually become nonreactive with 1 yr of adequate therapy for primary syphilis and within 2 yr of adequate therapy for secondary syphilis.Mycobacterium avium complex Mycobacterium marinum Explanation: "Swimming pool granuloma" or "fish tank granuloma" is caused by M. Treponemal tests (MHA-TP. 17th ed. 157. negative MHA-TP Negative VDRL. positive MHA-TP Explanation: Nontreponemal tests (VDRL. marinum and develops after inoculation of the organism at the site of a minor abrasion. negative MHA-TP VDRL 1:8. treatment with a nonpenicillin regimen. treatment given at less than 30 days before delivery.) Mycobacterium leprae Chlamydia trachomatis (LGV biovar) Question . 17th ed. or undocumented. (See Chapter 200 in Nelson Textbook of Pediatrics. even with adequate therapy. 156.) VDRL 1:16. RPR 1:8 VDRL 1:8. In a newborn whose mother was treated for syphilis during pregnancy. The newborn may be treated with a combination of ampicillin and gentamicin for 7 to 10 days. RPR negative. Neurosyphilis cannot be excluded in this newborn. 159. His CSF was clear. Only T. the causative organism of Lyme disease. (See Chapter 200 in Nelson Textbook of Pediatrics. Th nontreponemal tests (VDRL.) VDRL titer 1:32. 17th ed.) . 0 RBCs. are endemic in the United States.) All cases of neurosyphilis would have CSF pleocytosis and a reactive CSF VDRL result. the CSF VDRL result was nonreactive. (See Chapter 200 in Nelson Textbook of Pediatrics. which of the following is true? The newborn has symptomatic congenital syphilis but not neurosyphilis and can therefore be treated with benzathine penicillin. pallidum and Borrelia burgdorferi. protein of 80 mg/dL. and glucose of 49 mg/dL. Based on this examination. FTA-ABS) material with other spirochetes. Question . The most likely explanation is: Primary syphilis Secondary syphilis Latent syphilis Lyme disease Explanation: There is limited cross-reactivity of the treponemal test (MHA-TP. He also had a desquamative skin rash consistent with congenital syphilis. 17th ed. An MHA-TP test of the CSF is necessary to diagnose neurosyphilis. RPR) are uniformly nonreactive in Lyme disease. therefore. A sexually active adolescent who has never been treated for syphilis is found to have a negative result on a VDRL test and a positive result on an MHA-TP. (See Chapter 200 in Nelson Textbook of Pediatrics. He was anemic and had thrombocytopenia and mild hepatomegaly. 158. with 5 white blood cells (WBCs). Explanation: This infant requires a complete course of therapy for neurosyphilis. 17th ed. A full-term male newborn whose mother had reactive Venereal Disease Research Laboratory (VDRL) and microhemagglutination assay-Treponema pallidum (MHA-TP) results at the time of delivery was evaluated.decrease sufficiently (at least fourfold) to demonstrate a cure. he should be treated for neurosyphilis. Question . . The radiologist calls to report that a plain x-ray that you ordered to evaluate a hip-click shows periostitis. hepatomegaly. They live in an area in which Lyme disease is common. which all resolved briefly. Osteochondritis is a common finding in congenital cytomegalovirus.) Herpes simplex virus Parvovirus B19 Question . Which diagnosis is most likely? Relapsing fever Leptospirosis Explanation: A biphasic course is characteristic of icteric leptospirosis (Weil syndrome). The parents are unsure how long the tick had been attached. proteinuria. Hepatorenal dysfunction follows anicteric leptospirosis in less than 10% of cases. (See Chapter 200 in Nelson Textbook of Pediatrics. and icterus. is most typical of congenital syphilis. 17th ed. A 6-yr-old child is brought to your office because a tiny tick was found and removed from his forearm. (See Chapter 202 in Nelson Textbook of Pediatrics.False-negative results on VDRL testing Question . and syphilis infections. 160. rubella. emesis. The next step in the proper treatment of this patient should be to: Send the tick to be tested for evidence of B. although they thought that it probably had not been there for more than 1 day. This finding suggests congenital infection by which of the following agents? Cytomegalovirus and rubella virus Toxoplasma gondii Syphilis Explanation: Periostitis. and myalgias. 162. occurring in the long bones.) Infection mononucleosis Hantavirus pulmonary syndrome Acute HIV infection Question . 161. He now returns with hematuria. 17th ed. A sexually active adolescent who lives in a rural area with contact with farm animals presented 1 wk ago with an acute febrile illness associated with headache. burgdorferi. The most appropriate next step in treating this patient is to: Order a test for serum antibodies against Borrelia burgdorferi to . flat. The rash is not pruritic. The only abnormality on the examination is the circular. but amoxicillin (for 14-21 days) is the treatment of choice in children younger than 8 yr. and observing the patient for the development of a rash is reasonable.) Ceftriaxone Erythromycin Trimethoprim-sulfamethoxazole Question . which is about 6 cm in diameter and is not tender. children younger than 8 yr should not be treated with doxycycline because it may cause permanent discoloration of the teeth. 164. Erythromycin is an alternative for persons who cannot take either doxycycline or amoxicillin.Reassure the parents that the risk of infection is small and have them observe the area around the bite for the development of a rash. (See Chapter 204 in Nelson Textbook of Pediatrics. Where effective alternatives are available. (See Chapter 204 in Nelson Textbook of Pediatrics. Order a serologic test for antibodies against B. erythematous rash.) Begin prophylactic treatment with amoxicillin. burgdorferi in the child. The child has been afebrile and has had no other systemic symptoms. Most patients with Lyme disease experience erythema migrans even if a tick bite is not remembered. Explanation: The risk of Lyme disease is very small. The child's parents state that they have recently returned from a vacation in Massachusetts on Cape Cod and that a small tick had been removed from the same area where the rash is now. Wait 1 mo and then order a serologic test for antibodies against B. 17th ed. A 4-yr-old boy is brought to your office because of a circular reddish rash on the right side of his chest. burgdorferi in the child. 17th ed. Question . The preferred agent for treatment of Lyme disease in a child 13 yr of age is: Doxycycline Amoxicillin Explanation: Doxycycline (for 14-21 days) is the treatment of choice for Lyme borreliosis in children older than 8 yr of age. 163. Most patients with Lyme disease experience erythema migrans even if a tick bite is not remembered. Findings on physical examination are entirely normal.) Begin treatment with ceftriaxone Perform a lumbar puncture to be certain that the child's central nervous system is not involved Question . in a house surrounded by woods. although he does spend time outdoors. burgdorferi on the child in 1 mo. Explanation: The risk of Lyme disease is very small. She has not seen any ticks on her son. They live in northern Westchester County. New York.) Question . The most appropriate next step in management would be to: . she realizes that it is actually a small tick. There has been no weight loss. His mother is concerned about Lyme disease (they live in Connecticut and she recently heard a lecture about Lyme disease at the public library). Begin prophylactic treatment with a single dose of doxycycline. She is not sure how long the tick might have been on the child. She denies any stress at home or at school. Begin prophylactic treatment with a 10-day course of amoxicillin. (See Chapter 204 in Nelson Textbook of Pediatrics. (See Chapter 204 in Nelson Textbook of Pediatrics. A 13-yr-old boy comes to your office with a chief complaint of fatigue and intermittent arthralgia and headache for 8 mo. 17th ed. She removes it with tweezers and brings both the tick and the child to your office. and observing the patient for the development of a rash is reasonable. The mother of a 5-yr-old boy notices a new "mole" on her child's neck. 166. burgdorferi. Reassure the mother and instruct her to call if a rash subsequently develops at the site of the bite. 165. Upon closer inspection. The most appropriate next step in management would be to: Send the tick to be tested by the polymerase chain reaction assay to ascertain whether it is infected with B. 17th ed. The child is quiet and answers your questions in monosyllables. Reassure the mother and order a serologic test for antibodies against B. with no evidence of synovitis.confirm that the child has Lyme disease Begin treatment with doxycycline Begin treatment with amoxicillin Explanation: Amoxicillin is the treatment of choice for this early lesion of erythema migrans. there is a large (8-cm) erythematous annular rash on his upper back and a unilateral peripheral palsy of the facial nerve.Reassure the mother and order a serologic test for antibodies against B. His parents are concerned about his prognosis because of Lyme disease in a cousin that seems to be untreatable.) Begin treatment for Lyme disease with ceftriaxone administered intravenously. Question .) Reassure the mother and order tests for infectious mononucleosis and other possible chronic infections. The neck is supple. and the child appears otherwise well. (See Chapter 204 in Nelson Textbook of Pediatrics. Reassure the mother and order a CT scan of the brain. Reassure the mother and explain why it is very unlikely that her child has Lyme disease in the absence of any objective findings. There is no evidence that the clinical course of the facial palsy with Lyme disease is affected by antimicrobial treatment. Explanation: There is no evidence of Lyme disease. 17th ed. It would be advisable to discuss other possible non-organic causes for the symptoms. On physical examination. burgdorferi. burgdorferi. Obtain a bone marrow aspirate. Explanation: Paralysis of the facial (7th) cranial nerve is relatively common in children with Lyme disease and may be the initial or the only manifestation of infection. (See Chapter 204 in Nelson Textbook of Pediatrics. The paralysis usually lasts 2 to 8 wk and resolves completely in most cases. He has been afebrile and otherwise asymptomatic. The most appropriate next step in management would be to: Begin treatment for Lyme disease with doxycycline administered orally. 167. Obtain a CSF specimen by lumbar puncture. 17th ed. You have just diagnosed erythema migrans in a 14-yr-old boy. A 10-yr-old boy comes to your office with a chief complaint of facial nerve palsy. Which of the following is the most accurate characterization of the prognosis . Reassure the mother and explain that the child is completely normal. Defer treatment pending the results of a serologic test for antibodies against B. question . 168. He has never had fever blisters or cold sores. He lives in an area of New Jersey in which Lyme disease is endemic but has not had any recognized tick bites. even if treated appropriately. or cefotetan Erythromycin. The impression that Lyme disease requires prolonged treatment. Lyme disease among some families may be more resistant to treatment. Erythema migrans often proceeds to late Lyme disease. A Gram stain is most likely to reveal: Polymorphonuclear leukocytes and gram-negative diplococci .) Gentamicin or kanamycin Trimethoprim-sulfamethoxazole Question . Lyme disease is difficult to treat and may require therapy with the same antibiotic for several months. In fact. including intravenous antimicrobial therapy.) Question . (See Chapter 205 in Nelson Textbook of Pediatrics. 17th ed. cefotaxime. the most common reason for apparent treatment failure is misdiagnosis in patients who do not have Lyme disease. clarithromycin. Explanation: There is a widespread misconception that Lyme disease is difficult to treat successfully and that chronic symptoms and clinical recurrences are common. mucoid white urethral discharge. The prognosis for children treated for Lyme disease is excellent. Doxycycline is an alternative for patients older than 8 yr.for Lyme disease? Lyme disease is difficult to treat and may require multiple sequential courses of different antibiotics. (See Chapter 204 in Nelson Textbook of Pediatrics. An 18-yr-old sexually active boy complains of penile discomfort and dysuria that has developed slowly over the past 12 days. 169. and that treatment is often unsuccessful can be attributed to the treatment of patients whose symptoms were not due to Lyme disease. 170. 17th ed. or azithromycin Explanation: The newer macrolide antibiotics clarithromycin and azithromycin are preferred because they are tolerated better than erythromycin. Physical examination reveals a scanty. The recommended agent for treatment of pneumonia caused by Mycoplasma pneumoniae is: Amoxicillin or ampicillin Ceftriaxone. Sexual partners should also be treated to avoid recurrent disease. There is no fever.) Benzathine penicillin G weekly for 3 doses Cefotetan Ceftriaxone Gentamicin Question . 171. A 2-mo-old infant presents with conjunctivitis. The most likely etiologic agent is: Influenza virus Parainfluenza virus Respiratory syncytial virus Adenovirus . and a mild cough. mucoid-white urethral discharge are a classic presentation of nongonococcal urethritis in males.Polymorphonuclear leukocytes and gram-positive diplococci Polymorphonuclear leukocytes and gram-negative bacilli Polymorphonuclear leukocytes and gram-negative bacilli Polymorphonuclear leukocytes only Explanation: Penile discomfort and dysuria accompanied by a scanty.) Question . Physical examination reveals the presence of crackles bilaterally. which of the following is a recommended treatment? Azithromycin Explanation: Nongonococcal urethritis in adolescents and adults is treated with azithromycin (1 g PO as a single dose) or doxycycline (100 mg bid PO for 7 days). 17th ed. This is caused by Ureaplasma urealyticum and Mycoplasma hominis. (See Chapter 206 in Nelson Textbook of Pediatrics. 17th ed. The diagnosis of nongonococcal urethritis is confirmed by Gram stain of urethral discharge showing at least three polymorphonuclear leukocytes per oil-immersion field and the absence of gram-negative diplococci. For the patient described in Question 170. (See Chapter 206 in Nelson Textbook of Pediatrics. tachypnea. 172. Rales are common.3 in Nelson Textbook of Pediatrics. which helps distinguish C. trachomatis pneumonia is not associated with fever.) Trimethoprim-sulfamethoxazole for 7 days Ceftriaxone given as a single intramuscular dose Question . trachomatis infection is characterized by insidious onset of persistent cough and tachypnea. malaise. trachomatis from RSV pneumonia. with the notable absence of fever. trachomatis conjunctivitis or pneumonia is erythromycin 50 mg/kg/24 hr in 2 or 4 divided doses orally for 14 days. A chest radiograph reveals left upper and lower lobe infiltrates. Topical therapy of conjunctivitis does not reduce the high risk of subsequent pneumonia.) Question .3 in Nelson Textbook of Pediatrics.Chlamydia trachomatis Explanation: C.3 in Nelson Textbook of Pediatrics. A 15-yr-old boy presents to your office with complaint of fever. 174. (See Chapter 208. His WBC count is elevated. 173. 17th ed. The most likely diagnosis is: Mycoplasma pneumonia . (See Chapter 208. but wheezes are uncommon. 17th ed. headache. (See Chapter 208. cough.) Conjunctivitis Cough Tachypnea Eosinophilia Question . 17th ed. The recommended treatment for Chlamydia trachomatis conjunctivitis or pneumonia is: Erythromycin eye drops for 7 days Erythromycin eye drops and oral erythromycin for 7 days Oral erythromycin for 2 wk Explanation: The recommended treatment for C. and shortness of breath. He states that he recently received a pet cockatiel that became ill and died. Manifestations of Chlamydia trachomatis pneumonia include all of the following except: Fever Explanation: C. 175. but serologic studies are the diagnostic tests of choice for psittacosis. 177.Pneumococcal pneumonia Psittacosis Explanation: Psittacosis. now accompanied by a petechial rash that is prominent on the . An 8-yr-old child presents with headache.1 in Nelson Textbook of Pediatrics. (See Chapter 209 in Nelson Textbook of Pediatrics. All of the following laboratory findings are suggestive of Rocky Mountain spotted fever except: Normal to slightly low leukocyte count Shift to the left Reticulocytopenia Explanation: Early laboratory clues to Rocky Mountain spotted fever include normal to slightly low leukocyte count. 17th ed.) Q fever Legionnaires' disease Question . anorexia. 17th ed. and myalgias with onset 3 days ago. (See Chapter 209 in Nelson Textbook of Pediatrics.) Thrombocytopenia Low serum sodium Question . thrombocytopenia. and low serum sodium. The most appropriate next step in diagnosis for the case described in Question 175 is: Blood culture Throat culture Serologic testing Explanation: Culture is of poor value. 176. (See Chapter 210. produces severe pneumonia with systemic manifestations similar to those of Mycoplasma pneumoniae or Legionnaires' disease. fever. 17th ed.) Bronchoalveolar lavage and cultures MRI of the chest Question . a shift to the left (increased bands). a bird-borne chlamydial disease. 178. (See Chapter 210. 17th ed.1 in Nelson Textbook of Pediatrics.1 in Nelson Textbook of Pediatrics. (See Chapter 210. Peripheral edema of the wrists and ankles is also evident.extremities.) Henoch-Sch nlein purpura Systemic lupus erythematosus Question . including the palms and soles (Figure). 180. The recommended agent for treatment of Rocky Mountain spotted fever in a child younger than 8 yr is: Ceftriaxone Chloramphenicol Doxycycline Explanation: Chloramphenicol and tetracyclines have proven efficacy against Rocky Mountain spotted fever.) Clindamycin Quinidine Question . 17th ed. but chloramphenicol may be associated with higher mortality. 179. Doxycycline is recommended because the risk of dental staining is less than with other tetracyclines. All of the following may be characteristic of Rocky Mountain spotted fever except: History of tick exposure Presentation between April and September Headache and myalgias Rash that begins centrally and spreads to the periphery . Dental staining is unlikely with a single course of a tetracycline. The most likely diagnosis is: Infective endocarditis Meningococcemia Rocky Mountain spotted fever Explanation: The rash early in the illness of Rocky Mountain spotted fever is prominent over the extremities. including the ankles.300/mm3. His family had recently been camping in rural Wisconsin. and lower legs. He has no rash.4°F. A 5-yr-old boy has had fever. especially during the spring through the fall with a history of tick exposure. the cause of Rocky Mountain spotted fever. chloramphenicol is not effective against Ehrlichia spp. Dental staining is unlikely with a single course of a tetracycline.) Clindamycin Quinidine Question . and respiratory rate is 40/min. headache.1 in Nelson Textbook of Pediatrics. 17th ed. but there is no history of tick bite. 182. including the palms and soles.000/mm3. 181. The recommended treatment for human ehrlichiosis is: Ceftriaxone Chloramphenicol Doxycycline Explanation: Unlike with Rickettsia rickettsii. Laboratory findings include WBC count of 2. He appears acutely ill and dehydrated. heart rate is 130/min. The maculopapular rash appears on the extremities.Explanation: The diagnosis of Rocky Mountain spotted fever should be suspected in persons with an acute febrile illness with headache and myalgias. wrists. and platelet count of 57. abdominal pain. (See Chapter 210.) Rash involving the palms and soles Question . 65% bands. as the rash progresses to petechiae and purpura. and 8% lymphocytes. (See Chapter 213 in Nelson Textbook of Pediatrics. Doxycycline is recommended because the risk of dental staining is less than with other tetracyclines. The most likely diagnosis is: Meningococcemia Staphylococcal septicemia . 24% segmented neutrophils. His temperature is 103. The AST level is 465 IU/L. A peripheral blood smear reveals small blue clusters of bacteria-like bodies in an aggregate within the cytoplasm of 1% of circulating mononuclear leukocytes. The rash then spreads rapidly to involve the entire body. and muscle aches for the preceding 3 to 4 days. 17th ed. It is endemic and probably is more common than reported.) Ancylostoma Echinococcus Question . Which of the following pathogens is the most likely cause of this illness? Bartonella henselae Leptospira Coxiella burnetii Explanation: Acute Q fever is transmitted by inhalation of infectious aerosols. (See Chapter 213 in Nelson Textbook of Pediatrics. 17th ed. 17th ed. (See Chapter 214 in Nelson Textbook of Pediatrics. What is the most appropriate next step in diagnosis for the case described in Question 183? Blood culture Throat culture Serologic testing Explanation: The diagnosis of Q fever is most easily confirmed serologically by testing acute and convalescent sera (2-4 wk apart). Laboratory tests reveal leukopenia and elevated serum transaminase levels.) Bronchoalveolar lavage and cultures . 184. 183. There is no history of a tick bite. which should show a fourfold increase in indirect fluorescent antibody titers to phase I and phase II antigens. 17th ed. (See Chapter 214 in Nelson Textbook of Pediatrics. arthropod vectors are rarely implicated in human cases. which may be accompanied by a rash. A 17-yr-old adolescent who works on his family's sheep ranch presents in August with an influenza-like illness with interstitial pneumonitis.) Hemolytic-uremic syndrome Question .Rocky Mountain spotted fever Ehrlichiosis Explanation: Human ehrlichiosis is a rickettsial multisystem illness. 186.MRI of the chest Question . The recommended culture studies for this patient include: Blood cultures using only routine blood culture media Explanation: Candida grows readily on routine blood culture media. 17th ed.) Routine blood cultures and also fungal cultures using Sabouraud culture media . 185.) Question . A premature newborn on broad-spectrum antibiotics with a central venous catheter appears to be in shock.) Removal of the central line and flucytosine therapy Daily iodine antisepsis of the catheter site Question . The recommended treatment of a central line infection due to Candida is: Amphotericin B therapy Flucytosine therapy Removal of the central line and amphotericin B therapy Explanation: Treatment of Candida central line infections includes removal of the central line as well as parenteral amphotericin B for 23 wk. 17th ed. Cultures should also include the possibility of antibiotic-resistant bacteria. 17th ed. and you suspect either bacteria or Candida. (See Chapter 215 in Nelson Textbook of Pediatrics. (See Chapter 215 in Nelson Textbook of Pediatrics. and therefore special media are unnecessary. 187. which is a major predisposing factor for neonatal Candida infections. Which of the following is the most significant risk factor for fungal sepsis in a premature infant? Meconium aspiration Intravenous lipid infusion Previous administration of vancomycin Postconceptional age Heavy colonization with Candida Explanation: The inability of the newborn infant to localize Candida colonization facilitates overgrowth of Candida on mucocutaneous surfaces. (See Chapter 215 in Nelson Textbook of Pediatrics. The most likely diagnosis is: Aspergillosis Blastomycosis Cryptococcosis Explanation: Pigeon breeders and laboratory personnel who work with Cryptococcus are at greatest risk for cryptococcosis. Her past medical history is also remarkable for an episode of "bronchitis" 1 yr ago. An otherwise healthy 5-yr-old girl presents with focal back pain of insidious onset. A plain film of the back reveals destruction of the T8 vertebral body. 189.) Candida krusei Cryptococcus Malassezia furfur Question . Which of the following organisms has been identified? Aspergillus Candida albicans Explanation: Candida albicans is the only species that forms a germ tube when suspended in rabbit or human serum and incubated for 12 hr. and the child helps him care for the pigeons. The microbiology laboratory calls to report that an organism has been detected in a blood culture.) Histiocytosis Histoplasmosis . 17th ed. and the rapid germ tube test is positive. 188. (See Chapter 215 in Nelson Textbook of Pediatrics. Her father is a pigeon fancier. (See Chapter 216 in Nelson Textbook of Pediatrics. because the newborn is on antibiotics Question . 17th ed.Routine blood cultures and also fungal cultures using brain-heart infusion media Routine blood cultures and also fungal cultures using Bordet-Gengou culture media Only fungal cultures. ) Question .) Topical econazole applied to the catheter site Question . The use of lipid emulsions containing mediumchain triglycerides inhibits the growth of Malassezia. The most likely causative organism is: Burkholderia cepacia Stenotrophomonas maltophilia Aspergillus Explanation: Allergic bronchopulmonary aspergillosis complicates . The use of intravenous lipids predisposes most significantly to catheterrelated infections and fungemia due to which of the following organisms? Aspergillus Candida albicans Cryptococcus Histoplasma Malassezia furfur Explanation: Catheter-related infections and fungemia with Malassezia furfur occur almost exclusively in patients receiving intravenous lipids. 17th ed. 17th ed. including lipid emulsions. 190. The most important element of treatment for this infection is: Intravenous itraconazole Intravenous amphotericin B Oral fluconazole Discontinuing intravenous lipid infusion Explanation: Catheter-related infections and fungemia with Malassezia furfur occur almost exclusively in patients receiving intravenous lipids. A 17-yr-old boy with cystic fibrosis develops recurrent bronchospasm and transient pulmonary infiltrates. 191. Laboratory testing reveals mycelia in the sputum. and a peripheral eosinophilia. The use of lipid emulsions containing mediumchain triglycerides inhibits the growth of Malassezia. (See Chapter 217 in Nelson Textbook of Pediatrics. A blood culture yields Malassezia furfur. A newborn in the NICU is intubated and on total parenteral nutrition.Question . 192. (See Chapter 217 in Nelson Textbook of Pediatrics. 1 in Nelson Textbook of Pediatrics. (See Chapter 218. The most likely diagnosis is: Aspergillosis Blastomycosis Cryptococcosis Histiocytosis Histoplasmosis Explanation: Histoplasma capsulatum thrives in soil rich in nitrates such as areas heavily contaminated with bird droppings. A 16-yr-old boy is brought to your office for a second opinion. and with bat . All of the following are requirements for the diagnosis of allergic bronchopulmonary aspergillosis except: Asthma Immediate cutaneous reactivity to Aspergillus fumigatus antigens by RAST Elevated total serum IgE level Peripheral eosinophilia Pulmonary hyperinflation and hilar lymphadenopathy Explanation: Hyperinflation due to air trapping is not always noted with allergic bronchopulmonary aspergillosis. Candida controls were positive. 193. where a chest film demonstrated focal calcifications throughout both lungs with some hilar calcifications noted bilaterally. His history is remarkable for an episode of "bronchitis" 2 yr ago following a spelunking adventure in Kentucky. 17th ed. nor is hilar lymphadenopathy. 17th ed. He was seen at a local urgent care center. He was well until 2 wk ago when he developed a fever and cough.chronic pulmonary disease in approximately 10% of persons with cystic fibrosis. 194.1 in Nelson Textbook of Pediatrics.) Question . (See Chapter 218. A 5 U PPD placed at that time was negative. The inflammatory response does not necessarily produce either of these lesions.) Histoplasma Malassezia furfur Question . and fever of onset 2 wk previously. (See Chapter 219 in Nelson Textbook of Pediatrics.) Question . capsulatum thrives in soil rich in nitrates such as areas heavily contaminated bat guano. and southwestern Texas is: Aspergillus Blastomyces dermatitidis Coccidioides immitis . 17th ed.g. The most likely cause of fungal pneumonia in immunocompetent persons living in arid areas of California. Wisconsin).) Question . 197. Which of the following organisms is the most likely cause of fungal pneumonia in immunocompetent persons living in the Ohio and Mississippi River valleys? Aspergillus Blastomyces dermatitidis Coccidioides immitis Cryptococcus neoformans Histoplasma capsulatum Explanation: H. dyspnea. 17th ed. (See Chapter 219 in Nelson Textbook of Pediatrics. (See Chapter 219 in Nelson Textbook of Pediatrics. Chest film shows evidence of pneumonia. 195. The most likely causative organism is: Aspergillus Blastomyces dermatitidis Coccidioides immitis Cryptococcus neoformans Histoplasma capsulatum Explanation: H.) Question . 17th ed.. Arizona.guano in caves and along bridges frequented by bats. capsulatum is found in the soil throughout the Midwestern United States. in caves and along bridges frequented by bats. Blastomyces is found in the same areas but is less common except in the northern Midwestern states (e. fatigue. 196. An adolescent with a history of spelunking presents with weight loss. 17th ed. vector. 17th ed. 199.) Question .) Question . (See Chapter 223 in Nelson Textbook of Pediatrics. most frequently cats and armadillos. and the presence of only one serotype. Which of the following statements concerning measles virus is true? There is no animal reservoir There is no vector There is no transmissible latent virus There is only one serotype All of the above Explanation: Several features of measles support the possibility of global eradication. and southwestern Texas. 17th ed. (See Chapter 225 in Nelson Textbook of Pediatrics. central and southern Arizona.Explanation: C. A 4-yr-old girl develops nodular lymphangitis after being bitten by a cat. transmissible latent virus. including the absence of an animal reservoir.) Cryptococcus neoformans Histoplasma capsulatum Question . All of the following vaccines may be given to children with severe combined immunodeficiency syndrome except: . immitis is found in arid areas of California's San Joaquin Valley (Valley fever). (See Chapter 221 in Nelson Textbook of Pediatrics. 198. The most likely etiologic agent is: Aeromonas hydrophila Mycobacterium marinum Nocardia brasiliensis Pasteurella multocida Sporothrix schenckii Explanation: Sporothrix thrives in decaying vegetation but also can be transmitted by bites and scratches of animals. 200. 17th ed. (See Chapters 225 and 282 in Nelson Textbook of Pediatrics. 17th ed. 202.) Salk poliovirus vaccine Hepatitis B virus vaccine Pneumococcal vaccine Question . Nonetheless. (See Chapter 226 in Nelson Textbook of Pediatrics. (See Chapter 225 in Nelson Textbook of Pediatrics. Treatment with vitamin A reduces morbidity and mortality in children with severe measles in developing countries and is recommended for selected children with severe measles in the United States.Diphtheria. 17th ed. Snuffles is a sign of congenital syphilis. pertussis (DTaP) vaccine Measles virus vaccine Explanation: The live measles virus is contraindicated in patients with T-cell immunodeficiency because disseminated disease may occur. 201. it is recommended that HIV-infected patients be given this vaccine because measles itself is a serious illness once AIDS develops. All of the following are characteristic manifestations of congenital rubella syndrome except: Snuffles Explanation: Congenital rubella affects virtually all organ systems. Which of the following statements concerning measles and vitamin A is true? There is not a confirmed relationship of vitamin A to the prognosis of measles Measles causes vitamin A deficiency Treatment with vitamin A reduces measles severity for children in developing countries Explanation: There is an apparent correlation between retinol concentration and measles severity. tetanus.) Intrauterine growth retardation .) Subacute sclerosing panencephalitis (SSPE) is more likely in children with underlying vitamin A deficiency Treatment with vitamin A reduces the incidence of SSPE Question . ) Myocarditis Orchitis Pancreatitis Question . but appears to occur in more than two-thirds of cases.Cataracts Structural cardiac defects Sensorineural hearing loss Question . (See Chapter 227 in Nelson Textbook of Pediatrics. with clinical symptoms in more than 10% of cases. Which of the following statements most accurately describes the relationship between mumps and infertility? Infertility occurs as a sequela of mumps only among prepubertal children Infertility is more common among females than among males Infertility rate among males with mumps orchitis is approximately 3040% Infertility among males is rare even with bilateral orchitis Explanation: Infertility among males with mumps is rare. A 14-yr-old boy and his twin sister each developed mumps. and are more common among postpubertal men (14-35%) and women (7%). (See Chapter 227 in Nelson Textbook of Pediatrics.) . especially the possibility of infertility. 203. They ask about the prognosis for this disease in their children. 17th ed. even with bilateral orchitis. Their parents had repeatedly declined for them to receive MMR vaccination. The most common complication of mumps in childhood is: Arthritis Meningoencephalitis Explanation: Meningoencephalitis is the most common complication of mumps in childhood. 17th ed. There is no evidence of impairment of fertility among women with mumps oophoritis. 204. Mumps orchitis and oophoritis are rare among prepubescent boys and girls. 17th ed.) Nonspecific symptoms (e. and sensory changes (paresthesias) are common. the paralysis is characteristically symmetric. malaise. (See Chapter 228 in Nelson Textbook of Pediatrics. In Guillain-Barré syndrome. headache) Aseptic meningitis Aseptic meningitis with encephalitis (meningoencephalitis) Paralytic poliomyelitis Question . 206. The percentage of poliovirus infections that are inapparent is: 5-10% 20-25% 50% 75-80% 90-95% Explanation: Approximately 90-95% of poliovirus infections are inapparent.) Question . mild fever.Infertility rate among females with mumps oophoritis is approximately 30-40% Question . Pleocytosis is common in polio. 17th ed. 207. causing no paralytic disease and no sequelae. whereas the cerebrospinal fluid in Guillain-Barré syndrome usually shows only elevated protein and occasionally a few . 205. Most infections with poliovirus result in illness that is best characterized as: Subclinical illness Explanation: Approximately 90-95% of poliovirus infections are inapparent. (See Chapter 228 in Nelson Textbook of Pediatrics.. Which of the following features distinguishes paralytic polio from GuillainBarré syndrome? Pleocytosis is uncommon in paralytic polio Paralysis is usually asymmetric in paralytic polio Explanation: Paralytic polio is characterized by aseptic meningitis accompanied by asymmetric flaccid paralysis without sensory loss.g. causing no paralytic disease and no sequelae. The risk is higher in vaccinees than in contacts. The most probable diagnosis is: Acute paralytic poliomyelitis due to wild poliovirus Paralysis due to nonpolio enteroviruses Vaccine-associated poliomyelitis Guillain-Barré syndrome Explanation: Guillain-Barré syndrome. 208.) Tick-bite paralysis .cells. and absent ankle deep tendon reflexes. which follows reversion of the OPV strain to a neurovirulent strain. (See Chapter 229 in Nelson Textbook of Pediatrics. after the first dose. 17th ed. is symmetric and involves sensory but more so motor nerves. All of the following statements concerning the risk of vaccine-associated paralytic poliomyelitis (VAPP) are true except: The risk is associated only with OPV and not IPV The risk is higher after the second and third doses than after the first dose Explanation: Vaccine-associated paralytic poliomyelitis (VAPP). The child received only two immunizations with OPV. at the ages of 2 and 6 mo. A 5-yr-old boy who lives in an urban area is hospitalized because of lowgrade fever. Polio is an anterior horn cell disease and is purely motor. has accounted for all cases of polio in the United States since 1979. 17th ed. or autoimmune peripheral neuropathy. 17th ed. (See Chapters 228 and 282 in Nelson Textbook of Pediatrics.) The paralysis of polio is usually spastic Sensory changes are common in paralytic polio Paralytic polio only occurs in unimmunized persons Question . (See Chapter 228 in Nelson Textbook of Pediatrics. sensory changes. 209. and in immunocompromised persons. flaccid paralysis of both legs.) The risk is higher for immunocompromised persons The risk is present for household contacts as well as vaccinees The risk for VAPP in the United States has exceeded the risk for wildtype polio since 1979 Question . Question . Herpangina is usually caused by echovirus type 9. poor . 211. (See Chapter 229 in Nelson Textbook of Pediatrics. 17th ed. male sex. Hand-foot-and-mouth disease is usually caused by coxsackievirus A16. and mouth disease Aphthous stomatitis Explanation: Infections due to enteroviruses have a wide spectrum of clinical manifestations. All of the following may be manifestations of enterovirus infection except: Aseptic meningitis Herpangina Hand. (See Chapters 228 and 282 in Nelson Textbook of Pediatrics.) Vomiting and diarrhea Question . Advantages of inactivated poliovirus vaccine over live poliomyelitis vaccine include: Induction of a high level of intestinal mucosal immunity Oral administration No requirement for a booster dose at age 4-6 yr No association with vaccine-associated paralytic poliomyelitis (VAPP) Explanation: Vaccine-associated paralytic poliomyelitis (VAPP) is not associated with the inactivated poliovirus vaccine. foot. 210. 17th ed. Which of the following factors is associated with increased risk of enterovirus infection? Wintertime in temperate climates Upper socioeconomic class Recent receipt of poliovirus vaccine Young age Explanation: Factors associated with increased incidence and/or severity of enterovirus infection include young age. 212. Diarrhea is probably less common than is taught.) Longer duration of immunity Question . and mouth[HBJ1] disease Meningitis Question . unenveloped viruses that are able to replicate in the intestinal tract. 215. Osteomyelitis is not associated with enteroviruses. (See Chapter 229 in Nelson Textbook of Pediatrics.hygiene. 17th ed. negative-sense RNA virus A member of a genus containing 11 different serotypes An unenveloped virus able to replicate in the intestinal tract Explanation: Enteroviruses are small. foot. 17th ed. (See Chapter 229 in Nelson Textbook of Pediatrics. and low socioeconomic status. 213. overcrowding. however.) A small virus restricted to intestinal tract mucosal infection A virus whose infection is primarily limited by cellular immunity Question . Which of the following is a feature of epidemics of hand-foot-and-mouth disease associated with enterovirus 71? High rates of encephalitis and cardiopulmonary complications . All of the following are infections typically caused by enteroviruses except: Neonatal sepsis Osteomyelitis Explanation: Although more than 60 different serotypes of enteroviruses have been identified. 11 account for the majority of disease.) Myocarditis Hand. (See Chapter 229 in Nelson Textbook of Pediatrics. 17th ed. >25% of symptomatic enterovirus infections occur in children younger than 1 yr of age. certain manifestations are preferentially associated with specific serotypes. 214. Breast-feeding reduces the risk of infection in infants. Which of the following is an accurate description of an enterovirus? A large.) Breast-feeding Question . No enterovirus disease is uniquely associated with any specific serotype. especially in young children. myocarditis. and paralysis. 17th ed. hepatitis. and rapid progression to death.Hand. Which of the following statements regarding the diagnosis of enterovirus infections is true? Useful diagnostic techniques are not available Polymerase chain reaction assay is very sensitive Explanation: PCR assay detects the majority of enteroviruses (but frequently not echoviruses 22 and 23) and has been applied to a variety of specimens. serum. but a minority has severe disease that may be dominated by any combination of sepsis. (See Chapter 229 in Nelson Textbook of Pediatrics. including cerebrospinal fluid. encephalitis. coagulopathy. shock. 216. urine. with high rates of associated neurologic disease including aseptic meningitis. 17th ed. 217. Recent outbreaks in Malaysia. and Taiwan have been notable for brainstem encephalomyelitis.) High rates of concomitant hemorrhagic conjunctivitis Highest mortality in adolescents and young adults Occurrence of disease primarily in immunocompromised hosts High rates of severe gastrointestinal complications Question . foot. pulmonary hemorrhage. and pneumonitis.) They generally occur only in extremely low birthweight infants Question . (See Chapter 229 in Nelson Textbook of Pediatrics. Which of the following statements regarding neonatal enterovirus infections is true? They are much less common than infections due to herpes simplex virus and cytomegalovirus They are invariably mild. benign illnesses They are best treated with ribavirin They may cause life-threatening hepatitis and coagulopathy Explanation: Most symptomatic neonates with neonatal enterovirus infection have benign courses. Japan. neurogenic pulmonary edema. and mouth[HBJ2] disease caused by enterovirus 71 is frequently more severe than that due to coxsackievirus A16. and . meningoencephalitis. conjunctival, nasopharyngeal, throat, and rectal specimens. (See Chapter 229 in Nelson Textbook of Pediatrics, 17th ed.) Useful diagnostic specimens are limited to throat and rectal swabs All enteroviruses grow well in cell culture Antigen detection is a useful way to detect all enteroviruses Question . 218. All of the following may be manifestations of parvovirus B19 infection except: Facial rash ("slapped-cheek" appearance) Lacy, reticulated rash over the trunk and proximal extremities Arthritis Transient reversal of the CD4:CD8[HBJ3] ratio Explanation: The rash occurs in three stages, which are not always clinically distinguishable. After the initial "slapped-cheek" appearance, the rash spreads to become a lacy, reticulated rash over the trunk and proximal extremities. Arthritis and arthralgia are much more common in adults, especially in females, than in children. The transient arrest of erythropoiesis is usually clinically silent in previously healthy persons. (See Chapter 230 in Nelson Textbook of Pediatrics, 17th ed.) Reticulocytopenia Question . 219. During springtime, an infectious disease spread through a small community in the United States. The principal signs and symptoms were fever, mild rash, and arthralgia. One pregnant woman in late gestation contracted the illness but recovered without sequelae. However, 1 mo later she gave birth to a stillborn infant. The pathology report listed the diagnosis "hydrops fetalis." Which of the following congenital infections is the most likely etiology? Congenital rubella virus infection Congenital cytomegalovirus infection Congenital parvovirus B19 infection Explanation: Parvovirus B19, the agent of fifth disease (erythema infectiosum), produces congenital infection of the fetal erythrocyte precursor cells, producing transient fetal anemia. If the anemia is severe, it produces nonimmune hydrops with the possibility of intrauterine fetal demise. Intrauterine (umbilical venous) blood transfusion is curative but poses risk. (See Chapter 230 in Nelson Textbook of Pediatrics, 17th ed.) Congenital herpes simplex virus infection Congenital HIV infection Question . 220. All of the following statements regarding herpes simplex virus (HSV) infections in neonates are true except: Most cases are caused by HSV type 2 Women with primary HSV genital tract infection are more likely to transmit infection to their offspring than women with recurrent HSV infection Most mothers of newborns with perinatal HSV infection have a history of genital HSV infection Explanation: Only 15-20% of mothers of newborns with perinatal HSV have a history of obvious HSV infection, and only about 25% have any relevant symptoms at birth. (See Chapter 231 in Nelson Textbook of Pediatrics, 17th ed.) Most mothers of newborns with perinatal HSV infection are asymptomatic at delivery Most cases are transmitted at delivery and are not true congenital infections Question . 221. Recommended management for a mother with active genital HSV infection during labor is: Culture of blood from the newborn, with treatment based on culture results Culture of blood from the newborn, with empirical acyclovir therapy Intravenous acyclovir treatment for the mother Cesarean section within 4 hr of rupture of membranes Explanation: Both the American Academy of Pediatrics and the American College of Obstetrics and Gynecology recommend cesarean section if primary, first-episode, or recurrent HSV lesions are present on the mother at the onset of labor. Only 15-20% of mothers of newborns with perinatal HSV have a history of HSV infection. (See Chapter 231 in Nelson Textbook of Pediatrics, 17th ed.) Intravenous acyclovir treatment for the mother and cesarean section within 4 hr of rupture of membranes Question . 222. A 3-yr-old boy presents with a 7-day history of fever, cervical lymphadenopathy, foul breath, and painful oral lesions on his tongue, gums, and lips. For the past 3 days he has had a red, painful swollen area about the nail of his right thumb with an area of fluid by the nail bed, unresponsive to warm soaks and a first-generation cephalosporin. The most likely etiologic agent is: Staphylococcus aureus Mucocutaneous candidiasis Coxsackievirus Adenovirus Herpes simplex virus Explanation: The child has herpetic gingivostomatitis. He has autoinoculated his thumb by sucking, and herpetic whitlow has developed. (See Chapter 231 in Nelson Textbook of Pediatrics, 17th ed.) Question . 223. A 13-mo-old previously healthy child presents on New Year's Eve with a 2-day history of fever, lethargy, and irritability. Earlier this afternoon he began to have twitching movements of his left arm and on the left side of his face. His immunizations are up-to-date. Physical examination reveals fever with a temperature of 39°C and left-sided weakness, with no rashes. Examination of the cerebrospinal fluid (CSF) reveals 70 WBCs/mm3 with 85% lymphocytes, 400 RBCs/mm3, protein of 140 mg/dL, glucose 80 mg/dL, and negative results on Gram stain. MRI scan reveals right temporal abnormalities. The diagnostic study most likely to identify a treatable illness in a timely fashion is: Viral CSF culture Herpes simplex virus polymerase chain reaction (PCR) assay Explanation: The focal seizure, focal physical findings, the CSF profile, and temporal lobe lesion on MRI are highly suggestive of herpes simplex virus encephalitis. The fastest, most specific means of diagnosis is by PCR assay of CSF. (See Chapter 231 in Nelson Textbook of Pediatrics, 17th ed.) Acute and convalescent antibody titers Comparison of maternal and infant antibody titers CSF bacterial culture Question . 224. For the patient described in Question 223, the most appropriate empirical therapy to begin while awaiting definitive diagnosis is: Ceftriaxone Nafcillin, cefotaxime, and metronidazole Acyclovir Explanation: The focal seizure, focal physical findings, the CSF profile, and temporal lobe lesion on MRI are highly suggestive of herpes simplex virus encephalitis. Acyclovir must be included in the initial treatment. (See Chapter 231 in Nelson Textbook of Pediatrics, 17th ed.) Amphotericin B Isoniazid, rifampin, pyrazinamide, and streptomycin Question . 225. Most infants with localized neonatal herpes encephalitis become symptomatic at the age of: 0-2 days 2-8 days 8-12 days Explanation: Localized skin, eye, and mouth infection and also disseminated infection occur at a mean of 5-6 days post partum, whereas localized CNS infection occurs later at a mean of 8-12 days post partum. (See Chapter 231 in Nelson Textbook of Pediatrics, 17th ed.) 13-21 days Older than 21 days Question . 226. At 6 mo after stem cell transplantation, a 4-yr-old boy has had several episodes of recurrent oral herpes simplex virus infection, each responsive to episodic acyclovir treatment. For the past 2 wk he has been experiencing progressive oral and lip lesions that have spread to his face and neck, in spite of first oral and then high-dose intravenous acyclovir. The viral culture is positive for herpes simplex virus. The most appropriate treatment to begin during the wait for results of further studies on the virus is: Valacyclovir Famciclovir Ganciclovir Foscarnet Explanation: The repeated antiviral treatment of herpes simplex virus disease in an immunocompromised person promotes the development of resistant HSV. Foscarnet should be used during the wait for results of viral susceptibility testing. (See Chapter 231 in Nelson Textbook of Pediatrics, 17th ed.) Gamma interferon Question . 227. A 9-day-old neonate, born by vaginal delivery to a 21-yr-old healthy woman, is presented with fever, lethargy, and poor feeding. There are no diagnostic findings on physical examination, and results of sepsis evaluation, including a CBC and cerebrospinal fluid studies, are unremarkable. Ampicillin and cefotaxime are begun. Two days later all bacterial cultures are negative but the child's clinical condition worsens, with falling blood pressure, decreased level of consciousness, thrombocytopenia, and elevated liver enzymes. The indicated change in treatment is: Addition of amphotericin Addition of vancomycin Addition of ribavirin Addition of acyclovir Explanation: The history, inability to confirm bacterial infection, and worsening clinical condition are suggestive of neonatal herpes simplex virus infection. Acyclovir therapy should be initiated. Repeat cerebrospinal fluid analysis or MRI might also be recommended. (See Chapter 231 in Nelson Textbook of Pediatrics, 17th ed.) Substitution with meropenem and amikacin Question . 228. An otherwise healthy 16-yr-old high school wrestling star presents with a 6-mo history of a recurrent vesicular eruption on one side of his face. He has no other history of unusual infections and is HIV negative. His immunizations are appropriate for his age, and he has never had chickenpox. The most likely diagnosis is: Recurrent group A streptococcal infection Shingles Recurrent herpes simplex virus Explanation: This is a characteristic history of herpes gladiatorum. (See Chapter 231 in Nelson Textbook of Pediatrics, 17th ed.) Recurrent facial mat burns Traumatic candidiasis Question . 229. A 10-mo-old girl is presented in January with a 2-day history of fever with temperatures to 103.5°F and refusal to eat. Physical examination reveals a temperature of 40°C, general irritability, and numerous ulcers on the anterior portion of her buccal mucosa, gums, and tongue. Appearance of the throat is unremarkable. There are bilateral tender enlarged cervical lymph nodes. The recommended therapeutic agent is: Acyclovir Explanation: The fever, irritability, and mucosal ulcerations suggest acquired herpes simplex virus infection. Acyclovir therapy is the treatment of choice. (See Chapter 231 in Nelson Textbook of Pediatrics, 17th ed.) Azithromycin Nystatin Penicillin Pleconaril Question . 230. Which of the following statements concerning antiviral treatment of varicella-zoster virus infections is true? It carries a high risk of drug toxicity It significantly modifies the course of chickenpox in immunocompetent persons It significantly modifies the course of zoster Explanation: Acyclovir treatment of otherwise healthy persons with varicella is acceptable but is not recommended by the American Academy of Pediatrics. Antiviral treatment of zoster is associated with a less severe disease and greatly decreased risk for postherpetic neuralgia. (See Chapter 232 in Nelson Textbook of Pediatrics, 17th ed.) It is associated with a greater risk of recurrences of zoster Antiviral resistance by varicella-zoster virus is common Question . 231. A 12-yr-old girl develops varicella. Her parents ask about the risk for herpes zoster. Which of the following statements regarding herpes zoster is/are true? Increasing age and altered cell-mediated immunity are risk factors for herpes zoster The lifetime risk of herpes zoster is about 10-20 % Varicella-zoster virus can usually be cultured from zoster lesions The incidence and the duration of postherpetic neuralgia are directly correlated with age All of the above Explanation: Varicella-zoster virus establishes latent infection in sensory ganglia cells in all individuals who experience primary infection. Subsequent reactivation of latent virus causes herpes zoster. (See Chapter 232 in Nelson Textbook of Pediatrics, 17th ed.) Question . 232. A 2-yr-old healthy girl presents to your office for the first time. She was seen by another physician 2 wk ago and was told at that time that she was behind in her immunizations. Review of available records confirms that she has still not received varicella vaccine. All of the following are contraindications to varicella vaccine except: Past history of chickenpox, according to the mother Her younger sibling currently has chickenpox She currently has an upper respiratory tract infection with temperature of 100°F Administration of MMR vaccine 2 wk ago Explanation: MMR and varicella vaccine should be administered either simultaneously or at least 4 wk apart. Past or even current chickenpox is not a contraindication to vaccination (although physician-diagnosed chickenpox or serologic confirmation of immunity can be accepted in lieu of vaccination). HIV-infected children with CD4 percentage greater than 25% may also receive varicella vaccine. (See Chapter 232 in Nelson Textbook of Pediatrics, 17th ed.) HIV infection with a CD4 count of 50/mm3 Question . 233. Which of the following sets of serologic titers is most consistent with acute primary Epstein-Barr virus infection? IgM-VCA negative; IgG-VCA negative; EA 1:40; EBNA negative IgM-VCA negative; IgG-VCA 1:160; EA 1:40; EBNA negative IgM-VCA 1:16; IgG-VCA 1:160; EA 1:40; EBNA negative Explanation: The IgM-VCA is the best single test to identify acute EBV infection. Anti-EBNA antibodies appear 3-4 mo after infection and can be used to distinguish recent from past infection. (See Chapter 233 in Nelson Textbook of Pediatrics, 17th ed.) IgM-VCA 1:16; IgG-VCA 1:160; EA 1:40; EBNA 1:8 IgM-VCA 1:16; IgG-VCA 1:160; EA negative; EBNA 1:8 Question . 234. All of the following tumors are associated with Epstein-Barr virus except: Burkitt lymphoma Kaposi sarcoma Explanation: EBV is associated with several malignancies, including nasopharyngeal carcinoma, Burkitt lymphoma, Hodgkin disease, and lymphoproliferative diseases and leiomyosarcomas in immunocompromised persons. Kaposi sarcoma is associated with HHV-8. (See Chapter 233 in Nelson Textbook of Pediatrics, 17th ed.) Leiomyosarcoma Lymphoproliferative disease in immunocompromised persons Nasopharyngeal carcinoma Question . 235. All of the following organisms are recognized as potential causes of an infectious mononucleosis-like syndrome except: Cytomegalovirus Epstein-Barr virus Human immunodeficiency virus (HIV) Parvovirus B19 Explanation: Infectious mononucleosis-like illnesses may also be caused by primary infection with cytomegalovirus, T. gondii, adenovirus, hepatitis virus, HIV, and possibly rubella virus. (See Chapter 233 in Nelson Textbook of Pediatrics, 17th ed.) Toxoplasma gondii Question . 236. All of the following statements concerning congenital cytomegalovirus infection are true except: It is the most common congenital infection Approximately 5% of infected newborns have severe disease Approximately 60% of infected newborns have mild disease Explanation: Only 5% of infected newborns with congenital cytomegalovirus infection have severe disease, and another 5% have mild disease. Most infected newborns are asymptomatic. Treatment has not been shown to be beneficial and is considered experimental. (See Chapter 234 in Nelson Textbook of Pediatrics, 17th ed.) The diagnosis is best confirmed by neonatal urine culture Treatment with ganciclovir is recommended Question . 237. On examination of a full-term newborn, the physician notes mild hepatomegaly. Other physical findings are normal, including head circumference and appearance of the retinas. A urine culture grows cytomegalovirus (CMV). Results of head ultrasonography are normal. Subsequent testing discloses no metabolic disorders. The deficit most likely to occur in the next year is: Visual loss Hearing loss Explanation: Sensorineural hearing loss is a risk after asymptomatic congenital CMV infection. The incidence of neonatal CMV-positive urine is much greater than the incidence of symptomatic neonatal CMV inclusion disease (e.g., microencephaly, retinitis, being small for gestational age, petechiae). (See Chapter 234 in Nelson Textbook of Pediatrics, 17th ed.) Cirrhosis Patent ductus arteriosus Immunoglobulin deficiency Question . 238. Which of the following newborns is at greatest risk for symptomatic cytomegalovirus (CMV) infection? A 2,985-g boy born at 41 wk of gestation to a 27-yr-old G2P2 mother who had primary CMV infection during her first pregnancy Explanation: The incidence of congenital CMV infection ranges from 0.2 to 2.4% of all live births, with the higher rates among populations with a lower economic standard of living. The risk for fetal infection is greatest with maternal primary CMV infection (30%) and much lower with recurrent infection (<1%). (See Chapter 234 in Nelson Textbook of Pediatrics, 17th ed.) A 1-wk-old 2,412-g breast-fed girl born at 34 wk of gestation to a 21yr-old mother whose breast milk has detectable levels of CMV A 2,634-g exclusively bottle-fed girl born at 38 wk of gestation to a 16-yr-old G1P1 mother who smoked heavily throughout the pregnancy A 3,420-g boy born at 35 wk of gestation to a 38-yr-old G5P4 mother who works in a daycare setting and has school-aged children A 2,835-g boy born to a 26-yr-old mother who had no prenatal care and is a recent immigrant from Uganda Question . 239. An 18-yr-old male patient is followed in the infectious diseases clinic for human immunodeficiency virus infection. His most recent CD4 lymphocyte count is 97/mm3. Over the last week he has developed increasing dyspnea and coughing, visual changes, diarrhea, and fever. You suspect that cytomegalovirus infection is responsible for these symptoms. Which of the following laboratory tests would be most helpful in establishing the diagnosis? CMV IgG antibody titer CMV IgM antibody titer Glycoprotein H epitopes Viral culture of the blood buffy coat for CMV Explanation: In immunocompromised patients, excretion of CMV in urine, increases in IgG titers, and even the presence of IgM antibodies are common, making the distinction between primary and recurrent infections more difficult. Demonstrating viremia by buffy coat culture or detection of CMV DNA implies active disease and worse prognosis (See Chapter 234 in Nelson Textbook of Pediatrics, 17th ed.) Centrifugation-enhanced rapid culture system for early antigen (shell vial) from a urine specimen Question . 240. A 1,050-g male infant is born at 31 wk of gestation to a migrant farm worker who had poor prenatal care. The mother has a history of gonorrhea. He requires ventilatory support and has a mean arterial blood pressure of 27 mm Hg despite 3 intravenous boluses of normal saline. He has a petechial rash and is microcephalic. He thrusts his tongue forward and is jittery. Laboratory studies reveal an alanine aminotransferase level of 346 U/L, serum glucose of 61 mg/dL, and white blood cell count of 20,000/mm3, with neutrophils 60%, bands 15%, and lymphocytes 20%. The platelet count is 60,000/mm3. A computed tomography study of the head shows periventricular calcifications. The most likely diagnosis is: Group B streptococcal sepsis Congenital toxoplasmosis Congenital Neisseria gonorrhoeae sepsis Perinatal listeriosis Congenital cytomegalovirus infection Explanation: Petechial rash and thrombocytopenia in a small-forgestational-age newborn suggest congenital infection. The CT scan shows periventricular calcifications, which are characteristic of congenital CMV infection. (See Chapter 234 in Nelson Textbook of Pediatrics, 17th ed.) Question . 241. The patient described in Question 240 is discharged from the neonatal intensive care unit at 9 mo of age. His discharge diagnoses include chronic lung disease, cerebral palsy, a seizure disorder, resolved necrotizing enterocolitis, and retinopathy of prematurity. Because of the infection that was present at birth, the primary care physician should regularly order which of the following screening tests? Alanine and aspartate aminotransferase assays Auditory evoked response Explanation: CMV infection is a leading cause of sensorineural hearing loss, which occurs in approximately 7% of infected infants. (See Chapter 234 in Nelson Textbook of Pediatrics, 17th ed.) Lymphocyte subsets Neural imaging Platelet counts Question . 242. Which of the following statements regarding treatment of cytomegalovirus disease in children is true? Acyclovir and CMV IVIG constitute the regimen of choice for lifethreatening CMV infections Explanation: Ganciclovir combined with immune globulin, either standard intravenous immunoglobulin (IVIG) or hyperimmune CMV IVIG, has been used to treat life-threatening CMV infections in immunocompromised hosts (e.g., bone marrow, heart, and kidney transplant recipients and patients with AIDS). (See Chapter 234 in Nelson Textbook of Pediatrics, 17th ed.) Ganciclovir administration often results in severe neutropenia and liver dysfunction CMV retinitis and gastrointestinal disease are cured by ganciclovir Acyclovir is recommended to shorten the duration of heterophilenegative mononucleosis Cidofovir is recommended prophylactically in renal transplantation patients to prevent recurrent disease Question . 243. The peak incidence of HHV-6 infection is seen in which of the following age groups? Newborns, in whom it is a congenital infection Children 0-5 yr Explanation: Approximately 60-90% of children 12 mo of age and 80-100% of children 3-5 yr of age have antibodies to HHV-6. Most newborns are seropositive as a result of transplacental transfer of maternal antibodies. (See Chapter 235 in Nelson Textbook of Pediatrics, 17th ed.) Children 5-10 yr Children 10-15 yr Adolescents 16 yr and older, in whom it is acquired as a sexually transmitted infection Question . 244. A 7-mo-old child presents in late October with 3 days of fever with temperatures to 103.5°F, a mildly injected pharynx, mild cervical lymphadenopathy, and diarrhea. The child has been behaving normally and eating well and has no other symptoms. On the fourth day of the illness the fever resolves, and a generalized measleslike rash appears 12 hr later. The child appears normal on physical examination. The most likely diagnosis is: Measles Rubella Drug reaction to antipyretics HHV-6 infection Explanation: HHV-6 is the agent of roseola (erythema subitum), the childhood exanthem present in the infant described in the question. (See Chapter 235 in Nelson Textbook of Pediatrics, 17th ed.) Enteroviral infection The recommended antiviral treatment of HHV-6 infection complicated by seizures is: Administration of acyclovir Administration of ganciclovir Administration of famciclovir Administration of foscarnet Symptomatic treatment only Explanation: The generally benign nature of roseola precludes consideration of antiviral therapy. Further evaluation reveals that the child is HIV seropositive. cidofovir. 17th ed. and foscarnet (but not acyclovir). the most appropriate next step in management would be to: Perform a lumbar puncture Begin antiviral treatment with acyclovir Begin antiviral treatment with ganciclovir Report the case to the local public health department Reassure the parents Explanation: The generally benign nature of roseola precludes consideration of antiviral therapy. 245. The most appropriate next step in management of the Kaposi sarcoma lesions would be: Surgical excision Combination chemotherapy .) Question . (See Chapter 235 in Nelson Textbook of Pediatrics. For the patient described in Question 244. the clinical efficacy and benefit of antiviral therapy for roseola have not been established. oral candidiasis. Treatment for PCP and candidiasis is initiated. with development of Pneumocystis carinii pneumonia (PCP). cidofovir. Although HHV-6 is inhibited by ganciclovir. (See Chapter 235 in Nelson Textbook of Pediatrics. and cutaneous Kaposi sarcoma lesions. 246.) Question .000 copies/mL and a CD4 cell count of 150 cells/ L.Question . Although HHV-6 is inhibited by ganciclovir. 17th ed. the clinical efficacy and benefit of antiviral therapy for roseola has not been established. and foscarnet (but not acyclovir). A 2-yr-old child of an African mother is noted to have poor weight gain. with an HIV viral load of 100. 247. 249. However. cardiac. 248. for persons undergoing immunosuppression or receiving long-term aspirin therapy. 17th ed. including individuals with chronic pulmonary (e. A 4-yr-old child is brought to your clinic for well child care. renal.) Acute myositis Question . and cidofovir. for family members at risk for influenza.g. Reye syndrome may follow influenza. 17th ed. or metabolic disorders (e. usually if salicylates are given for antipyresis. asthma). foscarnet. including ganciclovir.. and forwomen who will be in the second or third trimester . (See Chapter 237 in Nelson Textbook of Pediatrics. Introduction of highly active antiretroviral therapy (HAART) has dramatically improved survival of AIDS patients with KS. which is why salicylates are not recommended for persons with influenza or flulike illnesses. diabetes mellitus) or with hemoglobinopathies. All of the following may be complications of influenza except: Pneumonia Otitis media Reye syndrome Hemolytic-uremic syndrome Explanation: Otitis media and pneumonia. (See Chapter 236 in Nelson Textbook of Pediatrics. the benefit of specific antiviral therapy for HHV-8-associated disease has not yet been established.. are common complications of influenza in children. Indications for influenza vaccine include all of the following except: Immunosuppression caused by medications Residence in a chronic care facility Family member who is at risk of complications of influenza Family history of SIDS Explanation: Annual influenza vaccine is recommended for individuals at high risk for influenza and its complications.g.) Question .Local radiation Intralesional injection with cidofovir Initiation of antiretroviral therapy Explanation: Several antiviral compounds inhibit HHV-8 in vitro. either viral or bacterial. Amantadine is associated with much more frequent central nervous systems adverse effects than rimantadine. 17th ed. Patient groups that should be targeted for annual influenza vaccination include: Persons aged 65 yr and older Persons with cardiovascular disease Otherwise healthy children with asthma Women who will be in the second or third trimester of pregnancy during influenza season All the above Explanation: All of these groups should receive annual influenza vaccination. both healthy. (See Chapter 237 in Nelson Textbook of Pediatrics. 251.) Mother in the second or third trimester of pregnancy Question . Which of the following is true concerning rimantadine and amantadine for treatment of influenza? They are both approved for treatment of influenza A illness in children They are equally effective against influenza A and B viruses They are both completely excreted unchanged in the urine They can reduce the severity and duration of influenza A illness when administered within 48 hr of onset of illness Explanation: When administered within the first 48 hours of illness. A 5-mo-old boy and an 18-mo-old girl. Which of the .) quastion . 17th ed. 250. are presented to your office in November for physical examination prior to entry to daycare. Amantadine and rimantadine are effective only against influenza A strains. (See Chapters 237 and 282 in Nelson Textbook of Pediatrics. 252. 17th ed. (See Chapters 237 and 282 in Nelson Textbook of Pediatrics. whereas rimantadine is indicated in children only for prophylaxis.of pregnancy during the influenza season.) They have identical adverse effect profiles Question . Amantadine is indicated for treatment and prophylaxis of influenza in both children and adults. amantidine or rimantadine hastens recovery from influenza by 1 to 2 days. The most likely etiologic agent is: Influenza virus Parainfluenza virus Explanation: Parainfluenza viruses account for approximately half of cases of croup and. (See Chapter 238 in Nelson Textbook of Pediatrics.following is true concerning the recommendation for influenza vacation of these children? Influenza vaccine is not recommended for either Influenza vaccine is recommended for the 5-mo-old but not the 18mo-old Influenza vaccine is not recommended for the 5-mo-old but is recommended for the 18-mo-old Explanation: Recently. unlike influenza virus and RSV infections. and anorexia. are not usually associated with fever or lower respiratory tract symptoms. and lower respiratory tract findings are normal. the recommendations for influenza vaccine have been extended to encourage vaccination. 17th ed.) Influenza vaccine is recommended for both Indications depend on whether either infant is in a vaccine target group Question . A 4-yr-old child has an acute illness with coryza. The vaccine is not given to children younger than 6 mo of age. to the extent feasible. There is no fever. 17th ed. tapering church . the radiologist calls you to report finding a steeple sign. It is the progressive narrowing of the subglottic region that has the appearance of a tall. of all healthy children who will be 6-23 mo of age during influenza season (October-March) because their risk of hospitalization is similar to that in the elderly. 254. 253. barky cough. hoarseness. The most likely etiologic agent is: Influenza virus Parainfluenza virus Explanation: The steeple sign is visible on an anteroposterior radiograph of the neck. (See Chapter 237 in Nelson Textbook of Pediatrics.) Respiratory syncytial virus Calicivirus Adenovirus Question . Following your request for a plain x-rays of the neck of a 2-yr-old child. 17th ed. 257. cough. A 7-mo-old infant has rhinorrhea.) Children younger than 2 yr with bronchopulmonary dysplasia who require oxygen Infants up to 12 mo of age who were born at 28 wk of gestation or less Infants up to 6 mo of age who were born at 32 wk of gestation or less Question . which has the highest incidence from 2 to 7 mo of age and usually occurs in the winter months. which is most frequently caused by parainfluenza viruses. Adenovirus Chlamydia trachomatis Question . Prophylaxis with palivizumab or RSV-IVIG against respiratory syncytial virus (RSV) is indicated for all of the following patients except: Children younger than 2 yr with congenital cyanotic heart disease Explanation: RSV-IVIG is contraindicated and palivizumab is not recommended for infants with cyanotic heart disease. audible wheezes. The most likely etiologic agent is: Influenza virus Parainfluenza virus Respiratory syncytial virus Explanation: Respiratory syncytial virus is the most common cause of bronchiolitis. Which of the following is/are frequently caused by adenoviruses? Pharyngitis . 256. In studies of RSV-IVIG given for prophylaxis. (See Chapter 238 in Nelson Textbook of Pediatrics.) Respiratory syncytial virus Adenovirus Chlamydia trachomatis Question . (See Chapter 239 in Nelson Textbook of Pediatrics. It is a sign of the subglottic edema that is typical of croup. and chest retractions.steeple. 17th ed. mortality is higher in treated patients with heart disease. There is a low-grade fever and irritability. 255. 258. conjunctivitis. a pertussis-like syndrome. 17th ed. Hemorrhagic cystitis. pneumonia. 17th ed. 260. 259. 17th ed. (See Chapter 241 in Nelson Textbook of Pediatrics. myocarditis. diarrhea.) Question .Pertussis-like syndrome Conjunctivitis Acute diarrhea All of the above Explanation: Adenoviruses cause a wide array of clinical illnesses. pneumonia.) Rhinovirus Herpes simplex virus Parainfluenza virus Question . The cell receptor for most rhinoviruses is: ICAM-1 Explanation: ICAM-1 (intercellular adhesion molecule 1) is present on the epithelium covering the lymphoepithelium of the adenoids and other epithelial cells of the nose and is the cell receptor for most rhinoviruses. pharyngoconjunctival fever. (See Chapter 240 in Nelson Textbook of Pediatrics. conjunctivitis. Leukocytosis and a high ESR may also be present. and diarrhea all have been linked to: Respiratory syncytial virus Adenovirus Explanation: Adenovirus 11 or 21 is probably the cause.) CD23 Tumor necrosis factor (TNF) IL-10 IFNQuestion . including pharyngitis. All of the following viruses may be associated with gastroenteritis except: Rotavirus . and hemorrhagic cystitis. (See Chapter 240 in Nelson Textbook of Pediatrics. adenoviruses. or increase in size and number. Norwalk virus) are the medically important pathogens of human viral gastroenteritis. remain unchanged. Condylomata acuminata are genital warts. Treatment of cervical warts may not decrease the risk of cervical cancer. especially hand warts. often resolve spontaneously Explanation: Untreated warts may spontaneously regress. Laryngeal papillomatosis follows acquisition of papillomaviruses during passage through an infected birth canal. and caliciviruses (e. (See Chapter 242 in Nelson Textbook of Pediatrics. astrovirus.g. 262. and some are painful and some are painless. some of which are patient-applied and some of which are physician-applied. 17th ed. Which of the following statements regarding treatment for papillomavirus infection is true? Warts.) Effective treatments are painful Effective treatments are physician-applied Most cervical warts should be treated by a generalist . There are many effective treatments. (See Chapter 243 in Nelson Textbook of Pediatrics. (See Chapter 243 in Nelson Textbook of Pediatrics. 17th ed.) Astrovirus Adenovirus Calicivirus Question . 261. Papillomaviruses may be associated with: Common skin warts Laryngeal papillomatosis Condylomata acuminata Cervical cancer All of the above Explanation: Papillomaviruses cause a variety of proliferative cutaneous and mucosal lesions. 17th ed..) Question .Polyomavirus Explanation: Rotaviruses. The case fatality rate is 33-75%. and headache. and coma. confusion. 264. His vaccinations are up to . irritability. The peripheral white blood cell count demonstrates a marked leukocytosis. 265. it carries a poor prognosis. No specific treatment is available. on September 3. 263. 17th ed. with many survivors having residual neurologic deficits. The history of outdoor activity in freshwater swamps in New Jersey suggests Eastern equine encephalitis. Unlike the other arboviral encephalitides. 17th ed. (See Chapter 244 in Nelson Textbook of Pediatrics.) Zanamivir is an effective agent but is not usually required Question .Treatment of cervical warts greatly decreases the risk of cervical cancer Question .) Question . followed by lethargy. The case-toinfection ratio is 1:8 in children and 1:29 in adults. (See Chapter 244 in Nelson Textbook of Pediatrics. He had accompanied his parents on a picnic and boating outing in the saltwater marshes along the New Jersey coastline 1 wk earlier. Which of the following is the most likely diagnosis? Meningococcal meningitis Tuberculous meningitis Dengue hemorrhagic fever Lyme disease Eastern equine encephalitis Explanation: The clinical picture is typical of viral encephalitis. This morning an 8-yr-old boy experienced the abrupt onset of fever. During the interview the mother tells you that the family shortly will leave the United States for several years' residence in northern Thailand. A 4 yr-old boy attends the clinic with a minor upper respiratory tract infection. seizures. which is also suggested by the time of year. Which of the following statements concerning eastern equine encephalitis is true? It is the most common of the arboviral encephalitides The area of highest incidence is the northeastern United States Asymptomatic infections are uncommon Death or residual neurologic deficits are common Explanation: Eastern equine encephalitis has a low incidence of a few cases each year along the Atlantic and Gulf States. shock. The child lived in Thailand for 2 yr prior to immigrating to the United States and has just returned from Thailand after visiting to see her grandparents. (See Chapter 246 in Nelson Textbook of Pediatrics. All of the following are typical symptoms of dengue fever except: Headache A transient macular rash for the first 1-2 days of fever A maculopapular rash after defervescence Mild interstitial pneumonia . 266. and a small effusion is detected in the right pleural space. and a petechial rash is characteristic of dengue hemorrhagic fever. a rapid and weak pulse. cool extremities. An 8-yr-old girl presents with high fever of 5 days' duration. The history of previously living in Thailand also suggests that the recent trip resulted in second infection with dengue virus. and a petechial rash. The most likely diagnosis is: Dengue hemorrhagic fever Explanation: The clinical picture of high fever. which is associated with more severe infections. What is your recommendation with regard to travel immunization? Vaccination against Japanese encephalitis Explanation: Travelers to countries endemic for Japanese encephalitis who will be staying for 1 mo or longer in rural areas should be vaccinated against Japanese encephalitis virus.2 in Nelson Textbook of Pediatrics. 267.) Vaccination against yellow fever Vaccination against Western equine encephalitis Immune globulin to prevent typhoid fever No additional immunizations are indicated Question . (See Chapter 245.) Hepatitis A Relapsing fever Typhoid fever Yellow fever Question .date for his age. 17th ed. 17th ed. Pulse pressure is less than 20 mm Hg. ) Pleural effusion (by chest radiograph) Hypoalbuminemia Thrombocytopenia (<=100. Departing next month with a group of classmates.000/mm3) Fever Question . 269. and a maculopapular rash. sometimes with desquamation. 17th ed. and environmental or occupational exposure may determine the specific risk and individual need for vaccination.) Arthralgias Question . myalgias and arthralgias. A 16-yr-old female high school student is seen for pretravel counseling. exact locations to be visited.Explanation: Dengue is characterized by high fever. (See Chapter 247 in Nelson Textbook of Pediatrics. a transient blanching rash for the first 1-2 days of fever. Your recommendation with regard to travel immunization is: Vaccination against malaria Vaccination against Lyme disease Vaccination against yellow fever Explanation: All persons traveling to areas of South America and Africa in which yellow fever is endemic are possible candidates for vaccination. 268. (See Chapter 246 in Nelson Textbook of Pediatrics. she will spend 7 days on a bird-watching excursion in the Amazonian portion of Peru. 17th ed.) Immune globulin to prevent typhoid fever Five days of chloramphenicol on arrival . but length of stay. Dengue shock syndrome is dengue hemorrhagic fever plus hypotension or a narrow pulse pressure. headache. (See Chapter 246 in Nelson Textbook of Pediatrics. that appears after defervescence. All of the following may be features of dengue hemorrhagic fever except: Hematocrit decreased by >=20% Explanation: Dengue hemorrhagic fever is associated with increased capillary permeability. 17th ed. which leads to hemoconcentration (an increased hematocrit) and hypoalbuminemia. 17th ed.Question . The most likely diagnosis is: Yellow fever Colorado tick fever Borreliosis Hantavirus pulmonary syndrome Explanation: The clinical manifestations of fever. and acute respiratory distress are characteristic of hantavirus pulmonary syndrome. A child suffers a provoked bite from a stray dog that was captured by animal control and appears healthy.) . The most appropriate action would be to: Confine and observe the dog for 10 days for signs suggestive of rabies Explanation: Healthy dogs. (See Chapter 251 in Nelson Textbook of Pediatrics. and feces of the deer mouse. vomiting. (See Chapter 250 in Nelson Textbook of Pediatrics. 272. Peromyscus maniculatus. and mid-abdominal pain. Which of the following is a risk factor for hantavirus pulmonary syndrome caused by Sin Nombre virus? Tick bites Mosquito bites Exposure to infected rodents Explanation: Sin nombre virus is shed from saliva. myalgia. 270. and ferrets should be held for a 10day observation. cough. His blood pressure is 90/55 mm Hg. diarrhea. On physical examination he is dyspneic and tachypneic. cats. 271. (See Chapter 250 in Nelson Textbook of Pediatrics. which should be euthanized immediately and tested. He presents with fever and myalgia accompanied by cough. 17th ed. 17th ed.) Swimming in stagnant or brackish water Contaminated food Question . urine. A 15-yr-old boy has been in Arizona on a camping trip. Postexposure prophylaxis for rabies should be given to the bitten person at the first sign of rabies in the animal.) Dengue hemorrhagic fever Question . 274. Which of the following describes the recommended use of HRIG for postexposure rabies prophylaxis in the United States? Administer one half of the HRIG dose at the exposure site and the remainder at another site intramuscularly. with the remainder administered intramuscularly at another site. (See Chapter 251 in Nelson Textbook of Pediatrics. with rabies vaccine for high-risk bites Rabies immune globulin is no longer recommended in the United States Question .Submit the dog's head for examination for rabies Begin rabies vaccination Administer human rabies immune globulin (HRIG) and begin rabies vaccination None of the above?no postexposure prophylaxis is necessary because it was a provoked attack Question . skunks. always with rabies vaccine Explanation: As much as possible of the dose of HRIG (20 IU/kg) should be infiltrated into the wound and the area around the wound. Animals that should be regarded in the United States as rabid include: Rabbits and hares Feral mice and rats Squirrels and chipmunks Bats Explanation: Bats. foxes.) Administer one half of the HRIG dose at the exposure site and the remainder at another site intramuscularly. with rabies vaccine for high-risk bites Administer as much of the HRIG dose as possible at the exposure site and the remainder at another site intramuscularly. always with rabies vaccine Administer as much of the HRIG dose as possible at the exposure site and the remainder at another site intramuscularly. 17th ed. HRIG should always be given with vaccine for postexposure prophylaxis of unimmunized persons. raccoons. and most other carnivores should be regarded as rabid unless the animal is proved . 273. visual field defects. (See Chapter 253 in Nelson Textbook of Pediatrics.) Question .) Woodchucks and beavers Question . An 18-yr-old patient with AIDS develops motor weakness. guinea pigs. (See Chapter 252 in Nelson Textbook of Pediatrics. 276. mice. and speech and cognitive impairment with dementia. 17th ed. and hares almost never require prophylaxis. chipmunks. Prion proteins from several species are similar but not identical in structure.rabies-negative by laboratory testing. Their primary structure is encoded by the host. The most likely etiologic agent is: Pneumocystis carinii Cryptococcus Mycobacterium avium complex Cytomegalovirus JC virus Explanation: Progressive multifocal leukoencephalopathy (PML) is caused by JC virus. rabbits. local public health officials should be consulted. (See Chapter 251 in Nelson Textbook of Pediatrics. 277. All of the following are features of prion proteins except: They are glycoproteins They are susceptible to chemical and physical treatments Explanation: Prion proteins cause the transmissible spongiform encephalopathies.) They are transmissible Their primary structure is encoded by the host Prions from different species are very similar in structure Question . 275. Bites of squirrels. rats. and personality change. confusion. other small rodents. 17th ed. They are very resistant to chemical and physical treatments. 17th ed. More than half of cases occur in HIV-infected individuals. All of the following statements regarding the spongiform encephalopathies are true except: . hamsters. gerbils. 17th ed. (See Chapter 253 in Nelson Textbook of Pediatrics.) Some forms show autosomal dominant inheritance There is no effective treatment Most patients die within 1 yr Question . She has never been on any antiviral medication because she has remained healthy without symptoms of AIDS. 278. (See Chapter 254 in Nelson Textbook of Pediatrics.) It is a transmembrane glycoprotein It is highly immunogenic It is a major component of the viral envelope It binds with the CD4 cell receptor Question .Creutzfeldt-Jakob disease (CJD) is the most common human spongiform encephalopathy Kuru now occurs primarily in young adults Explanation: Kuru once affected many children. so it should be avoided Treatment will decrease the likelihood of fetal infection by greater than 50% . which is one reason for the difficulty in developing an effective HIV vaccine. This indicates that ritual cannibalism was probably the only mechanism of spread. 17th ed. 280. All of the following statements regarding HIV gp120 are true except: It shows little heterogeneity among HIV strains Explanation: HIV gp120 has significant heterogeneity among HIV strains. Which of the following statements regarding such treatment is correct? Treatment will produce maternal thrombocytopenia Treatment will result in viral resistance in all infected infants Treatment will decrease the likelihood of fetal infection but adversely affect the woman. adolescents. However. she wonders whether antiviral treatment might diminish the likelihood of fetal HIV infection. An HIV-seropositive woman comes to her physician for advice because she is pregnant. and adults of Papua New Guinea but is now recognized only in older adults. (See Chapter 254 in Nelson Textbook of Pediatrics. In the absence of clinical signs or symptoms. 282.) . Mechanisms responsible for vertical transmission of HIV infection include: Intrauterine fetal infection Intrapartum (peripartum) transmission Breast-feeding All of the above Explanation: Most cases of vertical-transmitted HIV infection occur from intrapartum transmission. 17th ed. (See Chapter 254 in Nelson Textbook of Pediatrics. repeat testing at age 18 mo or older is necessary to confirm infection. All of the following statements about HIV infection are true except: HIV infection is the most common cause of immunodeficiency in adults and children HIV-infected children commonly have hypergammaglobulinemia Stem cell transplantation is not considered a useful therapeutic option for AIDS A positive HIV antibody test is diagnostic of HIV infection in children and adults Explanation: In infants and young children (<18 mo of age) it is not possible to distinguish readily between passively acquired and actively produced HIV antibody. 17th ed. 281. Treatment reduces the overall risk of infection from approximately 25-30% to 8%.) Treatment will adversely affect the fetus and should be avoided Question .) The risk of vertical transmission of HIV can be reduced by perinatal zidovudine administration Question . Treatment begins any time after 14 wk of gestation and continues during labor and delivery and for another 6 wk (in the infants). but HIV can also be vertically transmitted in utero or by breast-feeding. (See Chapter 254 in Nelson Textbook of Pediatrics.Explanation: Treatment of HIV-positive mothers can dramatically reduce the incidence of HIV infection in infants. 17th ed. ) .) Positive result on HIV DNA assay Positive result on HIV RNA assay Question . (See Chapter 254 in Nelson Textbook of Pediatrics. 283.Question . All of the following may be diagnostic of HIV infection in an 8-mo-old child except: Positive result on p24 antigen assay Positive HIV culture Positive HIV Western immunoblot assay Explanation: Serologic diagnosis of HIV infection by ELISA and Western immunoblot analysis is reliable only after 18 mo of age. 284. All of the following statements concerning treatment of HIV infection are true except: Multiple-drug regimens are superior to single-drug therapy Protease inhibitors prevent uncoating of infectious virions Explanation: Protease inhibitors act by preventing packaging of infectious virions before they leave the infected cell. (See Chapter 254 in Nelson Textbook of Pediatrics. All of the following statements concerning HIV infection are true except: HIV suppression is best achieved by regularly rotating antiretroviral regimens Explanation: Sustainable suppression of HIV is best achieved by combination antiretroviral therapy to which the patient has not been exposed previously and that is not cross-resistant to drugs given to the patient previously. (See Chapter 254 in Nelson Textbook of Pediatrics. 285. residual maternal antibodies acquired transplacentally may be responsible for the positive serologic test results. 17th ed.) Therapy can reduce HIV burden to undetectable levels Viral burden predicts disease progression CD4 cell counts reflect the risk of opportunistic infections Adherence to therapy is crucial Question . 17th ed. Before this age. 17th ed. intravenous zidovudine during labor. 287.) .Nucleoside and non-nucleoside reverse transcriptase inhibitors act at different sites of reverse transcription Drug-drug interactions are common with protease inhibitors Antiretroviral treatment is most successful in treatment-naive patients Question . 17th ed. DNA or RNA PCR assay. (See Chapter 268 in Nelson Textbook of Pediatrics. 286. prophylaxis is prescribed according to the CD4 cell count and percentage. carinii regardless of CD4 cell count and percentage. or p24 antigen assay Infants with an AIDS-defining diagnosis Infants who have had a first episode of P. (See Chapter 254 in Nelson Textbook of Pediatrics. After 12 mo of age. Regimens of only one or two of these components provide some reduction. 17th ed. Components of the recommended prophylactic regimen to prevent vertical HIV transmission include: Zidovudine given orally to the mother after the first trimester Zidovudine given intravenously to the mother during delivery Zidovudine given orally to the newborn for the first 6 wk of life All of the above Explanation: The regimen of prenatal oral zidovudine during the second and third trimesters. and postnatal zidovudine given to the infant reduces vertical HIV transmission to as low as 3-4%. Combination prophylaxis with drugs more potent than zidovudine and that may provide greater effectiveness is being studied. Which of the following infants born to HIV-infected mothers should receive prophylaxis for Pneumocystis carinii? Infants with a positive result on HIV culture.) Question . carinii pneumonia Infants with a CD4 cell count of less than750/mm3 or a CD4 percentage of less than 15% All infants 6 wk to 1 yr of age born to HIV-infected mothers Explanation: All infants between 6 wk and 1 yr of age should receive prophylaxis for P. asymptomatic 2-yr-old HIV-infected child with a CD4 cell count of 45/mm3 and HIV RNA of 110.000 copies/mL should include all of the following except: Trimethoprim-sulfamethoxazole prophylaxis for Pneumocystis carinii infection Clarithromycin prophylaxis for Mycobacterium avium complex infection Monthly IVIG prophylaxis for bacterial infections Explanation: Monthly IVIG to prevent bacterial infections is recommended only for HIV-infected children who have had two or more serious bacterial infections within 1 yr. Management for a newly diagnosed. The rate of transmission of maternal HIV infection to offspring without any antiretroviral treatment is: 1% 4% 12-30% Explanation: Most large studies in the United States and Europe have documented vertical transmission rates in untreated women of 12-30%. (See Chapter 254 in Nelson Textbook of Pediatrics. 17th ed. 17th ed.Question . (See Chapter 254 in Nelson Textbook of Pediatrics. Transmission rates in Africa and Haiti are higher (25-52%). have a documented inability to make antigen-specific immunoglobulin.) Tuberculin skin testing Antiretroviral therapy Question . or have hypogammaglobulinemia.) 50-60% 70-90% Question . 290. 288. The role of elective cesarean section in preventing perinatal HIV transmission can best be described as: Ineffective and not recommended Effective but not recommended Effective and selectively recommended . 289. (See Chapter 255 in Nelson Textbook of Pediatrics. The most important variables appear to be duration of ruptured membranes of greater than 4 hr and birthweight less than 2500 g. 292. 17th ed. (See Chapter 254 in Nelson Textbook of Pediatrics.Explanation: A meta-analysis of over 1. All of the following statements regarding infection with human T-cell lymphotrophic virus type I (HTLV-I) are true except: It is the cause of adult T-cell leukemia/lymphoma It is the cause of tropical spastic paraparesis (TSP) Breast-feeding is safe because HTLV-I is not transmitted in breast milk Explanation: HTLV-I is prevalent in Japan and the Caribbean. because these data predated the advent of highly active antiretroviral therapy (HAART). 17th ed. Factors that may affect the perinatal HIV transmission rate include: Preterm delivery Low maternal antenatal CD4 count Duration of ruptured membranes Vaginal versus cesarean delivery All of the above Explanation: Several risk factors influence the rate of vertical transmission: pre-term delivery (<34 wk of gestation).) HTLV-I is transmitted by sexual contact All donated blood in the United States is tested for anti-HTLV . 291.) Effective and routinely recommended Question . 17th ed. where breast-feeding is a major mode of transmission. (See Chapter 254 in Nelson Textbook of Pediatrics.000 pregnancies demonstrated that elective cesarean delivery decreased transmission by 87% if used in conjunction with zidovudine therapy in the mother and the infant. and use of illicit drugs during pregnancy. each of which doubles the transmission rate. Testing of all blood products for HTLV-I/II antibodies was implemented in the United States in 1997. the additional benefit of cesarean section is probably negligible if the mother's viral load is <500 copies/mL.) Question . However. a low maternal antenatal CD4 count. antibodies Question . Therapy is often difficult. Most patients are immunocompromised It is readily treated with metronidazole and amphotericin B It is indistinguishable from infection with Balamuthia mandrillaris Question . lakes. 293. 294. (See Chapter 256 in Nelson Textbook of Pediatrics. It occurs most commonly in previously healthy children and adults. and morbidity and mortality are high. and stagnant pools. Therapy is often difficult. Which of the following helps to distinguish Acanthamoeba from Balamuthia infection of the central nervous system? Acanthamoeba infection manifests within days of exposure Balamuthia is readily amenable to treatment while Acanthamoeba is nearly always fatal Patients with Acanthamoeba are more likely to be . 295. Which of the following statements concerning Naegleria infection of the central nervous system is true? It causes a granulomatous encephalitis Swimming in warm freshwater ponds is a risk factor for infection Explanation: Naegleria organisms are found in many freshwater sources. and morbidity and mortality are high. 17th ed. lakes.) Malaria Question . and stagnant pools. including ponds. including ponds. The most likely etiology is: Pseudomonas aeruginosa infection Herpes simplex virus infection Mucor infection Naegleria infection Explanation: Naegleria organisms are found in many freshwater sources. An 8-yr-old child presents with meningitis and encephalitis of acute onset in late summer after swimming in a pond of stagnant water. A 17-yr-old boy presents with symptoms suggestive of granulomatous amebic meningoencephalitis. immunocompromised Explanation: The clinical manifestations and course of Acanthamoeba and Balamuthia infection of the central nervous system are similar.) Naegleria Cryptosporidium Question . 17th ed. Most patients with Acanthamoeba are immunocompromised. 297. A 4-yr-old child presents with colicky abdominal pain. 17th ed.) Iodoquinol is recommended for asymptomatic persons Metronidazole is recommended for invasive intestinal amebiasis Metronidazole is recommended for hepatic amebiasis Metronidazole therapy should always be followed by iodoquinol .) Balamuthia is contracted from swimming in fresh water Balamuthia is associated with hypereosinophilia. blood-stained diarrhea. Acanthamoeba with lymphocytosis Question . There is no fever. dispar is associated only with an asymptomatic carrier state. (See Chapter 257 in Nelson Textbook of Pediatrics. and tenesmus. all of acute onset. The family has recently returned from a trip to Mexico. Amebiasis is uncommon in temperate climates. (See Chapter 256 in Nelson Textbook of Pediatrics. E. whether they have symptoms or not. 17th ed. in contrast to Balamuthia infection. All of the following statements concerning the treatment of Entamoeba histolytica amebiasis are true except: Asymptomatic cyst excreters usually do not need to be treated Explanation: All persons with Entamoeba histolytica trophozoites or cysts in their stools should be treated. 296. The most likely etiologic agent is: Giardia lamblia Entamoeba dispar Entamoeba histolytica Explanation: The highest incidence of amebic colitis caused by Entamoeba histolytica is in children 1-5 yr of age. Trophozoites are seen in the stool. Giardia lamblia does not cause bloody colitis. (See Chapter 257 in Nelson Textbook of Pediatrics. 17th ed. (See Chapter 257 in Nelson Textbook of Pediatrics. especially surface water treated by faulty or inadequate water purification systems.) question . 298.Question . 300. 7 cm. fluid-filled cavitary lesion in the right hepatic lobe. and hepatic tenderness. 299. The test most likely to confirm the diagnosis is: Upper gastrointestinal endoscopy and duodenal biopsy Bone marrow aspirate and culture Mesenteric lymph node biopsy and culture Fungal blood culture Serologic testing Explanation: The clinical presentation suggests amebic abscess. A CT scan reveals a single. A 7-yr-old girl whose family recently moved from Mexico to the United States presents with fever. abdominal pain and distention. mountain streams. and contaminated food. Recognized sources of Giardia lamblia include: Swimming pools Mountain streams Children in daycare centers Food All of the above Explanation: Contaminated water is the most common source of Giardia organisms. All of the following statements concerning the treatment of Giardia lamblia infections are true except: Asymptomatic cyst excreters usually do not need to be treated .) Question . 17th ed. The diagnosis is established by the characteristic CT or MRI findings and positive serologic results for antibodies to Entamoeba. Giardia is very common among children in child daycare centers and among male homosexuals. (See Chapter 258 in Nelson Textbook of Pediatrics. Other sources include contaminated swimming pools (Giardia is resistant to chlorination). 301. (See Chapter 259 in Nelson Textbook of Pediatrics. Asymptomatic persons usually do not need treatment except in specific instances such as outbreak control. with or without azithromycin. (See Chapter 259 in Nelson Textbook of Pediatrics. 17th ed. All of the following statements concerning Cryptosporidium parvum infections are true except: Infection is common in children It causes a bloody colitis Explanation: Cryptosporidium is prevalent in developing countries and in children younger than 2 yr of age. no specific therapy is required. and patients with hypogammaglobulinemia or cystic fibrosis. 302. All of the following are spore-forming intestinal protozoans except: Giardia lamblia Explanation: G. 17th ed. lamblia is a flagellated protozoan. nonbloody diarrhea that is usually self-limited but may persist for several weeks. Treatment of immunocompromised persons is with paromomycin.) Cryptosporidium parvum Isospora belli Cyclospora cayetanensis Microsporidia Question .) Diarrhea may persist for several weeks Treatment is not recommended for immunocompetent persons . prevention of household transmission to pregnant women. (See Chapter 258 in Nelson Textbook of Pediatrics. It causes watery. Because illness is self-limited in immunocompetent persons.Children with acute diarrhea require treatment Children with chronic diarrhea require treatment Metronidazole is the treatment of choice Metronidazole therapy should always be followed by iodoquinol Explanation: Symptomatic persons with G. lamblia should be treated.) Question . 17th ed. routine ova and parasite testing. (See Chapter 259 in Nelson Textbook of Pediatrics.) Order serologic testing Refer the patient to the gastroenterologist for colonoscopy Question . but the daycare staff report that most have had negative bacterial stool cultures. Giardia is diagnosed by documenting trophozoites. red. dysuria. vulvovaginal irritation. Recognized presentations of leishmaniasis include: A papular lesion on the face . 303. There is no blood present in the stool. Several of the other toddlers and adult workers in the center have also had diarrhea. or Giardia antigens in stool specimens. A sexually active adolescent girl presents with copious malodorous yellow vaginal discharge with vulvovaginal irritation. 17th ed. and dyspareunia. cysts. Cryptosporidium is diagnosed by modified acid-fast staining of the stool.Paromomycin is recommended for treatment of immunocompromised persons Question .) Giardia lamblia Haemophilus ducreyi Enterobius vermicularis Question . spherical bodies. 304. The most appropriate next step in diagnosis would be to: Repeat the bacterial stool culture requesting identification of E. coli O157:H7 Request bacterial stool culture and also routine ova and parasite testing Request bacterial stool culture. and there is no fever. and an acid-fast smear of the stool Explanation: The presentation suggests parasitic infection with Giardia or Cryptosporidium parvum. The most likely etiologic agent is: Candida albicans Trichomonas vaginalis Explanation: Trichomoniasis is characterized by copious malodorous vaginal discharge. 305. An 18-mo-old boy who attends out-of-home daycare develops intermittent watery diarrhea. (See Chapter 260 in Nelson Textbook of Pediatrics. appearing as small (2-6 µm). 17th ed. 17th ed. and cachexia All of the above Explanation: Leishmania causes diverse group of diseases.) P. A 12-yr-old girl who recently visited Haiti with her family presents with signs suggestive of cerebral malaria. (See Chapter 261 in Nelson Textbook of Pediatrics. 17th ed. vivax None of the above: all malaria species are similar in disease severity Question . diffuse cutaneous leishmaniasis. hepatomegaly. ovale P. and visceral leishmaniasis (kala-azar). malariae P.) Polymerase chain reaction (PCR) testing Question . 17th ed.Large non-ulcerating papules that resemble leprosy Nasal mucosal involvement with recurrent epistaxis Fever. 308. She did not take malaria prophylaxis. Which of the following Plasmodium species causes the most severe form of malaria and is associated with the highest fatality rate? P. 306. mucosal leishmaniasis. marked splenomegaly. (See Chapter 264 in Nelson Textbook of Pediatrics. including: localized cutaneous leishmaniasis. The most useful method for the diagnosis of malaria is: Serologic testing for IgM antibodies Serologic testing for IgG antibodies Rapid antigen testing Thick and thin blood smears Explanation: The diagnosis of malaria is best established by identification of organisms on Giemsa-stained thick and thin blood smears. 307. falciparum malaria is the most severe form and carries fatality rates of 25-35% in untreated persons. (See Chapter 264 in Nelson Textbook of Pediatrics. The .) Question . falciparum Explanation: P. (See Chapter 264 in Nelson Textbook of Pediatrics. but the cerebrospinal fluid is normal.) Question .) Visceral leishmaniasis Dengue hemorrhagic fever Question . especially if repeated. A child who returned 2 wk ago from equatorial Africa is admitted to the hospital in a coma. Laboratory manuals help identify the morphologic forms needed to distinguish the different malarial species. 309. 17th ed. The most likely diagnosis is: Pancreatic neoplasm Pneumococcal meningitis Falciparum malaria Explanation: Malaria is the first diagnosis to be considered in this patient. (See Chapter 264 in Nelson Textbook of Pediatrics.confirmatory procedure that should be performed immediately is: Serologic testing for specific antibodies CT scan of the brain Urinalysis to detect hematuria Temperature readings at 6-hr intervals to ascertain fever intermittency Microscopic examination of blood films Explanation: Cerebral malaria can be fatal in as little as 24 hr and is a medical emergency. All of the following are recognized routes of malaria transmission except: Blood transfusion Mosquito bite Use of contaminated needle . with high fever and a palpable spleen. The thick smear of peripheral blood should be diagnostic. The liver is not enlarged. Laboratory studies reveal hypoglycemia. who had the unfavorable prognostic features of hypoglycemia and coma. 310. 17th ed. from pregnant woman to her fetus Out-of-home daycare Explanation: Malaria is transmitted principally via blood mosquito bites. contaminated needles. and both he and his parents confirm complete adherence to the prescribed malaria prophylaxis regimen. 17th ed. pulmonary edema or renal failure. 311. (See Chapter 264 in Nelson Textbook of Pediatrics. The use of quinidine gluconate should be considered in any child experiencing malaria associated with: Neurologic dysfunction Pulmonary edema Inability to retain oral fluids or medication Parasitemia more than 5% of erythrocytes Any of the above Explanation: Intravenous quinidine gluconate should be administered for patients who: cannot retain oral fluids and medication because of vomiting. but has also been transmitted by blood transfusion. have neurologic dysfunction.) Question .Transplacental. (See . have a peripheral asexual parasitemia of >5% of erythrocytes. 17th ed. (See Chapter 264 in Nelson Textbook of Pediatrics.) Question . 312. He had traveled to India with his parents 6 mo previously. A 13-yr-old adolescent boy presents with unexplained febrile illness. Which of the following is true concerning the need for investigation for malaria as the cause of his illness? No investigation is necessary because prophylaxis was used No investigation is necessary if mefloquine was part of the prophylactic regimen Investigation is necessary only if another family member was ill Investigation is necessary only if he has had unexplained fevers since his return Investigation for malaria is necessary Explanation: Any person who has fever or unexplained systemic illness and has traveled or lived in a malarial endemic area within the previous year should be assumed to have life-threatening malaria until proven otherwise regardless of the use of chemoprophylaxis. or have a peripheral asexual parasitemia of 1-4% of erythrocytes with a severe attack. and vertical transmission. and discontinue on return home Begin prophylaxis 1 wk before departure. 17th ed. 315. 17th ed. and you consider a diagnosis of babesiosis. and seek medical attention immediately Question . and discontinue 1 wk after return home Begin prophylaxis 1 wk before departure. (See Chapter 264 and Table 285-2 in Nelson Textbook of Pediatrics. and discontinue 4 wk after return home Explanation: Weekly mefloquine is the drug of choice for malaria prophylaxis for children and adults traveling to areas with chloroquine-resistant P. 313. chills. For the patient described in Question 314. the recommended treatment is: .) Question . Appropriate guidance for timing of prophylaxis would be: Begin prophylaxis on arrival in Africa. and myalgias. He recalls a tick bite about 2 wk previously.) Begin prophylaxis with any fever while traveling. falciparum.Chapter 264 in Nelson Textbook of Pediatrics. Mefloquine is started 1 wk before departure and continued for 4 wk after the last exposure.) Question . 17th ed. Which of the following methods is useful to establish this diagnosis? Identification of Babesia on thin blood smears Amplification of Babesia DNA using polymerase chain reaction technique Identification of Babesia after injection of hamsters with patient's blood Identification of Babesia antibody Any of the above Explanation: Any of these methods may be used to diagnose babesiosis. 314. and discontinue on return home (to the United States) Begin prophylaxis 1 wk before departure. in 9 wk and is visiting your office for pretravel advice. in an area of chloroquine-resistant Plasmodium falciparum malaria. A 19-yr-old boy in Rhode Island presents with high fever. You recommend mefloquine. (See Chapter 265 in Nelson Textbook of Pediatrics. A family is planning on visiting Africa. (See Chapter 265 in Nelson Textbook of Pediatrics. 17th ed. Which of the following causes of congenital infections is associated with cats? Cytomegalovirus Rubella Toxoplasma gondi Explanation: Cats ingest Toxoplasma oocysts in infected meat or from the feces of other infected cats. Cats that are strictly kept . 317.) Erythromycin Tetracycline Question .Ceftriaxone with or without streptomycin Ciprofloxacin Clindamycin and quinine Explanation: The combination of clindamycin (20-40 mg/kg/day divided tid PO) and quinine (25 mg/kg/day divided tid PO) for 7 to 10 days is the therapy of choice for babesiosis in children. 316. Tinnitus and abdominal distress are common adverse reactions.) Central nervous system involvement is common Question . (See Chapter 266 in Nelson Textbook of Pediatrics. 17th ed. All of the following statements concerning congenital toxoplasmosis are true except: Fetal infection is more common if maternal infection is acquired in late pregnancy Severe disease is more common if maternal infection is acquired in early pregnancy Almost all persons with untreated congenital infection eventually experience symptoms or signs of infection Chorioretinitis is common at birth but infrequently develops afterward Explanation: Chorioretinitis occurs in only approximately 22% of newborns with congenital toxoplasmosis. but almost all untreated persons will have chorioretinal lesions by adulthood and about half will have severe visual impairment. gondii have no clinically recognizable disease. Toxoplasmosis causes all of the following patterns of disease except: Congenital infection manifested in neonates by chorioretinitis. Which is the most common result of primary Toxoplasma gondii infection in children? Cellulitis Lymphadenitis Meningitis Pneumonia No specific symptoms Explanation: Most immunologically normal children who acquire T.) Ingestion of cysts in undercooked meat Transplacental transmission from an acutely infected mother to her fetus in utero Organ transplantation of an infected to a previously uninfected recipient Question . maintained on prepared diets. (See Chapter 266 in Nelson Textbook of Pediatrics. Pregnant women who are seronegative for T.)i Syphilis Parvovirus B19 Question .indoors. . gondii should avoid contact with cat feces. 17th ed. 17th ed. uncooked meat should not contact encysted T. (See Chapter 266 in Nelson Textbook of Pediatrics. gondii oocysts. and not fed fresh. 318. Toxoplasma gondii may be acquired by all of the following means except: Ingestion of oocysts excreted by cats Ingestion of oocysts excreted by dogs Explanation: Dogs do not carry T. (See Chapter 266 in Nelson Textbook of Pediatrics. gondii or shed oocysts. The other sources listed in the question are compatible with transmission of toxoplasmosis. 17th ed. 319. 320.) Question . and paralysis in patients with acquired immunodeficiency syndrome (AIDS) Retinal lesions involving the macula and leading to blindness Question . thrombocytopenia. and 10% atypical lymphocytes. but not with true congenital anomalies. growth retardation. and gardened and played in the backyard sandbox. The child and the mother regularly cleaned the kitten's litter pan together.) Question . 100. 322. gondii-specific IgG in the 5-yrold boy's serum IgM ELISA AC/HS. hepatitis. and also PCR assay of amniotic fluid. 321. gondii.) Localized or generalized lymphadenopathy in previously healthy persons Brain abscess that results in confusion. and avidity tests to measure T. and microcephaly. Fetal ultrasound examination. 17th ed. The lymph nodes are nontender and have not suppurated.cerebral calcifications. where the kitten also defecated. can be used to evaluate the fetus. (See Chapter 266 in Nelson Textbook of Pediatrics. such as cleft palate or patent ductus arteriosus Explanation: Congenital toxoplasmosis is associated with organ dysfunction. seizures. inflammation. 17th ed. All of the following diagnostic tests to identify family members with or at risk for toxoplasmosis are indicated except: Sabin-Feldman [HBJ4]dye test to measure T. brings her 5-yr-old son for evaluation of fever and cervical lymphadenopathy. A pregnant woman. hepatosplenomegaly. (See Chapter 266 in Nelson Textbook of Pediatrics. at 15 wk of gestation. and hydrocephalus due to first-trimester infection Congenital malformations. An infant is born small for gestational age at 34 wk of gestation with a "blueberry muffin" rash. All of the following may be helpful in establishing the diagnosis of congenital toxoplasmosis for . gondii-specific antibodies in the mother's serum Fetal ultrasound examination CT scan of the head and neck for the 5-yr-old boy Explanation: Serologic tests are the best means to confirm infection with T.000 platelets/mm3. retinitis. His serum has a Sabin Feldman dye test titer of 1:4096. The child was given a new kitten 3 mo ago that frequently caught mice in their barn. 15% eosinophils. gondii-specific IgG in the mother's serum Sabin-Feldman dye test to measure T. ) Cell-mediated immunity is more important than humoral immunity in combating infection Severe P. gondii. (See Chapter 267 in Nelson Textbook of Pediatrics. All of the following statements concerning Pneumocystis carinii infections are true except: Most persons are infected before 4 yr of age Most primary infections are usually associated with only lowgrade fever and nonspecific symptoms Explanation: Most persons are infected with P.this infant except: IgM ISAGA to detect T. carinii before 4 yr of age. protein. and glucose Serum amylase Explanation: Serologic tests are the best means to confirm infection with T. carinii pneumonia occurs almost exclusively in immunocompromised persons All of the above Question . 323. gondii serum antibodies Inoculation of a preparation of the infant's placenta into mice and subsequent measurement of antibodies to T. Most primary infections in immunocompetent persons are asymptomatic. (See Chapter 267 in Nelson Textbook of Pediatrics. 17th ed. (See Chapter 266 in Nelson Textbook of Pediatrics. 17th ed. All of the following statements concerning Pneumocystis carinii infections are true except: Most persons are infected before 4 yr of age Most primary infections are usually associated with only lowgrade fever and nonspecific symptoms Explanation: Most persons are infected with P. gondii in the mouse's serum 6 wk after inoculation of the placenta CSF cell count. gondii serum antibodies in his serum IGA ELISA to detect T.) Question . 323. Most primary infections in immunocompetent persons are asymptomatic.) . carinii before 4 yr of age. 17th ed. Cell-mediated immunity is more important than humoral immunity in combating infection Severe P. carinii pneumonia occurs almost exclusively in immunocompromised persons All of the above Question . 17th ed. A 5-yr-old boy with AIDS and who is receiving zidovudine. prophylaxis is prescribed according to the CD4 cell count and percentage. 326. Patients who should receive prophylaxis for Pneumocystis carinii pneumonia include those with: X-linked agammaglobulinemia Severe combined immunodeficiency disorder Explanation: Severe combined immunodeficiency (SCID). and ritonavir presents with rapid respiratory rate and progressive dyspnea of 2 days' duration. 325. (See Chapter 267 in Nelson Textbook of Pediatrics. (See Chapters 254 and 267 and Table 254-4 in Nelson Textbook of Pediatrics.) Chronic granulomatous disease Sickle cell disease Congenital neutropenia Question . After 12 mo of age. The most appropriate next step in management would be to: Begin treatment with a protease inhibitor Order strict isolation of the infant Administer immune globulin to the infant monthly until HIV infection can be excluded Begin trimethoprim-sulfamethoxazole prophylaxis Explanation: All infants between 6 wk and 1 yr of age should receive prophylaxis for P. 324. . A 1-mo-old infant of an HIV-infected mother is classified as "indeterminate" with respect to HIV infection status.) Withhold all immunizations until HIV infection can be excluded Question . 17th ed. carinii pneumonia (PCP). carinii regardless of CD4 cell count and percentage. with its T lymphocyte deficiency. predisposes to P. lamivudine. A previously healthy 7-yr-old girl living in rural South Carolina presents with a 1-wk history of fever. 17th ed. or orally if there is mild disease and no malabsorption or diarrhea. (See Chapter 267 in Nelson Textbook of Pediatrics. but the absence of the organisms in induced sputum does not exclude the infection. Methods for obtaining appropriate specimens for detecting organisms include bronchoalveolar lavage. carinii pneumonia is trimethoprim-sulfamethoxazole administered intravenously. carinii in the lung in the presence of clinical signs and symptoms of the infection. cough. and open lung biopsy. pulse 80/min.) Question . Her temperature is 37.) Trimethoprim-sulfamethoxazole Primaquine and clindamycin Dapsone Question . Induced sputum samples are helpful if P. The open lung biopsy is the most reliable method. The most important diagnostic test would be: CT scan of the chest Pharyngeal swab for viral and bacterial cultures Blood gas profile Serum lactate dehydrogenase level Bronchoalveolar lavage for cytology Explanation: The presentation is characteristic of Pneumocystis carinii pneumonia.5oC. carinii is found. tracheal aspirate. 17th ed. A 10-yr-old child with Pneumocystis carinii pneumonia is allergic to sulfonamide. The chest radiograph reveals bilateral diffuse alveolar disease. transbronchial lung biopsy.There is no fever. although bronchoalveolar lavage is more practical in most cases. and blood pressure 95/60 mm Hg. pentamidine isethionate may be used. 327. bronchial brushings. Which of the following drugs should be used for treatment? Fansidar Pentamidine isethionate Explanation: The recommended therapy for P. Chest film reveals bilateral pulmonary infiltrates. For patients who cannot tolerate or fail to respond to trimethoprimsulfamethoxazole after 5-7 days. percutaneous transthoracic needle aspiration. 328. The complete blood count is normal except for . and shortness of breath. respiratory rate 30 breaths/min. (See Chapter 267 in Nelson Textbook of Pediatrics. A definitive diagnosis of requires the demonstration of P. Hemoptysis was observed within the previous 12 hr. with a repeat dose in 2 wk. 100 mg orally. The most appropriate therapy is: Bacitracin ointment to the perianal area Diphenhydramine orally as needed for itching Single oral dose of mebendazole repeated in 2 wk Explanation: The recommended treatment regimen for pinworms (Enterobius vermicularis) is mebendazole. with a repeat dose in 2 wk. An 8-yr-old boy from Tennessee received a liver transplant 6 mo ago and is on corticosteroids and azathioprine. An alternative treatment is albendazole. in a single dose Question . There are no other symptoms. caused by Ascaris lumbricoides. (See Chapter 268 in Nelson Textbook of Pediatrics. He presents to the emergency department with a 2day history of cough and wheezing. 400 mg orally. 330. A 6-yr-old girl who was previously healthy presents with a 1-wk history of nocturnal perianal itching.) Enterobius vermicularis Mycobacterium tuberculosis Streptococcus pneumoniae Question .) A 2-wk course of amoxicillin/clavulanate Ketoconazole.eosinophilia. A stool specimen is most likely to reveal which of the following microorganisms? Salmonella enteritidis Histoplasma capsulatum Toxocara canis Pneumocystis carinii . 17th ed. and findings on physical examination are normal. Which of the following organisms is the most likely cause of the patient's illness? Trichuris trichiura Ascaris lumbricoides Explanation: The presentation is characteristic of L ffler syndrome. 329. (See Chapter 271 in Nelson Textbook of Pediatrics. 17th ed. caused by Toxocara canis. The most likely etiologic agent is: Trichuris trichiura Trichinella spiralis Explanation: In trichinosis. and fever. Laboratory findings include a white blood cell count of 60. A 4-yr-old boy who recently moved from Puerto Rico to a northern city in the continental United States presents with a 1-wk history of fever and malaise. The most likely diagnosis is: Eosinophilic gastroenteritis Visceral larva migrans Explanation: This is a characteristic presentation of visceral larva migrans. pulse 130/min. and hepatomegaly. 17th ed. especially while chewing and with breathing.000/mm3 with 66% eosinophils. migrating larvae of Trichinella spiralis cause symptoms of fever and myalgias and elicit an eosinophilic response. (See Chapter 272 in Nelson Textbook of Pediatrics. 17th ed. 17th ed.) Question . There is a history of eating undercooked meat. Laboratory testing shows 8% eosinophilia.) Angiostrongylus cantonensis infection Ascaris lumbricoides infection Enterobius vermicularis infection Question . 331. His past history includes extensive use of community playground facilities. blood pressure 100/60 mm Hg. A child presents with several days of increasing myalgias. (See Chapter 276 in Nelson Textbook of Pediatrics. (See Chapter 275 in Nelson Textbook of Pediatrics. respiratory rate 22 breaths/min.5oC. Physical examination is notable for temperature 38.) Strongyloides stercoralis Toxocara canis . 332.Strongyloides stercoralis Explanation: This is a characteristic presentation of hyperinfection syndrome with Strongyloides stercoralis. which have now resolved. 333. 335.) Question . Which of the following schistosome species is unlikely to be the cause of his illness? Schistosoma mansoni Schistosoma japonicum Schistosoma intercalatum Schistosoma haematobium Explanation: Schistosoma haematobium is not endemic to the areas visited. She emigrated with her family from a rural area of China to the United States 4 wk ago. The most useful test for diagnosis of neurocysticercosis is: Serologic testing Stool examination . His kidneys and bladder are normal.Ascaris lumbricoides Question . Ultrasound examination of his abdomen shows periportal fibrosis consistent with schistosomiasis. Kampuchea. and Mali. (See Chapter 276 in Nelson Textbook of Pediatrics. 334. and periorbital edema. (See Chapter 277 in Nelson Textbook of Pediatrics. preferring the ureteral and bladder veins. myalgias. During the past 5 yr his family has lived in the Philippines. The most likely cause of the patient's condition is: Typhoid fever Trichuriasis Cysticercosis Visceral larva migrans Trichinosis Explanation: This is a characteristic presentation of trichinosis.) Schistosoma mekongi Question . 17th ed. and hepatosplenomegaly. Her 16-yr-old brother had similar symptoms. 17th ed. migratory arthralgias. A communal meal that included a locally butchered pig was part of the family's farewell party. A 13-yr-old girl presents with a 1-wk history of fever. caused by Trichinella spiralis. fever. Senegal. The 12-yr-old son of diplomat parents presents with crampy abdominal pain. It also does not affect the portal circulation. but may miss the cysts of cysticercosis.Rapid antigen testing Computed tomography (CT) Explanation: The most useful diagnostic test for neurocysticercosis is either computed tomography (CT) or magnetic resonance imaging (MRI). 336. were found to have cysticercosis. and he has poor coordination and intention. the boy's headache is noted to increase on standing. In S o Paulo. The four other household members have normal findings on imaging studies. MRI is the most sensitive test to determine if there are fresh or active lesions. The symptoms could be consistent with cerebellar and/or ventricular cysts. 17th ed. The CT shows a single calcified lesion in the left hemisphere. On physical examination.) Polymerase chain reaction (PCR) testing Question . a head CT study was performed on a 9-yr-old boy for evaluation of headache following a fall. and never indicated if all of the lesions are calcified. solium infection (cysticercosis) comes from ingestion of eggs excreted by infected humans and cannot be directly related to pigs. which typically reveals a solitary cyst with or without contrast enhancement. To establish the source of this infection and the risk to others in the community. 337. (See Chapter 280 in Nelson Textbook of Pediatrics. 17th ed. (See Chapter 280 in Nelson Textbook of Pediatrics. Ventricular cysts may require surgical intervention. Brazil. the single best investigation would be to: Test all pork in the household for cysts Explanation: CT will show calcification. A mother and son from a Midwestern community. 17th ed. The parasite is often present in the central nervous system while absent from the gastrointestinal tract. Chemotherapy is usually not indicated in uncomplicated disease with a solitary lesion.) . who have never traveled outside of the United States or Canada.) Measure Toxoplasma titers Question . The most appropriate next step in management would be to: Perform MRI Perform brain biopsy Order stool examination for ova and parasites Administer albendazole Explanation: The cyst stage of T. (See Chapter 280 in Nelson Textbook of Pediatrics. 17th ed. 17th ed. The other choices mentioned are not relevant to cystic liver disease. and liver enzyme values are normal. Ultrasonography of the liver shows one cyst approximately 3 cm in diameter. She has lived her whole life in Salt Lake City.Ask about visits to county fairs or employment on pork farms Check pork in the grocery and meat stores frequented by the family Examine the stool of anyone who has prepared meals for the family.) .g. has never traveled to areas endemic for hydatid transmission (e.) Negative echinococcal serology Negative hepatitis serology Normal liver enzymes Normal menstrual periods Question . areas where there are moose). On physical examination. The sensitivity of an echinococcal serologic study ranges from 60% to 90%. The most important factor in allowing her to play this sport is: Her travel history Explanation: A patient who has only been in an urban area and has not participated in agricultural activities.. Except for Fasciola hepatica. (See Chapter 281 in Nelson Textbook of Pediatrics. 338. in North America. (See Chapter 281 in Nelson Textbook of Pediatrics. Praziquantel is effective in the treatment of all of the following infections except: Fish tapeworm infection Hydatid cysts Explanation: Hydatid disease (echinococcosis) does not respond to praziquantel. all other trematode and cestode infections (flukes and tapeworms) will respond to praziquantel. and has not had contact with sheep or dogs is unlikely to acquire echinococcal infection. and has never traveled abroad. Healthy hydatid cysts may not stimulate much of an immune response. 339. probably due to failure to penetrate the liver cyst. Results on serologic testing for Echinococcus and hepatitis are negative. Utah. Check the stool of the family dog Question . Her menstruation is normal. her liver edge is palpable and seems minimally enlarged. A 15-yr-old girl wishes to play soccer in school and needs medical clearance. ) Manufacturer Lot number Name of the person administering the vaccine Question .) Lyme disease vaccines Question . with the exception of DTaP vaccines. (See Chapter 282 in Nelson Textbook of Pediatrics. 340. and name of the health care provider administering the vaccine. for vaccines mandated in childhood. 341. Diphtheria and tetanus toxoids and acellular pertussis vaccine (DTaP) should be administered to healthy children at which ages? . manufacturer. 342. Vaccines against the same infectious agent but produced by different manufacturers are considered interchangeable with the exception of: Haemophilus influenzae type b vaccines Hepatitis A vaccines Hepatitis B vaccines Acellular pertussis vaccines (as DTaP) Explanation: Vaccines made by different manufacturers but directed against the same infections are generally considered interchangeable. 17th ed. lot number.Cysticercosis Schistosomiasis Hymenolepis nana infection Question . 17th ed. health care providers document the date of administration. (See Chapter 282 in Nelson Textbook of Pediatrics. All of the following information about vaccination is required to be documented in the patient's medical record except: Date of administration Site of administration Explanation: The National Childhood Vaccine Injury Act requires that. and 15-18 mo. 343. (See Chapter 282 in Nelson Textbook of Pediatrics. should be vaccinated at the same chronological age as for full-term infants and according to the routine childhood immunization . 344. and pertussis. Which of the following is a contraindication to administration of MMR? History of anaphylactic reaction to eggs Serious allergic reaction to gelatin Explanation: Measles and mumps vaccines contain insignificant amounts of egg proteins. 17th ed. 4-6 yr. and every 5 yr thereafter 2. 4. 6. and persons with hypersensitivity to eggs are at negligible risk of anaphylactic reactions from these vaccines.2 and 4 mo and 1-6 yr 2. 4. and every 10 yr thereafter Question . and 12 mo and 10-12 yr 2. 4. 17th ed. 6. 6. 4-6 yr. with a booster at 4-6 yr.) Otitis media with fever of 101°F Febrile convulsion following prior DTaP immunization History of thrombocytopenia Question . and 15-18 mo. with the following exception: DTaP Pneumococcal conjugate Hepatitis B Explanation: Preterm infants. and 4-6 yr Explanation: DTaP is the preferred vaccine for diphtheria. including those of very low birthweight. 4. 6. as indicated. Preterm. It is recommended for all children at 2. (See Chapter 282 in Nelson Textbook of Pediatrics. Children younger than 7 yr of age and adults should receive only DT or dT vaccine. low birthweight infants should be vaccinated at the same chronological age as for full-term infants and in accordance with the routine childhood immunization schedule.) 2. and 15-18 mo. Hypersensitivity reactions to MMR that were previously attributed to egg protein may have been caused by allergic reactions to the gelatin stabilizer in the vaccine. 6. and 15-18 mo. 4. tetanus. 17th ed. The only absolute contraindication to subsequent administrations of acellular pertussis vaccine is: History of anaphylaxis to egg or egg proteins Collapse or shock-like state within 2 days of pertussis vaccination Persistent. 346. (See Chapter 282 in Nelson Textbook of Pediatrics. or an antibiotic. such as egg proteins.e. 17th ed. One exception to this recommendation is hepatitis B vaccination of those born of HBsAg-negative mothers with low birthweights (i.schedule. gelatin. For which of the following infections is an immune globulin product available that is a monoclonal antibody? Tetanus Hepatitis B Varicella Respiratory syncytial virus Explanation: Palivizumab is a monoclonal antibody against RSV that is recommended for protecting high-risk children against serious complications from RSV disease..) Convulsions with or without fever within 3 days of pertussis vaccination .) Hepatitis A Question . Initiation of vaccination in this case should be delayed until the infant is 1 mo of age. (See Chapter 282 in Nelson Textbook of Pediatrics. inconsolable crying lasting 3 hr or longer within 2 days of pertussis vaccination An anaphylactic reaction to a previous dose of whole-cell pertussis vaccine Explanation: Generic contraindications to vaccination are an anaphylactic reaction to a previous dose of the same vaccine and an anaphylactic reaction to a vaccine constituent. 345. (See Chapter 282 in Nelson Textbook of Pediatrics. 17th ed. <2 kg).) IPV Haemophilus influenzae conjugate combination vaccine Question . 17th ed. (See Chapter 282 in Nelson Textbook of Pediatrics. and 15-18 mo. 4. (See Chapter 283 in Nelson Textbook of Pediatrics.) Antibiotic therapy . Which of the following is associated with development of autism? Measles vaccination Mumps vaccination Rubella vaccination Combined measles. 4. 348. mumps. and 15-18 mo. and every 10 yr thereafter Question . 4-6 yr. 6. 17th ed. Polio vaccine (IPV) should be administered to healthy children at which ages? 2. as inactivated polio vaccine (IPV). 349. and 6-18 mo and 4-6 yr of age. 4. 6.) Question . 4. 17th ed.) 2. 347. and 15-18 mo and 10-12 yr 2 and 4 mo and 4-6 yr 2. 4. (See Chapter 282 and Table 282-1 in Nelson Textbook of Pediatrics. should be administered at 2. All of the following are risk factors for hospital-acquired infection except: Surgical procedures Foley catheters Intravascular catheters Immunizations Explanation: Immunizations are not a risk factor for hospitalacquired infections.Question . and rubella vaccination None of the above Explanation: There is no association between vaccination and autism. and every 10 yr thereafter 2. 6. and 6-18 mo and 4-6 yr Explanation: Polio vaccine. (See Chapter 284 in Nelson Textbook of Pediatrics. but by far the most common and important route is via the hands. 351. Likely etiologic agents that you should consider include all of the following except: Cryptosporidium parvum Giardia lamblia Rotavirus Ascaris lumbricoides Explanation: Ascaris is not a known cause of daycare-associated diarrheal epidemics.) Calicivirus Question .) Giardia lamblia infection Pharyngitis .) Needle stick Small airborne droplet Food Water Question . 17th ed. You are asked to investigate an outbreak among children with profuse watery diarrhea in a daycare center. All of the following infections are readily transmissible among children attending child daycare except: Cytomegalovirus infection Rhinoviral infections Human immunodeficiency virus (HIV) infection Explanation: No cases of HIV transmission in out-of-home daycare have been reported. (See Chapter 283 in Nelson Textbook of Pediatrics. but choices A-C are. 350.Question . 352. 17th ed. The most common route of transmission of nosocomial infection is: Hands Explanation: Transmission of infectious agents occurs by various routes. 17th ed. (See Chapter 284 in Nelson Textbook of Pediatrics. . eczema. Features of tyrosinemia type I include all of the following except: Hepatic carcinoma Coagulopathy Acute peripheral neuropathy Hepatomegaly Congenital heart disease Explanation: Tyrosinemia often presents acutely with signs of overwhelming hepatic failure. The musty-mousy odor is phenylacetate.) Question . and a musty odor. Long-term results of hepatic disease may result in cirrhosis and hepatic carcinoma (which is . 1. 2. a metabolic by-product of the blocked pathway due to deficiency of phenylalanine hydroxylase. A blonde. (See Chapter 74 in Nelson Textbook of Pediatrics. 17th edition.) Biotinidase deficiency Question . 3. (See Chapter 74 in Nelson Textbook of Pediatrics. 17th edition. Fair complexion. microcephaly. who is at risk for congenital heart disease.Question . and developmental delay.5 mo of age with severe emesis to rule out pyloric stenosis. The baby will also be a carrier. If a woman with poorly controlled PKU becomes pregnant. emesis. and mental retardation are common features. The child described in Question 1 survives to adulthood and becomes pregnant. blue-eyed girl was admitted at 2. eczema. The most likely diagnosis is: Wiskott-Aldrich syndrome Galactosemia Cystinosis Phenylketonuria Explanation: This is a classic picture of phenylketonuria (PKU). her high phenylalanine levels will be reflected in the fetus. She had poor development. Risk to her fetus if her diet is poorly controlled include all of the following except: Microcephaly Congenital heart disease Mental retardation Cataracts Explanation: Adults who have the autosomal recessive disease PKU are at risk for mild to moderate neurointellectual problems if they are on an unrestricted diet. ketosis. Laboratory data reveal metabolic acidosis. and flaccidity. Laboratory data reveal hypoglycemia. hypotonia. 4. (See Chapter 74 in Nelson Textbook of Pediatrics. due to a deficiency of cystathionine synthase activity. 6.) Phenylketonuria Homocystinuria Galactosemia Question . opisthotonos posture. (See Chapter 74 in Nelson Textbook of Pediatrics. Plasma levels of leucine. A 6-wk-old girl has had feeding problems. metabolic acidosis. isoleucine. The most likely diagnosis is: Acrodermatitis enteropathica Multiple carboxylase deficiency Explanation: Multiple carboxylase deficiency in its infantile or early form is an autosomal recessive disorder and presents with these features in addition to tachypnea or apnea. and emesis for 3 wk. Physical examination reveals muscle rigidity. 17th edition.) Scoliosis Question . and valine are elevated. and cerebral edema.seen in [tilde ] 40% of cases). resembles Marfan syndrome. 17th edition. and hyperammonemia. The rash helps to distinguish this from other organic acidemias. periods of hypertonicity. . 5. (See Chapter 74 in Nelson Textbook of Pediatrics. Peritoneal dialysis to remove the excessive metabolites of branch-chain amino acids and means to reverse catabolism are priorities in treating a metabolic crisis in MSUD.) Question . During the last week she developed an erythematous exfoliative generalized rash and partial alopecia. Failure to thrive and lethargy or coma may also develop with time. Treatment with vitamin B6 is helpful in the subgroup of patients who are responsive. A previously healthy 2-wk-old now has progressive lethargy. Classic homocystinuria is characterized by all of the following except: Mental retardation Iridodonesis Ectopia lentis Short arms Explanation: This autosomal recessive disorder. 17th edition. The most likely diagnosis is: Hartnup disease Maple syrup urine disease Explanation: Maple syrup urine disease (MSUD) in the severe form presents in the first few weeks of life and is often lethal. The most likely diagnosis is: Methylmalonic acidemia Carbamoylphosphate synthase deficiency Ornithine transcarbamylase deficiency Explanation: Ornithine transcarbamylase (OTC) deficiency is an X-linked disorder and is most severe in males. Her head is enlarged. A 2-day-old boy manifests poor feeding. 8. (See Chapter 74 in Nelson Textbook of Pediatrics. The treatment of the patient in Question 6 is best accomplished by administration of: Biotin Explanation: Treatment with biotin will result in rapid improvement. 17th edition. 9. and in older children. infants with a severe neonatal onset have a very poor prognosis. Which of the following diseases would you consider in the diagnosis? Birth asphyxia .) Phenylketonuria Pellagra Vitamin B1 deficiency Question . An 8-mo-old girl is brought to your office with the complaint that she has developmental delay. 17th edition.) Vitamin B1 Vitamin B6 Vitamin B12 Ascorbic acid Question . Biotin is an important cofactor for the fourcarboxylase enzymes in this disorder. and lethargy leading to coma. vomiting. (See Chapter 74 in Nelson Textbook of Pediatrics.(See Chapter 74 in Nelson Textbook of Pediatrics. and her head circumference is above the 95th percentile. OTC deficiency may mimic Reye syndrome. The urine orotic acid level is also elevated.) Galactosemia Reye syndrome Question . She has considerable head lag. and she feeds poorly and does not focus. 17th edition. You examine the baby and find her hypotonic and unable to sit. Superficially it looks like Reye syndrome. Laboratory data reveal respiratory alkalosis and hyperammonemia. 7. Unfortunately. and an MRI shows diffuse white matter disease.Familial hydrocephalus Rubella infection in utero Canavan disease Explanation: Answers B. seen most often in Ashkenazi Jews. Medium-chain acyl-coenzyme A (CoA) dehydrogenase deficiency is characterized by all of the following except: Autosomal recessive inheritance Hypoketotic hypoglycemia Cardiomyopathy Explanation: Muscle weakness or cardiomyopathy is not seen in medium-chain acyl-CoA deficiency (MCAD). a patient is brought to you with increased spasticity. This lethal disease has no specific therapy. and E are possibilities. above the 95th percentile. D. 17th edition. (See Chapter 74 in Nelson Textbook of Pediatrics.) Northern European ancestry Episodes associated with fasting . Which of the following tests would be most likely to be relevant in this case? Urinary glycosaminoglycans measurement Blood phenylalanine levels N-acetylaspartic acid (NAA) assay Explanation: Canavan disease produces severe white matter disease as noted on the MRI. 10. The patient has a large head circumference. (See Chapter 74 in Nelson Textbook of Pediatrics. This disorder is the most common disorder of fatty acid oxidation defects and is best treated by avoiding prolonged periods of fasting or catabolic states. The poor vision is due to optic atrophy in this autosomal recessive disorder.) Thyroid profile Biotinidase activity in the blood Question . which was not present at birth or at 6 mo of age. 17 edition. 17th edition. N-Acetylaspartic acid is present in large amounts in the blood and urine and in the CNS. 11.) Hurler disease Question . (See Chapter 75 in Nelson Textbook of th Pediatrics. but the presence of visual problems and a large head suggest Canavan disease. At the age of 1 yr. If her fetus is male. prenatal testing should be performed on cultured amniocytes or cultured chorionic villus cells. Therefore. A 7-mo-old boy manifests muscle weakness and cardiomyopathy in addition to hypoketotic hypoglycemia with fasting. An older female sibling also has periods of muscle pain and rhabdomyolysis. The sister of a 40-yr-old man with adrenomyeloneuropathy seeks genetic counseling in regard to her risk of having a son with the rapidly progressive and fatal form of childhood cerebral adrenoleukodystrophy. 17th edition. Mutation analysis should be performed on blood samples from the sister and the affected brother. This can only be done by analysis and comparison of the brother's specific mutation with the sister's gene. (See Chapter 75 in Nelson Textbook of Pediatrics. the sister and brother should be evaluated to determine if the sister carries the family's mutation. He is married and manages a small business successfully. The advice is: The risk of her having a severely affected son is low because her plasma VLCFA levels are normal. which does not have associated skeletal or cardiac muscle involvement. Short-chain acyl-CoA deficiency (SCAD) does not manifest hypoketotic hypoglycemia. Prenatal studies are not required if VLCFA levels in fibroblasts are also normal.2 in Nelson Textbook Question . Prenatal studies are not required if she does not carry the pathogenic mutation demonstrated in her brother. and the phenotype in her affected brother is mild. 13. 12. (See Chapter 75. The affected brother has slowly progressive moderately severe spastic paraparesis. It is more severe than MCAD. The most likely diagnosis is: Medium-chain acyl-CoA dehydrogenase deficiency Long-chain acyl-CoA dehydrogenase deficiency Carnitine deficiency Long-chain-very-long-chain acyl-CoA dehydrogenase deficiency Explanation: Long-chain acyl-CoA deficiency (LCAD)-verylong-chain acyl-CoA deficiency (VLCAD) is one disease.Question . The VLCFA levels should be tested in the sister's cultured skin fibroblasts. The levels of very-long-chain fatty acids (VLCFAs) in the sister's plasma are normal. which was originally described as LCAD only. Mutation analysis should be performed on the sister's blood. this is not reassuring. 14.) Short-chain acyl-CoA dehydrogenase deficiency Question . A 43-yr-old woman with pigmentary degeneration of the retina and . Explanation: Because the levels of very-long-chain fatty acids (VLCFAs) may be normal in some carriers. Both have a urinary organic acid profile with hypoketotic dicarboxylic aciduria. Prenatal testing is not required if a pathogenic mutation is not demonstrated. Prenatal studies are not required. Plasma VLCFA levels are moderately elevated. The paternal grandmother has a high cholesterol level and had a heart attack at the age of 48 yr.) Unknown Dysbetalipoproteinemia Question . triglyceride. How do you respond? The statin medications are not approved for use in potentially childbearing women The statin medications are not approved for use in children No medication should be tried until the response to dietary modification is assessed . 242 mg/dL. and total cholesterol.) Peroxisomal bifunctional enzyme deficiency Question . MRI shows symmetric demyelination in the parieto-occipital regions. 17th edition. His sister has an elevated triglyceride level. (See Chapter 75. 50 mg/dL.) The most likely diagnosis for this child is: Familial hypercholesterolemia Polygenic hyperlipidemia Familial combined hyperlipidemia Explanation: The family demonstrates significant elevations in cholesterol and triglycerides and a relatively benign childhood course. (Note: In this case. 41 mg/dL. elevated levels refer to levels greater than 95th percentile. HDL-C. He takes one of the so-called statin medications (HMG-CoA reductase inhibitor). (See Chapter 75. but early coronary artery disease (<50 yr of age). In her early 40s she has developed a dementing illness. A 10-yr-old boy is found to have an elevated low-density lipoprotein cholesterol (LDL-C) level. and wants his daughter to be given the same medication.2 in Nelson Textbook of Pediatrics. 16. and 84% of catalase activity in cultured skin fibroblasts is located in the cytosolic fraction. A 15-yr-old girl is found to have an LDL-C level of 191 mg/dL. The father has elevated LDL-C and triglyceride levels. 17th edition.impaired hearing since childhood has otherwise developed normally. The child's father had a heart attack at age 45 yr. 15.3 in Nelson Textbook of Pediatrics. The diagnosis is: Manifesting heterozygote for X-linked adrenoleukodystrophy Classical Refsum disease Usher syndrome Late-onset peroxisome biogenesis disorder Explanation: This is a classic appearance of a late-onset disorder. She had good athletic skills in adolescence and young adulthood and worked as a physical therapy assistant. Most others present either in the neonatal period or later during childhood. Explanation: Although cholesterol levels are elevated, the recommended first approach to the teenaged patient is to begin with dietary restrictions of cholesterol and fat. (See Chapter 75.3 in Nelson Textbook of Pediatrics, 17th edition.) Medication should be prescribed right away, because the response to diet modification is likely to be limited in this case Niacin is the treatment of choice Question . 17. Dietary fat should be restricted to 20-30% of calories: In all persons except children younger than 2 yr, as this age group needs a higher-fat diet to support rapid growth Explanation: Fat is an essential source of calories, essential fatty acids, and cholesterol, which are all needed in the rapidly growing toddler. Parents who restrict fat by using skim milk in children <2 yr have caused failure to thrive in their children. (See Chapter 75.3 in Nelson Textbook of Pediatrics, 17th edition.) In all persons except children younger than 4 yr, as this age group needs a high fat intake to support the rapid growth of the brain that occurs in the first 4 yr of life In all persons except children younger than 15 yr, as a higher fat intake is needed to support normal pubertal development Only for individuals with hyperlipidemia Only for individuals with hypertriglyceridemia Question . 18. A 5-yr-old child has a paternal grandmother who had a heart attack at the age of 45 yr. The child's father has hypercholesterolemia and takes medication to lower his cholesterol level. How should the child be screened? Non-fasting total cholesterol, as the total cholesterol level does not vary significantly between fasting and non-fasting states Fasting total cholesterol, as the cholesterol level varies significantly after eating Fasting total cholesterol, triglyceride, HDL-C, and calculated LDL-C to evaluate for elevated cholesterol or triglycerides, or low HDL-C levels Explanation: These laboratory tests are the recommended approach to evaluate this 5-yr-old at risk for familial hypercholesterolemia. (See Chapter 75.3 in Nelson Textbook of Pediatrics, 17th edition.) Should not be screened until older than 10 yr Should be screened only if the child has other risk factors for cardiovascular disease Question . 19. GM1 gangliosidosis is characterized by all of the following except: Psychomotor retardation Angiokeratomata Hepatosplenomegaly Frontal bossing Peripheral neuropathy Explanation: This lethal lipidosis disorder is a progressive neurovisceral storage disease that has some features similar to those of some severe mucopolysaccharidoses. In addition to dysmorphic features (coarsened facies), deafness, blindness, seizures, and hepatosplenomegaly occur. (See Chapters 74.4 and 74.5 in Nelson Textbook of Pediatrics, 17th edition.) Question . 20. A 4-mo-old girl presents with developmental delay, an exaggerated startle response to loud noise, and macrocephaly. On physical examination, the child has decreased eye contact and a cherry red spot in each retina. The most likely diagnosis is: GM2 gangliosidosis Explanation: Also known as Tay-Sachs disease, this lethal autosomal recessive lipidosis manifests with progressive neurologic deterioration. There is no treatment, but prenatal diagnosis is possible. (See Chapters 74.4 and 74.5 in Nelson Textbook of Pediatrics, 17th edition.) Gaucher disease Fabry disease Galactosemia Glycogen storage disease, type I Question . 21. A 15-yr-old presents with chronic fatigue and severe bone pain of 1 year's duration. He has hepatosplenomegaly and a normal retinal examination. Laboratory studies reveal normocytic anemia and thrombocytopenia. Radiographs of the distal femur reveal Erlenmeyer flask deformities. The most likely diagnosis is: Lymphoma Gaucher disease Explanation: This is a classic presentation for type I Gaucher disease, a lipidosis most commonly seen in Ashkenazi Jews. The central nervous system is not involved, but bone pain and hematologic problems can be severe. Treatment is effective with recombinant enzyme replacement infusions and is quite successful. (See Chapters 74.4 and 74.5 in Nelson Textbook of Pediatrics, 17th edition.) Sandhoff disease GM3 gangliosidosis Sickle cell anemia Question . 22. Fabry disease is characterized by all of the following except: Acroparesthesias Corneal opacities Cardiomyopathy Mental retardation Explanation: This X-linked disorder causes severe debilitating pain but no mental retardation. Treatment is effective with recombinant enzyme replacement therapy. (See Chapters 74.4 and 74.5 in Nelson Textbook of Pediatrics, 17th edition.) Angiokeratomas Question . 23. A 14-mo-old girl presents with irritability, poor walking, genu recurvatum, and hypotonia. Physical examination reveals absent deep tendon reflexes. Laboratory data reveal an increased CSF protein level and decreased nerve conduction velocities. The most likely diagnosis is: Metachromatic leukodystrophy Explanation: This is the classic preservation of metachromatic leukodystrophy (MLD), an autosomal recessive disorder with no treatment. The patient's condition deteriorates progressively. Metachromatic granules may be seen in the urine as well as sural nerve biopsies. (See Chapters 74.4 and 74.5 in Nelson Textbook of Pediatrics, 17th edition.) I cell disease GM3 gangliosidosis Fabry disease Neuromyopathic Gaucher disease Question . 24. A 3-mo-old previously healthy and well infant presents to an emergency room with a generalized seizure. The physical examination shows a temperature of 36.5°C, respiration rate of 50/min, and marked hepatomegaly, with the liver edge palpable down to the umbilicus. Laboratory findings show Na, 141 mEq/L; Cl, 95 mEq/L; K, 4.5 mEq/L; BUN, 12 mg/dL; Cr, 0.2 mg/dL; glucose, 25 mg/dL; CO2, 12 mEq/L; uric acid, 7.1 mg/dL; cholesterol, 270 mg/dL; and triglyceride, 550 mg/dL. The most likely diagnosis is: Hyperinsulinemia Adrenal insufficiency Growth hormone deficiency Glycogen storage disease Explanation: The child most likely has glycogen storage disease, type I, also called glucose-6-phosphatase deficiency. During fasting from a viral illness or by sleeping longer between feeds, the metabolic defect becomes manifest as an inability to release glucose from glycogen. Hypoglycemia, lactic acidosis, ketosis, and hyperlipidemia are noted, in addition to high uric acid levels. (See Chapter 76 in Nelson Textbook of Pediatrics, 17th edition.) Mitochondrial disorder Question . 25. A 6-day-old breast-fed infant has had poor feeding and vomiting since birth. On physical examination, he is jaundiced, has hepatomegaly, and shows excessive bruising. Laboratory investigations show elevated transaminase levels, prolonged clotting time, and aminoaciduria. Galactosemia is suspected; however, urine-reducing substance is negative. What is the most appropriate next step in diagnosis? Oral galactose tolerance test Challenge with lactose-containing formula Blood analysis to measure galactose-1-phosphate uridyltransferase enzyme activity Explanation: Because of perceived feeding intolerances, the child may have been fed with a non-lactose containing formula (soy type), or with fasting due to the metabolic decompensation, the intake of lactose may have been insufficient to demonstrate a reducing substance-positive urine. Measuring the enzyme is the most reliable test to diagnose galactosemia under any conditions, unless the patient has received a blood transfusion. (See Chapter 76 in Nelson Textbook of Pediatrics, 17th edition.) Liver biopsy Kidney biopsy Question . 26. A 17-yr-old boy has a history of recurrent episodes of exercise intolerance with muscle cramps. He is otherwise healthy and well. Which of the following is most likely to develop during these episodes in this patient? Myoglobinuria Explanation: The history is compatible with muscle glycogenosis. With decreased ability to release glucose from muscle glycogen, myocyte injury occurs and places the patient at risk for myoglobinuria and renal failure. (See Chapter 76 in Nelson Textbook of Pediatrics, 17th edition.) Abnormal liver function Aminoaciduria Cardiac arrhythmia Metabolic acidosis Question . 27. A patient with recurrent hypoglycemia and hepatomegaly is diagnosed to have type I glycogen storage disease. Which of the following is unlikely to raise the blood glucose levels during the hypoglycemia episode? IV glucose Oral uncooked cornstarch Oral Polycose Oral Pedialyte Oral fructose Explanation: Fructose must first be converted to glucose within the hepatocyte. Under normal circumstances, the intermediary metabolite, glucose-6-phosphate, will then be released to the circulation after enzymatic conversion to glucose by hepatic glucose-6-phosphatase. This enzyme is deficient or missing in this form of glycogen storage disease. (See Chapter 76 in Nelson Question . 28. A 5-mo-old infant with gross motor delay, hypotonia, and cardiomegaly has a creatine kinase level of 860 IU/L. Of the following, which is the most likely diagnosis? Spinal muscular atrophy Hypothyroidism Prader-Willi syndrome Pompe disease Explanation: Pompe disease is a glycogen storage disease that affects the heart and skeletal muscle. Cardiac involvement causes heart failure and a shortened P-R interval on the ECG. Treatment with recombinant enzyme replacement has shown promise in the management of Pompe disease. (See Chapter 76 in Nelson Textbook of Pediatrics, 17th edition.) Down syndrome Question . 29. Hurler disease is characterized by all of the following except: Hepatosplenomegaly Coarse facies Dysostosis multiplex X-linked inheritance Explanation: All the mucopolysaccharidoses are inherited as autosomal recessive disorders except Hunter syndrome (MPS II), which is inherited as an X-linked trait. (See Chapter 77 in Nelson Textbook of Pediatrics, 17th edition.) Corneal clouding Question . 30. Potential cardiac problems in patients with mucopolysaccharidoses include all of the following except: Coronary artery disease Mitral regurgitation Pericarditis Explanation: Storage in endothelial cells can cause coronary artery disease and even myocardial infarction. Cardiomyopathy may be present in infancy and is associated with endocardial fibroelastosis. Valve disease necessitating valve replacement usually occurs in older children. (See Chapter 77 in Nelson Textbook of Pediatrics, 17th edition.) Aortic valve disease Cardiomyopathy Endocarditis Question . 31. A 3-yr-old boy was normal at birth but developed progressive coarsening of the face and developmental delay. In addition to coarse facies and mental retardation, the physical examination reveals a cardiac systolic regurgitant murmur, hepatomegaly, joint stiffness, and short stature. CT scan reveals hydrocephalus. There is no corneal clouding. All affected family members have been boys. The most likely diagnosis is: Hurler syndrome Galactosemia Metachromatic leukodystrophy Hunter syndrome Explanation: This X-linked disorder affects only males, who do not reproduce. The phenotype of severe Hunter syndrome (MPS II) is similar to that of Hurler syndrome (MPS I), but it does not include corneal clouding and it progresses at a slightly slower rate. (See Chapter 77 in Nelson Textbook of Pediatrics, 17th edition.) Mucopolysaccharidosis type III (Sanfilippo syndrome) Question . 32. A 14-mo-old white boy presents with chorea of 2 months' duration. Prior to this, he had failure to thrive and recurrent emesis. He was normal until age 4 mo, when hypotonia and vomiting began. Physical examination reveals hyperreflexia and erosions on his lips and fingers. Laboratory data reveals a normal BUN and ammonia levels and an elevated uric acid level. An older male sibling is in an institution for severely disabled teens. The most likely diagnosis is: Glycogen storage disease, type VI Mucopolysaccharidosis, type XII GM1 gangliosidosis Familial X-linked gout Lesch-Nyhan syndrome Explanation: This is a classic presentation of this X-linked genetic disorder, which is characterized by complete deficiency of the hypoxanthine-guanine phosphoribosyltransferase enzyme. Partial enzyme deficiency may cause some cases of severe gout with normal cerebral function. Treatment with allopurinol reduces the hyperuricemia but does not alter the CNS manifestations Question . 33. A 4-yr-old girl presents with severe failure to thrive with weight of 18 lb and height of 30 inches, a high-pitched voice, a sclerodermatous appearance to skin overlying the abdomen, nearly total alopecia, a small jaw and face relative to the head circumference, and normal mental and motor development. The most likely diagnosis is: Cystic fibrosis Child abuse and malnutrition Hutchinson-Gilford progeria syndrome Explanation: This disorder of unknown etiology is associated with rapidly progressive characteristics of aging, with normal motor and mental development. (See Chapter 79 in Nelson Textbook of Pediatrics, 17th edition.) Marfan syndrome. Hallermann-Streiff syndrome Question . 34. An 18-yr-old girl complains of abdominal pain with menses that seem out of proportion to the menstrual pain of her peers. She has intermittent constipation and diarrhea. During a recent episode of pain, she was noted to have hypertension, tachycardia, and urinary retention. Her urine was noted to have a port wine discoloration when the urinary catheter was placed to relieve her acute urinary retention. The most likely diagnosis is: Acute intermittent porphyria Explanation: Acute intermittent porphyria is one of the most common of the porphyrias and presents with pain and neurologic symptoms. It is rare before puberty and is exacerbated by menstruation and other precipitating factors such as drugs, other illness, and stress. (See Chapter 80 in Nelson Textbook of Pediatrics, 17th edition.) Congenital erythropoietic porphyria Porphyria cutanea tarda Mucopolysaccharidosis System lupus erythematosus Question . 35. A 3-day-old white girl was found unresponsive in the normal newborn nursery. The mother was a 35-yr-old (gravida 3, para 2) with a normal pregnancy, including screening tests negative for herpes simplex, group B streptococci, HIV, diabetes (normal glucose tolerance test), and -fetoprotein. Her other children are normal. The baby weighed 4.5 kg at birth and had Apgar scores of 9 and 9. There is no hepatosplenomegaly or respiratory distress, but the baby has a weak cry, is hypotonic, and responds poorly to stimuli. Immediate evaluation should include all of the following except: Blood magnesium level Explanation: Hypermagnesemia, usually associated with maternal magnesium sulfate therapy for preeclampsia, is usually most severe on the first day of life. Hypomagnesemia presents like hypocalcemia with neuromuscular irritability, hyperreflexia, and clonus. (See Chapter 81 in Nelson Textbook of Pediatrics, 17th edition.) Blood glucose level Blood culture Arterial blood gas analysis Blood electrolytes Question . 36. The infant described in Question 35 has a blood glucose level of 22 mg/dL. The most appropriate therapy is: Intravenous glucose infusion at 2-4 mg/kg/min Oral administration of 5% dextrose in water (D5W) Intravenous glucose bolus of D10W at 2 mL/kg Explanation: A mini-bolus of glucose is indicated in all hypoglycemic infants with symptoms. (See Chapter 81 in Nelson Textbook of Pediatrics, 17th edition.) Administration of prednisone Administration of diazoxide Question . 37. The infant described in Questions 35 and 36 initially becomes more responsive and has normal tone. Three hours later she has a seizure, and her blood glucose level is 18 mg/dL. The most appropriate next step in management is to: Begin prednisone administration Repeat a mini-bolus of D10W Give a mini-bolus of D10W plus continuous intravenous glucose infusions Explanation: The first mini-bolus corrected the hypoglycemia but was not followed by any exogenous source of glucose, and hypoglycemia recurred. (See Chapter 81 in Nelson Textbook of Pediatrics, 17th edition.) Give continuous intravenous glucose infusion Administer diazoxide Question . 38. The infused glucose concentrations in the patient describedin Question 35 have to be increased to 25% glucose, given by a central venous catheter to maintain blood glucose levels of just 45 mg/dL. The most likely diagnosis is: Galactosemia Glycogen storage disease Hyperinsulinemic hypoglycemia Explanation: This is a classic case of persistent hyperinsulinemic hypoglycemia, which was previously called nesidioblastosis. Defects in the regulatory KATP channel, which stimulates insulin release from the pancreatic islet cells, is the most common of these disorders and may be familial or sporadic. Treatment with diazoxide or octreotide may be successful; some patients require pancreatic resection. (See Chapter 81 in Nelson Textbook of Pediatrics, 17th edition.) Infant of diabetic mother Congenital herpes simplex infection Question . 39. A 9-day-old full-term infant is admitted to the hospital with lethargy, fever, and increasing jaundice. Physical examination also reveals hepatomegaly. Laboratory results reveal a blood glucose value of 10 mg/dL, total and direct bilirubin values of 15 and 7 mg/dL, respectively, and liver enzyme test shows AST??700 units/L, and ALT, 650 units/L. The next day, the blood culture is positive for a gramnegative rod. The most likely diagnosis is: Necrotizing enterocolitis Galactosemia Explanation: The combination of hypoglycemia, jaundice, elevated liver enzyme values, and Escherichia coli sepsis is classic for early-onset severe galactosemia. A white blood cell defect may predispose to E. coli sepsis, and the toxic effects of galactose-1-phosphate explain the hepatotoxicity. In this child, who was fed cow's milk-based formula, the urine-reducing substances (galactose) were positive. (See Chapter 76.2 in Nelson Textbook of Pediatrics, 17th edition.) Neonatal hepatitis Glycogen storage disease Biliary atresia Question . 40. A previously healthy 6-mo-old presents with hepatomegaly, lethargy, increasing jaundice, and severe emesis. The child appears dehydrated; the urine also has a positive reaction for reducing substances. The child's diet has been solely breast milk until age 5 mo, when fruit juices and baby food were added to the diet. The most likely diagnosis is: Galactosemia Gycogen storage disease Benign fructosuria Hereditary fructose intolerance Explanation: The acute or subacute presentation of hereditary fructose intolerance is very similar to that of galactosemia. The inability to metabolize fructose can produce shock, hypoglycemia, hepatic dysfunction, and emesis. In the child described in the question, the urine-reducing substance was fructose. Before the initiation of fructose (sucrose)-containing foods (e.g., fruit juices), such a child will be asymptomatic. (See Chapter 76.3 in Nelson Textbook of Pediatrics, 17th edition.) Pyruvate carboxylase deficiency Question . 41. A 2-mo-old presents with failure to thrive, emesis, alopecia, rash, and chronic metabolic acidosis. An older sibling died at age 3 mo with hypotonia and chronic lactic acidosis. An important diagnostic study is determination of: Blood galactose level Serum lactate dehydrogenase level Serum biotinidase level Explanation: Biotinidase levels may reflect enzymatic deficiencies affecting carbohydrate and amino acid (organic acid) metabolism. Both enzymatic pathways require biotinidase; deficiencies produce manifestations, as noted in this case. Treatment with oral biotin may overcome this defect in some affected patients. (See Chapter 74 in Nelson Textbook of Pediatrics, 17th edition.) Plasma triglyceride levels Serum calcium and magnesium levels Question . 42. A previously healthy 4-mo-old now manifests increasing hypotonia and poor feeding. Physical examination reveals macroglossia, a gallop rhythm, tachycardia, and marked flaccidity, but mental status is normal. Laboratory studies reveal a blood glucose level of 85 mg/dL and sinus tachycardia with a shortened P-R interval on an electrocardiogram. The most helpful diagnostic study would be: MRI of the spine Glucagon infusion test Muscle biopsy Explanation: Muscle biopsy may confirm the diagnosis of Pompe disease (glycogen storage disease, type II). Deficiency of acid -glucosidase results in marked lysosomal glycogen accumulation and primarily affects the heart and skeletal muscle. Death in the infantile form is due to respiratory muscle failure. (See Chapter 76.1 in Nelson Textbook of Pediatrics, 17th edition.) Lumbar puncture Question . 43. A 15-yr-old Ashkenazi Jewish girl is seen because of chronic fatigue. On examination, she seems pale and thin and has a somewhat large abdomen. Her spleen is felt in the iliac fossa. She is mentally alert and has a history of normal development and normal school performance. Her blood count shows hemoglobin, 9.0 g/dL and a white blood cell count of 3,000/cu mm, with normal differential cell count and no abnormal cells. Platelet count is 60,000/ L. The most likely diagnosis is: Tay-Sachs disease Niemann-Pick disease type A Gaucher disease type I Explanation: All five diseases listed in the question are prevalent among Ashkenazi Jews. Niemann-Pick disease, type A, and Gaucher disease, type I, are the only two diseases that present with a very large spleen. Children with Niemann-Pick, type A, disease are likely to be retarded, but this patient is not. Anemia, leukopenia, and thrombocytopenia are typically found in Gaucher disease. Patients with Canavan disease, Tay-Sachs disease, and mucolipidosis, type IV, are neurologically impaired, but this patient is not. (See Chapter 75 in Nelson Textbook of Pediatrics, 17th edition.) Mucolipidosis type IV Canavan disease Question . 44. A 13-mo-old infant is found comatose in bed after sleeping later than usual. On physical examination, the infant is afebrile and of normal size, and the liver is palpated 4 cm below the costal margin. The plasma glucose level is 15 mg/dL; bicarbonate, 20 mEq/L;BUN, 35 mg/dL; ammonia, 295 mol/L; AST, 320 units/L; and ALT, 425 units/L. Bilirubin is normal. Urinalysis is negative for glucose, ketones, protein, and reducing substances. Which of the following is the most likely diagnosis? Medium-chain acyl-CoA dehydrogenase deficiency Explanation: Nonketotic hypoglycemia, with or without hyperammonemia, in a patient of this age is often due to this relatively common cause. In addition to hypoglycemia, mediumchain acyl-CoA deficiency (MCAD) is associated with a Reyelike syndrome, and some cases are initially diagnosed as sudden infant death syndrome (SIDS). (See Chapter 73 in Nelson Textbook of Pediatrics, 17th edition.) Glucose-6-phosphatase deficiency (type I glycogenosis) Congenital hyperinsulinism Growth hormone deficiency Isovaleric acidemia Question . 45. Plasma very-long-chain fatty acids are elevated in all of the following peroxisomal disorders except: Zellweger syndrome Rhizomelic chondrodysplasia punctata Explanation: In rhizomelic chondrodysplasia punctata, plasma levels of phytanic acid and erythrocyte plasmalogens are elevated. (See Chapter 75.2 in Nelson Textbook of Pediatrics, 17th edition.) Pseudo-Zellweger syndrome Neonatal adrenoleukodystrophy Infantile Refsum disease Question . 46. Of the following physical findings noted in some patients with hyperlipidemia, which is most likely in a 17-yr-old boy with heterozygous familial hypercholesterolemia? Arcus corneae Xanthelasma Cutaneous xanthomas over the buttocks Tuberous xanthomas causing Achilles tendinitis Explanation: The Achilles tendon is a first site for the development of xanthomas, an area that most of us do not usually examine other than to test deep tendon reflexes. (See Chapter 75.2 in Nelson Textbook of Pediatrics, 17th edition.) Tuberous xanthomas over the elbow 49. A 7-mo-old white girl presents with severe developmental delay and episodes of vomiting. Physical examination reveals blue eyes.) Galactosemia Question . even during feedings. The infant also startles very easily when there is a loud noise in the house. (See Chapter 74 in Nelson Textbook of Pediatrics. The most likely diagnosis is: Child neglect TORCH infection VATER syndrome Phenylketonuria Explanation: Phenylketonuria presents with the gradual appearance of these symptoms. A 7-mo-old boy has been healthy and developing normally since birth. lethargy. Of the following. 48. and hyperactive deep tendon reflexes. His mother now reports that he has decreased eye contact with her.Question . There is failure to thrive and microcephaly. (See Chapter 75 in Nelson Textbook of Pediatrics. Overtreatment may result in: Obesity and striae A musty. Appropriate treatment of the child described in Question 49 before the onset of serious symptoms is helpful in preventing severe retardation. All children appear normal at birth. and rash Explanation: Appropriate treatment lowers the serum level of the essential amino acid phenylalanine. mousy odor Headache and pseudotumor cerebri Rectal prolapse and colitis Anorexia. the most appropriate diagnostic test to confirm the etiology of these findings is the measurement of: Leukocyte -hexosaminidase A activity Explanation: Tay-Sachs disease classically presents with these clinical features about this time of infancy.) Serum concentration of amino acids Serum concentration of ammonia Urinary mucopolysaccharides Urinary organic acids Question . A cherry red spot on the fundi would also be present. Too severe restriction . 17th edition. 17th edition. 47. light skin with an eczematoid rash. (See Chapter 74. Complications of tyrosinemia include all of the following except: Cataracts Explanation: Cataracts are unusual in tyrosinemia. The most likely explanation is: . and death. mild mental retardation. treatment is initiated with high-dose vitamin B6. The hypercoagulable state may even be present if homocysteine levels are elevated without any other signs of homocystinuria. they are common in galactosemia. 17th edition. Additional complications that may occur in the patient described in Question 51 include all of the following except: Seizures Thromboembolism Osteoporosis Scoliosis Cardiomyopathy Explanation: A through D are all correct. (See Chapter 74.3 in Nelson Textbook of Pediatrics.) Question . iridodonesis. and Marfan-like features (tall. but no response is observed. 17th edition. (See Chapter 74. anorexia.) Renal tubular acidosis Peripheral neuropathy Cirrhosis Hepatic carcinoma Question .3 in Nelson Textbook of Pediatrics. 51. ( Question . subluxation of the ocular lens. A 4-yr-old blue-eyed white girl manifests a malar flush.of this amino acid produces lethargy. 50. diarrhea. arachnodactyly).2 in Nelson Textbook of Pediatrics. rash.) Angelman syndrome Question . 52. 17th edition. In the patient described in Questions 51 and 52. thin. The most likely diagnosis is: Hawkinsinuria Alcaptonuria Piebaldism Homocystinuria Explanation: Homocystinuria is due to a deficiency of cystathionine synthase. 53. The most likely diagnosis is: . A 10-mo-old Ashkenazi Jewish girl manifests hypotonia and macrocephaly. 17th edition.9 mm Hg. and hypotonia. (See Chapter 74. she has hyperreflexia and optic atrophy. In addition to routine supportive care. MRI demonstrates diffuse white matter degeneration of the cerebral cortex. Laboratory findings reveal 4+ ketonuria. 6. Additional problems include failure to thrive and swallowing difficulties. 95 mm Hg. 17th edition. (See Chapter 74. even if the child survives the acute episode. and absolute neutrophil count. the patient responds to the primary therapy of high-dose vitamin B6.) Phenylketonuria Primary carnitine deficiency Question . 54.6 in Nelson Textbook of Pediatrics. (See Chapter 74. PCO2. 37 mEq/L. A 10-day-old child manifests profound coma 1 day after an illness characterized by emesis. what additional therapy would be most appropriate for the child described in Question 54? Vitamin B12 Explanation: Some cases of MMA are responsive to large doses (1-2 mg/24 hr) of vitamin B12.Folate deficiency Explanation: After high-dose folate treatment. By 18 mo of age.) Vitamin B1 Biotin Vitamin C Folate Question .6 in Nelson Textbook of Pediatrics. 17th edition. 55. PO2.3 in Nelson Textbook of Pediatrics. 19 mm Hg.) Malabsorption Gastric hypersecretion Vitamin B1 deficiency Vitamin C deficiency Question . anion gap. arterial blood gas. 56. poor oral intake. 400/cu mm. The most likely diagnosis is: Galactosemia Glycogen storage disease type II Methylmalonic acidemia Explanation: Methylmalonic acidemia (MMA) presenting like this in a newborn has a poor prognosis. Seizures develop 6 mo later. Alexander disease is similar but progresses at a slower rate. 59.2 in Nelson Textbook of Pediatrics.) Charcot-Marie-Tooth neuropathy Question .14 in Nelson Textbook of Question . A 7-yr-old boy. Physical examination reveals slight diffuse hyperpigmentation. MRI cerebral white matter lesions (symmetrical periventricular sites). Despite therapy. a peroxisomal disorder. spasticity. 58. 57. (See Chapter 75. his school performance deteriorates. The most likely diagnosis is: Acute demyelinating encephalitis Guillain-Barré syndrome Zellweger disease X-linked adrenoleukodystrophy Explanation: Adrenoleukodystrophy. and adrenal insufficiency (as determined by a poor cortisol response to intravenous ACTH). demonstrates progressive hyperactivity and is diagnosed with attention deficit hyperactivity disorder. Physical examination reveals . presents with progressive dementia. previously an "A" student. seizures. 17th edition. He has difficulty understanding people when spoken to on the telephone.) MRI angiography Cerebrospinal fluid myelin basic protein measurement Question . 17th edition. and reduced deep tendon reflexes. (See Chapter 74. (See Chapter 75.3 in Nelson Textbook of Pediatrics. He has a 3-yr history of bone pain and poor growth. A 16-yr-old boy presents with easy bruisability and chronic fatigue. The best diagnostic test for the patient described in Question 57 is Serum amino acid assay Nerve conduction studies Serum very-long-chain fatty acids determination Explanation: Very-long-chain fatty acids are elevated in patients with X-linked adrenoleukodystrophy.Adrenal insufficiency Cerebral palsy Encephalomyelitis Alexander disease Canavan disease Explanation: Canavan disease due to excessive amounts of brain N-acetylaspartate acid produces leukodystrophy with macrocephaly and loss of developmental milestones. and the course is somewhat protracted for acute leukemia. X-ray films of the skeleton reveal long bone lytic lesions and osteosclerosis.) Lovastatin administration Plasmapheresis Sphingomyelinase of lysosomal origin .4 in Nelson Textbook of Pediatrics. The most likely diagnosis is: Leukemia Sickle cell anemia Tay-Sachs disease Hunter syndrome Gaucher disease Explanation: Although leukemia is of concern. 17th edition. (See Chapter 75. (See Chapter 75. Laboratory studies reveal normal liver enzyme levels but a hematocrit of 25% and a platelet count of 25.hepatosplenomegaly.) Ophthalmoscopy for cherry-red spots Urine long-chain amino acids measurement Urine long-chain fatty acids measurement Question . 60. The best diagnostic test for the patient described in Question 59 is: Bone marrow biopsy Assay of acid -glucosidase in leukocytes Explanation: Gaucher disease is due to a deficiency of enzyme with the resultant build-up of glucocerebrosides in the reticuloendothelial system and subsequent dysfunction or enlargement of the respective organs.4 in Nelson Textbook of Question . It is hoped that in the future. Treatment of the patient described in Questions 59 and 60 is best achieved with: Liver transplantation Purified placental acid -glucosidase Explanation: Enzyme replacement therapy has been very effective in reducing extraskeletal symptoms.4 in Nelson Textbook of Pediatrics. (See Chapter 75. 61. 17th edition. there are no blasts in the blood or bone marrow.000/cu mm. recombinant enzyme replacement therapy will eliminate the small risk of infection associated with the disease. . This patient actually had systemic lupus erythematosus (SLE). and thrombophlebitis or diabetes is very unusual.) . False-positive VDRL test results are most likely to occur in: Mononucleosis Systemic lupus erythematosus Endocarditis Intravenous drug misuse Tuberculosis All of the above Explanation: All of the conditions listed in the question produce a false-positive result in the VDRL or other nontreponemal test for syphilis. (See Chapter 110 in Nelson Textbook of Pediatrics.SPECIAL HEALTH PROBLEMS DURING ADOLESCENCE Question . Oral contraceptive agents are associated with increased risk of all of the following except: Thrombophlebitis Carbohydrate intolerance High levels of high-density lipoproteins Premature epiphyseal closure Explanation: Oral contraceptive agents in the available doses contain too little estrogen to close growth plates.) Question . 1. 2. (See Chapter 108 in Nelson Textbook of Pediatrics. 17th ed. In addition. Other complications of oral contraceptives are quite rare in adolescent patients. 17th ed. most females use oral contraceptive agents after the adolescent growth spurt. the antidote for opiates. The most effective therapy is: Intravenous glucose Flumazenil Disulfiram Naloxone (Narcan) Explanation: Narcan is indicated in this patient with intravenous drug misuse and a heroin overdose. 3. (See Chapter 110 in Nelson Textbook of Pediatrics. vaginalis or other pathogens typically is associated with these biochemical and microscopic features. (See Chapter 105 in Nelson Textbook of Pediatrics.) Candida infection pulmonary edema. 17th ed. 17th ed.Question . He has hypertrophic lesions over the dorsum of the left hand and the antecubital fossa. Treatment in nonpregnant women is with metronidazole. the patient will need the ABCs of resuscitation addressed if he is cyanotic. and so forth.) Intravenous calcium . A foul-smelling vaginal discharge that emits a fishy odor on contact with 10% potassium hydroxide and that demonstrates clue cells on the wet preparation is most likely to be due to: Gonorrhea Chlamydia infection Chancroid Gardnerella vaginalis infection Explanation: Bacterial vaginosis due to G. Nonetheless. Securing an airway while providing oxygen and artificial ventilation is the first priority-followed by Narcan. Toluene (glue. Anabolic steroid use is associated with all of the following except: Enhanced school performance Explanation: School performance usually deteriorates as a result of mood changes and aggressive behavior.Question . 17th ed. 5.) Question . (See Chapter 105 in Nelson Textbook of Pediatrics. 17th ed. solvents) misuse is associated with: Hallucinations Tolerance Pulmonary edema Peripheral neuropathy Rhabdomyolysis All of the above Explanation: Both acute and chronic effects of organic solvent misuse produce significant morbidity and mortality. 6.) Testicular atrophy Aggressive behavior Cholestasis Increased low-density lipoprotein levels . (See Chapter 105 in Nelson Textbook of Pediatrics. Leading causes of hospitalization for adolescents include all of the following except: Pregnancy Mental disorders Asthma Anemia Explanation: Anemia is rare in adolescent males and common in females.) Gastrointestinal disorders Injuries . The most likely cause of this patient's condition is: Inflammatory bowel disease Anorexia nervosa Explanation: Such severe wasting is compatible with anorexia nervosa.Question . and erythrocyte sedimentation rate is 3 mm/hr. Additional features of anorexia nervosa include bradycardia. (See Chapter 104 in Nelson Textbook of Pediatrics. 17th ed. She denies vomiting. A 15-yr-old girl has experienced a loss of 30 pounds during the past 6 mo and has amenorrhea. and abdominal pain and claims to feel well. 7. diarrhea. amenorrhea. hospitalization is usually not warranted.0 mEq/L and bicarbonate of 30 mEq/L. Electrolyte determination reveals a serum potassium level of 3.) Bulimia nervosa Addison disease Pituitary adenoma Question . (See Chapters 99 and 100 in Nelson Textbook of Pediatrics. hypothermia. Physical examination reveals cachexia and a pulse of 40. 17th ed. 8. Bulimia does not produce such severe weight loss. and hypokalemia. Nonetheless. Hematocrit is 30. 9. however. They are part of the AMA's annual screening. High-risk characteristics of adolescent sexuality include all of the following except: Young adolescents Late-maturing boys Explanation: Late-maturing boys are at moderate risk.(See Chapters 99 and 100 in Nelson Textbook of Pediatrics.) Drug use Same-sex partner .) Graduated licensing systems Enforcement of drinking age laws Night-time driving restrictions Drivers' education classes Question . (See Chapter 100 in Nelson Textbook of Pediatrics. stable relationships. 17th ed. 10.) Safety practices Scoliosis Breast examination Eating disorders Question . 17th ed. 17th ed. and regular contraceptive use. 11. Successful preventive measures to avoid morbidity and mortality from adolescent automobile accidents include all of the following except: Peer pressure TV commercials Explanation: B-E all are correct. The American Academy of Pediatrics recommends annual health visits for adolescents for all of the following except: Learning problems Explanation: Learning problems were probably identified earlier than in adolescence and are not part of the AAP guidelines.Question . (See Chapter 100 in Nelson Textbook of Pediatrics. Low-risk categories are older age. military. (See Chapter 100 in Nelson Textbook of Pediatrics. military Explanation: Children who are less than 18 years old but who are no longer subject to parental control (married. Examination of two successive cervical scrapings increases the yield and avoids false-negative results. (See Chapter 100 in Nelson Textbook of Pediatrics. 17th ed.Coercive sex Question . 12. (See Chapter 101 in Nelson Textbook of Pediatrics.) Attention deficit disorder Substance abuse . 17th ed. 13. An emancipated minor is one who: Is able to understand health issues Lives with stepparents Has graduated from high school Is a member of the U. 17th ed.) Travels overseas Question .S. 14. economic self-sufficiency) are considered emancipated.) None of the above Question . Papanicolaou smears are indicated in: All young women older than 16 yr A 14-yr-old mother with cervical cancer A 12-yr-old girl exposed to diethylstilbestrol All sexually active teenage girls Explanation: Sexual activity is an absolute indication for a Pap smear. Depression in adolescents is characterized by all of the following except: Two to three times higher incidence in females Growth spurts Explanation: Depression is probably under-reported in the teenager. you determine that she has inappropriate guilt. (See Chapter 101 in Nelson Textbook of Pediatrics. On further questioning of the patient in Question 15. 16.) Chronic fatigue syndrome Depression Substance abuse Question . poor exercise participation. 15. Untreated. and low selfesteem. poor concentration. 17th ed. and hypersomnia. The differential diagnosis includes all of the following except: Hypothyroidism Mononucleosis Exercise-induced asthma Explanation: The other choices are important considerations in the differential diagnosis. weight loss. most patients become depressed again in the next 7 years. 17th ed. (See Chapter 101 in Nelson Textbook of Pediatrics.Death of a family member Question . it may last 7-9 mo. The most likely diagnosis is: Pseudotumor cerebri Depression Explanation: Depression requires a comprehensive treatment plan. A 17-yr-old girl manifests diminished interest in school.) Lead poisoning Alcohol withdrawal Schizophrenia . bone marrow hypoplasia. others include postural hypotension. Is less of a concern if there is no specific plan Does not require psychiatric consultation Question .Question . 17. family discord) and a specific plan raise the level of concern. Suicide ideation: Is a significant risk factor for completed suicide in females Is a significant risk factor for attempted suicide in young teens (age 12-14 yr) Is experienced by about 25% of teens Explanation: Ideation is relatively common. 19. Risk factors for adolescent suicide include all of the following except: Native American race Chronic illness Access to guns Gender Explanation: Males are more likely to complete suicide.) Gay and bisexual activity Question . (See Chapter 104 in Nelson Textbook of Pediatrics. whereas females are more likely to attempt suicide. constipation. 17th ed. Diagnostic criteria for anorexia nervosa include all of the following except: Bradycardia and hypothermia Explanation: These are consequences of weight loss. 17th ed. (See Chapter 102 in Nelson Textbook of Pediatrics. but more serious cofactors such as an adverse event (breaking up with a loved one. 18. and dry lanugo-type skin.) Fear of becoming obese A feeling of being fat despite being emaciated Refusal to maintain body weight . Her last normal menstrual period was 4 mo ago. 22.) Self-induced vomiting and laxatives Fear of not being able to stop eating during a binge At least two binge episodes per week for at least 3 mo Binges lasting less than 2 hr Question . A 17-yr-old white boy presents with an exacerbation of acne and with aggressive behavior and poor school performance. 21. Other potential causes of galactorrhea include oral contraceptive pills. (See Chapter 104 in Nelson Textbook of Pediatrics. Adrenal hypertrophy Marijuana use 17-Hydroxylase deficiency Testicular feminization syndrome Question . This student was on the wrestling team and. testes that are smaller than expected for his sexual maturity rating. Elevated estrogen levels Migraines . had used anabolic steroids.Absence of three consecutive menstrual cycles Question . A 16-yr-old female has had headaches for 3 mo and visual changes for 2 wk and now has galactorrhea. some antihypertensive medications. 20. 17th ed. and some tranquilizers. Physical examination reveals gynecomastia. The most likely diagnosis is: Anabolic steroid abuse Explanation: Athletes of both sexes use anabolic steroids to enhance athletic performance. Bulimia is characterized by the following except: Isocaloric nutrient malnutrition Explanation: B-E are diagnostic of bulimia. like over 500. and oily hair. The most likely cause of her galactorrhea is: Stress of having amenorrhea Elevated prolactin levels Explanation: The patient had a prolactin-secreting pituitary adenoma detected by a CT scan.000 high school students. ) FSH pituitary tumor Hypothyroidism Cervical stenosis Autoimmune endocrinopathy .Her serum FSH level is high. Her mother had her first period at the age of 12 yr. 17th ed. 17th ed. The adolescent is short and has poor breast development. All of the following may cause gynecomastia except: Klinefelter syndrome Phenothiazines Anabolic steroids Heroin Albuterol Explanation: All the rest plus at least a dozen other drugs may produce gynecomastia in males. (See Chapter 107 in Nelson Textbook of Pediatrics.Adrenal insufficiency Question . and there is no withdrawal bleeding after administration of intramuscular progesterone. A 16-yr-old girl has not had her first menstrual period. The most likely diagnosis is: Primary ovarian failure Explanation: The patient has hypergonadotropic hypogonadism-primary ovarian failure. (See Chapter 106 in Nelson Textbook of Pediatrics. Her phenotype is compatible with Turner syndrome (45. 23.) Question . A vaginal smear shows no estrogen effect.X). 24. which are irregular and first started 13 mo ago. and her tachycardia responds to the administration of 1 liter of normal saline (intravenous) Determine the estrogen level Determine the platelet count Question . The most likely diagnosis for the patient described in Question 25 is: Ovarian tumor Fibroids Dysfunctional uterine bleeding Explanation: Dysfunctional uterine bleeding occurs after anovulatory cycles and is due to lack of estrogen build-up of the endometrium. A 14-yr-old girl presents with vaginal bleeding that is more prolonged and profuse than her usual periods.Question . 26. The next step in her evaluation is to: Administer medroxyprogesterone (Provera) Administer conjugated estrogens (Premarin) Determine the hematocrit Explanation: The hematocrit is 27% (MCV 65). Some affected women also have von Willebrand disease. 25. Hemophilia Leukemia . She has pallor and tachycardia. Serious complications of exogenous estrogens in oral contraceptive pills include all of the following except: Thrombophlebitis Myocardial infarction Hepatic adenoma Diabetes mellitus Uterine rupture Explanation: A-D are correct but fortunately are quite rare events and are even rarer in nonsmokers.Question . The iron-deficiency anemia must be treated with iron.) . 27. 28. The most appropriate therapy for the patient described in Questions 25 and 26 is administration of: Premarin Explanation: Estrogens such as Premarin or as part of a combination oral contraceptive pill constitute the treatment of choice. 17th ed. (See Chapter 108 in Nelson Textbook of Pediatrics. Prednisone 17-Hydroxyprogesterone Ibuprofen DDAVP Question . 17th ed. irregular periods) and often ask to have it removed. but many adolescents have trouble with its side effects (weight gain. 30.) A cervical cap .) Lo-Ovral 2 tablets now and in 12 hr Question . It is also important to provide advice about proper nonemergency contraception. A 19-yr-old female patient tells you she had unprotected sex yesterday with her steady partner. The best contraceptive for a 16-yr-old girl with a history of two pregnancies while on combination estrogen-progestin oral contraceptives and a steady sexual partner is: An intrauterine device An all-progestin oral contraceptive Condoms Levo-norgestrel (Norplant) Explanation: Norplant is a reasonable choice. (See Chapter 108 in Nelson Textbook of Pediatrics. Your emergency treatment should be to give: Morgestrel (Ovral) 2 tablets once Spermicide now and in 12 hr Intravenous FSH and hCG Ovral 2 tablets now and in 12 hr Explanation: The goal is at least 200 g of ethinyl estradiol and 2 mg of norgestrel given twice 12 hr apart. 17th ed. 29. She does not want to become pregnant.Question . (See Chapter 108 in Nelson Textbook of Pediatrics. Physical examination reveals an irritable. D. preferring to lie in bed.5°F. A 3-year-old child from a suburban community presents with vomiting. Laboratory confirmation of the diagnosis in Question 1 can best be obtained by measuring the toxic compound's concentration in: A. Which of the following tests will establish the correct diagnosis? A. Tremor of the tongue is evident. erythematous rash on the hands and feet. and respiratory rate is 23/min. Hair D. is characteristic of mercury toxicity. B. pale child with photophobia. Which of the following is the most likely diagnosis? A. salivation. Feces Question . C. The mother reports that he has become ill tempered and refuses to walk about. A 2-yr-old boy presents with a peeling. diarrhea. Blood C. Measles Fifth disease Kawasaki disease Acrodynia Explanation: Acrodynia. Photosensitivity Question . peeling. Pinpoint pupils in patients with altered mental status are not always due to narcotics. heart rate is 80/min. Urine Explanation: Urine collection makes this a more useful test to determine mercury poisoning and because the differential diagnosis may include other heavy metals or drug toxicity. and blurred vision. Further history and follow-up evaluation reveal that the child's elder brothers have been playing with liquid mercury. 2. Physical examination reveals an afebrile child with pinpoint pupils. producing cholinergic crises. Plasma cholinesterase level Explanation: Insecticides are commonly acetylcholinesterase inhibitors. and muscle fasciculations. Saliva E. 24-hr urine mercury level C. characterized by erythemia. and neurologic signs. (See Chapter 702 in Nelson Textbook of Pediatrics.Environmental Health Hazards Nelson Self Assessments website 17th Edition Question . (See Chapter 701 in Nelson Textbook of Pediatrics.Nelson Self Assessments website 17th Edition 1 . Urine malathion level E.) D. Blood lead level B. The family's lawn was treated yesterday for insects. Temperature is 98. 17th ed. 3. Urine morphine level Environmental Health Hazards . (See Chapter 702 in Nelson) E. 17th ed. 1.) B. Obtain chest films from all of the students and teachers Question . 4. Latex paint F. 17th ed. Swordfish B. Milk Explanation: Milk is an unusual source of mercury. (See Chapter 701 in Nelson Textbook of Pediatrics. Mental retardation C. Immediately remove all of the insulation B. Potential sources of mercury include all of the following Except: A. Quicksilver C. Steroids are of no value and may predispose to infection. and they call on you.) E. The best approach to his treatment is to: A.Nelson Self Assessments website 17th Edition 2 . if asbestos is confirmed. B. D. 17th ed.) D. Anemia D. (See Chapter 702 in ) D. Cerebral edema Explanation: Cerebral edema is often noted with blood lead levels exceeding 100 µg/dL. 6. The most serious manifestation of lead intoxication is: A. Confirm the presence of asbestos by laboratory evaluation. and C are dangerous and may induce pulmonary aspiration if emesis occurs.Question . Confirm the presence of asbestos by laboratory evaluation. (See Chapter 703 in Nelson Textbook of Pediatrics. Reassure the parents that the hazards of asbestos have been overstated E. B. A 2-yr-old boy is noted to be drinking from a container filled with kerosene. Because of the risk of asbestos. Deteriorating insulation is found in the ceilings of a local school built in 1958.) Environmental Health Hazards . and is brought to the hospital. Parents and teachers are extremely concerned that it may be asbestos. and do nothing further C. 7. Peripheral neuropathy B. Pesticides E. the local pediatrician. to give advice in an open meeting. Induce emesis Perform nasogastric tube lavage Instill mineral oil Administer steroids None of the above Explanation: Choices A. 5. Folk remedies Question . He immediately coughs. Supportive therapy is indicated and includes oxygen and fluids. call a certified engineer for expert assessment Explanation: It is important always to be calm but to make decisions based on accurate information. becomes tachypneic. Lead lines Question . E. C. 17th ed. an accurate assay is needed before ways are identified to isolate and confine or remove the hazard. (See Chapter 704 in Nelson Textbook of Pediatrics. Which of the following approaches to the problem do you recommend? A. Rattlesnake envenomations Scorpion envenomations Black widow envenomations Stonefish envenomations Hymenoptera envenomations Explanation: Antivenin is not necessary for Hymenoptera bites or stings. 8. D. 10. (See Chapter 700 in Nelson Textbook of Pediatrics. At Three Mile Island. 17th ed. 17th ed. The other choices listed in the question are not associated with severe acute radiation reaction. because of the associated hereditary DNA repair defect. Do nothing acutely. D. 9. It is important to diagnose ataxia-telangiectasia in young patients with lymphoma to avoid radiotherapy. and the sooner it is given the better. Wiskott-Aldrich syndrome Question . About 500 children have just been exposed to radiation from a nuclear power plant accident. Results of the CBC will probably be normal. Order thyroid function tests C. Chédiak-Higashi syndrome Neurofibromatosis Chronic mucocutaneous candidiasis Ataxia-telangiectasia Explanation: A child with ataxia-telangiectasia (AT). B. and children have died. evaluate in 4-6 wk E. and potassium iodide administration was the first measure taken. (See Chapter 700 in Nelson Textbook of Pediatrics. C. The most appropriate first step in management of these children should be to: A. B.) B. is unable to repair acute radiation damage. Antivenins should be considered in the treatment of all of the following Except: A. but immune prophylaxis may be indicated for systemic reactions. Look for signs or symptoms of acute radiation sickness Question . 17th ed. Thyroid function tests are performed much later. (See Chapter 708 in Nelson Textbook of Pediatrics. C. E.Nelson Self Assessments website 17th Edition 3 .) Environmental Health Hazards .) E. Signs of acute radiation sickness will not appear for several days and probably will not result at all from this fallout exposure. Order complete blood counts (CBCs) D. Conventional doses of radiotherapy to treat lymphomas (to which AT predisposes) have caused acute radiation sickness. a serious problem was the inability to obtain eyedroppers to administer the drug by mouth.Question . Prescribe potassium iodide to protect the thyroid Explanation: Potassium iodide protects the thyroid against radiation. Which of the following diseases may be associated with a severe acute reaction to radiotherapy? A. and therefore no problems should occur E. Charcoal may reduce absorption. Administer ipecac at home and bring the child in to see you Question . 17th ed. The child's mother telephones you for help.Question . most clinicians recommend esophagoscopy within 12-24 hr of such an ingestion. A 2-yr-old child is found playing with a can of crystalline drain cleaner.) D. 11. Send the patient home because an ingestion of this magnitude is not toxic E. 165-lb patient reports consuming 20-40 325-mg capsules containing acetaminophen 1 hr ago. 13. 12. The most likely diagnosis is: A. E. Measure the plasma level and determine potential toxicity from the level on the nomogram B. Have the mother administer water or milk and call you back in 2 hr C. Pemphigus Stevens-Johnson syndrome Thermal burn Insect bite Radiation burn from handling a highly radioactive source Explanation: (See Chapter 700 in Nelson Textbook of Pediatrics.) Environmental Health Hazards . which you have the mother wash out.Nelson Self Assessments website 17th Edition 4 . B. The next step in treatment should be to: A. (See Chapter 704 in Nelson Textbook of Pediatrics. A 5-yr-old child presents with a painful blister of unknown origin with surrounding erythema on the palm of the hand and with low white cell and platelet counts. A 16-yr-old. Have the mother administer lemon juice or orange juice to neutralize the alkaline crystals and come to your office B. D. and the ultimate predictor of toxicity (the 4-hr serum level) will determine the need for N-acetylcysteine therapy. Steroids are of little value in the presence of esophageal lesions and do not prevent stricture formation. The ability to predict who will experience esophageal disease is poor. 17th ed. Wait until 4 hr after ingestion to measure the plasma level and do nothing else C. 17th ed. There are several crystals in the mouth. (See Chapter 704 in Nelson Pedia. There is also a history of mild nausea and vomiting occurring about 4-6 days ago. Have the mother administer water or milk and bring the child in for esophagoscopy Explanation: Immediately dilute the ingested drain cleaner with water or milk. Administer N-acetylcysteine at a dose of 140 mg/kg Question . thus. The most appropriate approach to treatment is to: A. Simply observe the child because the crystals are so bitter that the child was trying to spit them out when the mother called.) D. Administer activated charcoal immediately and measure the plasma level of acetaminophen 4 hr after ingestion Explanation: The patient described in the question has ingested a potentially significant amount of acetaminophen. C. Mercury exposure in a pregnant woman who eats tuna fish twice a day as part of a weight loss diet Environmental Health Hazards .) C. Asbestos exposure in a child whose home has sealed asbestos insulation covering pipes in the ceiling of an unfinished basement used for storage Explanation: (See Chapter 701 in Nelson Textbook of Pediatrics. Obtain an x-ray film of a long bone to look for "lead lines" D. All of the following are potential exposure routes to consider in an environmental history Except: A. 16. Perform a bone lead analysis B. Begin chelation therapy E. 17th ed. Organophosphate exposure in a child whose mother applies pesticides for a termite control company D. this level of lead exposure is not harmful Question .) D. 17. Their developmental processes are sensitive and easily disrupted C. Multiple CT scans Living in homes with a high radon level Living in an area of high natural background Exposure to radioactive iodine Explanation: (See Chapter 700 in Nelson Textbook of Pediatrics. C. They have a greater minute ventilation Question . Fine-particulate exposure in an asthmatic child who lives in a home heated by a wood-burning stove C. Reassure the parents that because their child has no symptoms. 17th ed. Compared with adults. Benzene exposure in a child who drinks tap water from a private well B. 17th ed. Children can be exposed to environmental pollutants through a number of routes. D.) E.Nelson Self Assessments website 17th Edition 5 . A 13-mo-old healthy child presents for a follow-up visit to address a capillary blood lead level of 18 ?g/dL obtained routinely at a recent well child visit at age 1 yr. The most appropriate next step in the evaluation and management of this patient is to: A. 15.Question . Paternal preconception radiation exposure Question . 17th ed. 14.) E. They have more future years of life B. They eat less food daily Explanation: (See Chapter 701 in Nelson Textbook of Pediatrics. B. young children are uniquely vulnerable to chemical pollutants for all of the following reasons Except: A. Take a detailed environmental history and obtain a venous blood lead level Explanation: (See Chapter 701 in Nelson Textbook of Pediatrics. They often play near the ground and engage in hand to mouth behavior E. An increase in childhood cancer after radiation exposure has been clearly demonstrated after: A. ) Question . Elemental mercury liquid is rapidly and completely absorbed from the gastrointestinal (GI) tract Explanation: Elemental mercury liquid is poorly absorbed from the GI tract. A patient with chronic arsenic poisoning may present with fatigue. cardiovascular collapse. severe abdominal pain. and encephalopathy C. D. gastroenteritis. Acute ingestion of inorganic mercury salts may result in corrosive gastroenteritis. B. Inhalation of arsine gas results in hemolysis B. All of the following statements regarding mercury absorption are true Except: A. hypotonia. neuropsychiatric disturbances. 19. 20. and gingivostomatitis D. cough. E. irritability. (See Chapter 702 in Nelson Textbook of Pediatrics.17th ed. (See Chapter 702 ) Question . 18. (See Chapter 701 in Nelson Textbook of Pediatrics. 17th ed. The organic forms of arsenic found in seafood are highly toxic to the central nervous system Explanation: The organic forms of arsenic found in seafood (primarily arsenobetaine) are nontoxic but may confound the results of urinary assays for arsenic. Acute ingestion of inorganic arsenic salts results in hemorrhagic gastroenteritis. anemia. with less than 0. 21. peripheral sensorimotor neuropathy.Question . and alopecia D. paresthesias. and an acral rash Environmental Health Hazards . chest pain. acute tubular necrosis. dyspnea. Acrodynia is an idiosyncratic hypersensitivity reaction to mercury characterized by generalized pain.1% being absorbed. Methylmercury is the most avidly absorbed of the organic mercury compounds. All of the following statements regarding arsenic poisoning are true Except: A. C.Nelson Self Assessments website 17th Edition 6 . and respiratory failure B. leukopenia. Acute inhalation of elemental mercury vapor may result in fever. Chronic inorganic mercury intoxication produces the classic triad of tremor. Pesticides Seafood Folk remedies Occupational exposures All of the above Explanation: Exposure to arsenic or mercury may occur with all of the sources listed. Sources of arsenic or mercury exposure include: A. All of the following statements regarding mercury poisoning are true Except: A. Inorganic mercury salts are partially absorbed from the GI tract C. with about 90% absorbed from the GI tract Question .) B. and cardiovascular collapse C. The most appropriate next step in management is to: A.Nelson Self Assessments website 17th Edition 7 . Order a chest film Environmental Health Hazards .45% normal saline. Order an acetaminophen level Explanation: (See Chapter 704 in Nelson Textbook of Pediatrics. Ignore these changes because they are still within normal limits B. 17th ed.) E. Add sodium bicarbonate to his IV fluids to try to raise his serum pH above 7. 17th ed.E. Order an abdominal x-ray study to look for pills in the stomach Question . she confessed to her mother what she had done. Administer syrup of ipecac Administer activated charcoal Remove any ingested lamp oil by gastric lavage Admit the child for observation and supportive care Explanation: (See Chapter 704 in Nelson Textbook of Pediatrics. On examination the patient has normal vital signs and is asymptomatic except for the complaint of nausea. 22. Begin a lidocaine infusion at an appropriate dose E. and a temperature of 37. after she began vomiting.2oC. Repeat a dose of activated charcoal D. You observe a change in his cardiac monitor display with a widening of his QRS complex to 0. The delayed neurotoxicity of methyl mercury intoxication is best treated by chelation with dimercaprol. C. Request a psychiatric consultation D. The most appropriate therapy for this child is to: A. A chest film is read as normal. The child reportedly vomited once at home. The child has a heart rate of 160 beats/min.) C. Discharge the patient home B.) C. 24. a respiratory rate of 48/min. Send a second sample for salicylate determination E. Discharge the child home with a follow-up office visit in the morning Question . One hour afterward. 17th ed. The patient is placed in the ICU for monitoring and remains stable. He is receiving only 0. The most appropriate next step in management is to: A. A 15-yr-old boy is admitted to your care following the intentional ingestion of 2 g of his own amitriptyline in a suicide gesture. B.12 second and occasional ectopic beats. (See Chapter 701 in Nelson Textbook of Pediatrics.4 Explanation: (See Chapter 704 in Nelson Textbook of Pediatrics.) Question . but the laboratory reports no salicylates in her blood. resulting in increased neurotoxicity. A serum salicylate level is ordered. He received activated charcoal for GI decontamination. also known as British antilewisite (BAL) Explanation: Dimercaprol is contraindicated for chelation of methylmercury because BAL redistributes mercury to the brain from other tissue sites. 23. 17th ed. A teen-age girl presents in the emergency room with the story that she got upset with her boyfriend and swallowed a "handful of aspirin" about 4 hr previously. A 2-yr-old child presents in the emergency department following the reported ingestion of a mouthful of lamp oil. D. In the United States the only significant morbidity from spiders is caused by the black widow spider B. His laboratory values are follows: arterial pH 7. 17th ed.Question . potassium 3. 26.) Question .2oC. All antivenins are animal-derived immunoglobulins and cause only delayed hypersensitivity reactions Explanation: (See Chapter 708 in Nelson Textbook of Pediatrics. sodium 148 mmol/L. heart rate 200 beats/min.200/mm3. WBC count 10. A 2-year-old boy arrives in the emergency department following a seizure. arterial PCO2 18 mm Hg. 27. Most bites and stings by spiders. Pit vipers may be identified by their triangular heads.) C. On presentation his vital signs are as follows: temperature 40. you suspect that an accidental ingestion has occurred. 95% of poisonous snakes are pit vipers E. and other venomous animals do not require attention B. The black widow spider is glossy black and has bright red or orange markings on the ventral surface of the abdomen Explanation: (See Chapter 708 in Nelson Textbook of Pediatrics. snakes. respiratory rate 52 breaths/min. Most scorpion bites are life-threatening E. and identifiable pit between the eyes and nose Question . The most likely toxin is: A. All of the following statements are true Except: A. On the basis of the history obtained and the presentation. C. In the United States. 17th ed. Sea bather's eruption is caused by stingrays Environmental Health Hazards .1 mmol/L.) C. Which of the following statements is true? A. B. A cyclic antidepressant Acetaminophen Cocaine An organophosphate insecticide A salicylate Explanation: (See Chapter 704 in Nelson Textbook of Pediatrics. elliptical eyes.Nelson Self Assessments website 17th Edition 8 . D.2. E. serum bicarbonate 6 mmol/L. 17th ed. 25. Delayed hypersensitivity or serum sickness develops in up to 65% of patients who receive equine-derived antivenin D. Antivenin should be used first to control the pain of a black widow bite D. because one is not related to the other. SIDS should not be confused with apnea of prematurity. mortality. 1. All of the following are features of postneonatal mortality except: Death between age 28 days and 1 yr Caused in part by sudden infant death syndrome Caused in part by bronchopulmonary dysplasia Caused in part by premature rupture of membranes Explanation: Premature rupture of membranes (PROM) is associated with risks of early-onset sepsis and premature birth. This can be explained only in part by a rising number of multiple births following reproductive technology advances. not postnatal. Historical family features that suggest risk in a current pregnancy include all of the following except: .Question . Major causes of neonatal mortality include all of the following except: Sudden infant death syndrome Explanation: Sudden infant death syndrome (SIDS) usually does not occur in the first month of life. 17 edition.) Low birthweight may be due to prematurity Low birthweight may be due to intrauterine growth retardation Low birthweight mortality is higher in black infants Question . 4. th (See Chapter 82 in Nelson Textbook of Pediatrics. (See Chapter 82 in Nelson Textbook of Pediatrics. (See Chapter 82 in Nelson Textbook of Pediatrics. All of the following regarding the low-birthweight rate are true except: Low birthweight contributes to neonatal mortality The low birthweight mortality has decreased between 1981 and 2000 Explanation: Indeed the low-birthweight rate has increased during this interval. Apnea of prematurity occurs in the first month of life but does not contribute to mortality.) Respiratory distress syndrome Congenital malformations Asphyxia Sepsis Question .) Caused in part by trauma Question . 17th edition. Both should contribute to neonatal. 17th edition. 2. 3. 17th edition. The most important next step is: Perform a sepsis evaluation Perform a CBC followed by a sepsis work-up if neutropenia is found Keep the infant warm Explanation: Acrocyanosis is normal at this time.) Maternal diabetes mellitus Poor prenatal care Question .) Congenital herpes simplex. The flaccid vesicles contain neutrophils. born after a spontaneous vaginal delivery. Deformational plagiocephaly is associated with all of the following except: . which is in contrast to the eosinophils seen in erythemia toxicum. Warming is all that may be needed. The most likely diagnosis is: Erythema toxicum Pustular melanosis Explanation: This is the classic presentation of this benign neonatal rash. (See Chapter 83 in Nelson Textbook of Pediatrics. Adolescents and women over the age of 40 yr are considered high risk. and if the physical examination is also normal there is no risk of a serious underlying disorder. A 30-min-old term 3. 17 edition.) Begin oxygen administration Hold oral feedings until the acrocyanosis resolves Question . An African-American male is born after a normal pregnancy and delivery with rupture of the fetal membranes for 3 hr and no fetal monitoring.A sibling with neonatal jaundice Maternal history of urinary tract infection Maternal age 19-35 yr Explanation: These are the most ideal ages for safe maternal reproduction. 6. 7. 17th edition. type II Incontinentia pigmentosa Herpes gestationalis Question .4-kg baby. 5. (See Chapter 83 in Nelson Textbook of Pediatrics. is noted to have acrocyanosis. he is noted to have multiple small pustules over his chin and neck that are on a pigmented macular base. Immediately after birth. (See Chapter 83 in Nelson Textbook th of Pediatrics. are most likely due to: Birth trauma Nuchal cord Vacuum extraction Being large for gestational age Child abuse Explanation: Child abuse is often associated with bilateral retinal hemorrhages. 17th edition. You come to see the baby and note that the grunting has stopped.5-kg female born following repeat cesarean section is noted by the nurses to be grunting at 10 min of age.) Question . The most appropriate next step is to: th . 17th edition. A 3. (See Chapter 83 in Nelson Textbook of Pediatrics. the pulse oximetry reading is 99%. 10. presenting in the 6 week of life in a lethargic neonate without any other physical findings. (See Chapter 83 in Nelson Textbook of Pediatrics. and the child looks vigorous. 17th edition.) Question . Bilateral and multiple retinal hemorrhages. the respiratory rate is 36/min. 8. especially in what is called the "shaken baby syndrome. (See Chapter 83 in Nelson Textbook of Pediatrics. Nonetheless. Indications for cutting the oral frenulum include: Short frenulum Cleft palate Cleft lip Bifid uvula None of the above Explanation: There is little evidence for the clipping of a short frenulum. they should all resolve by 1 mo of age.Breech presentation Explanation: Vertex positioning with possible pressure of the developing skull by the mother's bony pelvis contributes to this deformation." Retinal hemorrhages may be noted immediately after birth.) Torticollis Asymmetrical skull Ear malalignment Asymmetric face Question . 9. while neonates with low Apgar scores do not universally get cerebral palsy. (See Chapter 84 in Nelson Textbook of Pediatrics. The best description of the Apgar score is that it: Accurately predicts who will develop cerebral palsy Assesses neonates in need of resuscitation Explanation: The Apgar score helps to rapidly assess the need to resuscitate neonates after birth. (See Chapter 83 in Nelson Textbook of Pediatrics. related in part to preterm birth. 17th edition. 11.) Accurately predicts a low umbilical cord pH Is unaffected by maternal opiate pain relief Accurately predicts neonates who will die in the neonatal period Question . Most children with cerebral palsy have had normal Apgar scores. there is an increased risk of cerebral palsy. respiratory distress syndrome (RDS). Reproductive technologies are associated with all of the following pregnancy-related risks except a : Higher risk for multiple fetuses Higher risk for group B streptococcal sepsis Explanation: Group B streptococci are not increased in these pregnancies. If grunting persists beyond 30 min or if there are other signs of distress. or congenital heart disease. 17th edition.Perform a sepsis evaluation Obtain a chest film Observe.) Higher risk for low birthweight Higher risk for very low birthweight . as well as multiple-fetus pregnancies. it has a poor positive predictive value. particularly after a cesarean section without prior labor. and if grunting returns. the child should be evaluated for sepsis. Although it has some value in predicting neonatal mortality and cerebral palsy.) Begin surfactant therapy Begin nasal continuous positive airway pressure Question . (See Chapter 83 in Nelson th Textbook of Pediatrics. 12. 17 edition. admit to the neonatal intensive care unit Explanation: Grunting is common. In addition to the risks of low birthweight (LBW) and very low birthweight (VLBW). Higher risk for congenital anomalies Question . Fetal ultrasonography may estimate fetal gestational age.) Screening for trisomy 21 Predicting intrauterine growth retardation Question . but it cannot ascertain the actual L/S ratio. 17th edition. 15. Fetal ultrasonography is of use for all of the following except: Determining multiple fetuses Identifying congenital heart disease Determining the lecithin-sphingomyelin (L/S) ratio Explanation: The lecithin-sphingomyelin ratio (L/S) predicts pulmonary maturity and is a functional test that is based on maturation of the fetal type II pulmonary alveolar cells. Oligohydramnios is associated with all of the following except: Esophageal atresia Explanation: Esophageal atresia is most often associated with polyhydramnios.) Pulmonary hypoplasia Potter syndrome Posterior urethral valves Skeletal contractures Question . 16. Fetal tachycardia may be caused by all of the following except: . 17th edition. it has a poor sensitivity and a low positive predictive value for premature birth. and thus the potential for lung maturity. Spontaneous uterine contraction monitoring is most useful to: Accurately identify risk for very low birthweight Determine risk of premature rupture of the membranes Make diagnosis of abruptio placentae Identify a short cervix None of the above Explanation: Although monitoring for the rate of spontaneous uterine contractions before the onset of labor is popular. (See Chapter Question . 13. 14. (See Chapter 84 in Nelson Textbook of Pediatrics. as are other upper intestinal obstructive lesions or disorders of fetal swallowing. (See Chapter 85 in Nelson Textbook of Pediatrics. and it produces bradycardia by the development of an antibody immune-related process. 17. (See Chapter 85 in Nelson Textbook of Pediatrics.) Filtered versus non-filtered cigarettes Educational level of the mother Question . splenectomy reduces the destruction of maternal platelet antibody complexes by the spleen.) Question . Splenectomy does not reduce the IgG antiplatelet antibodies. An important influence on the effects of cigarette smoking on fetal growth restriction is: Brand of cigarettes Altitude Polymorphisms of hydrocarbon-metabolizing enzymes Explanation: Polyhydric aromatic hydrocarbons may alter fetal growth. Immunologic diseases of the mother that may affect the fetus include all of the following except: Systemic lupus erythematosus Idiopathic thrombocytopenic purpura years after splenectomy Myasthenia gravis Graves disease Multiple sclerosis Explanation: It is interesting that B might be a choice. 18. which can readily cross the placenta. 19. 17th edition. (See Chapter 85 in Nelson Textbook of Pediatrics. Cordocentesis (percutaneous umbilical blood sampling) is useful for all of the following except: . 17th edition. However.Maternal lupus erythematosus Explanation: Maternal lupus may affect the fetal cardiac conduction system. It is very possible that enzymes that metabolize these to either toxic or-more likely-nontoxic metabolites modify the impact of the fetal growth restriction due to cigarette smoking. thus increasing the mother's platelet count.) Maternal fever Prematurity Fetal anemia Fetal supraventricular tachycardia Question . 17th edition. (See Chapter 85 in Nelson Textbook of Pediatrics. it was not performed via PUBS but via fetal cardiac puncture. The incidence of twins detected at 40 wk of gestation is characterized by all of the following except: Family history of monozygotic twins Explanation: Monozygotic twinning has little or no genetic determination.) Measuring antibody titers Measuring fetal pH Question . (See Chapter 86 in Nelson Textbook of Pediatrics. Many twins detected at 12 wk of gestation are singletons at 40 wk owing to fetal twin demise.g.. 17th edition. incidence of cerebral palsy is higher in the surviving twin). (See Chapter 86 in Nelson Textbook of Pediatrics. 20. (See Chapter 85 in Nelson th Textbook of Pediatrics.) Fetal mortality of twins noted at 12-wk of gestation Family history of dizygotic twins Race Infertility treatment Question . 17th edition.) Renal agenesis Pulmonary hypoplasia Flattened facies Question . Amnion nodosum is associated with all of the following except: Oligohydramnios Fetal Candida infection Explanation: Fetal-placental amniotic Candida infection may demonstrate nodules on the umbilical cord and a neonatal macular-papular. disseminated rash. erythematous. 17 edition. This process may affect the surviving twin (e. 21. 22. Very-low-birthweight infants are best described as: Predominantly growth restricted .Measuring fetal hemoglobin Determining fetal genome Performing fetal cardiac catheterization Explanation: This is future shock! Although one case of fetal valvotomy has been reported. Predominantly premature Explanation: The very low birthweight (VLBW) rate is an excellent marker for prematurity (<37 wk of gestation). D. (See Chapter 86 in Nelson Textbook Question .) Hyperbilirubinemia Hypernatremia Oliguria Question . 24. Answers A. 17th edition. 17th edition. (See Chapter 86 in Nelson Textbook of Pediatrics. but almost all infants weighing <1500 g are premature. 23. the infant is placed at risk for BPD or a PDA.) Predominantly post dates The result of maternal illness The result of placental infarction Question . but at the same time. C. Breast milk from a donor may have insufficient amounts of all of the following nutrients except: Protein Calcium Phosphorus Vitamin D Essential fatty acids Explanation: Essential fatty acid deficiency has never been reported in a human milk-fed infant in the absence of a hepatic cholestasis malabsorption syndrome. (See Chapter 86 in Nelson Textbook of Pediatrics. Hypotension in a newborn suggests all of the following except: Fetal-maternal hemorrhage Hypoplastic left heart syndrome . Excessive intravenous fluid intake in very-low-birthweight infants is associated with: High BUN level Bronchopulmonary dysplasia Explanation: Both bronchopulmonary dysplasia (BPD) and a patent ductus arteriosus (PDA) are associated with excessive intravenous fluids in premature infants. Some VLBW infants may have growth restriction. or E are some reasons why fluid intakes may be increased. 25. A 900-g infant with respiratory distress syndrome has persistent hypotension despite administration of fluid boluses and dopamine. 26. (See Chapter 87 in Nelson Textbook of Pediatrics Question . 27. or opiate withdrawal. and is stopped by simple pressure. 4. A 1-day-old infant born to a diabetic mother is noted to have tremors of all extremities while awake. They disappear during sleep and are not associated with eye movements. (See Chapter 87 in Nelson Textbook of Pediatrics. spleen). If the child appears normal and has no abnormal eye movements. Epinephrine also fails to elevate the blood pressure. and sepsis can all produce hypotension. The next step to manage the blood pressure is to administer: Albumin Hydrocortisone Explanation: Many studies have demonstrated the value of administering hydrocortisone (Solu-Cortef) to VLBW infants who respond poorly to fluid boluses and dopamine. it could be associated with a serious problem such as hypocalcemia. and if the motion is sensory-dependent. 17th Question . IDMs are often more jittery than other infants but are also at risk for hypoglycemia and hypocalcemia. it is probably not a seizure. All jittery children should be examined carefully and concern for a seizure considered. (See Chapter 87 in Nelson Textbook of Pediatrics.500-g boy has a bilirubin level of 21 mg/dL. large-for-gestational-age. is only present during active wakefulness. other congenital heart defects.Adrenogenital syndrome Tension pneumothorax Jaundice Explanation: Answers A to D. but on examination he has a . The most likely diagnosis is: Hypoglycemia Kernicterus Renal vein thrombosis Caudal regression syndrome Jitteriness Explanation: This is a classic problem in infants of diabetic mothers (IDMs). postnatal hemorrhage (CNS. liver. Nonetheless. The glucose and calcium levels are normal. hypoglycemia. There is no anemia or polycythemia. A 5-day-old. 17th edition.) Hypertonic saline Norepinephrine Dobutamine Question . 28. (See Chapter 88 in Nelson Textbook of Question . (See Chapter 88 in Nelson Textbook of Pediatrics.500 g to detect intracranial hemorrhage is best described as: Performed between 7 and 14 days and at 36-40 wk Explanation: In addition. The management of post-hemorrhagic hydrocephaly includes all of the following except: Serial head circumferences Serial head ultrasound examinations External ventricular drainage Ventricular-peritoneal shunt Repeated lumbar punctures Explanation: Although repeat lumbar puncture (LP) is often done. The next therapeutic activity should be to: Aspirate the hematoma Perform an incision and drainage of the hematoma Undertake prophylactic blood transfer Administer phototherapy Explanation: Phototherapy is clearly indicated. 31. large-for-gestational-age infant is noted to have Erb palsy.large cephalohematoma. 17th edition. most physicians do not believe that they avoid the need for a ventricular peritoneal (VP) shunt. Most cases of dilated ventricles after IVH do not necessitate later placement of a shunt. nonroutine ultrasonography should be performed for symptoms of intraventricular hemorrhage (IVH) and for the follow-up of abnormalities noted on the first ultrasound.) Performed for anemia Performed for seizures Performed at birth and at 40 wk Performed between 7 and 14 days and at 1 yr of age Question . Routine head ultrasonography in infants <1. 17th edition. You should do all of the following except: . 29. 30. A 12-day-old.) Perform exchange transfusion Question . (See Chapter 88 in Nelson Textbook of Pediatrics. Aspiration or incision and drainage (I + D) should not be done to manage a cephalohematoma. ) Administer naloxone in the endotracheal tube Begin chest compressions Obtain a cord pH Question . The baby is apneic and limp. A term female is born by spontaneous vaginal delivery to a primiparous woman who received two doses of meperidine 30 min and 2 hr prior to an abrupt delivery.) Refer for physical therapy Reassure the family Determine if the clavicle is fractured Look for additional nerve involvement (phrenic) Question . endotracheal intubation should be performed. 17th edition. one expects that with a successful intubation and ventilation.Refer for immediate neuroplasty Explanation: Most Erb palsies resolve rapidly with immobilization. immediate management is to: Administer naloxone in the umbilical vein Perform bag-mask ventilation Explanation: Apnea from any cause is treated with securing a patent airway and instituting ventilation. Successful ventilation is determined by all of the following except: Zero reading of end-tidal CO2 measurement Explanation: Indeed. (See Chapter 88 in Nelson Textbook of Pediatrics. 32.) Pink color Rising heart rate Symmetric breath sounds Good chest rise . 17th edition. (See Chapter 89 in Nelson Textbook of Pediatrics. (See Chapter 89 in Nelson Textbook of Pediatrics. If bag mask ventilation is ineffective. rehabilitation. 33. a referral for surgical evaluation is indicated. and positioning. the end-tidal (exhaled) CO2 will rise dramatically. Complete cardiac onset with poor perfusion may also cause a low end-tidal CO2. Naloxone (Narcan) can be given only after the baby is ventilated. 17th edition. If there is no improvement between 3-6 mo. The most important. This is now used in many neonatal resuscitations. and mild abdominal distention is noted. 17th edition. premature infant has sudden onset of apnea and bradycardia that occurs repeatedly. The approach to the patient's management is best performed by: Considering this a case of apnea of prematurity Being concerned about a potential medical condition Explanation: A previously healthy "gainer and groaner" should not develop apnea of prematurity at 3 wk of age and at a corrected age of 35 wk. The next step in management. The others (B-E) are important to look for other causes of respiratory distress and prepare to treat empirically for the possibility of early-onset group B streptococcal pneumonia. is to: Institute inhalation of nitric oxide Perform a blood transfusion Begin nasal continuous positive airway pressure (CPAP) Explanation: Nasal CPAP is often quite effective when larger premature infants develop respiratory distress syndrome (RDS). 36. 17th edition. (See Chapter 90 in Nelson Textbook of Pediatrics. The abdominal distention also suggests an ileus or necrotizing enterocolitis. In addition to administration of CPAP. A 32-wk gestational age infant develops grunting. flaring. the infant appears to be asleep. would include all of the following except: Head ultrasonography Explanation: In a neonate with no signs of intraventricular hemorrhage (IVH) and on the first day of life. and retraction after birth. On examination. 17th edition. one does not need a head ultrasound. 34. (See Chapter 90 in Nelson Textbook of Pediatrics. if he requires more O2. A 3-wk-old. immediate approaches to the patient described in Question 35.) Blood culture Chest x-ray . (See Chapter 90 in Nelson Textbook of Pediatrics. 35. 32-wk-gestation. He requires 50% oxygen (O2) by hood to keep his oxygen saturation above 95%.Question .) Begin penicillin infusion Begin dexamethasone infusion Question . Sudden onset and repeated episodes strongly suggest an underlying illness.) Obtaining a head ultrasound study Doing an echocardiogram Starting caffeine Question . All of the following should be done to evaluate and manage her except: Administer prostaglandin E2 Explanation: In a neonate with no signs of intraventricular hemorrhage (IVH) and on the first day of life. and hypoxia with hypercarbia. 37. The immediate treatment of choice for the patient described in Question 37 is administration of: Digoxin Dobutamine Dexamethasone Indomethacin Explanation: Indomethacin. 790-g female infant had been ventilated for respiratory distress syndrome and was being weaned effectively from the ventilator. 38. 17th edition. 17th edition.) Vitamin E Question .) Obtain a chest film Obtain an echocardiogram Restrict fluids Treat the hypoxia Question .Complete blood count Blood gas analysis Question . The others (B-E) are important to look for other causes of respiratory distress and prepare to treat empirically for the possibility of early-onset group B streptococcal pneumonia. 39. bounding pulses. The differential diagnosis of persistent pulmonary hypertension of the newborn includes all of the following except: Polycythemia Total anomalous pulmonary venous return Meconium aspiration pneumonia . A 3-day-old. If medical treatment is unsuccessful. Today she is noted to have an active precordium. (See Chapter 90 in Nelson Textbook of Pediatrics. facilitating medical closure of a PDA. inhibits prostaglandin synthesis. (See Chapter 90 in Nelson Textbook of Pediatrics. one does not need a head ultrasound. surgical ligation is performed. and perhaps ibuprofen when approved by the FDA. 17th edition. immature colonic function). (See Chapter 91 in Nelson Textbook of Pediatrics. All of the following are true about necrotizing enterocolitis except: Epidemics occur Prematurity is the greatest risk factor Human milk reduces the incidence Surgeons need not be involved until there is evidence of intestinal perforation Explanation: Surgeons should be involved early to help in the . A plain film or KUB film is not sufficient to detect all abnormalities but should be done before a barium intestinal series. A term baby girl has two episodes of bile-stained emesis at 24 hr after birth. 40.) Question . (See Chapter 91 in Nelson Textbook of Pediatrics. The most appropriate diagnostic test is: Blood culture Barium swallow and upper gastrointestinal tract x-ray series Explanation: All infants with bile stained emesis should be evaluated radiologically. There is a history of excessive amniotic fluid volume. 17th edition.Hypoglycemia Renovascular hypertension Explanation: Systemic hypertension may be a complication of treating persistent pulmonary hypertension of the newborn (PPHN) by ECMO or from umbilical artery catheterization.) Barium enema CT scan Head ultrasound study Question . 42. it is not associated with a higher incidence of meconium plugs. Meconium plug is associated with all of the following except: Hypermagnesemia Infants of diabetic mothers Cystic fibrosis Hirschsprung disease Prematurity Explanation: Although prematurity may cause delayed passage of meconium (NPO. 41. (See Chapter Question . (See Chapter 91 in Nelson Textbook of Pediatrics. 44. On physical examination. breast-feeding. 43. including evidence of perforation and failure to respond to medical management. prematurity. 17th edition. Placement of an abdominal drain with peritoneal lavage may also help in the management of a sick VLBW infant. the liver is 3 cm below the right costal margin. The hour-specific bilirubin assay (see accompanying figure) best predicts: Hemolysis Exaggerated (indirect) hyperbilirubinemia Explanation: This test helps determine who is at risk for . it is unknown whether the infant has cholestatic or indirect hyperbilirubinemia. (See Chapter 91 in Nelson Textbook of Pediatrics. A 3-wk-old breast-fed infant has deepening jaundice. She had biliary atresia. High risk factors for neonatal jaundice include all of the following except: Neonatal polycythemia A sibling with jaundice Poor enteral intake Asian heritage Post dates Explanation: Postmature infants have a lower incidence of jaundice unless polycythemia is present. In this child. (See Chapter 91 in Nelson Textbook of Pediatrics. diabetic mother.) Pneumatosis intestinalis is an important diagnostic radiologic sign Question . hypothyroidism. and diseases producing cholestatic disorders.management and to discuss indications for surgery. bruising. 45. Other risk factors for jaundice include hemolysis. The most important laboratory test in this child at this time is: Serum ceruloplasmin determination Direct and total bilirubin level Explanation: Until this test is done.) Question . 17th edition. intestinal obstruction.) Hepatic ultrasonography Complete blood count Urine urobilinogen determination Question . 17th edition. Gilbert disease. the total bilirubin was 20 mg/dL and the direct was 10 mg/dL. dehydration) may be present this early. All of the following are true about tin protoporphyrin except: It may have value for Jehovah's Witnesses It may have value for patients with glucose-6-phosphate dehydrogenase deficiency It may avoid the use of phototherapy It may cause hemolysis Explanation: Although not extensively tested. 48. 46. and the review of the labor and delivery do not reveal any sources of blood loss. The family history is not contributory. The vital signs reveal tachycardia. kernicterus. she is noted to be pale. which may predispose to kernicterus if untreated. A term female infant is born with Apgar scores of 9 and 9. but breast milk jaundice traditionally appears during the second week of life (and of nursing). there is no hepatosplenomegaly or jaundice. But the graphic risk designation based on hour-specific bilirubin levels (Fig.) Kernicterus Sepsis Biliary atresia Question . 17th edition. (See Chapter 91 in Nelson Textbook of Pediatrics. it is a very useful graph.) It may cause erythema with concurrent administration of phototherapy Question . Her hematocrit at 16 hr of age is 30%. 17th edition. XI-1) does not predict. as is the blood smear. 47. The bilirubin is 2 mg/dL. . nor was it designed to predict. (See Chapter 91 in Nelson Textbook of Pediatrics.exaggerated indirect hyperbilirubinemia. At 15 hr of age. (See Chapter 91 in Nelson Textbook of Pediatrics. this single dose inhibition of heme oxygenase may be an important method to prevent extreme jaundice in specific patient populations. whereas the platelet and WBC counts are normal. Kernicterus has been reported with breast-feeding and bilirubin levels between 21 and 50 mg/dL. All of the following are true about breast milk jaundice except: It is associated with a risk of kernicterus It peaks on the 3rd day of life Explanation: Breast-feeding jaundice (poor intake. The reticulocyte count is 15%. 17th edition.) It resolves within 24 hr of temporarily stopping breast-feeding It is most common in the second week of life The cause is unknown Question . Nonetheless. antibodies to erythropoietin may develop. There may be a role in special circumstances. which will bind to endogenous erythropoietin. in older patients.The next important step in her evaluation is to do: Red blood cell fragility test Coombs test Kleihauer-Betke test Explanation: The Kleihauer-Betke test is performed on maternal blood and tests for the presence of fetal hemoglobin containing erythrocytes from a fetal-to-maternal transfusion. Both 2. ABO B. Neither 1. A low bilirubin suggests that there is no hemolysis. 17th edition. Kernicterus Explanation: C. 50. and a normal examination. (See Chapter 92 in Nelson Textbook of Pediatrics. Fetal-to-maternal bleeding can be chronic or acute. the risks exceed the benefits. Indeed. such as when a patient is a Jehovah's Witness. except for tachycardia. suggests no internal blood loss. Both D. Both 3. Rh 4. A. Jaundice on day 1 Explanation: C. Anemia . At this time. ( Question . thus producing a severe anemia.) Apt test Serum ferritin determination Question . 49. Increasing severity with subsequent pregnancies Explanation: B. Rh hemolytic disease. Matching: ABO vs. The role of recombinant human erythropoietin in the management of very low birthweight is best defined as: Providing prophylaxis in jaundiced infants Providing prophylaxis in infants only <750 g Preventing of bronchopulmonary dysplasia Preventing of retinopathy of prematurity None of the above?it has no routine role Explanation: Recombinant human erythropoietin (rHuEpo) has not consistently demonstrated benefit. Rh C. postmature infants. Rh 10. Hydrops fetalis Explanation: B. Amniotic fluid volume is adequate. Rh. and the bladder size looks normal. an obstruction may be missed if only one ultrasound is performed. is seen occasionally with ABO incompatibility (See chapter 92 in Nelson Textbook of pediatrics. Neither. (See Chapter 92 in Nelson Textbook of Pediatrics. In utero treatment Explanation: B. 17th edition. it is recommended to repeat the ultrasound examination at 1 mo of age. Both 5.) Renal ultrasound study at birth .) Neonatal Graves disease Adrenogenital syndrome Infant of diabetic mother Question . 17th edition. Neither. (See Chapter 93 in Nelson Textbook of Pediatrics. After birth. 17th edition. The most appropriate diagnostic approach to this child is: Renal ultrasound study at birth and at 1 mo Explanation: Because fluid intake and urine flow may be reduced in the first day or two after birth. Additional risks for polycythemia include intrauterine growth retardation. and being born at high altitude. Therefore. 51. 52. Polycythemia is associated with all of the following except: Trisomy 21 Donor twin in a twin-twin transfusion Explanation: The recipient twin develops polycythemia. unless exchange transfusion 9. Intracranial hemorrhage Explanation: D. Iron deficiency Explanation: D. Hepatosplenomegaly Explanation: B. Beckwith-Wiedemann syndrome. unless associated with premature death 6.Explanation: C.) Question . Rh 7. the child voids spontaneously within 24 hr. Preventable with immunoglobulin Explanation: B. Rh 8. A term fetus is noted to have bilateral dilation of the renal pelvis and ureter. Urine test results may vary. The best approach to prevent congenital anomalies in infants of diabetic mothers is to: Discontinue insulin and begin glyburide . In the emergency department he seems awake but has tremors and feels rigid. 17th edition. The best method to identify prenatal drug misuse is: Careful maternal history Meconium testing Explanation: Meconium drug testing has become the standard and reflects long-term drug misuse at a site of drug excretion and storage.) Testing of neonatal urine Abstinence withdrawal score Maternal urine testing Question . rickets. He feeds often and well compared with her other children who are 18 mo. 53. (See Chapter 95 in Nelson Textbook of Pediatrics. depending on the timing between drug intake and testing and the dose.Renal ultrasound study at 1 mo Intravenous pyelogram None of the above?no diagnostic evaluation is needed because he voided spontaneously within 1 day of birth Question . He is being breast-fed by a mother who is a strict vegetarian. his growth and development were normal. However. 55. later in infancy. 17th edition. The most likely diagnosis is: Nesidioblastosis Intracranial hemorrhage Hypoglycemia Pyridoxine deficiency Hypocalcemia Explanation: This infant has late neonatal onset tetany. (See Chapter 95 in Nelson Textbook of Pediatrics. 54. which is usually due to ingesting cow's milk with a very high phosphate content.) Question . some infants breast-fed by a mother at nutritional risk may develop hypocalcemia or. Prior to this. A 2-mo-old African-American term male has a seizure at home. The patient responded to intravenous calcium and was sent home on supplemental vitamin D. and 10 yr old. Infants with hypocalcemia often have only peripheral neuromuscular signs and appear alert during what appears to be a seizure. 5 yr. Provide continuous insulin infusion during labor and delivery Switch from an oral hypoglycemic agent to insulin until 36-wk of gestation Do serial glucose tolerance tests Maintain periconceptional tight control of maternal blood glucose levels Explanation: Periconception control and control during the early period of organogenesis help reduce the risk of congenital malformations. and micropenis. tachypnea. 17th edition. A newborn boy is noted on physical examination to have a unilateral cleft lip. Inotropic agents that increase the force of ventricular muscle contraction exacerbate the obstruction and are contraindicated. You are unable to make a specific overall diagnosis. bilateral postaxial polydactyly. 17th edition. Multiple malformations however may be of unknown etiology. microcephaly. A fetal echocardiogram does not reveal any evidence of congenital heart anomalies. 17th edition. At 40 hr of age. and a normal blood glucose level. the baby has tachycardia. intrauterine growth retardation. (See Chapter 97 in Nelson Textbook of Pediatrics. Metabolic disorders rarely cause multiple malformations but may result in normal formation with subsequent changes. cyanosis. (See Chapter 96 in Nelson Textbook of Pediatrics. This is a form of ventricular septal hypertrophy that can obstruct the left ventricular outflow tract and cause heart failure and poor cardiac output.) Epinephrine Dopamine Question . The therapeutic agent of choice for this infant is: Digoxin Dobutamine Propranolol Explanation: This infant has cardiomyopathy associated with infants of diabetic mothers. (See Chapter 96 in Nelson Textbook of Pediatrics. and you believe that the infant is in heart failure.5-kg baby girl of a mother with classic diabetes develops grunting after cesarean delivery. A 4. such as coarsening of the facial features or microcephaly or macrocephaly.) Question . Common anomalies affect the heart and musculoskeletal system. 57. A chest x-ray reveals cardiomegaly. What is the most appropriate initial diagnostic test for this patient? Quantitative urinary organic acid measurement Plasma amino acid determination Karyotype Explanation: This constellation of findings suggests a chromosomal abnormality. 56.) . you determine that she was not taking any medications during her pregnancy. (See Chapter 97 in Nelson Textbook of Pediatrics. A 2-day-old female infant is noted to have a bilateral cleft lip and cleft palate. Neither parent has a cleft lip or cleft palate. you determine that she is otherwise completely normal from a structural standpoint. you further determine that no other family member is affected. Upon careful questioning of the mother. is noted to have multiple joint contractures and to be microcephalic. Based on a careful family history. The most effective way to prevent early onset of neonatal group B streptococcal sepsis is: Screen all pregnant women for group B streptococci Explanation: Most studies suggest that screening for group B Streptococcus in the mother and then selectively treating affected women will reduce the risk of neonatal sepsis. 17th edition.) 75% Question . (See . Which of the following recurrence risk figures would you give this baby's parents? 25% 0% 50% 3-5% Explanation: Recurrence risks of what is traditionally thought to be a polygenetic disorder are between 3-5%.17-hydroxyprogesterone assay Urinary mucopolysaccharide screen Question . 58. 60. (See Chapter 97 in Nelson Textbook of Pediatrics. A 1-day-old male infant. The pregnancy was complicated by polyhydramnios. Based on a careful physical examination. 17th edition. Which of the following is the most likely explanation for these clinical findings? A serious defect in CNS development Explanation: Polyhydramnios suggests obstruction of the gastrointestinal tract or disorders of swallowing due to poor CNS function in utero. born to an 18-yr-old primigravida female. 59. Contractions suggest poor motor activity in the fetus or being fixed in one position (this is unlikely in the presence of polyhydramnios). This is seen in congenital heart disease and other single-organ malformations without a positive family history.) Deformational defects associated with a primigravida mother Uterine fibroids leading to deformational defects A chromosomal disorder associated with deformational defects Renal agenesis Question . 62. (See Chapter 98 th in Nelson Textbook of Pediatrics. If the blood culture is positive.) Begin intravenous antibiotics Obtain blood and CSF for culture and begin intravenous antibiotics Question . the blood culture is the only needed initial culture. Pierre Robin syndrome E. Williams syndrome 2. Deletion of band q11-12 Explanation: E. Waiting for culture results in this situation would be dangerous. he develops grunting. Prader-Willi syndrome . A 30-wk-old average-for-gestational-age male infant is born after 24 hr of ruptured membranes and another 10 hr of labor. He requires intubation. Deletion of elastin allele Explanation: C. 17th edition. as well as cyanosis. Matching: Malformations A. None of the above 1. as the infant is hypotensive and hypoxic.Chapter 98 in Nelson Textbook of Pediatrics. and surfactant therapy. as the infant will die without therapy before the culture is reported positive. ventilation. After birth. Despite these measures he is still hypoxic and hypotensive.3 Explanation: A. a lumbar puncture can be performed. when the patient's condition becomes more stable.) Employ higher risk criteria Give intramuscular penicillin to all newborns Institute tocolysis to prevent preterm birth There is no effective prevention Question . You should now: Do a blood culture and await results Do a Gram stain of the tracheal fluid Do a blood culture and begin intravenous antibiotics Explanation: Premature infants with respiratory distress syndrome (RDS) may look indistinguishable from infants with early-onset sepsis. 17 edition. Trisomy 21 C. and restrictions. 61. A lumbar puncture is not wise in this case. VATER syndrome G. Microdeletion 16p13. flaring. Rubinstein-Taybi syndrome 3. Prader-Willi syndrome F. Williams syndrome D. Rubinstein-Taybi syndrome B. He oozes blood from all venipuncture sites. Endocardial Cushing effects Explanation: B. 17th edition.2 mg/dL) in a 3-wk-old.) 10 Question . cyanotic hands and feet. Seizures began 8 hr later. (See Chapter 91 in Nelson Textbook of Pediatrics. (See Chapter 97 in Nelson Textbook of pediatrics. 64. Tracheoesophageal fistula Explanation: F. 63.) Question .) Question . VATER syndrome. Trisomy 21 5. This infant's Apgar score is: 7 8 9 Explanation: One point in the Apgar score is taken off for color. A 2-wk-old infant is brought to the emergency room in coma with retinal hemorrhages and severe pallor. 130/min. VATER syndrome 8.4. assuming normal growth and development. Mandibular hypoplasia Explanation: D. Anal atresia Explanation: F. An infant has the following findings at 5 min of life: pulse. Diagnostic tests should include all of the following except: . Radial hypoplasia Explanation: F. His diet is breast milk. good muscle tone. 17th edition. VATER syndrome 7. (See Chapter 83 in Nelson Textbook of Pediatrics. The day before admission. breast-fed infant with normal growth and development? Phototherapy Exchange transfusion Phenobarbital None of the above Explanation: No treatment is necessary for the infant described in the question. 65. Which of the following is most appropriate for treating hyperbilirubinemia (11. He was born at home and was first seen by a physician at 10 days of age and placed on amoxicillin for otitis media. and a strong cry and grimace. his parents took him in a four-wheel-drive vehicle on a hot day over a rough road in the mountains. and his condition steadily deteriorated for the next 16 hr. 17th edition. Pierre Robin syndrome 6. 17th edition. 17th edition.) Question . 68. Immediate therapy for the infant described in Question 63 should include administration of: Vitamin A Vitamin B6 (pyridoxine) Vitamin C Vitamin E Vitamin K Question . The death of the infant described in Question 63 could have been prevented by which one of the following measures: . 67.4 in Nelson Textbook of Pediatrics.) Child abuse Hypervitaminosis A Question .4 in Nelson Textbook of Pediatrics. The most likely diagnosis for the child described in Question 63 is: Pyridoxine deficiency Severe scurvy Hemorrhagic disease of the newborn Explanation: (See Chapter 92. 66.Coagulation studies Skeletal survey CT scan Complete blood count Lumbar puncture Explanation: (See Chapter 92. The hyperbilirubinemia is indirect (unconjugated). Treatment with thyroxine improved the jaundice and the other signs.3 in Nelson Textbook of Pediatrics. (See Chapter 92. indicating high TSH and low T4 levels. breast-fed. A-positive. Biliary atresia is always a concern in infants with delayed clearance of jaundice or worsening jaundice after 2 wk of life. breast-feeding (low vitamin K content). Of all possible preventive measures. 17th edition. The next most likely diagnosis is child abuse. administration of vitamin K at birth would have been the most effective. the infant is found to have not gained weight since birth. A 3. Crigler-Najjar syndrome is a possibility and is either autosomal dominant or recessive (check the family history). (See Chapter 91. and the amoxicillin treatment that eliminated normal intestinal bacterial synthesis of vitamin K led to the tragic demise of an otherwise normal infant. hepatomegaly. or ascites. The hyperbilirubinemia is predominantly direct (conjugated). 70. However. there are signs other than jaundice that suggest another disease. African-American infant (former 40-wkgestational-age) was born to an O-positive mother and experienced hyperbilirubinemia. The combination of home delivery (no AquaMEPHYTON administered). white girl with a gestational age of 42 wk is noted to have persistent hyperbilirubinemia at 2 wk of age.600-g. an umbilical hernia. The infant appears apathetic and demonstrates pallor. most infants in coma with retinal hemorrhages have been shaken. 17th edition.Administration of AquaMEPHYTON (vitamin K) at birth Explanation: This infant has a case of severe vitamin K deficiency-hemorrhagic disease of the newborn. she has decreased tone. a grade 2/6 . On physical examination. A 4-wk-old. Galactosemia should be considered.) Home visitor services Discontinuance of antibiotics Proper use of an infant seat An air conditioner Question . and a skeletal survey thus is appropriate. especially in the presence of hypoglycemia. directreacting jaundice. and an anterior fontanel measuring 4 x 6 cm. The most likely diagnosis is: Crigler-Najjar syndrome Gilbert disease Biliary atresia Hypothyroidism Explanation: Hypothyroidism was confirmed by the late arrival of the newborn screening results.4 in Nelson Textbook of Pediatrics. which required 2 days of phototherapy in the newborn nursery after birth.) Galactosemia Question . 69. CMV 4. CMV B. thus. Rubella E. A thorough family history and examination of the child's and parents' blood smear are helpful (because most cases of spherocytosis are inherited as an autosomal dominant trait). Parvovirus F. Myocarditis Explanation: C. Hereditary spherocytosis is a possibility but is relatively rare. The most likely diagnosis is: Anemia of chronic disease Cholestasis secondary to neonatal hepatitis Hereditary spherocytosis Sickle cell anemia hemolytic crisis ABO incompatibility with continued hemolysis Explanation: Jaundice usually resolves in all infants with hyperbilirubinemia due to ABO incompatibility in the first week of life. Fetal anemia Explanation: E. Megaesophagus Explanation: G. Limb hypoplasia Explanation: B. Varicella C. Trypanosoma cruzi 1. Late-onset anemia must be watched for and treated with a packed red blood cell transfusion if the infant is symptomatic. Hydrocephalus . there are few -chains to sickle. Enteroviruses D. the hemolysis continues without evidence of jaundice because the liver can now excrete the bilirubin load. Parvovirus 7. Varicella 5. (See Chapter 92. Sickle cell anemia hemolytic crisis is not encountered this early in life because a considerable amount of fetal hemoglobin remains. Rubella 3. 71. Cerebral periventricular calcifications Explanation: A.) Question .2 in Nelson Textbook of Pediatrics. Toxoplasmosis G. Nonetheless. 17th edition. Patent ductus arteriosus Explanation: D. Trypanosoma cruzi 2. Matching: Congenital infections A. Enteroviruses 6.systolic ejection murmur. and a heart rate of 175 beats/min. Ibuprofen Explanation: B. (See Chapter 83 in Nelson Textbook of Pediatrics. whereas these other issues raise concern and need further evaluation or therapy. (See Chapter 98 in Nelson Textbook of Pediatrics. Neonatal heart failure B. 17th edition.) No stools Question . Misoprostol Explanation: D. Oligohydramnios C. Goiter 1. Valproate Explanation: C. Spina bifida 6. A term baby of an uncomplicated pregnancy is born limp. Arthrogryposis E. Spina bifida D. 74. 17th edition) Question . 72. Possible considerations in the differential diagnosis include all of the following except: Prolapsed umbilical cord Central nervous system trauma . Facial-ear anomalies F. Matching:Maternal drugs and the fetus and newborn A. (See Chapter 85 in Nelson Textbook of Pediatrics. 17th edition Question . Goiter 4. and apneic after a difficult vaginal delivery. Accutane Explanation: E. Oligohydramnios 5. 73. Reasons to avoid the early discharge of a normal term infant include all of the following except: Jaundice evident on day 1 Positive VDRL Bleeding after circumcision Two successful feedings Explanation: At least two successful feedings are reassuring. cyanotic. Arthrogryposis.Explanation: F. Toxoplasmosis. Blue cohosh herbal tea Explanation: A. Facial-ear anomalies 2. Neonatal heart failure 3. Iodide Explanation: F. (See .35-dihydroxyvitamin D Administer magnesium sulfate Explanation: Infants of diabetic mothers (IDM) often experience hypocalcemia within 24-48 hr after birth. 17th edition. due to traction injury of the brachial plexus. With time. calcium gluconate (10%). Thereafter. The IDM also develops hypomagnesemia. The most appropriate step to manage this infant's condition is to: Administer glucose Administer pyridoxine Administer 1. hypothermia. The Apgar scores were 8 and 9 at 1 and 5 minutes. After intubation and resuscitation. 76. 75. 2 mL/kg.Administration of morphine to the mother Klumpke paralysis Explanation: Klumpke paralysis involves injury to the 7th and 8th cervical nerves and the 1st thoracic nerve. if uncorrected. It is usually unilateral. Pregnancy was complicated by severe diabetic vasculopathy and placental insufficiency. due in part to an attenuated parathyroid gland response to reduced calcium levels.0 mg/dL. although the baby does not cry when the toes are pinched.)46 Congenital myasthenia gravis Neurosyphilis Question . The most likely diagnosis is: Congenital botulism Narcotic overdose Transection of the spinal cord Explanation: Transection of the spinal cord may occur in vertex and breech positions and may be noted with normal vertebral body anatomy. A 2. It would manifest as in this patient.100-g infant of a diabetic mother experiences seizures on the first day of life. respectively. (See Chapter 88 in Nelson Textbook of Pediatrics. During the first 6 hr of life.) Administration of local anesthetic into the fetal scalp Question . 17th edition. and bowel and bladder dysfunction. Laboratory studies revealed blood glucose of 80 mg/dL and calcium of 6. hypotonia resolves into hypertonia and hyperreflexia. (See Chapter 88 in Nelson Textbook of Pediatrics. the child was well and tolerated formula feedings well. with the development of late decelerations and the need for emergency caesarean section. results in persistent hypocalcemia due to the dependency of parathyroid function on magnesium. Jitteriness developed at 10 hr of age and progressed to tonic-clonic seizures at 18 hr of age. which. the patient in Question 70 remains limp but appears aware and looks around. was given repeatedly without affecting the frequency of seizures. and also with shock. 17th edition.Chapter 95 in Nelson Textbook of Pediatrics.) Administer lorazepam . Opioids and benzodiazepines are being used for analgesia and sedation. Pneumothorax Question . Increased vital capacity C. Intrathoracic airways Explanation: Airway edema or inflammation will produce this constellation of findings. 1. 3. Increased residual volume D. The child has no spontaneous respirations. Respiratory depression Explanation: As a result of sedation and analgesia.Oxygenation may not be affected if he is breathing enriched oxygen.) E. (See Chapter 357 in Nelson. A 3-mo-old infant is brought to the emergency room because of lethargy and poor feeding.) E.The Respiratory System Question Nelson Self Assessments website 17th Edition Question . Decreased functional residual capacity Explanation: Cardiac failure produces interstitial and alveolar edema. Her breathing rate is 30/min. Alveolar surfactant B. 17th ed. Medullary respiratory neurons Question . No wheezing or crackles are audible.The reduced breath sounds and crackles could be due to atelectasis. 17th ed. Cardiogenic shock D.With smaller airway involvement. Lung interstitium C. Increased peak expiratory flow B.13. Occasional crackles can be heard over both bases. (See Chapter 357 in Nelson Textbook of Pediatrics. Pulmonary edema B. Pediatrics. Pneumonia C. She has a prolonged expiratory phase. Diaphragm E. The mandatory ventilatory rate has been decreased from 20 to 10 breaths/min in preparation for removal of the endotracheal tube. Which of the following functional findings is most likely in a 12-yr-old girl who has developed acute rheumatic carditis with severe mitral insufficiency? A.An abnormality in which of the following components of the respiratory system is most likely to be involved in the genesis of these manifestations? A. The arterial PO2 is 120 mm Hg and the arterial PCO2 is 75 mm Hg. Breath sounds are decreased bilaterally. (See Chapter 357 in Nelson Text. which is manifested by an acute respiratory acidosis and hypercarbia. Increased CO diffusion capacity The Respiratory System Nelson Self Assessments website 17th Edition 1 .) D. A 3-yr-old boy is undergoing mechanical ventilation 12 hr after repair of an atrial septal defect. Auscultation of the chest demonstrates that the breath sounds are slightly reduced on the left side. The most likely reason for this child's acidosis is: A. he is hypoventilating. 2. this patient may eventually demonstrate wheezing. which will reduce the FRC. Other findings include subcostal retractions and use ofthe abdominal muscles during expiration. She appears pale. 17th ed. The arterial pH is 7. B.30 are measured in a blood sample obtained from the left radial artery.Assuming the FIO2 in the hood is 45%. Intercostal retractions are caused by: A. A 1-mo-old infant is breathing supplemental oxygen from a hood at a measured concentration of 45% after developing respiratory distress.(See Chapter 357 in Nelson Textbook of Pediatrics. Which of the following is most likely to be the cause of the respiratory distress episodes? A. A 2-wk-old infant begins to experience episodes of acute respiratory distress after undergoing repair of esophageal atresia. D. C. D. a PCO2 of 50 mm Hg.) E. 17th ed. D. B. anxiety. and a prolonged expiratory phase. The infant becomes agitated and demonstrates decreased breath sounds bilaterally. B. Which of the following interpretations is most consistent with these findings? A.Question . E.Weakness of both the extrathoracic and intrathoracic trachea can produce episodes of cyanosis and respiratory distress often triggered by crying. retractions will develop. C. 5. 17th ed. E. or pain. The episodes appear to be triggered by crying. The blood sample is venous The infant has a right-to-left shunt via the ductus arteriosus Oxygen diffusion across the alveolar-capillary membrane is impaired The patient is hypoventilating The blood gas anomalies are caused by ventilation-perfusion inequality Explanation: The patient has both hypercarbia and hypoxia. 6. Recruitment of the scalene and sternocleidomastoid muscles Question .) The Respiratory System Nelson Self Assessments website 17th Edition 2 . with the development of cyanosis and bradycardia. (See Chapter 357 in Nelson Textbook of Pediatrics. C.The intercostal space is even more compliant than the chest wall. one would expect a PaO2 much higher than 60 mmHg. (See Chapter 357 in Nelson. Patent ductus arteriosus Recurrent laryngeal nerve injury Choanal atresia Pulmonary hypertension Tracheomalacia Explanation: Tracheomalacia is quite common after repair of esophageal atresia. 4. Direct traction applied by the diaphragm on the ribs Contraction of the internal intercostal muscles Contraction of the external intercostal muscles Decreased pleural pressure Explanation: When the compliance of the chest is greater than the negative intrathoracic pressure generated during inspiration. 17th ed. bilateral rhonchi. A PO2 of 60 mm Hg.The most common cause of hypoxia in children with acute respiratory disorders is a ventilation/perfusion mismatch. Physical examination conducted when he is calm reveals mild subcostal retractions with a respiratory rate of 45 breaths/min.) Question . and a pH of 7. A 5-mo-old infant develops signs of respiratory distress after coughing and sneezing for 3 days.CPAP may be tried under very controlled circumstances but rarely avoids intubation. Arterial PO2 is 80 mm Hg. but persistent hypoxia (80% saturation on pulse oximetry) while the patient is on 100% FIO2 is an indication for intubation and mechanical ventilation. Administration of diuretics The Respiratory System Nelson Self Assessments website 17th Edition 3 . 17th ed.Poor peripheral perfusion and weak pulses (and presumably low blood pressure) in this setting should respond to expansion of the intravascular volume with normal saline. but may have also been exacerbated by the PEEP.) C. Intubation of the trachea and mechanical ventilation Explanation: This child is in respiratory failure. Reduce PEEP to 3 cm H2O Reduce peak inspiratory pressure to 28 cm H2O Reduce ventilatory rate to 26 breaths/min Administer 10 mL/kg of normal saline Explanation: The PaO2 and PCO2 are quite appropriate and in the target range for appropriate therapy. manifested as a prolonged capillary refill time and weak arterial pulses. He has marked subcostal and intercostal retractions and a respiratory rate of 80 breaths/min.Which of the following measures is most likely to result in an improvement in this infant's perfusion? A. Diffuse crackles can be heard bilaterally. The central venous pressure measured at the right atrium with an umbilical venous catheter is 2 mm Hg (or approximately 3 cm H2O). Breath sounds are markedly diminished on both sides.Which of the following is the most appropriate immediate course of action? A. 17th ed. 7. C. and ventilatory rate is 30 breaths/min. Administration of corticosteroids B. A premature infant is undergoing mechanical ventilation for respiratory distress syndrome.The poor perfusion may have preceded the initiation of PEEP. Administration of normal saline E. There is no stridor. and arterial PCO2 is 38 mm Hg. B. Begin an infusion of dopamine at 5 µg/kg/min Question . positive end-expiratory pressure (PEEP) is 5 cm H2O. Arterial oxygen saturation in 100% oxygen by non-rebreather mask is 80%.(See Chapter 357 in Nelson Textbook of Pediatrics.An arterial blood gas determination may be helpful.Question . D. The skin is pale and peripheral arterial pulses are weak. Peak inspiratory pressure is 32 cm H2O. 8. The infant has decreased peripheral perfusion. Sampling of arterial blood and measurement of arterial pH and blood gases D. (See Chapter 357 in Nelson Textbook of Pediatrics.) E. At a well-child examination the previous day. Which of the following statements defines this situation most accurately? A.) D.If the drive for ventilation was inhibited by the hyperoxia. 17th ed. Decreased hypoxic drive after correction of the hypoxemia has resulted in acute hypercapnia B. His medical history was otherwise unremarkable. the cause of death was determined to be sudden infant death syndrome (SIDS). the PCO2 would be even higher and the pH lower. He responds only to painful stimuli.Question . A fixed intrapulmonary right-to-left shunt is responsible for the limited response to administration of oxygen Question .420 g. (See Chapter 360 in Nelson Textbook of Pediatrics. 9. A 4-mo-old African-American infant was found unresponsive in his crib by his mother in the early morning and could not be resuscitated. Pulmonary hypertension caused by chronic hypoxemia has produced increased ventilation-perfusion inequality C. Immunizations Explanation: Multiple studies have looked at the potential associations between immunizations and SIDS.Compensation cannot totally correct the pH to normal. Movement to a prone position after having been placed supine to sleep C.) D. and the arterial pH is 7. Low birth weight The Respiratory System Nelson Self Assessments website 17th Edition 4 .30. Renal tubular compensation of prolonged hypercapnia has resulted in an elevation of serum bicarbonate levels Explanation: Owing to poor ventilatory muscular effort. He had been placed for sleep on his back but was found on his stomach. After administration of supplemental oxygen. the arterial PO2 is 150 mm Hg. The patient has become dehydrated E. autopsy.None has ever demonstrated a relationship with SIDS. Prematurity B. the patient has been hypoventilating for a long enough time to allow renal tubular reabsorption of bicarbonate to compensate for the prolonged hypercarbia (respiratory acidosis). (See Chapter 357 in Nelson Textbook of Pediatrics. and review of the medical history. Which of the following factors has not been found to be associated with greater risk of SIDS? A. His respiratory rate is 40 breaths/min. 17th ed. the arterial PCO2 is 70 mm Hg. he had been found to be in good health and received his routine immunizations. 10. His arterial oxygen saturation in room air is 87%. African-American heritage E. After a thorough scene investigation. A 16-yr-old boy who has been diagnosed with a yet uncharacterized form of muscular dystrophy develops increased somnolence. He was born at 36 wk of gestation and weighed 2. Exposure of the infant to environmental tobacco smoke after he or she is born D.) B. as some believe that it reduces the risk of SIDS. Brainstem autonomic control of heart rate and blood pressure Question . All of the following measures are recommended by the American Academy of Pediatrics to reduce the risk of SIDS except: A.) E.Question . 17th ed.) E.(See Chapter 360 in Nelson Textbook of Pediatrics. There is no association between smoking and SIDS Question . Smoking by the father in the prenatal period C.Although C is also important.This together with rebreathing in the prone position may explain many cases. 17th ed. the prenatal exposure is more dominant. D.(See Chapter 360 in Nelson Textbook of Pediatrics. D. Smoking by the mother in the prenatal period Explanation: This is the epidemiologically correct answer. Placing babies on their back to sleep Avoiding waterbeds. (See Chapter 360 in Nelson Textbook of Pediatrics. 13.It is controversial and is not consistently agreed on as a protective factor and is not recommended by the AAP. Smoking by the mother prenatally only in association with alcohol use E. B. 17th ed.Both B and C should be suspected when more than one case of SIDS occurs in a family or if there are non-healthy affected family members. 11. sofas and other soft surfaces for sleep Avoiding overheating during sleep Using a pacifier if the infant is not breast feeding Explanation: Pacifier use is interesting. Avoiding pillows in the infant's sleep environment The Respiratory System Nelson Self Assessments website 17th Edition 5 . B. C. the strongest risk factor associated with SIDS is: A. 12. C. C has also been associated with SIDS but its incidence is unknown. In the clinical scenario described in Question 10. which of the following physiologic abnormalities is most likely to be related to the child's sudden. Of the following. B has been associated with SIDS but is uncommon. unexpected death due to SIDS? A. Increased susceptibility to bacterial infection Prolonged Q-T interval Medium-chain fatty acid metabolic abnormality Arousal responsiveness from sleep Explanation: Arousal responsiveness from sleep is thought to be the most common mechanism for SIDS. ) E. 17th ed.) C. Neuropsychological testing Lateral soft tissue radiograph of the neck CT study of the upper airway Diagnostic testing for obstructive sleep apnea Explanation: Although all of these are useful. and his BMI is 20 kg/m2. B.What is the most appropriate next step in diagnosis? A. the diagnosis of airway obstruction is made by history and physical examination E. The father is obese and on CPAP for obstructive sleep apnea. 17th ed. difficulty breathing. 15. Findings on physical examination are completely unremarkable except for 2+ tonsillar hypertrophy and some mouth breathing. and obstructed breathing at night.Question . Chronic nasal infection (chronic adenoiditis) Chronic sinus infections that have failed medical management Recurrent bouts of acute otitis media Recurrent otorrhea in children with tympanostomy tubes Recurrent pharyngotonsillitis Explanation: In this situation tonsillectomy alone is effective treatment. or a history of expelling foul-tasting and foul-smelling cheesy lumps D. chronic sore throat. C. 16. a frequent clinical presentation is halitosis.Peritonsillar abscesses may obstruct the airway and are treated with intravenous antibiotics (penicillin is OK) and incision and drainage or aspiration. A 7-yr-old African-American boy is brought to your office by his parents. C. Most episodes of acute pharyngotonsillitis are viral B. E. which occurs acutely. Tonsillectomy alone is usually performed for recurrent or chronic pharyngotonsillitis Question . 14. 17th ed. Which of the following is not an indication for adenoidectomy alone? A.(See Chapter 368 in Nelson Textbook of Pediatrics.) Question .(See Chapter 369 in Nelson Textbook of Pediatrics.(See Chapter 368 in Nelson. but his parents think he is just a "high-energy" child. Rapid enlargement of one tonsil is typical of pharyngotonsillitis Explanation: Enlargement of one tonsil. In many children. All of the following statements are true except: A. With cryptic tonsillitis. who describe loud snoring. is typical of a peritonsillar abscess and not routine pharyngotonsillitis. His teacher has complained that he seems inattentive and hyperactive. D. B. at some point it is most wise to perform dynamic testing for obstructive sleep apnea in a sleep laboratory. D. ECG and echocardiogram The Respiratory System Nelson Self Assessments website 17th Edition 6 . he stays up until midnight and sleeps until 9:00 A. Narcolepsy E. He goes to bed at 10:30 P. Check pre-catheterization hemoglobin level. and tonsillar hypertrophy strongly suggest obstructive sleep apnea. and falls asleep immediately. His brother will no longer share a room with him because of loud snoring. electrolytes. Delayed sleep phase syndrome C.) D. has new findings of pulmonary hypertension. 18. It is difficult for his parents to wake him for school in the morning at 7:00 A.M. Your colleague is planning a cardiac catheterization to assess the nature and severity of the pulmonary hypertension. The child had a VSD that spontaneously closed by age 2 yr. He reports that he struggles to stay awake and pay attention during the day. but there is mild tricuspid regurgitation with an increased jet velocity across the tricuspid valve. 3+ tonsillar hypertrophy. hematocrit. On the weekends. or sleep paralysis. Findings on physical examination are remarkable for a BMI of 30 kg/m2 and 3+ tonsillar hypertrophy. He usually naps for 1-2 hr after school. The heart size was normal on x-ray study. The child. 17th ed. He denies sudden losses of muscle tone. Order an MRI study of the upper airway E.) C. Order a lateral soft tissue radiograph of the neck D.You see the child in your office for a precatheterization general health assessment. He also orders thyroid function studies. and poor school performance.(See Chapter 369 in Nelson Textbook of Pediatrics. behavior problems.(See Chapter 362 in Nelson Textbook of Pediatrics. who is usually very cooperative.It would be interesting if in addition to snoring the family noted pauses between his noisy sleep-related breathing. Echocardiogram shows normal intracardiac anatomy with no evidence of a PDA. dreaming during the day. BMI. is sleepy and irritable. A 10-yr-old African-American boy is referred to you by the school psychologist for medical management of suspected ADHD because of inattention. Start the child on steroids to shrink the enlarged tonsils The Respiratory System Nelson Self Assessments website 17th Edition 7 . and a prominent pectus deformity. ECG shows normal sinus rhythm and right ventricular hypertrophy.M.In this case. His room air pulse oximetry value is 98%. You receive a follow-up note from your community cardiology colleague that a mutual patient. and clotting factors B. 17. Idiopathic hypersomnia Question . You note prominent mouth breathing. Insufficient sleep B. Obstructive sleep apnea syndrome Explanation: His snoring. but not in school. the pulmonary hypertension is not due to cardiac problems but is most likely to be due to prolonged hypoxia and hypercarbia during sleep.M. Request diagnostic studies for obstructive sleep apnea Explanation: Children with trisomy 21 have an increased risk for obstructive sleep apnea.Question . Which of the following is the most likely diagnosis? A. a 7-yr-old boy with trisomy 21. 17th ed.What is the most appropriate next step in management? A. The second heart sound is loud and the intensity of the P2 component is increased. Dry winter air Question . Family history of epistaxis D. Nose drops should be given until it can be removed E. D. 17th ed. Maxillary sinus Question . B. 22. The patient should be referred electively to a specialist for removal B. 17th ed. Turbinates Nasopharynx Posterior septum Kiesselbach's plexus (anterior septum) Explanation: This is an easily reachable area that is easily irritated by picking or inflammation. B.(See Chapter 362 in Nelson.(See Chapter 363 in Nelson Textbook of Pediatrics. Digital trauma B. Nosebleeds are commonly associated with all of the following except: A.) C. With severe recurrent non-infection-related epistaxis. Hard to distinguish from nasal turbinates The Respiratory System Nelson Self Assessments website 17th Edition 8 . Nosebleeds in children most commonly arise from A.Immediate removal is indicated. C. C. 21. it is also seen with other conditions such as allergies. Menstruation Explanation: The five most common causes of epistaxis are on your hand (the fingers!).) E.Menstruation-related epistaxis is extremely uncommon. Never found to arise in the ethmoid sinus Common in infancy Seen only in children with cystic fibrosis Associated with allergic rhinitis Explanation: Although cystic fibrosis is a common cause of nasal polyps.Question .(See Chapter 362 in Nelson Textbook of Pediatrics. 17th ed.) E. Nasal polyps in children are: A.(See Chapter 362 in Nelson Pediatrics. The parents should remove it immediately D. 17th ed. Removal may be simply done in the office Question . 20. the child may have a coagulopathy such as von Willebrand disease.) C. especially in children younger than 12 yr. When a disk battery is seen as a foreign body in the nose of a child. which of the following is the most important consideration in management? A. Sinus infections E. D. It may leak and cause local tissue damage Explanation: These are particularly dangerous because of the risk of a chemical burn or pressure necrosis in a small space and because attempts to remove them could actually push them from the anterior space to the more distal posterior space. 19. On physical examination he has a barky cough and stridor only with crying. Which of the following is the most likely etiologic agent? A. his temperature is 39oC. During the night. 25. He is well hydrated. pharyngitis. His respiratory rate is 36/min. Grow and develop during the first seven years of life Explanation: Some sinuses are present at birth. 24. C.(See Chapter 365 in Nelson Textbook of Pediatrics. and consolable. and inspiratory stridor.) D. What is the appropriate next step in patient management? A. Influenza virus type A B.) The Respiratory System Nelson Self Assessments website 17th Edition 9 . Are all present at birth B. Parainfluenza virus Explanation: This is the classic presentation of croup. and lowgrade fever. Nasal washing for influenza virus and respiratory syncytial virus Lateral radiograph of the neck Nebulized racemic epinephrine Complete blood count and blood culture Dose of dexamethasone Explanation: In this patient with croup and manifesting stridor only with crying. racemic epinephrine and dexamethasone would be indicated. Develop during the teenage years C. and his pulse oximetry reading is 96%. Mycoplasma pneumoniae Question . E.) D. dexamethasone is indicated. The paranasal sinuses in children: A. 23. Adenovirus E.Aeration as seen on x-ray may occur after sinus formation. Are unlikely to be infected before the age of 12 yr Question . An 18-mo-old girl has a 2-day history of rhinorrhea. Respiratory syncytial virus C. 17th ed.Involvement of the vocal cords (laryngitis in adults) is most often due to parainfluenza virus but may also be due to any of these pathogens. D. A 2-yr-old boy is presented to the emergency department at 3 A.Question . (See Chapter 371 in Nelson Textbook of Pediatrics. whereas others develop after birth.(See Chapter 371 in Nelson Textbook of Pediatrics. with a chief complaint of fever and cough.If there were stridor at rest. she wakes with a barky cough. 17th ed. hoarseness. Easily visualized on plain radiographs E.M. B. able to drink. 17th ed. You once again prescribe dexamethasone.) C. Although this disorder is uncommon in the era of immunization against H. and drooling but without a barking cough. Lateral radiograph of the neck C. Stridor D. C. You are the pediatric consultant for a community emergency department. (See Chapter 370 in Nelson Textbook of Pediatrics. 17th ed.Epiglottitis is uncommon in children immunized against H. fever. D. The third-year medical student asks about the data for the use of steroids in croup. physicians must be aware of this dangerous disease with its requirement for immediate airway protection. A 4-yr-old boy presents with sore throat and fever of sudden onset. and stridor when agitated. You have just seen the fifth patient that day with a classic clinical presentation for croup: barky cough. Reduced hospitalization Question . The department's physician calls to ask advice about a 3-yr-old boy with fever and a cough. He is drooling and sitting upright and leaning forward in a tripod position. 26. toxicity.(See Chapter 371) E. and if it occurs. Direct laryngoscopy in the operating room Explanation: This is the classic presentation for epiglottitis. it does so in unimmunized children or those with an unusual bacterial etiology. Complete physical examination including inspection of the oral cavity The Respiratory System Nelson Self Assessments website 17th Edition 10 . He thinks the patient has croup but is also concerned about epiglottitis. Dose of oral dexamethasone D. Respiratory distress Question . 28. air hunger. Fever B. Shorter hospitalization Decreased need for subsequent medical interventions Oral dexamethasone is as effective as intramuscular administration Decreased need for oxygen Explanation: Dexamethasone has been quite effective in the management of children with mild to moderate croup. 17th ed. Which of the following physical findings is most helpful in attempting to differentiate croup from epiglottitis? A. influenzae type b. Barky cough Explanation: In an unimmunized child. B. Complete blood count and blood culture followed by immediate prophylactic intravenous antibiotics B. influenzae type b. What is the appropriate next step in patient management? A.Question . epiglottitis usually manifests with high fever. Which of the following has not been demonstrated in studies of the use of steroids in croup? A. He has difficulty swallowing and his breathing is labored.(See Chapter 371 in NelsonPediatrics. You are a primary pediatrician in an office where a third-year medical student is doing her clerkship.Its efficacy in reducing the need for oxygen in more severely affected children has not been demonstrated.) E. 27. Drooling E. Blood culture Explanation: The child has bacterial pneumonia. although other organisms may be responsible. E. She had a low-grade fever. B. she has a high fever and brassy cough.The most likely bacterial organism is Staphylococcus aureus. and peribronchial cuffing. 29. (See Chapter 379 in Nelson Textbook of Pediatrics.8oC. White blood cell count is 18.A blood culture is quite appropriate. Over the past 24 hours. and rhinorrhea. Lung puncture C. 17th ed.1o C. Which of the following is the most appropriate antibiotic for this condition? A. On physical examination she is toxic-appearing with a fever of 39. but does have some evidence of respiratory distress with increased work of breathing and retractions. a respiratory rate of 40/min.(See Chapter 371 in Nelson Textbook of Pediatrics. and crackles in the right anterolateral chest. 30. she does not drool and has no dysphagia. She appeared to be improving. She can lie flat. A 2½-yr-old girl has had symptoms of an upper respiratory infection for 1 week.) Question . Viral culture E. D. C. Sputum culture B. which is the most appropriate next step in diagnosis? A. On auscultation her lungs are clear bilaterally.A child this young usually does not produce sputum or require a lung puncture. Erythromycin Ciprofloxacin Ampicillin Gentamicin Nafcillin Explanation: This child has bacterial tracheitis as a complication of a previous viral respiratory tract infection.Viral PCR assay may be better than a viral culture. 17th ed. inspiratory rhonchi. fever and tachypnea developed.) D. and mild to moderate intercostal retractions. generalized hyperinflation. now.Question .000/mm3with 70% granulocytes. however. Chest film shows a shaggy right heart border. Oxygen saturation is 94-95%. Her examination reveals diffuse wheezing. cough. A 2-yr-old girl had an upper respiratory tract infection approximately 5-7 days ago. Cold agglutinins titer The Respiratory System Nelson Self Assessments website 17th Edition 11 .Some physicians might use ceftriaxone to cover these pathogens.Of the following. with worsening cough and increased work of breathing. She did not require any medical intervention. She has a temperature of 39. Question . E. B. and blunting of the right costophrenic angle. Amoxicillin PO C. Cefuroxime IV The Respiratory System Nelson Self Assessments website 17th Edition 12 . No antibiotic therapy D. Oxygen saturation is 93-94%. White blood cell count is 11.More effective diagnostic tests include Mycoplasma PCR and IgM assays. 33. a respiratory rate of 24/min. D. B. C. Which of the following is the most appropriate next step in the management of the patient described in Question 32? A. 32.) B. Erythrocyte sedimentation rate Question . treatment with intravenous vancomycin is necessary. Outpatient management of the patient described in Question 30 is considered in view of the patient's clinical status. Sputum culture Tuberculin skin testing Throat culture Cold agglutinins titer Explanation: It is likely that this patient has Mycoplasma pneumonia. A previously healthy 12-yr-old boy presents with upper respiratory symptoms of 8 days' duration. Ceftriaxone IM E. and inspiratory crackles in both lung fields.Which of the following is the most appropriate next step in diagnosis? A. 17th ed.) E. Clinical features suggest a bacterial pneumonia. more focal consolidation in the right lower lobe. 17th ed. C. Which of the following is the most appropriate treatment option? A.With highly resistant pneumococci. 31.) Question .000/mm3 with a normal differential. with worsening cough and fever. Chest film shows scattered infiltrates in multiple lung fields.(See Chapter 379 in Nelson Pediatrics. 17th ed. The season is autumn. Erythromycin PO Explanation: Erythromycin or azithromycin is quite effective in improving the clinical course of Mycoplasma pneumonia. D. Penicillin PO Cefixime PO Erythromycin PO Cephalexin PO Amoxicillin PO Explanation: High-dose oral amoxicillin will be effective against most pneumococci. (See Chapter 379 in Nelson Textbook of Pediatrics. Examination reveals a temperature of 39°C.(See Chapter 379 in Nelson Textbook of Pediatrics. (See Chapter 379 in Nelson) Question . A 4-yr-old boy has had rhinorrhea and cough for 4 days. Nasopharyngeal swabs for viral antigens E. B. Chest film shows opacification of the right hemithorax. and chest discomfort over the past 2 days. Sweat chloride analysis in siblings The Respiratory System Nelson Self Assessments website 17th Edition 13 . with crackles heard over the right upper posterior chest.This patient obviously had a symptomatic effusion and improved dramatically after withdrawal of 700 mL of cloudy fluid. and poor growth has two sweat chloride values of 36 and 41 mEq/liter. Examination reveals splinting. 34.Which of the following is the most appropriate next step in diagnosis and management? A.It could be pneumonia due to S. with fever. decreased breath sounds.) B. Nasal potential difference measurement Explanation: This is a useful test that has abnormal results in CF. but there are no pneumatoceles.1oC. vancomycin should be added. and oxygen saturation is 88%.000/mm3 with a predominance of granulocytes. C. and dullness to percussion over the right posterior chest. Ampicillin IV Cefuroxime IV Erythromycin IV Erythromycin and ampicillin IV Cefotaxime and vancomycin IV Explanation: The child probably has pneumococcal pneumonia. intermittent wheezing. Fat balance measurement (72-hr stool collection) D. 35. D. E. Cold agglutinins titer Question . A 7-yr-old child with a 3-yr history of cough.(See Chapter 402 in) C. worsening cough. His white blood cell count is 30.Which of the following is the most appropriate treatment? A. Tube thoracostomy drainage Explanation: Thoracentesis is of value both as a diagnostic aid and as a therapeutic procedure.(See Chapter 379 in Nelson Textbook of Pediatrics.If it is a lifethreatening illness. 17th ed. Gram stain of the pleural fluid from the patient described in Question 34 reveals gram-positive cocci in clusters. Additional diagnostic testing to rule out cystic fibrosis should include: A. 36. Bronchoscopy C. A right lateral decubitus film of the chest reveals significant pleural fluid. aureus. DNA analysis for the F508 mutation E. CT imaging of the chest B. Today's DNA testing for the many (in the hundreds) of mutations in the CFTR gene is available and is of great value in equivocal test results. respiratory rate is 28/min.The pneumococcus is becoming resistant to penicillins and even to cephalosporins.Question . His temperature is 40. Sputum culture D. The chest radiograph reveals a normal cardiothymic silhouette but a diffuse ground-glass appearance to the lung fields. Intussusception Peritonitis Pancreatitis Distal intestinal obstruction syndrome (DIOS) Explanation: DIOS. Manometry E.5-kg male infant born at term after an uncomplicated pregnancy and delivery develops respiratory distress shortly after birth and requires mechanical ventilation. A 3. 37. A 12-yr-old child with confirmed cystic fibrosis has experienced cramping abdominal pain intermittently for the past 8 days. A newborn infant fails to pass meconium for the first 48 hr. Over the first week of life. Surfactant replacement fails to improve gas exchange. Contrast imaging of the lower gastrointestinal tract Explanation: The examination will look for a meconium plug or a small left colon (which suggests proximal intestinal obstruction as in atresias). B. 39. The pain is diffuse and unrelated to eating and is not attended by guarding or rebound. (See Chapter 402 in Nelson) E.If the patient has been receiving high-dose pancreatic enzymes. a suction biopsy should also be performed. Serum immunoreactive trypsin assay Question . also called meconium ileus equivalent. 17th ed. a fibrosing colonopathy must also be considered.Question . Sweat chloride assay B. Neonatal pulmonary alveolar proteinosis Explanation: RDS in a term infant not responding to surfactant replacement therapy is most likely to represent neonatal pulmonary alveolar proteinosis.If Hirschsprung disease is considered. C. Total anomalous pulmonary venous return B. 38. 17th ed. The next diagnostic steps would include: A. Results of routine cultures and echocardiographic findings are negative. the hypoxemia worsens.(See Chapter 389 in Nelson Textbook of Pediatrics. The patient denies emesis or diarrhea. Disseminated herpes simplex infection E. The most likely cause of the abdominal pain is: A. Medium-chain acyl-dehydrogenase deficiency The Respiratory System Nelson Self Assessments website 17th Edition 14 . D. Meconium aspiration C. Genotyping the child for CF C. is an obstruction due to impacted stool. CF must be considered.) D.) D.(See Chapter 402 in Nelson Pediatrics. A term female sibling died at 1 mo of age with "respiratory distress. Abdominal distention and emesis have occurred overnight."Which of the following is the most likely diagnosis? A. Cholecystitis Question .In both examples. 17th ed. an endotracheal tube is placed. 41.(See Chapter 370 in Nelson Textbook of Pediatrics.2-kg full-term female infant is delivered by vaginal delivery. Repeat echocardiogram C.) D. which potentially can cure this disorder. Which of the following laboratory evaluations should be obtained in the case in Question 39? A.Question . which acts as a spaceoccupying lesion in the chest. RFLP analysis of the MCAD gene E. If the laboratory analysis confirms SP-B deficiency. A chest radiograph to assess placement of endotracheal tube D. 17th ed. She is initially cyanotic and is in significant respiratory distress. Auscultation of the chest reveals diminished breath sounds in the left hemithorax and a scaphoid abdomen. Urine organic acid screen Question . what is the most appropriate next step in management of the patient described in Question 39? A. 40. Immediate surgery E. 42.(See Chapter 389 in Nelson Pediatrics. Obtain a lung biopsy B. (See Chapter 389 in Nelson Pediatrics. Administer surfactant E. Administration of epinephrine The Respiratory System Nelson Self Assessments website 17th Edition 15 . 17th ed. A 3. The point of maximal impulse (PMI) is shifted to the right side of the chest. After bag and mask ventilation. Blood and tracheal cultures for virus and yeast B. Begin inhaled nitric oxide C. Discuss lung transplantation with the family Explanation: ECMO is a temporizing bridge to lung transplantation. Immediate bronchoscopy B.) D. Placement of a nasogastric tube Explanation: This patient potentially has a diaphragmatic hernia and needs gas to be removed or prevented from entering the bowel.) C. RFLP analysis of the SP-B gene Explanation: Neonatal alveolar proteinosis is due to a genetic mutation causing a deficiency of surfactant protein B. Begin corticosteroids Question .The most important initial intervention is: A. Measles virus and adenovirus may be potential agents. He denies any exercise intolerance or cough but does experience intermittent wheezing on exertion. Physical therapy to correct scoliosis Question .) Question . nonproductive cough.) E. Reassurance that surgery will not be required C. His vital capacity is 80% of predicted. 17th ed. After treatment with an oral antibiotic. and findings on physical examination are normal except for a mild thoracic scoliosis. E. A 14-yr-old boy with a pectus excavatum deformity presents for evaluation. and mild dyspnea. C. Which of the following abnormalities on diagnostic work-up suggests the need for surgical correction? A.(See Chapter 378 in Nelson Textbook of Pediatrics. and wheezing. the child began to show some signs of improvement. or if he continues to grow. FEV1/FVC ratio of 0. A chest radiograph demonstrates hyperlucency. Spinal fusion surgery D. A 15-yr-old boy with thoracic scoliosis undergoes evaluation for surgery. 43. a productive cough. the curve may not change. 45. A 7½-yr-old girl presents with a history of low-grade fever. the curve may worsen. The most likely diagnosis is: A. Repeat assessment in 6 mo Explanation: Depending on where he is in his puberty growth spurt. He states he is not concerned about how his chest appears. Physical examination reveals a mild pectus deformity. Bone density assessment (DEA scan) E. 17th ed. C. However.) B. History is unremarkable. 44. A Cobb angle of 25 degrees is noted on the chest radiograph. Spirometry shows a severe obstructive pattern. the child subsequently experienced increasing dyspnea. Pulmonary alveolar microlithiasis Wilson-Mikity syndrome Follicular bronchitis Bronchiolitis obliterans Explanation: Bronchiolitis obliterans may follow a viral bronchitis or pneumonia. and his exercise tolerance is minimally reduced.What is the most appropriate next step in treatment of this patient? A. Postviral syndrome The Respiratory System Nelson Self Assessments website 17th Edition 16 . B. D.Question .(See Chapter 410 in Nelson 17th ed. D.(See Chapter 410 in Nelson Textbook of Pediatrics.60 on spirometry A Wolff-Parkinson-White pattern on ECG Low ventilatory reserves during a maximal exercise test A total lung capacity of 80% of predicted A peak work capacity of 60% of predicted Explanation: Many children with a pectus excavatum do not need surgery according to results of work capacity studies. B. as many children are later found to have asthma.Question . but the clinical picture is highly suggestive of a sequestration. Next steps in the evaluation should include: A. E. C. Bacterial culture of the nasopharynx C. None of the above The Respiratory System Nelson Self Assessments website 17th Edition 17 . Bronchopulmonary dysplasia Explanation: Children with BPD usually are successfully weaned. Allergy skin testing E.Bronchitis in children is often an incorrect diagnosis. B. fever. Barium esophagram D. but a follow-up chest film at 8 wk demonstrates even more dense consolidation involving the right lower lobe. Spinal muscular atrophy D. The fever abates. On physical examination both height and weight are in the 50-75th percentile.The others in choices A and C often become dependent on their ventilator. Lung biopsy Question .(See Chapter 411 in Nelson Textbook of Pediatrics.) Question . D. 47.) C. which medical condition is most often associated with successful wean off all ventilatory support? A. A previously healthy 2-yr-old girl is given oral antibiotic therapy for a cough. and chest examination is unremarkable. 46. Doppler flow studies of the artery supplying the sequestration will show the artery coming from the aorta. 48. the cough improves. Central hypoventilation B. The cough is nonproductive and occurs during sleep in the early morning hours as well as during the day.(See Chapter 376 in Nelson Pediatrics. No crackles are heard on chest auscultation. A chest roentgenogram is interpreted as normal. The diagnostic procedure most likely to ascertain the cause is: A. A 3-yr-old boy has been coughing daily for 2 mo.) B. and patchy consolidation of the right lower lobe.If a pulmonary sequestration is found.(See Chapter 370 in Nelson Textbook of Pediatrics. There is no evidence of digital clubbing. 17th ed. 17th ed. For the mechanically ventilated child. 17th ed. Sputum cytology and culture Sweat chloride testing Bronchoscopy Complete blood count Trial of bronchodilator therapy Explanation: This is a common pattern seen in some children with asthma. Bronchoscopy Explanation: This child could have many problems such as a foreign body. particularly when the child is active. 49. and hypoxemia. 50. Within 1-2 min he develops tachypnea. 51.) E. Chest CT scan E. Bronchoscopy The Respiratory System Nelson Self Assessments website 17th Edition 18 .) D. D.(See Chapter 380 in Nelson Textbook of Pediatrics. Upper gastrointestinal series B. Lack of committed caregivers C.) C. Lack of appropriate housing B.(See Chapter 380 in Nelson Textbook of Pediatrics. 17th ed. Acute blood loss from postoperative bleeding Anesthetic reaction Bacterial infection and toxin release Acute airway obstruction from aspirated material Explanation: Acutely after an aspiration there is a mechanical obstruction with irritation.(See Chapter 411 in Nelson Textbook of Pediatrics. A 2-yr-old boy undergoing outpatient surgery for tonsillectomy vomits while recovering from general anesthesia in the postoperative recovery area. Of the following diagnostic tests. the most sensitive for detecting recurrent airway aspiration is: A. which can cause bronchospasm. Lack of telephone Question . C. 17th ed. A gastroesophageal radionuclide scintiscan D. Lack of private insurance Explanation: Communication and being able to provide constant care are keys to home ventilator management.Later a chemical pneumonia and possibly bacterial infection may supervene.Question . chest retractions. Atelectasis Question . 17th ed. Modified barium swallow with video fluoroscopy Explanation: This dynamic study will often demonstrate aspiration during swallowing but may not show aspiration from emesis or reflux. B. Which of the following is the most likely explanation for these findings? A.Insurance status could be private or public. All of the following may present a barrier to home discharge on a ventilator except: A. "Small" is relative to the size of the bronchus and is thus age dependent.) D. Popcorn C. He feeds adequately and is gaining weight but frequently spits up. 54.Airway lesions may also occur in the absence of any cutaneous lesions. 17th ed. Tracheomalacia B. sunflower seeds. Asymmetric subglottic narrowing Explanation: Airway hemangiomas may produce stridor and crouplike symptoms with viral upper respiratory tract infections. 52. The noisy breathing is accompanied by moderate signs of inspiratory obstruction including suprasternal and subcostal retractions. 53. Laryngeal cyst Question . Tonsil and adenoid hypertrophy E.Facial hemangiomas distributed in a "beard pattern" carry the highest risk for an associated airway hemangioma.(See Chapter 372 in Nelson Textbook of Pediatrics.) B. Enlarged adenoids D. Raw carrot fragments B. Gastroesophageal reflux C. Airway radiographs are most likely to show: A. A 4-wk-old healthy-appearing term infant is evaluated in the office for stridor. Her parents report two brief episodes of croup. which has persisted since birth.(See Chapter 375 in Nelson Textbook of Pediatrics. particularly peanuts Explanation: Anything small enough can get into the bronchus. Nails E. Laryngomalacia Explanation: Laryngomalacia is common and often produces noisy breathing that worsens with viral upper respiratory tract infections or in the supine position. Coins The Respiratory System Nelson Self Assessments website 17th Edition 19 . Vascular ring C. Tracheomalacia E.Question . 17th ed.) D. A 1-cm-diameter hemangioma is present on her right thigh. and the like should not be given to small children. The most common bronchial foreign body is: A. The most likely cause of his symptoms is: A. A 3-mo-old infant has had progressively worsening biphasic stridor. 17th ed.(See Chapter 370 in Nelson Textbook of Pediatrics. Subglottic hemangioma Question . Nut fragments.Nuts. The most likely cause of his respiratory distress is: A.) B. 17th ed. B. 17th ed. A 6-mo-old boy presents with biphasic stridor and a recent episode of croup. Laryngopharyngeal reflux with reflux laryngitis Recurrent respiratory papillomatosis A malignant laryngeal neoplasm A congenital laryngeal cyst Vocal nodules (screamer's nodes) Explanation: Chronic hoarseness or deepness of the voice that is exacerbated with talking.) The Respiratory System Nelson Self Assessments website 17th Edition 20 . He has no symptoms of airway obstruction.) D. Cough-variant asthma B. which has been present for several months. Viral laryngotracheobronchitis D.(See Chapter 370 in Nelson Textbook of Pediatrics. C. E.Some patients also develop reactive airways and reflux. His past history reveals that he was a premature infant who was intubated and ventilated for 6 wk.Treatment is symptomatic.(See Chapter 375 in Nelson Textbook of Pediatrics. Gastroesophageal reflux E. The cough is dry and barking and occasionally associated with expiratory wheezing. no surgery is needed.Question . He has had minimal response to bronchodilator therapy. or crying is common.(See Chapter 373 in Nelson Textbook of Pediatrics. Acquired subglottic stenosis Explanation: Subglottic stenosis may be congenital or acquired. A 4-yr-old boy with two older sisters is evaluated for symptoms of chronic hoarseness and strained voice. Sinusitis C. Direct laryngoscopy will confirm the diagnosis. The hoarseness is worse in the evening and lessens in the morning. Reactive airways disease/chronic lung disease E. The cough has persisted since he was discharged from the hospital after his TEF repair. The most likely cause of his symptoms is: A. D. singing. 57. Tracheomalacia Explanation: Tracheomalacia is very common after a TEF repair. Subglottic stenosis Question . 17th ed. A 4-yr-old boy with a history of tracheoesophageal fistula (TEF) repair at birth is evaluated for a chronic cough. 56. The most likely cause of the chronic cough is: A. Vascular ring C. 55. Reflux laryngitis Question . She herself has moderate asthma and mild seasonal allergies and would like to know if this means her son will have asthma also. he develops progressive clear nasal drainage. Pulmonary function tests C. cough. Mild eczema E.) D. but always outside. A high index of suspicion based on the clinical presentation in association with a suspected exposure The Respiratory System Nelson Self Assessments website 17th Edition 21 . Chest radiograph and complete blood count B. Trial of nebulized albuterol B.Which of the following is not a risk factor for the persistent wheezing phenotype in this infant? A. Maternal history of allergies Explanation: Drug or food allergies in themselves are not a risk factor for asthma. and malaise. Examination reveals an ill-appearing child who is mildly tachypneic with bibasilar crackles. then 1 mg/kg twice daily for 5 days Question . 17th ed. intercostal retractions. His parents state that his 4-yr-old sibling has had cold symptoms for about 4 days. Recurrent episodes of wheezing Question . humidified oxygen regardless of the diagnosis. chills.) C. mild respiratory distress with a respiratory rate of 48/min. 59. he did help his grandmother clean her pigeon coop the day before presentation.(See Chapter 359 in Nelson Textbook of Pediatrics.Question . A 2-mg/kg load of oral corticosteroids. This was his only hospitalization. A sweat chloride test E. An 11-mo-old infant is admitted to the hospital in December because of failure to thrive and recurrent pneumonia. Within 24 hr of his hospitalization. Passive smoke exposure B. 17th ed. however.(See Chapter 376 in Nelson Textbook of Pediatrics. 58. but he has been seen on several occasions in the emergency department of their local hospital. There are no ill contacts. and diffuse wheezing throughout the lung fields. Nasopharyngeal wash for RSV antigen and viral culture D. She adds that there are no pets in the home and that his father does smoke. His oxygen saturation is 88% on room air.The diagnosis of hypersensitivity pneumonitis would be based primarily on: A.What is the most appropriate next step in the acute treatment of this patient? A. dyspnea. Maternal history of asthma C. A 23-mo-old male infant with mild eczema is presented for evaluation of recurrent wheezing episodes. An 8-yr-old boy presents with a 24-hr history of fever. His mother reports that he developed recurrent wheezing with colds following acute RSV infection at age 6 mo for which he was hospitalized and received oxygen by nasal cannula and nebulizer treatments. 60. Oxygen supplementation via nasal cannula Explanation: Any previously well patient who becomes hypoxic and is in respiratory distress must be given warmed. You inform the child's mother that her son may be at risk for persistent wheezing. and an episode of hemoptysis.) B.. D. Restrictive pattern on pulmonary function tests Question . Oxygen saturation of 91% on room air E. Silo filler's disease is typically a result of: A. Examination reveals a normal-appearing young man who is mildly tachypneic with normal findings on lung auscultation. Flexible fiberoptic bronchoscopy with bronchoalveolar lavage E. C.Smoking this agent when it is used as a herbicide is also toxic. Cystic fibrosis D.) C.Which of the following is the most ominous prognostic indicator? A.) D. B.g. 17th ed. Complaint of vomiting and diarrhea D.Eosinophilia is a major clue!(See Chapter 384 in Nelson Textbook of Pediatrics. Cyanide toxicity Question . lung ausculation reveals diffuse end-expiratory wheeze. 61. A 17-yr-old boy previously well presents with a history of cough and dyspnea 24 hr after working in a corn silo. intermittent wheezing.(See Chapter 383 in Nelson Textbook of Pediatrics. Ingestion of dilute solution B. Skin testing and serum precipitins to the suspected antigen Question . Asthma C.Silo filler's disease results from fermentation and gas production in a closed space. Carbon monoxide poisoning Hydrocarbon toxicity Ammonia toxicity Nitrogen dioxide toxicity Explanation: Inhaled chemical or gaseous agents produce airway inflammation. 17th ed.(See Chapter 383 in Nelson Textbook of Pediatrics. A 16-yr-old boy attempted suicide by ingesting the herbicide paraquat. Pulmonary infiltrates with eosinophilia (L?ffler syndrome) Explanation: This disorder has many potential causes. There is marked peripheral eosinophilia on complete blood count. intermittent low-grade fever.Explanation: The history and clinical course are the key. or it may be a primary disease. especially if the patient gets better every time he or she is removed from the antigen (e. A 6-yr-old girl presents with a 1-mo history of mild cough. Pulse rate is 100/min. 63. 17th ed. on weekends for factory workers). There are no ill contacts. 17th ed. Mycoplasma pneumonia E. Pulmonary lymphoma The Respiratory System Nelson Self Assessments website 17th Edition 22 . respiratory rate is 25/min. Ingestion of 40 mg of paraquat per kg of body weight Explanation: The dose is the most important.) E.(See Chapter 384 in Nelson Text Pediatrics.The most likely diagnosis is: A. Chest film shows nonspecific bilateral diffuse infiltrates. 62. cough. and room air oxygen saturation is 89%.(See Chapter 379 in ) Question . 64. A 15-yr-old boy presents with a history of chronic cough productive of yellow-green sputum. Admit for IV antibiotics providing aerobic and anaerobic coverage Explanation: Most lung abscesses are the result of aspiration and contain anaerobic bacteria. respiratory rate is 35/min and room air oxygen saturation is 90%. A 6-yr-old girl presents 72 hr after undergoing a tonsillectomy/adenoidectomy with fever. Change antibiotics to IV ampicillin/sulbactam Obtain chest CT scan Add oral macrolide antibiotic Send blood for an immune work-up Begin IV vancomycin Explanation: This child probably has a pneumococcal infection with resistant organisms that has not responded to cefuroxime. D.(See Chapter 397 in Nelson Textbook of Pediatrics. and poor weight gain. Findings on lung examination and chest film are unchanged from admission. Ventilation-perfusion scan C. Complete blood count demonstrates marked leukocytosis with left shift. 66.CBC reveals marked leukocytosis.Two days later the child is still febrile with a temperature of 102oF. with crackles. pulse rate is 125/min.The child was hospitalized and provided with supplemental oxygen and started on high-dose IV cefuroxime. Chest radiograph shows a left lower lobe consolidation. with dullness to percussion. and respiratory rate 40/min. Bronchography E. Bronchoscopy with bronchoalveolar lavage B. dyspnea on exercise. his pulse rate is 95/min and respiratory rate is 24/min. respiratory rate is 40/min. nonproductive cough. 17th ed.Which of the following is the most appropriate next step? A.The gold standard technique for demonstrating bronchiectasis is: A. He has a history of recurrent sinopulmonary infections.(See Chapter 396 in Nelson Pediatrics. Examination reveals an ill-appearing child. with no need for drainage. A 4-yr-old previously well girl presents with fever (temperature of 103°F). digital clubbing. dyspnea. 17th ed. Thin-section high-resolution CT scan of the chest Explanation: CT scanning of the chest is the diagnostic test of choice to demonstrate bronchiectasis. and right-sided chest pain. and left-sided chest pain.) The Respiratory System Nelson Self Assessments website 17th Edition 23 .Which of the following is the most appropriate initial management? A. dyspnea. her lungs have decreased air movement on the right. Initial examination reveals an ill-appearing child. The patient is a thin-appearing young man. pulse rate 110/min. her temperature is 104°F. and appearance on chest radiograph is consistent with a pulmonary abscess.) D.Question . B. auscultation of the lungs demonstrates diffuse fine crackles and end-expiratory wheezing. Auscultation demonstrates decreased air movement on the left side. Chest MRI study Question .They can usually be managed with high-dose antibiotics. 65. E. C. Hysterical hyperventilation Question . and cyanosis of sudden onset 1 wk after the birth of her first child. neonatal mucus plus syndrome. Inguinal hernias The Respiratory System Nelson Self Assessments website 17th Edition 24 . and percussion Explanation: This patient has atelectases and needs assistance to achieve good inspiratory efforts. and cyanosis. A spiral CT study is a useful test to determine the presence of a pulmonary embolism. inguinal hernia. C. intussusception. An 18-yr-old female patient presents with chest pain.) E. B. Then dyspnea and tachycardia develop. D. A 3-yr-old child does well for the first 6 hr following surgery. E. Colonic mucosal thickening D. D. C. Arrange for percutaneous drainage in interventional radiology department Prescribe oral macrolide antibiotic and follow-up evaluation within 24 hr Arrange for thoracotomy and decortication Arrange for bronchoscopic drainage of the abscess Question . (See Chapter 402 in Nelson Textbook of Pediatrics. (See Chapter 395) E. 68. A normal-appearing chest radiograph with significant hypoxia is classic for pulmonary embolism.) Question . Appendicitis C.(See Chapter 392 in Nelson Textbook of Pediatrics.B. Chest tube placement Intravenous antibiotics Positive pressure ventilation Fiberoptic bronchoscopy Cough. 17th ed. Her chest radiograph is nondiagnostic. with rapid shallow respirations. C. Gastric outlet obstruction Explanation: Other causes of intestinal obstruction are possible and include neonatal meconium ileus. D. E. deep breathing. dyspnea. 17th ed. B.Which of the following is the most appropriate first step in treatment? A.Careful use of pain medications to control chest or abdominal pain and to avoid oversedation is also valuable. All of the following are gastrointestinal manifestations of cystic fibrosis except: A. Preeclampsia Legionella pneumonia A fractured rib A pulmonary embolism Explanation: Pulmonary embolism must be considered with the sudden onset of chest pain. 69. The most likely diagnosis is: A. meconium ileus equivalent (from insufficient use of pancreatic enzyme replacement). tachypnea. 67. etc. congenital ileal atresia. On physical examination the patient has decreased breath sounds and coarse rales on the right. and appendiceal obstruction. Intussusception B. but her PaO2 is 60 mm Hg on 40% oxygen. 500 black and 1 case per 17. Cyanosis B. Nasal polyps The Respiratory System Nelson Self Assessments website 17th Edition 25 . WI282X mutation in 60% of Ashkenazi Jews F. Clubbing C. The most common manifestation of preliminary involvement in children with cystic fibrosis is: A. F508 as the dominant mutation E. 17th ed. Severity of lung disease is not predictable by the gene mutation Question .Question . Autosomal recessive inheritance C. (See Chapter 402 in Nelson Textbook of Pediatrics. More than 700 gene mutations D. 70. 71.) B. Cough Explanation: The cough begins as dry and nonproductive but progresses to loose with production of purulent sputum.000). All of the following statements regarding cystic fibrosis are true except: A. Incidence of 1 case per 3. Wheezing E.) D. (See Chapter 402 in Nelson Textbook of Pediatrics.000 white infants Explanation: The incidence of cystic fibrosis is highest in northern European whites (1 in 3500) and lowest in Asian infants in Hawaii (1 in 90. 17th ed. IV verapamil C. Antibiotic therapy Blood transfusion The CVS System Nelson Self Assessments website 17th Edition 1 .1. Parents report that the child was well until 3 days earlier. not the result. Reentrant SVT Explanation: This is a classic presentation of supraventricular tachycardia in infancy.The Cardiovascular System Question . Blood culture Question . but in infants in heart failure. Anemia E. Adenosine therapy Explanation: Intravenous rapid push adenosine is the drug treatment of choice for all cases of SVT. Always be sure the parents are not giving the child over-the-counter cold remedies that contain sympathomimetic agents. It may also be present in utero. (See Chapter 428 in Nelson Pediatrics. when poor feeding began. 17th ed. Verapamil was once used in older patients. Pulse rate is 280 beats/min. 17th ed. The most appropriate diagnostic test for this patient would be: A.) B. 2. and blood pressure is 80/50 mm Hg. 3. The child is in heart failure. What would be the most likely diagnosis in the case in Question 1? A. ECG Explanation: This is a very high heart rate for a simple sinus rhythm. Sepsis D. of heart failure. (See Chapter 428 in Nelson Textbook of Pediatrics.) D. Pneumonia Question . It is usually idiopathic and transient but does require rapid diagnosis and treatment. in infants and in patients of all ages. ABG analysis E. Chest radiograph B. but the excessively high heart rate may be the cause. (See Chapter 428 in Nelson Textbook of Pediatrics. verapamil has serious adverse effects (cardiac arrest) owing to its negative inotropic effects on an already failing myocardium.) D. A 1-mo-old female infant is presented with a chief complaint of poor feeding and lethargy. 17th ed. Ventricular tachycardia C. CBC C. Physical examination shows a gallop rhythm and an enlarged liver palpable 2-3 cm below the right costal margin. which may include "herbal" remedies that could contain ephedra. Electrical defibrillation B. What would be the most appropriate next step in treatment for the case in Question 1? A. respiratory rate is 50/min. This lesion is common as a dysplastic valve in Noonan syndrome and in Alagille syndrome.) E. 17th ed. (See Chapter 415 in Nelson Textbook of Pediatrics. or unless symptoms are exacerbated by exercise.) D. Alagille syndrome E. which is due to mutations in Jagged 1. Noonan syndrome B. Tetralogy of Fallot C. experience with device closure in the cardiac catheterization laboratory has been quite successful. and this procedure is now approved by the FDA. Isolated PS is also noted to have an increased frequency of Jagged 1 mutations without Alagille syndrome. The best management approach in a 10-yr-old with an ostium secundum atrial septal defect with a 3:1 shunt is: A. C. Transvenous device closure Explanation: Although surgical closure is highly successful. dyspnea. 5. Mutations of Jagged 1 gene The CVS System Nelson Self Assessments website 17th Edition 2 . 6. Phlebotomies for recurrent polycythemia Question . Observation until pulmonary hypertension develops E. Pulmonary stenosis is associated with all of the following Except: A. chest pain in children is often due to musculoskeletal or pulmonary disorders. This is less likely to be coronary vessel disease and more likely to be anxiety. Children may complain of chest pain after an adult family member has a myocardial infarction. D.Question . 17th ed. Chest pain at rest in a 10-yr-old boy is most likely to be due to all of the following Except: A.) D. or syncope with exercise. (See Chapter 420 in Nelson Textbook of Pediatrics. 17th ed. (See Chapter 419 in Nelson Textbook of Pediatrics. Rheumatic fever Explanation: Pulmonary valve lesions in rheumatic fever are very unusual. 4. Nonetheless. B. Asthma Overuse injury Musculoskeletal trauma Coronary atherosclerosis Explanation: Unless there is a strong family history of premature myocardial infarction or sudden death. Anxiety Question . Observation until heart failure is present B. Prophylaxis for endocarditis each month C. the clinician should be aware of more serious causes in children with chest pain. On examination. Short P-R interval B. The initial management of a hemodynamically stable 2-mo-old infant with supraventricular tachycardia should include: A. Recurrent emesis Chronic wheezing Tracheomalacia Heart failure Explanation: The vessels involved in vascular rings do not produce a shunt or excessive blood flow. This is a soft systolic murmur. Renal tubular acidosis Explanation: There are rarely any extracardiac problems in patients with WPW. C. breath holding. Ebstein anomaly E. 17th ed.Question . Slow upstroke of QRS complex C. Complications of vascular rings include all of the following Except: A. The day of admission she had a syncopal event. If this is unsuccessful. she has a systolic ejection click and a loud. Vagotonic maneuvers in older children include doing a Valsalva maneuver.) Question .) E. A 10-yr-old white girl has a history of increasingly severe exercise intolerance and fatigability. 8. The peripheral pulmonary vascular markings are greatly decreased. 9. D. 10. B. intravenous adenosine is the next step. 7. Vagal stimulation Explanation: In infants. The ECG features of the delta wave (slow upstroke of the QRS complex. straining. C. Tetralogy of Fallot Stills murmur Rheumatic fever Cor pulmonale Primary pulmonary hypertension Explanation: Primary pulmonary hypertension is an idiopathic disorder. D. the vagal maneuver of choice is placing a plastic bag containing iced saline completely over the nose and mouth. Wolff-Parkinson-White syndrome is associated with all of the following Except: A. also known as preexcitation) are not seen during the SVT and are visible only in sinus rhythm.) D. B. 17th ed. (See Chapter 426 in Nelson Textbook of Pediatrics. (See Chapter 425 in Nelson Textbook of Pediatrics. E. Episodes of supraventricular tachycardia Question . The main problems are mechanical impingement on the trachea and/or esophagus and the symptoms resulting from these points of obstruction. The chest radiograph demonstrates prominent pulmonary arteries and an enlarged right ventricle. Cough Question . (See Chapter 428 in Nelson 17th ed. The CVS System Nelson Self Assessments website 17th Edition 3 . Some familial cases have mutations in the gene for bone morphogenetic protein receptor II. narrowly split second heart sound. The most likely diagnosis is: A. ) Cardioversion Defibrillation Digitalization Intravenous verapamil Question . D. (See Chapter 429 in Nelson Textbook of Pediatrics. the next step in her evaluation should be: A. Bone scan E. 17th ed. Maternal systemic lupus erythematosus B.) D. C. any person with a positive blood culture for S. which did not reveal any heart murmur. (See Chapter 428 in Nelson Textbook of Pediatrics. C. Disorders associated with complete heart block include all of the following Except: A. Two separate positive blood cultures for common bacteria Intracardiac mass on a valve seen with echocardiography Dehiscence of a prostatic valve Osler nodes Explanation: Immune complex phenomena and embolic events are minor criteria. A previously healthy 7-yr-old girl has a 3-wk history of fever. 17th ed. Endocarditis Question . drinking iced water. Echocardiogram Explanation: This patient has bacterial endocarditis of a previously normal valve. and gagging.) E. The past medical history is negative including a normal camp physical examination 6 mo ago. 17th ed. Two major or one major and 3-5 minor criteria suggest definite endocarditis. E. Chest radiograph B. and in the absence of bone or soft tissue findings. myalgias. 12. After repeating a blood culture. S. vomiting. (See Chapter 429 in Nelson Textbook of Pediatrics. The duration of the illness is too long for a simple viral illness. D. (See Chapter 428 in Nelson. Cardiac rhabdomyoma E. Rheumatoid arthritis Explanation: Rheumatoid arthritis primarily involves the pericardium and not the conduction system. aureus is a common pathogen producing endocarditis of a native valve. aureus should be considered as having endocarditis until proven otherwise. 11. and a positive blood culture for Staphylococcus aureus. Dental clinic appointment Question . Kearns-Sayre syndrome C. Major findings in the Duke criteria for the diagnosis of endocarditis include all of the following Except: A. More than two positive blood cultures for unusual bacteria The CVS System Nelson Self Assessments website 17th Edition 4 .B. Complete blood count C. 13. 17th ed.) D. B. coughing. Another cause of complete heart block is injury to the conduction system during reparative surgery for congenital heart disorders. squatting. B. and erythema marginatum (not in this patient). and knees unrelated to edema. and a new systolic murmur. ankle edema. A 3-mo-old female infant has a history of poor feeding. and chronic cough. C. These all represent major criteria for the diagnosis. (See Chapter 431 in Nelson Textbook of Pediatrics. Urine pH Question . Rheumatic fever presents acutely as a migrating polyarthritis. failure to thrive. 16. the clinical picture and history are not compatible with A-C. Electrocardiogram C. Blood culture D. A chest radiograph reveals cardiomegaly. the child described in Question 15 should also be started on: A. which resolved spontaneously. (See Chapter 431 in Nelson) B. Endocarditis Rheumatoid arthritis Meningococcal sepsis Glomerulonephritis Rheumatic fever Explanation: The sore throat represented an untreated episode of group A streptococcal pharyngitis.Question . shortness of breath during feedings. pericardium). 17th ed. Physical examination reveals tachycardia and a gallop rhythm but no murmur. Many believe that all infants with cardiomyopathy should receive a trial of carnitine after being evaluated for metabolic inborn errors of metabolism associated with cardiomyopathy. Selenium Vitamin B1 Thyroid hormone Carnitine Explanation: Although deficiencies of all of these nutrients (or hormone) may produce heart failure. One month ago he had a sore throat that lasted for 5 days. D. After further evaluation and treatment for heart failure. C. Delayed or chronic manifestations include chorea or subcutaneous nodules. 17th ed. shortness of breath. but documentation of a previous streptococcal infection is also required. An 8-yr-old boy presents with fever of 2 weeks' duration. Echocardiogram Explanation: The differential diagnosis includes myocarditis. E. pancarditis (valves. elbows. There is hepatomegaly but no cyanosis. The most likely diagnosis is: A.) Question .) E. The echocardiogram reveals poor contractibility and a dilated cardiomyopathy. 14. D. The most appropriate diagnostic test is: A. intermittent painful swelling of the wrists. 15. anomalous coronary arteries. cardiomyopathies. Serum amino acids E. myocardium. and arteriovenous malformations in the liver or brain. None of the above The CVS System Nelson Self Assessments website 17th Edition 5 . B. (See Chapter 430 in Nelson Textbook of Pediatrics. Sodium bicarbonate B. (See Chapter 431 in Nelson Textbook of Pediatrics. Blood culture Question . In addition to medications to treat the heart failure. Intravenous immunoglobulins E. Oseltamivir Question . Positive pressure ventilation The CVS System Nelson Self Assessments website 17th Edition 6 . The echocardiogram for the patient in Question 17 reveals myocarditis. 17. Heart transplantation B. and death. 17th ed. resulting in acidosis. (See Chapter 423 in Nelson Textbook of Pediatrics. Morphine C. and hepatomegaly. Physical examination reveals a grade 2-3/6 systolic murmur and a single loud second heart sound. The electrocardiogram (ECG) reveals left ventricular dominance. The next step in the management of this neonate is to administer: A. Pleconaril Explanation: Pleconaril is a potent antiviral agent that is very effective against enteroviruses. A 20-day-old previously well full-term infant is presented with fever. Liver function tests F. Digoxin E. tachypnea. a gallop rhythm. Prostaglandin E1 (PGE1) maintains patency of the ductus arterious between the pulmonary artery and the aorta. 17th ed. Important steps in the evaluation of this neonate include all of the following Except: A. Viral cultures and PCR assay D. If the PDA closes.) B. tachycardia of severity out of proportion to the fever. (See Chapter 431 in Nelson Textbook of Pediatrics. and the viral cultures reveal an enterovirus. ECG E. Head ultrasonography Explanation: This neonate has a febrile illness with heart failure. shock. Echocardiography C. A 1-day-old infant is noted to be cyanotic. 19. It is the agent of choice for treatment of severe enteroviral infections. The chest radiograph reveals a normal-sized heart and decreased pulmonary vascular markings. which additional therapy is now indicated? A. 17th ed. marked cyanosis would supervene.) C.Question .) D. Ribavirin D. Prostaglandin E1 Explanation: The murmur may represent a patent ductus arteriosus (PDA). 18. Therapy of a "blue" or "tet" spell could include all of the following Except: A. Phenylephrine The CVS System Nelson Self Assessments website 17th Edition 7 . Breath-holding spell Question . Oxygen D. 22.) D. The most likely diagnosis for the patient described in Question 19 is: A. An 18-mo-old child is noted to assume a squatting position frequently during playtime at the daycare center. decreased pulmonary vascular markings. early and marked cyanosis without heart failure. Sodium bicarbonate F. propranolol has been used to treat "tet" spells. 17th ed. (See Chapter 423 in Nelson Textbook of Pediatrics. Persistent pulmonary hypertension Transposition of the great arteries Truncus arteriosus Pulmonary atresia Explanation: Pulmonary atresia is manifested by a small right ventricle. hyperpneic. 17th ed. and ductal dependence to maintain some pulmonary blood flow. (See Chapter 423 in Nelson Textbook of Pediatrics. Anomalous coronary artery C. B. Cardiomyopathy B. Total anomalous venous return Question . Epinephrine Explanation: Epinephrine is potentially dangerous because it may exacerbate inotropy and contractile forces. Knee-chest position C. 17th ed. blue. The day of admission. The more serious episode is a cyanotic. which may obstruct the right ventricular infundibulum. Indeed. Constipation E. The most likely underlying lesion is: A. Morphine E. Tetralogy of Fallot Explanation: The child described has tetralogy of Fallot with exercise-induced cyanosis. and deeply cyanotic.) E. Within 10 min. The mother also notices occasional episodes of perioral cyanosis during some of these squatting periods. 20. The murmur of tetralogy (the pulmonary stenosis) often disappears or lessens during a spell. increased pulmonary artery pressure. or "tet" spell and may be due to decreased systemic vascular resistance.) B. 21. (See Chapter 423 in Nelson Textbook of Pediatrics. D. the child becomes restless. the child becomes unresponsive. C.Question . or right ventricular outflow tract obstruction. The age at onset usually corresponds to the time when the normally high fetal pulmonary vascular resistance declines in the first 1-3 mo of life. Open surgical valvotomy C. Myocarditis B. The most likely diagnosis: A. Tetralogy of Fallot Question . Anomalous coronary arteries D. and poor radial and femoral pulses. and poor growth. Coarctation of the aorta E. VSD Explanation: A large VSD with a large left-to-right shunt produces significant heart failure. The initial treatment of choice for a symptomatic patient with isolated pulmonic stenosis is: A. and metabolic acidosis. and severe cyanosis are typical of hypoplastic left heart syndrome. 17th ed. the left-to-right shunt increases. reduced left ventricular forces on ECG. (See Chapter 419. Valve replacement The CVS System Nelson Self Assessments website 17th Edition 8 . Transposition of the great arteries Question . The most likely diagnosis is: A. (See Chapter 420 in Nelson Textbook of Pediatrics. hypotension. 17th ed. Vital signs reveal a respiratory rate of 70/min.6 in Nelson Textbook of Pediatrics. 24. Closed surgical blade valvotomy B. 25. 17th ed. With decreasing pulmonary artery pressure. tachycardic. and an ECG demonstrates right ventricular dominance with markedly reduced R waves in V5 and V6. Cardiomyopathy B.) D. A grade 4 holosystolic murmur and a mid-diastolic rumble are noted. On examination.) C. a grade 2-3/6 systolic murmur. and blood pressure of 90/65 mm Hg in the upper and lower extremities. The cardiac examination reveals a palpable parasternal lift and a systolic thrill. and gray-blue. diaphoresis during feeding. cardiogenic shock. Balloon catheter valvuloplasty Explanation: Balloon valvuloplasty has greatly improved the management of stenotic lesions of the pulmonic and aortic valves. with hepatomegaly. (See Chapter 424 in Nelson Textbook of Pediatrics. A chest radiograph reveals cardiomegaly.Question . 23. Total anomalous venous return E. pulse of 175/min. The chest radiograph reveals cardiomegaly. A 2-day-old infant experiences cyanosis. A previously well 3½-mo-old presents with poor feeding. Myocarditis C.) D. Hypoplastic left heart syndrome Explanation: Poor pulses. the infant is lethargic. Blalock-Taussig shunt E. ) E. (See Chapter 419 in Nelson Textbook of Pediatrics. (See Chapter 432 in Nelson Textbook of Pediatrics.5 cm2) produces a loud murmur. Coarctation of the aorta Explanation: Rib notching is caused by increased collateral arteries trying to supply the lower trunk and extremities and bypass the aortic coarctation.) B. Postperfusion syndrome Question . An atrioventricular septal defect is different from an ostium secundum ASD because the AV septal defect: A. Endocarditis C. rumble. Tetralogy of Fallot E. The ECG of an A-V septal defect is characteristic because of the superior left deviation of the QRS axis. and hepatomegaly is not noted.6 in Nelson Textbook of Pediatrics. (See Chapter 420.) B. A VSD with a 4:1 shunt D. owing to the limited left-to-right shunt. Systemic hypertension E. Single ventricle The CVS System Nelson Self Assessments website 17th Edition 9 . 29. The child feeds well and has grown adequately. with perhaps a thrill but no other abnormalities. Pulmonary hypertension B. C. Anomalous left coronary artery C. 17th ed. Anomalous pulmonary venous return above the diaphragm C. The radiographic finding of notching of the ribs is associated with: A. You suspect: A. A restrictive VSD Explanation: A small VSD (<0. Rheumatic fever D. (See Chapter 419 in Nelson. 17th ed. 17th ed. Many of these defects will close spontaneously.) D. B. Does not manifest heart failure Does not create volume overload Has the same ECG findings Produces an early tendency for pulmonary hypertension Explanation: The atrial and the atrial-ventricular defects both are associated with a higher tendency for an earlier and more rapid onset of pulmonary hypertension. Myocarditis E. 27. The first and second heart sounds are normal. Creates an atrial-level shunt Question . Aortic insufficiency Question . 17th ed. or gallop. 28. A 5-mo-old previously well infant is found to have a loud holosystolic murmur (4/6) at the left sternal border. Pulsus paradoxus is associated with: A. D. 26. there is no tachycardia.Question . Pericarditis Explanation: Paradoxic pulse is also noted in asthma. 31. treatment is best performed by which of the following method(s)? A. Physical examination reveals a continuous machinery murmur and a wide pulse pressure with a prominent apical impulse. For the patient described in Questions 30 and 31. Truncus arteriosus Aorticopulmonary window Sinus of Valsalva aneurysm rupture Aortic valve insufficiency Critical aortic stenosis from a bicuspid valve Explanation: Aortic stenosis without significant aortic regurgitation (insufficiency) produces an inactive precordium. 32. (See Chapter 419 in Nelson. and pressure overload of the left ventricle. (See Chapter 419 in Nelson. Anomalous coronary artery Question . B. Pulmonic stenosis Aortic stenosis Ventricular septal defect Patent ductus arteriosus Explanation: A PDA often presents like a large VSD except that there is a continuous murmur as well as the diastolic runoff.) Question . A 6-mo-old is presented with tachycardia. tachypnea. Intravenous indomethacin B. Immediate cardiac catheterization D. the availability of transcatheter closure with coils or umbrellas has reduced the need for surgery in most patients. and poor feeding for 3 mo. However.) E. MRI E. The best method to evaluate this patient is to perform: A. C. Surgical closure Explanation: A PDA manifesting in any patient other than a premature infant does not close spontaneously. In many circumstances. B. which produces the wide pulse pressure and bounding pulses. Chest x-ray examination B. Angiotensin-converting enzyme inhibitor Question .Question . (See Chapter 419) C. D. spiked P waves and right ventricular hypertrophy. Catheter coil closure D. The ECG reveals tall. Echocardiogram Explanation: Echocardiology has replaced most methods of visualizing the anatomy of congenital and other heart lesions. C. 17th ed. There is a grade 4/6 systolic ejection murmur without an audible ejection click. 33. A neonate manifests cyanosis and hepatomegaly. D. the The CVS System Nelson Self Assessments website 17th Edition 10 . 17th ed. Digoxin until the PDA closes spontaneously E. normal or weak pulses. The most likely diagnosis is: A. E. Surgery carries low morbidity and even lower mortality. 30. Vector cardiography C. The differential diagnosis for the lesion in Question 30 includes all of the following Except: A. Tilt table testing The CVS System Nelson Self Assessments website 17th Edition 11 . pulse. estimate gradients. The treatment of choice for the lesion in the neonate described in Questions 33 and 34 is: A. 17th ed. B. The most likely diagnosis in the patient described in Question 33 is: A. Critical pulmonic stenosis Explanation: Critical pulmonic stenosis often presents in the neonate. He has a history of a heart murmur as an infant. 17th ed. D. (See Chapter 420 in Nelson Textbook of Pediatrics. (See Chapter 420 in Nelson of Pediatrics. he has a normal rhythm.) E. and detect signs of endocardial fibroelastosis. Patent ductus arteriosus B. he experiences severe dyspnea and becomes light-headed. Critical aortic stenosis C. During the tryout. Chest x-ray examination Exercise test Digitalization Echocardiography Explanation: Echocardiography is essential to identify valve disease. Digoxin Propranolol (Inderal) Surgical shunt Balloon valvuloplasty Explanation: Balloon pulmonary valvuloplasty is a remarkable nonsurgical method to dilate the valve and partially relieve the obstruction to right ventricular outflow. Tetralogy of Fallot E. and blood pressure and is no longer dizzy. Valve replacement Question . Truncus arteriosus Question . (See Chapter 420 in Nelson of Pediatrics. (See Chapter 416 in Nelson Textbook of Pediatrics. C. There is a grade 4/6 systolic ejection murmur that radiates to the neck. but the doctor thought it would go away. A 12-yr-old boy tries out for a middle school hockey team.) Question .) E. Cyanosis is due to elevated right-sided pressures and right-to-left shunting at the patent foramen orale. 35. At your office. D. There is also an ejection click. 17th ed. The next step in his management should be: A.echocardiographic diagnosis is all that is needed before surgery or other therapies. 17th ed. C. 36. 34.) D. B. An ECG reveals left ventricular hypertrophy. C. He has a grade 4/6 holosystolic murmur. The most likely diagnosis is: A. 39. (See Chapter 420 in Nelson Textbook of Pediatrics. once hyperviscosity develops. and his hematocrit is 70%.) D. E. 17th ed. Tetralogy of Fallot Explanation: TOF includes a VSD. and an overriding aorta between the two ventricles and above the VSD. Severe aortic stenosis Explanation: Severe aortic stenosis may develop gradually. 37. 38. Hyperviscosity aids flow to the pulmonary artery The CVS System Nelson Self Assessments website 17th Edition 12 . 25% of patients have a right-sided aortic arch. treatment is best accomplished with A. Chronic hypoxia Explanation: Chronic hypoxia produces both the clubbing and the polycythemia. Immediate valve replacement A shunt Digoxin Propranolol Balloon valvuloplasty Explanation: Balloon dilatation valvotomy and surgically splitting the valve are treatments of choice. pulmonary stenosis. (See Chapter 423 in Nelson Textbook of Pediatrics. Single ventricle Question . B. In addition. Total anomalous venous return B. Therefore. 17th ed. It was taken in a low cardiac output state C. 40. right ventricular hypertrophy. Mild aortic stenosis C. the raised hematocrit improves oxygen delivery. Williams syndrome Question . The reason that the hematocrit is so high in the patient described in Question 39 is: A. His chest film reveals a normal pulmonary blood flow as well as a right-sided aortic arch.) Question . Pulmonic stenosis B.) C.Question . The patient described in Question 36 most probably has: A. High hematocrits (usually >65% on a central blood sample) usually produce hyperviscosity. (See Chapter 420 in Nelson of Pediatrics. Patent ductus arteriosus E. Polycythemia improves flow through VSD E. An 8-yr-old boy from the former Soviet Union is brought to your office because of a heart murmur and dusky blue skin. For the patient described in Question 36. D. 17th ed. It was a peripheral finger stick D. up to a point. 17th ed.) B. Eisenmenger syndrome D. He has marked clubbing of the fingers and toes. the tissue microcirculation is compromised. which impairs tissue perfusion. progressing over 5-10 yr. Transposition of the great vessels E. (See Chapter 427 in Nelson Textbook of Pediatrics. 17th ed. 41. 42.) D. tachycardia.) E. 17th ed. Cerebral abscess Explanation: Cerebrovascular accidents result from a combination of polycythemia and the right-to-left shunting in patients with TOF. defibrillation may improve the outcome if done within minutes of the episode. D. The most likely diagnosis is: A. which is unfortunately refractory to almost all resuscitative efforts. B.Question . The patient described in Question 39 experiences a headache and is unable to move the left side of his body. The most likely cause of sudden death in an athlete immediately after forceful blunt trauma to the chest is: A.) B. The chest radiograph demonstrates cardiomegaly. C. she had fever. Cerebral abscess formation is more common in children older than 2 yr. and myalgias. Ruptured cardiac tendineae E. Physical examination reveals an uncomfortable. no murmur. afebrile patient with tachycardia. There is a paradoxical pulse of 22 mm Hg. Strokes are more common in children younger than 2 yr and in those with iron-deficiency anemia. Sepsis The CVS System Nelson Self Assessments website 17th Edition 13 . and a pulsus paradoxus. and distant heart sounds. (See Chapter 428 in Nelson Textbook of Pediatrics. often presents as chest pain. The most likely cause is: A. Embolic stroke B. 17th ed. (See Chapter 427 in Nelson Textbook of Pediatrics. Moyamoya disease Question . The chest radiograph shows cardiomegaly without pulmonary edema. Myocarditis Cardiomyopathy Kawasaki disease Pericarditis Explanation: Pericarditis. anxious. An 8-yr-old girl presents with chest pain of 1 day's duration. Commotio cordis Explanation: This unusual event places the heart in asystole. Migraine E. (See Chapter 432 in Nelson Textbook of Pediatrics. Five days before onset of the pain. Myocardial infarction C. At the scene. in this child due to coxsackievirus infection. 43. narrow pulse pressure. Cerebral thrombosis C. Lacerated coronary artery D. which can be enlargement due to myocardial dilatation or expansion of the pericardial space from fluid. chills. Hemorrhagic pericarditis Question . D. B.) The CVS System Nelson Self Assessments website 17th Edition 14 . pericarditis. (See Chapter 432 in Nelson Textbook of Pediatrics. 17th ed. The differential diagnosis for the patient described in Question 43 includes all of the following Except: A. E.Question . and rarely cardiac tamponade. Rheumatic fever SLE Juvenile rheumatoid arthritis Post-meningococcal immune complexes Chronic atrial fibrillation Explanation: Immune complex and other immune mechanisms may produce a pericardial effusion. C. 44. B. Tetracycline administration during tooth formation may lead to: A. Necessitate immediate orthodontic correction The Digestive System Nelson Self Assessments website 17th Edition 1 .) D. Necessitate future orthodontic correction E. Age 5 yr E.The Digestive System Nelson Self Assessments website 17th Edition. 3.) C. An 8-yr-old child presents with enamel defects on the permanent central and lateral incisors. 1. At what time would a metabolic disturbance affecting the ameloblast have occurred? A. Last trimester of pregnancy C. (See Chapter 288 in Nelson Textbook of Pediatrics. Age 7 yr Question . A persistent thumb sucking habit in a 7-yr-old child will: A. Prenatal disturbances often affect primary teeth. 2. 17th edition. Not affect the dentition because the child is continuing to grow B. (See Chapter 288 in Nelson Textbook of Pediatrics. First trimester of pregnancy B. Enamel defects Tooth shape abnormalities Enamel hypoplasia Gingival enlargement Tooth discoloration Explanation: Yellow-brown discoloration or even color changes that are visible with a fluorescent lamp have been reported with long periods of tetracycline use before the age of 8 yr. C. 17th edition. 17th edition. (See Chapter 290 in Nelson Textbook of Pediatrics.) Question . Cause the maxillary primary incisors to protrude D. E. Cause the maxillary permanent incisor teeth to protrude Explanation: Prolonged thumb-sucking after the eruption of permanent teeth may produce maxillary tooth protrusion. D. Question . Age 2 yr Explanation: This is the critical time for enamel defects on permanent incisors. ) B. Cleidocranial dysostosis C. C. This oral flora ferments carbohydrates and produces the local metabolic environment necessary to produce and expand a cavity. These teeth may need to The Digestive System Nelson Self Assessments website 17th Edition 2 . Enamel defects C. High sucrose consumption and Streptococcus mutans infection Explanation: This is classic for a child who consumes excessive carbohydrate drinks (soda. but not in number. Hereditary enamel hypoplasia B. 5.) D. The most likely etiology of this condition is: A. of teeth are noted in the other syndromes. Metabolism of Streptococcus mutans Explanation: A. Dental caries (cavities) in children are due to: A. Failure to brush teeth E. Ectodermal dysplasia Explanation: Variations in size or shape. D. Osteogenesis imperfecta B. Pierre Robin syndrome E. 17th edition. All of the above Question . 7. Enamel defects in teeth C. 4. Deformed or absent teeth Deficiencies of the nasal alar cartilage Protrusion of the premaxillary process Notching of the vermilion border All of the above Explanation: Cleft lip combined with cleft palate is a major developmental anomaly of the mid-face and produces all of these manifestations. 17th edition.) Question . Hereditary factors D. Clinical manifestations of a cleft lip or palate may include: A. Syndrome(s) with significant reduction in the number of teeth is (are): A. All of the above Question . (See Chapter 293 in Nelson Textbook of Pediatrics. (See Chapter 291 in Nelson Textbook of Pediatrics. Surgical repair must take each of these into consideration in addition to long-term sequelae. B. (See Chapter 292 in Nelson Textbook of Pediatrics. A 2-yr-old presents to your office with black spots and missing enamel on the maxillary anterior teeth. milks) and has cariogenic bacteria. 6. E.Question . juices. 17th edition. D. (See Chapter 295 in Nelson Textbook of Pediatrics. 8. crying from tooth pain. You tell the coach to: A. 17th edition. C. Gingival enlargement Explanation: Gingival hyperplasia is classically seen in patients receiving phenytoin (Dilantin).) Question .) C. such as tetracycline. A football coach calls you about a 15-yr-old who just had a central incisor knocked out on the field. Medications. Do not worry about finding the tooth. Oral trauma from falling Question . All of the above Question . Your plan includes: A. Put the tooth in milk and bring the child to your office B.) D. (See Chapter 294 in Nelson Textbook of Pediatrics. Not all patients get this complication. but take the child immediately to the emergency department The Digestive System Nelson Self Assessments website 17th Edition 3 . Try to reimplant the tooth and take the child to the dentist Explanation: The best place to put a dislodged tooth is back in its socket.) C. Malocclusion B. B. Notching of the vermilion border E. Oral complications associated with seizures in children include: A. 17th edition. (See Chapter 293 in Nelson Textbook of Pediatrics. 17th edition. Caries are usually painless unless an abscess develops. A 5-yr-old child presents to your practice with pain from a tooth. Reimplantation of the tooth (rinse it first. Referral to a dentist Prescription for an antibiotic Prescription for an analgesic All of the above Explanation: This child most likely has an apical tooth abscess. Increased rate of dental caries D. that become absorbed in the enamel E.be protected with sealants. (See Chapter 293 in Nelson Textbook of Pediatrics. Penicillin is an appropriate antibiotic. Wrap the tooth in a damp paper towel and bring the tooth and child to your office D. 10. 17th edition. 9. The parent says that for the last week the child has been waking up at night. but don't scrub it) is the best approach in a cooperative patient. (See Chapter 328 in Nelson Textbook of Pediatrics. Pancreas divisum Explanation: Pancreas divisum is a common finding now that CT. 11. It may or may not predispose to the development of pancreatitis. 17th edition.Question . MRI. B. but it is unlikely. C. (See Chapter 296 in Nelson Textbook of Pediatrics. or chronic diarrhea and failure to thrive.) Question . 13. Long-term xerostomia is a high risk factor for dental caries. Pancreatic pseudocyst E. Dehydration Anticholinergic drugs Sj gren syndrome Tumoricidal dose of radiation in the area of the salivary glands All of the above Explanation: Dry mouth is uncommon in children but must be considered in highrisk populations as noted in this question. 17th edition. E. It can be associated with: A. Choledochal cyst The Digestive System Nelson Self Assessments website 17th Edition 4 . E. Oral candidiasis can be found in: A. C. D. Cystic fibrosis B. and ultrasonography readily identify this anatomic abnormality. 17th edition. The most common developmental abnormality of the exocrine pancreas is: A. Most people with pancreatitis do not have pancreas divisum. 12. Annular pancreas D.) C. recurrent otitis media. (See Chapter 297 in Nelson Textbook of Pediatrics. Whether most people with pancreas divisum develop pancreatitis is unknown. Infants Children with AIDS Children receiving antibiotics Children with nutritional deficiencies All of the above Explanation: Oral candidiasis is quite common in all infants and should be a concern if it doesn't respond to topical therapy or if it occurs in the presence of other opportunistic infections. B.) Question . D. Question . 15. Congenital lipase deficiency C. A 10-yr-old boy develops pancreatitis following a blow to the abdomen. D. 17th edition. C. Shwachman-Diamond syndrome E. he is treated with intravenous fluids and analgesics and is discharged home on a low-fat diet. (See Chapter 330 in Nelson Textbook of Pediatrics. 16. cystic fibrosis is the most common etiology of pancreatic insufficiency. 17th edition.) The Digestive System Nelson Self Assessments website 17th Edition 5 . C. D. (See Chapter 332 in Nelson Textbook of Pediatrics. Chronic pancreatitis B. Serum amylase Serum lipase Abdominal ultrasound examination Fecal fat Explanation: Determination of fecal fat content is useful in the diagnosis of fat malabsorption from hepatobiliary or pancreatic disease.) E. Serum amylase isoenzymes Question . E. The most common cause of pancreatic insufficiency in childhood is: A. Cystic fibrosis Explanation: In the developed countries. The most likely diagnosis is: A. Pearson syndrome Question . malnutrition is the most common cause of pancreatic insufficiency.) D. 17th edition. B. 14. Four weeks later he has recurrence of abdominal pain and vomiting. Acute pancreatitis is not associated with significant steatorrhea. one must also consider a duodenal hematoma. This is present in more than 90% of children with cystic fibrosis and is usually present at birth. In developing countries. If symptoms develop within a week of blunt trauma. All of the following tests or determinations may be useful in the diagnosis of acute pancreatitis Except: A. (See Chapter 333 in Nelson Textbook of Pediatrics. Symptomatic pseudocysts require drainage. During a 5-day hospitalization. Chronic pancreatitis may cause fat malabsorption. Functional abdominal pain Duodenal ulcer due to Helicobacter pylori Chronic pancreatitis Hemorrhagic pancreatitis Pancreatic pseudocyst Explanation: A pancreatic pseudocyst is a common sequela to acute pancreatitis. B. D. and vomiting. A 2-mo-old Bangladeshi boy. with improvement in her stools for 11 mo. hematochezia. has been coughing since birth. He is breast-fed and is given occasional supplements of cow's milk-based formula. His immunizations are up-to-date and there is no history of travel. Treatment with cimetidine resolved his constant crying and even seemed to improve his spitting. B. born in the United States. Extended pH monitoring B. His mother also mentions occasional choking and gagging episodes. his parents report that for the past 2 wk he once again has begun frequent regurgitation. A 15-yr-old girl with cystic fibrosis and difficult-to-manage pancreatic insufficiency is treated with pancreatic enzyme supplementation containing lipase. Reassurance Fundoplication Upper endoscopy with biopsies Adjustment of the dose of cimetidine Explanation: This child responded well to the initial therapy with an H2-blocking agent. and more efficient acid suppression is required to heal the esophagitis and improve the child's condition. 10. Crohn disease C. A 5-mo-old boy was clinically diagnosed with gastroesophageal reflux at 3 mo of age. a meconium ileus-like picture could develop. he has outgrown his dose of cimetidine. (See Chapter 331 in Nelson Textbook of Pediatrics. 17.) E.Question . (See Chapter 304 in Nelson Textbook of Pediatrics. and irritability. 17th edition. The most likely diagnosis is: A. hiccups. however. With his continued growth along the 50th percentile. and omeprazole. If insufficient enzymes were used. C. Tuberculin skin test C. He still has reflux. Pressure-injection barium swallow The Digestive System Nelson Self Assessments website 17th Edition 6 . She then develops severe constipation. Which of the following is the most appropriate intervention? A. His weight continues in the 50th percentile. Fibrosing colonopathy Explanation: Excessive quantities of oral pancreatic enzyme replacements can produce fibrosing colonopathy. 17th edition. Intussusception Question . On a follow-up visit.) B. Daily use of antacids Question . 18. Acute pancreatitis E. His examination is normal.000 IU/kg. 20 mg/day. Distal intestinal obstruction syndrome (meconium ileus equivalent) D. abdominal pain. The coughing episodes are particularly pronounced during feeding. Which of the following will be the most appropriate diagnostic test? A. 19. D. A 6-yr-old girl with documented ?1-antitrypsin deficiency presents to the emergency department for assessment of large-volume hematemesis. and the child is well between episodes. C. Which of the following complaints would be a cause for concern? A. 21. and splenomegaly. producing a gag-cough response. Endoscopy of esophagus and stomach Question . Growth is normal. is abnormal and suggests gastrointestinal bleeding. Appendicitis The Digestive System Nelson Self Assessments website 17th Edition 7 . 22. Hepatomegaly and petechiae are absent. milk goes through the fistula to the trachea. Which of the following is most likely to develop in this patient? A. D. even in microscopically detectable quantities. B. 17th edition. Inflammation due to milk protein allergy is a common cause of heme-positive stools in formula-fed infants. (See Chapter 287 in Nelson Textbook of Pediatrics. With each feeding. Erosive reflux esophagitis Question . Modified barium swallow ("cookie swallow") E.) E. Regurgitation of 15-30 mL of formula three times a day One bowel movement every other day Three bowel movements per day Liver edge palpable 10 cm below the right costal margin Hemoccult-positive stool Explanation: A through D are physiologic variations of normal infants. A 3-mo-old infant presents for a well-child evaluation. E.) Question . Blood in the stool. There have been four episodes over the last 8 mo. Pancreatitis B. B. 20. portal hypertension.Explanation: This is a classic history for the most uncommon type of tracheoesophageal fistula: the H type. (See Chapter 300 in Nelson Textbook of Pediatrics. Which of the following is the most likely source of the hematemesis? A. Peptic ulcer disease Thrombocytopenia secondary to hypersplenism Swallowed blood from pulmonary hemorrhage Esophageal varices Explanation: Severe ?1-antitrypsin deficiency presenting in early childhood often involves the liver and could lead to cirrhosis. (See Chapter 337 in Nelson Textbook of Pediatrics. Vomiting episodes last 24 to 48 hr and usually end abruptly. and esophageal varices. 17th edition. C. pallor. 17th edition. Physical examination is remarkable for clear lung fields.) D. An 8-yr-old boy presents with a recurrent history of vomiting. Long chain fat-predominant formula Question . Chest x-ray shows a pleural effusion. Treatment includes intravenous fluids acutely and antimigraine medication if long-term treatment is needed. 17th edition. (See Chapter 310 in Nelson Textbook of Pediatrics. potassium.) D. 32 mEq/L. 2. Medium chain tryglycerides-predominant formula Explanation: Inadvertent injury to the thoracic duct during cardiac surgery may produce a chylous effusion and diarrhea. Serum electrolytes are as follows: sodium. 17th edition. Fecal pH is 6. and hyperkalemia. A 6-wk-old infant has a thoracotomy for transposition of the great vessels. Gastroesophageal reflux E. and chloride. Carbohydrate-free formula E. Soy formula C. (See Chapter 332 in Nelson Textbook of Pediatrics. The hypochloremic hypokalemic metabolic alkalosis helps in the diagnosis. Cholecystitis Question . The differential diagnosis of episodic recurrent emesis includes metabolic and mitochondrial disorders. HCO3 -. hyponatremia. Medium-chain triglycerides are absorbed through the venous plexus and not the lymphatics and are the traditional nutritional approach to this problem. while ultrasonography or an upper gastrointestinal contrast study will demonstrate the hypertrophied pylorus as the site of obstruction. Migraine equivalent Explanation: Migraine equivalent manifesting as cyclic episodes of vomiting ("cyclic vomiting") is the most likely diagnosis. The infant fails to gain weight. overfeeding. Stool cultures are negative and the stool is Hemoccult-negative. Physical examination immediately after feeding may reveal the olive-shaped pyloric obstruction. Following surgery. Gastric volvulus B. 89 mEq/L. A 6-wk-old male infant presents with a 3-wk history of intermittent vomiting and a weight loss of 300 g.) D. Adrenogenital syndrome often presents with emesis and diarrhea with acidosis. Malrotation with volvulus E. (See Chapter 287 in Nelson Textbook of Pediatrics. The most likely diagnosis is: A. Hypertrophic pyloric stenosis Explanation: This is the classic history of pyloric stenosis in a child. 24. Which of the following would be the most appropriate dietary intervention? A. Lactose-free formula D.) B. 23.9 mEq/L. and intracranial pathology.C. Cyclic vomiting The Digestive System Nelson Self Assessments website 17th Edition 8 . the infant develops loose stools. 147 mEq/L. intestinal obstruction. 17th edition. Adrenogenital syndrome C.0. especially if it was known that he needed a glycerin suppository and rectal stimulation to stool in the nursery after 72 hours of not passing meconium. E. foul-smelling stool. 17th edition.8 g/dL. This is a classic presentation. 17th edition. (See Chapter 312 in Nelson Textbook of Pediatrics. and the infant subsequently passes an explosive. The most appropriate step to establish the diagnosis is: A. C. She appears comfortable and in no distress or pain but is pale. 17th edition. Physical examination reveals a thin. (See Chapter 311 in Nelson Textbook of Pediatrics. 27. Rectal motility The Digestive System Nelson Self Assessments website 17th Edition 9 . Which of the following findings are diagnostic studies most likely to show? A. Target sign on abdominal ultrasound examination E. Colonoscopy Explanation: Colonoscopy is not indicated in children suspected of having Hirschsprung disease. The prenatal history is unremarkable. Rectal suction biopsy D.) B.Question . B. Pulse is 110/min. Hemoglobin level is 9. The disease is more common in males. (See Chapter 313 in Nelson Textbook of Pediatrics. Dilation of the rectum and colon on barium enema Question . Rectal examination reveals normal tone. Un-prepped barium enema C. 25. Calcifications on KUB B. Dilated distal small bowel loops on KUB D.) Question . A 6-mo-old infant has a 6-wk history of intermittent vomiting and poor weight gain. Inversion of the superior mesenteric artery and vein noted on abdominal ultrasound examination Explanation: This child has a malrotation and may have obstruction due to a volvulus or Ladd bands. Full-thickness rectal biopsy E. All the following may be useful in establishing the diagnosis Except: A. Abdominal ultrasound examination Air-contrast enema Angiography Colonoscopy Meckel scan Explanation: A Meckel diverticulum often presents with painless rectal bleeding in young children. The presence of gastric tissue in the diverticulum predisposes the patient to ulceration and painless bleeding. 26. The gastric tissue is identified by the scan as it picks up the technetium. An associated volvulus may produce bowel infarction and is a surgical emergency. poorly nourished child with abdominal distention.) C. A 4-yr-old girl presents with a 1-day history of a painless. A 2-mo-old infant has a history of infrequent bowel movements and poor weight gain. D. An upper gastrointestinal contrast study will reveal an abnormal position of the duodenum and any obstructing lesions. Meconium was passed on the second day of life. maroon-colored stool. The most appropriate next step in treatment would be: A. No blood has been noted in the stool. 28.) D.Question . (See Chapter 314 in Nelson Textbook of Pediatrics. 29. Over the next 12 hr the infant passes stool containing blood and mucus and becomes progressively lethargic. (See Chapter 315 in Nelson Textbook of Pediatrics. B. There has been no fever. Colonoscopy with polypectomy Stool culture Meckel scan Air-contrast enema Explanation: This child most likely has intussusception. lasting 20 to 30 min. is also the treatment of choice. D. 17th edition. The pain is relieved following the passage of stool. 30. It is unknown why children with this disorder become lethargic. with hydrostatic pressure. or weight loss. and the child is noted to be drooling. Observation is not a correct choice. C. 17th edition.) E. Following fluid resuscitation. The most appropriate next step in the evaluation or management of this patient would be: A. Empiric antibiotic therapy Question . Colonoscopy E. B. The infant has occasional vomiting. Trial of dietary fiber Explanation: Both mom and daughter probably have irritable bowel syndrome. The disease is of unknown etiology but is not "psychological. No battery can be found. joint symptoms. D. Ensuring consumption of fluids Ipecac administration Activated charcoal administration Chest x-ray study Explanation: Hearing aid batteries are quite dangerous and if ingested they should be removed as soon as possible. Observation and follow-up in 24 hr Question . C. with a frequency of two or three times per week. Her mother has had similar symptoms intermittently since the age of 12 yr. Air or barium enema is the diagnostic tool of choice and. rash. (See Chapter 323 in Nelson Textbook of Pediatrics. Gluten-free diet The Digestive System Nelson Self Assessments website 17th Edition 10 . the most appropriate next step in management is: A.) E." Dietary fiber is a good first line of therapy. Abdominal CT scan B. A 14-yr-old girl has a 6-mo history of recurrent abdominal pain. A 3-yr-old hearing-impaired child is found playing with his hearing aid. 17th edition. Endoscopic retrograde cholangiopancreatography (ERCP) C. The pain is characterized as periumbilical to epigastric. but many are incorrectly thought to have central nervous system pathology and wind up getting a head CT rather than a radiologic enema. A previously healthy 6-mo-old child develops paroxysmal colicky abdominal pain. B. D. 33. In a newborn baby with an anorectal malformation. Sacral anomalies may also be present. E. Radiograph of the sacrum Question . B. C. None of the above Question . 17th edition. (See Chapter 325 in Nelson Textbook of Pediatrics. <10% 30% 60% 90% Explanation: This is a high-risk anomaly and is associated with a major risk for urologic problems of the bladder and kidneys. A baby is born with no anal opening and obvious features of Down syndrome. A female newborn has a single perineal orifice.Question . 17th edition.) The Digestive System Nelson Self Assessments website 17th Edition 11 . D. which of the following diagnostic tests is most important during the first 24 hr of life? A. (See Chapter 325 in Nelson Textbook of Pediatrics. 17th edition. D.) E. especially the kidneys. C. MRI of the entire body CT of the pelvis Electroencephalogram Ultrasound examination of the abdomen Explanation: Associated internal anomalies in children with anorectal malformations often include the genitourinary system. The most likely type of anorectal malformation in this baby is: A. Recto-bladder neck fistula Rectourethral fistula Rectoprostatic fistula Rectoperineal fistula Imperforate anus with no fistula Explanation: Imperforate anus without a fistula is a common pattern of anal anomalies in children with trisomy 21. B. but it is critical to know if there are associated renal anomalies prior to surgery. C. (See Chapter 325 in Nelson Textbook of Pediatrics. The chance that this baby has associated urologic problems is: A. 32.) E. 31. The boy's father and grandfather have undergone colectomies as young adults. None of the above Question . Heart rate and blood pressure are normal. Clinical follow-up until age 25 yr Question . CT scan E. (See Chapter 326 in Nelson Textbook of Pediatrics. The child may "pass" meconium as well. Repeat genetic testing in 1 yr The Digestive System Nelson Self Assessments website 17th Edition 12 . (See Chapter 325 in Nelson Textbook of Pediatrics.) D. Proctocolectomy C. Cloacal defect B. Proctocolectomy C. The incidence of familial adenomatous polyps is 1:8000 people.) B. more serious is the risk of developing colonic cancer within one or more of the polyps. but no mutations were identified in the girl's APC gene. COX-2 inhibitor therapy D. the imperforate anus may be missed. The most common type of anorectal malformation seen in females is: A. 35. Patients require annual colonoscopy. Colonoscopy B. and rectal examination reveals velvety polyps. (See Chapter 326 in Nelson Textbook of Pediatrics. The girl requires: A. A 10-yr-old girl is brought for evaluation because her mother was found to have familial adenomatous polyposis coli (APC). He appears well and denies nausea or vomiting. aunt. 17th edition.) D. her risk is no greater than that of the general population. 17th edition. 34. Genetic screening identified identical APC gene mutations in mother. If the gene is known in the family and the patient does not have that gene. Without direct observation of the perineum. Annual physical examination Explanation: This autosomal dominant gene is responsible for familial adenomatous polyps with a high risk of malignant transformation. Rectoperineal fistula E.Question . CT scan E. 17th edition. and grandfather. The next appropriate step is: A. Rectovaginal fistula C. 36. but notes occasional crampy abdominal pain. A 9-yr-old boy complains of lower gastrointestinal bleeding and mucous drainage. Colonoscopy Explanation: This child most likely has familial adenomatous polyposis coli and is at risk for developing multiple polyps for the rest of his life. Rectovestibular fistula Explanation: This fistula may not be completely obstructed. The next step in her care is: A. but more so because she is at increased risk for lymphoma or a lymphoproliferative disease. Partly because of her age. The abdomen is not tender on examination. He appears to have a limp. On examination. A 12-yr-old girl presents with severe crampy abdominal pain. On examination. At his age and given the size of the tumor. 17th edition. Start -adrenergic blockade and obtain spine MRI scan E. The next step in the care of this boy is to: A. and gums. 39. Refer to physical therapy Question .) C. he has a poorly defined mass in the left flank and a blood pressure of 170/100 mm Hg. it is most likely a neuroblastoma.) C. Start therapy with a long-acting somatostatin analogue B. which favors the left leg.Question . Pheochromocytomas are usually too small to be palpable. Discharge home with a diagnosis of constipation B. The pain resolves after 30 min in the emergency department. Abdominal CT scan demonstrates a large paraspinous mass on the left. Perform genetic screening E. 38. buccal mucosa. perhaps secondary to Peutz-Jeghers syndrome. 17th edition. she is found to have freckles on the lips. 17th edition. Obtain upper and lower gastrointestinal contrast studies to rule out intussusception Explanation: The clinical picture suggests a complicated intussusception. Hydrostatic reduction in radiology B. The next most appropriate step is to: A. 37. This is an autosomal dominant disorder that carries a high risk of eventual development of colonic cancer. A 5-yr-old girl with a history of heart transplant presents with nausea and severe intermittent abdominal pain. a hydrostatic reduction may not be safe or successful and will not be diagnostic of malignancy. Surgical exploration Explanation: This child is at increased risk of intussusception from a specific pathological lead point. Enteroclysis E. (See Chapter 326 in Nelson Textbook of Pediatrics. Refer for immediate surgical exploration C. A 4-yr-old boy presents with diarrhea and headaches. (See Chapter 326 in Nelson Textbook of Pediatrics. Start -adrenergic blockade and obtain MRI scan Explanation: This child has a catecholamine-secreting or other endocrinologically active tumor. Perform proctocolectomy The Digestive System Nelson Self Assessments website 17th Edition 13 .) D. Remove all polyps endoscopically to prevent cancer D. CT scan is consistent with ileocolic intussusception. Pain medication and observation in the hospital D. Colonoscopy Question . (See Chapter 326 in Nelson Textbook of Pediatrics. 100-200 times D. Serum serotonin E. which usually does not contain bowel and is thus usually identified as an asymptomatic mass. as noted in this case.) D. 10-20 times C. 41. 17th edition. Metastatic neuroblastoma B. Lipoma E. is eating normally. Serum ACTH D.Question . the dose associated with mottling in young infants is how many times the dose from fluoridated water? A. The laboratory test that would suggest the presence of a carcinoid tumor is assay of: A. 2-5 times Explanation: The therapeutic window for mild mottling of teeth from fluoride is narrow. He is afebrile. A 4-yr-old boy in good health presents with a 3-day history of a tender mass. midway between the umbilicus and the xyphoid process. The most likely diagnosis is: A. 42.) B. (See Chapter 326 in Nelson Textbook of Pediatrics. Serum parathyroid hormone Question . 17th edition.) C. Incarcerated epigastric hernia Explanation: This is a typical site for an epigastric hernia. Although the toxic dose of fluoride is many times greater than the dose needed to cause mild fluorosis (mottling of dental enamel). Infected dermoid cyst Question . 2 cm in diameter. A 15-yr-old boy presents with frequent episodes of flushing and watery diarrhea. Urinary 5-hydroxyindoleacetic acid (5-HIAA) Explanation: Carcinoid is uncommon in children and may present as a nonsecreting tumor or one with identical features to those in adults. 40. Rhabdomyosarcoma of the abdominal wall C. (See Chapter 293 in Nelson Textbook of Pediatrics. 1000 times The Digestive System Nelson Self Assessments website 17th Edition 14 . (See Chapter 354 in Nelson Textbook of Pediatrics. 17th edition. and has an otherwise normal physical examination. Urinary catecholamines B. which developed at age 10 yr. Chronic active hepatitis Explanation: Choices A. 45. anorexia. is excluded by the presence of anemia and weight loss. D. The direct bilirubin level is 6 mg/dL. A chronic or recurrent course of abdominal pain. and emesis. no icterus. Pelvic inflammatory disease E. nausea. Wilson disease Question . Hepatitis C. where he received an over-the-counter medication for an illness characterized by abdominal pain. (See Chapter 317 in Nelson Textbook of Pediatrics. (See Chapter 343 in Nelson Textbook of Pediatrics. weight loss. Henoch-Sch nlein purpura B. B.) C. 17th edition. Functional abdominal pain (irritable bowel). Irritable bowel disease Question . 1-Antitrypsin deficiency D. which is part of the differential diagnosis. High titers of liver-kidney microsomal antibodies are present. Chronic active hepatitis of the autoimmune type (lupoid hepatitis) is often associated with other autoimmune diseases (the patient described in the question had Hashimoto thyroiditis) and is more prevalent in adolescent females. Four weeks before admission.5 g/L. 44. Physical examination reveals hepatomegaly. and jaundice of 4 months' duration. The most likely diagnosis is: A. Hepatoblastoma E. An 18-yr-old boy complains of right upper respiratory quadrant pain and fever for 2 wk. and the total bilirubin value is 11 mg/dL.) C. 43. and right lower quadrant fullness. Inflammatory bowel disease Explanation: Inflammatory bowel disease is one of the most common causes of significant chronic or recurrent abdominal pain in adolescents. 17th edition. amenorrhea. Amebiasis B. B. Anorexia nervos D. C. and anemia in an adolescent girl suggests: A. Hepatic abscess (bacterial) Explanation: In the patient described in the question. and the serum IgG level is 16. he returned from Mexico. The most likely diagnosis is: A. now manifests fever. A 16-yr-old girl with a past history of hypothyroidism. and C must be considered because each may produce similar hepatic and extrahepatic manifestations. the hepatic abscess developed as a complication of a partially treated ruptured appendix after acute The Digestive System Nelson Self Assessments website 17th Edition 15 . Mononucleosis B. Results of the tests for hepatitis A.Question . The antinuclear antibody response is also positive in many patients. and E are negative. When you begin your examination. subsequently. Six hours later. but not unheard of. Pelvic inflammatory disease Ruptured ectopic pregnancy Crohn disease Appendicitis Explanation: All of the choices in the question must be included in the differential diagnosis. Kernicterus Question . C. and ataxia. Hepatic encephalopathy Vitamin A deficiency Encephalitis Vitamin E deficiency Explanation: Vitamin E deficiency has a long latency (years) before it eventually produces ataxia. A ruptured ectopic pregnancy would be unlikely. You notice guarding and tenderness throughout her abdomen. experiences intermittent periumbilical abdominal pain 2 days before an emergency visit to her pediatrician.appendicitis. The over-the-counter medication was an oral antibiotic that suppressed some of the signs of appendicitis. the most tender area is the right lower quadrant. and signs of peritoneal irritation. Amebiasis is in the differential diagnosis. The organisms are usually enteric anaerobes in this form of liver abscess. Cholangitis E. D. (See Chapter 334 in Nelson Textbook of Pediatrics. C. 47. B. D. An 11-yr-old girl. A 10-yr-old girl who had biliary atresia treated with the Kasai procedure in infancy now manifests increasing clumsiness. (See Chapter 337 in Nelson Textbook of Pediatrics. Early treatment with water-soluble vitamin E may prevent and reverse these neurologic processes. the liver. and peripheral neuropathy. 17th edition. Treatment includes antibiotics (including metronidazole or clindamycin) and percutaneous drainage under ultrasonographic or computed tomographic guidance.) E. The most likely diagnosis is: A. reduced deep tendon reflexes. The right lower quadrant fullness is a walled-off appendiceal abscess. Mesenteric adenitis may present in a manner similar to appendicitis The Digestive System Nelson Self Assessments website 17th Edition 16 . B. The most likely diagnosis is: A. She also has had two soft bowel movements without blood or relief of symptoms. The hepatic abscess developed after septic embolization into the portal vein (during the acute appendicitis) and. Crohn disease Question . 46. she is nauseated and has one or two episodes of emesis. 17th edition. This is treated with antibiotics and resected at a later date. vaginal discharge. she is apprehensive and watches every move of your examining hand. Tanner stage 2. posterior (spinal cord) column signs. She walks cautiously into your office and lies still on your examining table. in a girl at Tanner stage 2.) D. Pelvic inflammatory disease often presents with lower abdominal suprapubic pain. (See Chapter 352 in Nelson Textbook of Pediatrics. All of the following statements regarding cyclic vomiting are true Except: A. 17th edition. Culture of the ascitic fluid is most likely to reveal: A. 50. The 2-day history is typical of appendicitis. 48. Reflux dyspepsia Question . Nystagmus should suggest serious central nervous system disease or vestibular disease. A 5-yr-old girl with cirrhosis and portal hypertension experiences increasing abdominal distention and fever. as is the apprehension about the examiner's hand during the abdominal palpation. D. B. 17th edition. C. cirrhosis). Mesenteric adenitis Question . Onset occurs between ages 3 and 5 yr Episodes last 2-3 days There are four or more emesis episodes per hour It is a migraine equivalent It is associated with nystagmus Explanation: A-D are all correct.) E. are noted on physical examination. Candida albicans C. as well as abdominal tenderness.) The Digestive System Nelson Self Assessments website 17th Edition 17 . (See Chapter 287 in Nelson Textbook of Pediatrics. Shifting dullness and a puddle sign.) E. B.) D. This patient has the characteristic appendicitis sequence of periumbilical pain. Paracentesis reveals cloudy fluid. Pain on swallowing Emesis without nausea Drooling due to obstruction Difficulty swallowing Explanation: Dysphagia is a sensation that swallowing is difficult because of something being "stuck. C. (See Chapter 287 in Nelson Textbook of Pediatrics.(pseudoappendicitis) and can be detected by abdominal ultrasonography or CT scan. 17th edition. followed more often by nausea and less often by emesis. Haemophilus influenzae Question . Pseudomonas B. D. (See Chapter 324 in Nelson Textbook of Pediatrics. Next in frequency is Escherichia coli." Odynophagia is pain on swallowing. 49. Serratia E. E. Dysphagia is best described as: A. followed by right lower quadrant pain. Pneumococci Explanation: The pneumococcus is a common pathogen producing peritonitis (primary) in any condition causing ascites (nephrosis. 17th edition. d. Crohn disease The Digestive System Nelson Self Assessments website 17th Edition 18 . t. Obtain a dental film to look for intrusion Explanation: Intrusion of a tooth into the maxillary bone with the appearance of avulsion may be a common problem in children. a 12-yr-old appears to have a missing incisor. After blunt facial trauma. There is no evidence of the tooth at the scene.) B. and the child does not remember swallowing the tooth. f. palatopharyngeal incompetence may first become evident or be exacerbated by adenoid removal. b. Indeed. Eosinophilic gastritis E. 53. h. The next step in evaluating the child is to: A. (See Chapter 295 in Nelson Textbook of Pediatrics.) E. Difficulty whistling Question . Prostaglandin E infusions D. D. C.Question . Palatopharyngeal incompetence is characterized by all of the following Except: A. 17th edition. 17th edition. 17th edition. s Improvement after adenoidectomy Explanation: This is quite false. Cystic fibrosis Explanation: B-E are all correct. Administration of penicillin to prevent facial cellulitis Question . Acquired causes of gastric outlet obstruction include all of the following Except: A. Plan for a bridge B. v. Epidermolysis bullosa C.) D. 52. Hypernasal speech Presence of a submucosal cleft Difficulty in pronouncing p. Obtain an abdominal x-ray study to search for the avulsed tooth C. B. 51. Placement of a ceramic tooth E. (See Chapter 310 in Nelson Textbook of Pediatrics. (See Chapter 291 in Nelson Textbook of Pediatrics. Males are affected more commonly than females C. diarrhea. 56. It is associated with trisomy 21 E. poor appetite. and irritability. A diagnosis of cyclic vomiting was made at 3 yr of age. Superior mesenteric artery syndrome E. a tender distended abdomen. Celiac disease Explanation: Imperforate anus is associated with VATER syndrome. pH. (See Chapters 311 and 313 in Nelson Textbook of Pediatrics. 54. 17th edition. was over 80 yr. His growth curve is flattening. It is uncommon in preterm infants B. It is associated with VATER syndrome Explanation: Imperforate anus is associated with VATER syndrome. The most likely diagnosis is: A. Most cases present before age 3 yr. Therapy with antimigraine medications was unsuccessful. however. Superior mesenteric artery obstruction of the intestines may also produce bile-stained emesis. He now manifests bile-stained emesis. Malrotation Explanation: Malrotation. The most likely diagnosis is: A. The oldest reported patient. usually presents with bowel obstruction before 1 yr of age. however. Cystic fibrosis D. Stress ulcers B. Breast-fed infants may appear less ill than formula-fed infants Question . and in this patient volvulus (a bowel and lifethreatening complication). All of the following statements regarding Hirschsprung disease are true Except: A.Question . he was weaned from breast milk to regular foods. glucose.) D. Mitochondrial myopathy The Digestive System Nelson Self Assessments website 17th Edition 19 . A 5-yr-old boy has a history of chronic recurrent abdominal pain and emesis. Meconium ileus equivalent Question . Hirschsprung disease is the most common anatomic cause of lower intestinal obstruction in neonates. (See Chapter 313 in Nelson Textbook of Pediatrics. abdominal distention. Giardiasis B. (See Chapter 313 in Nelson Textbook of Pediatrics. Pancreatitis E.) D. 17th edition. and bicarbonate levels were normal. and bright-red blood per rectum. 55. He had been well until 9 mo of age. An 18-mo-old white boy manifests failure to thrive. During each episode. Hirschsprung disease is the most common anatomic cause of lower intestinal obstruction in neonates. intestinal infarction is not observed. 17th edition. Intussusception C. Thereafter.) C. the serum ammonia. ) C. An 18-mo-old is discovered with his mouth over a storage bottle containing a strong alkali. (See Chapter 338 in Nelson Textbook of Pediatrics. Administer milk Explanation: Administration of milk helps calm a child and helps dilute the alkali in the esophagus and stomach. Some fluid is missing from the bottle. Dystonia and tremor E. Antineutrophil cytoplasmic antibody assay Question . is to: A. which may be absent in younger patients.Question . Antinuclear antibody (ANA) assay B. The best diagnostic serum test for the disease in Question 56 is: A. E. D. who usually manifest hepatic disease. Hemolysis F. The disease in Question 56 is also noted with higher prevalence in patients with all of the following conditions Except: A. 57. (See Chapter 320 in Nelson Textbook of Pediatrics. and the boy seems well.) B. but no external signs are found on the child's clothing. 59. Ocular disease is detected by Kayser-Fleischer corneal rings. IgA endomysial antibody assay Explanation: This is the serum diagnostic test of choice and is almost 100% sensitive and specific. IgA deficiency Down syndrome Juvenile rheumatoid arthritis Diabetes mellitus Systemic lupus erythematosus Explanation: SLE is not a predisposing condition for celiac disease. Fanconi syndrome Question . (See Chapter 308) C. 17th edition. The parents remove the bottle. Administer acetaminophen The Digestive System Nelson Self Assessments website 17th Edition 20 . 58. Administer toast D. except in IgA-deficient patients. and renal (plus hematologic) findings. Hepatomegaly C. Cardiomyopathy Explanation: B-F represent the hepatic. and the child has no burns on his face or his lips. 17th edition. C. IgA antigliadin antibody assay E. The most appropriate advice to give the parents. neurologic. Ascites and portal hypertension D. who are on their way to the hospital. B. (See Chapter 320 in Nelson Textbook of Pediatrics. 60.) Question . Wilson disease is associated with all of the following Except: A. IgA antigluten antibody assay D. Administer ipecac B. 17th edition. B. particularly in younger children and in those with an appendix in an unusual place. the most appropriate next step in management would be: A. The most appropriate next step in management is to: A. He has mild tenderness in both lower quadrants.400/cu mm with 75 PMNs and 20 lymphocytes. Peripheral smear The Digestive System Nelson Self Assessments website 17th Edition 21 . The most likely diagnosis is: A. abdominal imaging should be performed. and platelet count. 62.) B. 17th edition. C. D. Administer prednisone to decrease stricture formation Administer penicillin to prevent infection Administer an acidic fluid to neutralize the alkali Perform endoscopy to assess the severity of the ingestion Explanation: Endoscopy is indicated to assess the severity of inflammation and necrosis. Abdominal ultrasonography has been useful. 63.000/cu mm. 9. Place a nasogastric tube to feed the child Question . WBC count. such as mesenteric adenitis and gynecologic pathology in females. Laparotomy D. 61. Viral gastroenteritis C. He now has had 24 hr of abdominal pain that appears maximal in the lower abdomen. (See Chapter 325 in Nelson Pediatrics. When in doubt. including an examination of his posterior pharynx. Prednisone is of no value in preventing strictures. 36. Intussusception B. results of his physical examination.) D.Question . hematocrit. Barium enema C. A 4-yr-old boy has had vomiting and diarrhea for 4 days. Stool culture E. 160. 17th edition.4°C. Appendicitis Explanation: Appendicitis does not always follow the classic pattern. normal rectal examination. Even without oral lesions. His temperature is 38. For the patient described in Question 62. significant esophageal involvement is possible.) E. but CT has shown superior results. Bacterial enteritis E. are found to be unremarkable. (See Chapter 325 in Nelson Textbook of Pediatrics. 17th edition. Henoch-Sch nlein purpura Question . (See Chapter 308 in Nelson Textbook of Pediatrics. After the child described in Question 60 arrives in the emergency department. Imaging is most useful in identifying other causes of pain. and mild muscular guarding in the lower quadrants. CT scan Explanation: Abdominal CT scanning is the imaging study of choice but is not necessary for all patients with appendicitis. Urinalysis is normal. with the exception of 3+ ketonuria. This infant needs to be resuscitated immediately and have the airway protected and ventilation begun. Congenital diaphragmatic hernia (hiatal) E. Order an ultrasound examination E. with diminished breath sounds on the left and heart tones displaced to the right.) E. A 3. 66. Therefore. Perform fecal leukocyte examination C. Respiratory distress is apparent in the delivery room. 17th edition. 64. 160. with 75 PMN and 20 lymphocytes. 17th edition. Begin oral antibiotics Question .9-kg girl is born at term after having carried a prenatal diagnosis of left congenital diaphragmatic hernia since 19 wk of gestation. Chest film reveals a normal abdominal gas pattern and multiple lucent areas in the left lower thorax.5-kg girl is delivered by spontaneous vaginal delivery. with mild tachypnea and retractions developing at 24 hr of age. Congenital diaphragmatic hernia (Morgagni) C. The WBC count is 9. (See Chapter 353 in Nelson Textbook of Pediatrics. The most appropriate next step in treatment is to: A. a CT scan is definitely indicated to look for an abscess. A 2. Congenital diaphragmatic hernia (Bochdalek) B.) D. The abdomen is scaphoid. 17th edition. with Apgar scores of 2 and 3. Mask-bag ventilation would increase gas in the intestines. B. he may have local peritonitis or may be developing an intra-abdominal abscess. Perform umbilical artery catheterization Question . The most appropriate next step in patient management would be to: A.8°C but is tolerating a regular diet and has two loose stools per day. 36. which could increase the mass effect in the left thorax.400/cu mm. If fever continues despite administration of antibiotics. hematocrit.000/cu mm. immediate endotracheal intubation is indicated. C. D.) B. 65. Delivery and the early postnatal period are uneventful. Order a CT scan D. (See Chapter 353 in Nelson. The most likely diagnosis is: A. Cystic fibrosis The Digestive System Nelson Self Assessments website 17th Edition 22 . 3. or if localizing signs develop. and platelet count. Administer sodium bicarbonate. (See Chapter 335 in Nelson Textbook of Pediatrics. A 5-yr-old boy underwent open appendectomy for perforated appendicitis 4 days ago. He continues to be febrile with temperatures to 38. Congential cystic adenomatoid malformation Explanation: Congenital cystic adenomatoid malformation (CCAM) is in the differential diagnosis of congenital diaphragmatic hernia.Question . Continue intravenous antibiotics Explanation: At this point.5 mEq Perform left tube thoracostomy Perform emergency laparotomy Perform endotracheal intubation Explanation: This is a classic presentation for a left-sided diaphragmatic hernia. Quantitative immunoglobulin level D. Congenital diaphragmatic hernia (hiatal) Explanation: This anomaly may be confused with reflux or gastritis. the hidden mortality of CDH. A 16-yr-old boy is being evaluated for an incidentally documented increase in alanine aminotransferase (ALT). RT/PCR is the best method to confirm the diagnosis of hepatitis C. Diagnostic evaluations to date include positive results on hepatitis A IgG assay and hepatitis C ELISA and an anti-liver-kidney microsomal antibody titer of 1:80. Hepatitis A IgM assay B.) E. Over the past 6 mo the ALT level has fluctuated between 100 and 200 IU/L. which demonstrated a single gas bubble in the medial aspect of the left lower hemithorax. The most appropriate next diagnostic test would be: A. Which of the following is most likely to occur in this situation? A. Premature delivery C. Group B streptococcal sepsis is. A 4-yr-old boy has a history of intermittent non-bilious vomiting since infancy. associated with right-sided CDH. Cystic fibrosis Question . It requires surgical repair. He was evaluated for suspected pneumonia with a chest film. Congential cystic adenomatoid malformation D. the liver and stomach appear to be in the abdomen. for some unknown reason. (See Chapter 353) D. Congenital diaphragmatic hernia B. incorrectly suggesting a diagnosis of an autoimmune process. Serum creatinine level The Digestive System Nelson Self Assessments website 17th Edition 23 . Fetuses with other anomalies have a high intrauterine fetal demise rate. 68. Hepatitis C reverse transcriptase-polymerase chain reaction (RT/PCR) assay Explanation: Testing for chronic active hepatitis with LKM antibodies in patients may result in a false-positive test result. Group B sepsis in fetus Question . 17th edition. 67. Fetal demise B. Successful repair of fetal hernia Explanation: In children with congenital diaphragmatic hernia (CDH) and no other anomalies. and an ultrasound study at 16 wk did not reveal the lesion. The gestation is 23 wk. 69. There is no evidence of fetal hydrops.6. Congenital diaphragmatic hernia (Morgagni) C. It is important to document persistent hepatitis C infection. fetal survival is quite good. Prothrombin time E. He has remained below the 5th percentile in weight. (See Chapter 353 in Nelson Textbook of Pediatrics. but has been at the 50th percentile for height and head circumference. as chronicity is associated with the risk of cirrhosis.Question . A 29-yr-old woman is identified as carrying a fetus with a right congenital diaphragmatic hernia. (See Chapter 343) C. and the lung-to-head ratio is reported as 1. The most likely diagnosis is: A. Severe pulmonary hypertension in fetus E. The most appropriate initial therapy would be: A.) B. 40 mg of prednisone daily Explanation: Daily therapy with corticosteroids is essential for initial management of autoimmune hepatitis. 335 IU/L. A 12-yr-old girl presented with nonspecific abdominal pain and hepatosplenomegaly and was found to have an ALT of 185 IU/L. both on alternate days The Digestive System Nelson Self Assessments website 17th Edition 24 . 17th edition. 265 IU/L: and antinuclear antibody titer. positive HBcAb. 70. 40 mg of prednisone and 50 mg of azathioprine (Imuran). 50 mg of azathioprine (Imuran) on alternate days C. Slit-lamp examination Question . (See Chapter 343 in Nelson. alkaline phosphatase. which has a high risk of subsequent hepatocellular cancer. and positive HBeAg.4 mg/dL. 640 IU/L. 72. Which of the following statements is true about interferon therapy? A.2 mg/dL. B. Patients with chronic active autoimmune hepatitis respond quite well to daily prednisone. 1:80. On screening. 1. Hepatitis C reverse transcriptase-polymerase chain reaction (RT/PCR) assay Repeat liver biopsy Abdominal ultrasound examination with Doppler interrogation Endoscopic retrograde cholangiopancreatography (ERCP) Explanation: There are clinical similarities between autoimmune hepatitis and sclerosing cholangitis in children. This is also called lupoid hepatitis. and an anti-smooth muscle antibody titer. (See Chapter 343 in Nelson Textbook of Pediatrics. total bilirubin. Duration of therapy is typically 4 wk E. C. 1:80. a 6-yr-old adopted child has the following biochemical and serologic findings: ALT. 17th edition.) D. total bilirubin. which does not respond to prednisone but requires interferon and antiviral drugs.) E. positive HBsAg. A realistic goal of therapy is to reduce the risk of the development of hepatocellular carcinoma Explanation: This child has evidence of being actively infected with hepatitis B virus. A 9-yr-old girl presented with persistent fatigue and hepatomegaly and was found to have an ALT of 275 IU/L. 40 mg of prednisone on alternate days E. alkaline phosphatase. 71. 50 mg of azathioprine (Imuran) daily B. Liver biopsy revealed a lymphoplasmacytic infiltrate of the portal tracts. Therapy is not approved and is not indicated D. This child could also have chronic viral infection. 2. Specific treatment reduces this risk. She has been treated for 4 mo with prednisone without significant improvement.Question . The next most appropriate diagnostic study would be: A. Liver biopsy revealed a lymphoplasmacytic infiltrate of the portal tracts. (See Chapter 343 in Nelson Textbook of Pediatrics. 17th edition. D. A realistic goal of therapy is to induce hepatitis B surface antibodies C. Therapy is typically not associated with significant side effects Question . g. 74. (See Chapter 337 in Nelson Textbook of Pediatrics. hemolytic. No changes The Digestive System Nelson Self Assessments website 17th Edition 25 . Scratch marks were seen on the skin secondary to pruritus. Bile duct paucity Explanation: This is a classic presentation of a bile duct paucity syndrome. The patient has a history of renal tubular acidosis and peripheral pulmonic stenosis.Question . the hyperbilirubinemia must be characterized as prehepatic (indirect) (e. Giant cell transformation C. A liver biopsy will most likely show: A. The patient had been breast-fed since birth. breast milk. PAS-positive diastase-resistant globules E. E. The stools were described by the mother to be yellow in color. The patient had a normal physical examination. B. Pruritus is secondary to poor excretion of bile salts. physiologic. (See Chapter 337 in Nelson Textbook of Pediatrics. 73. Total bilirubin was 11 mg/dL. The mother was concerned that the baby might have biliary atresia. Bile duct proliferation B. Abdominal ultrasound examination Hepatobiliary scintigraphy Percutaneous liver biopsy Urine culture Total and direct bilirubin Explanation: First and foremost. On examination. C.) D. A 5-wk-old white boy was noted to be jaundiced. widely spaced eyes. such as Alagille syndrome. Biliary atresia usually causes pale stools and hepatomegaly and is always accompanied by cholestatic jaundice. which induce itching when deposited in the skin. The most reasonable initial step is to obtain the following diagnostic test: A.. Gilbert disease) or cholestatic. 17th edition. 17th edition. A 3 yr-old white girl was referred to a pediatric gastroenterologist because of elevated levels of liver enzymes and total and direct bilirubin. the patient was noted to have a broad forehead and deep-set. D.) Question . The anatomic site of hematopoiesis changes during gestation and the population of cells generated at those sites are distinct B. The red cell distribution width (RDW) is elevated in iron deficiency but not in and thalassemia trait The Diseases of the Blood Nelson Self Assessments website 17th Edition 1 .Iron deficiency may have effects on neurologic and intellectual function D. Recovery from severe anemia due to parvovirus B19 infection is usually spontaneous D. 2. Question . All of the following statements regarding iron deficiency are true Except: A. 17th ed. All of the following statements about anemia are true Except: A. 17th ed.Diseases of the Blood Nelson Self Assessments website 17th Edition Question . Pediatrics. the cause usually is a consequence of bleeding or ongoing hemolysis B. The major component of anemia in chronic renal disease is decreased erythropoietin production due to damage of renal tubular cells Explanation: (See Chapter 439 and 443 in Nelson Text. 17th ed. Transient erythroblastopenia of childhood is the most common acquired red cell aplasia occurring in children C.) B. a diet containing 80-100 mg of iron daily is necessary for optimal nutrition Explanation: (See Chapter 447 in Nelson Textbook of Pediatrics. 3. In anemia with an appropriate reticulocyte response. C. The level of serum ferritin provides a relatively accurate estimate of body iron stores in the absence of inflammatory disease E. Erythropoiesis in utero is controlled by erythroid growth factors produced solely by the fetus E. Because absorption of dietary iron is assumed to be about 10%. the onset of anemia and/or jaundice may occur at any age. Some HbA can be detected in even the smallest embryos Question . In congenital dyserythropoietic anemia type 1. Few neutrophils are found in the fetal circulation until the third trimester C.) E. All of the following statements are true Except: A. Intense exercise conditioning may result in iron depletion in adolescent girls C.) D. 1. Thrombopoietin is the physiologic regulator of platelet production but does not act as a potent stimulator of all stages of megakaryocyte growth and development Explanation: (See Chapter 438 in Nelson Textbook of Pediatrics. 6. All of the following statements about paroxysmal nocturnal hemoglobinuria (PNH) are true Except: A. free hemoglobin appears in the plasma C. 7. 4. When the capacity of the heme-binding proteins in the plasma is exceeded. Thrombosis and thromboembolic phenomena are serious complications of PNH D. hereditary pyropoikilocytosis (HPP).) D. Splenectomy is not indicated for PNH The Diseases of the Blood Nelson Self Assessments website 17th Edition 2 .) B.Question . The most severe form of hereditary elliptocytosis. The most common cause of aplastic crisis is parvovirus B19 infection D. The usual marrow response to a chronic hemolytic anemia is reflected by a reticulocyte index of 3-4 Question .) Question . 17th ed. Patients with chronic hemolysis should receive folic acid to prevent secondary folic acid deficiency Question . All of the following statements regarding hereditary spherocytosis are true Except: A. B. All of the following statements regarding elliptocytosis are true Except: A. Hereditary elliptocytosis may produce neonatal jaundice even though characteristic elliptocytosis may not be evident at the time C. Splenectomy does not eliminate most of the hemolysis of hereditary spherocytosis Explanation: (See Chapter 450 in Nelson Textbook of Pediatrics. Chronic hemolysis is more common than nocturnal and morning hemoglobinuria in patients with PNH C. The marrow can increase its output of RBCs two. 5. No treatment is necessary unless hemolysis is present E. Approximately 10% of RBCs are normally removed each day and replaced by the marrow to maintain the RBC count Explanation: (See Chapter 449 in Nelson Textbook of Pediatrics. 17th ed. Bone marrow failure is a rare presentation of PNH Explanation: (See Chapter 453 in Nelson Textbook of Pediatrics. 17th ed. 17th ed.) B. The newborn with hereditary spherocytosis may present with anemia and hyperbilirubinemia severe enough to require phototherapy and exchange transfusions C. is characterized by macrocytosis Explanation: (See Chapter 451 in Nelson Textbook of Pediatrics. Hereditary elliptocytosis is inherited as a dominant disorder. Individuals with hereditary spherocytosis may be asymptomatic without anemia B. Flow cytometry is the best diagnostic test for PNH E.to threefold acutely E. Isoimmune hemolytic anemia due to ABO incompatibility may mimic hereditary spherocytosis D. All of the following statements are true Except: A. Thermal injury can cause spherocytosis E. 9.) D. PK deficiency is the most common glycolytic enzyme deficiency B. Nonincubated osmotic fragility is normal in PK deficiency E. Febrile infants with sickle cell disease should be managed in the hospital C. 8. C. All of the following statements regarding pyruvate kinase (PK) deficiency are true Except: A. D. E. Symptoms usually develop in patients with G6PD deficiency 24-48 hr after ingesting a substance with oxidative properties B. iron deficiency or a combination of Hb S with or thalassemia must be considered a possible diagnosis B. A pregnant woman who ingests oxidative drugs may cause hemolytic anemia in a fetus with G6PD deficiency D. 17th ed. The thalassemia traits present as microcytic anemia. 17th ed. which is essential for oxygen transport Explanation: (See Chapter 454 in Nelson Textbook of Pediatrics. 10. Infection may result in hemolysis in patients with G6PD deficiency C. Children with homozygous 0 thalassemia usually become symptomatic from progressive hemolytic anemia during the second 6 mo of life if not treated E. B.) D. Severe jaundice and anemia may occur during the neonatal period C. which can be mistaken for iron-deficiency anemia Question . 11. 17th ed.) The Diseases of the Blood Nelson Self Assessments website 17th Edition 3 . Epstein-Barr virus Systemic lupus erythematosus Agammaglobulinemia Methyldopa Penicillin Explanation: (See Chapter 456 in Nelson Textbook of Pediatrics.) Question . Spherocytes are common in PK deficiency Question . 17th ed. All of the following statements are true Except: A.Question . The iron in hemoglobin is normally in the ferric state. The usual dose of aspirin causes clinically relevant hemolysis in the A variety of G6PD deficiency Explanation: (See Chapter 455 in Nelson Textbook of Pediatrics. In patients who have Hb SS electrophoresis pattern and concomitant microcytosis. All of the following statements regarding G6PD deficiency are true Except: A. Splenectomy for severe PK deficiency is curative Explanation: (See Chapter 455 in Nelson Textbook of Pediatrics. Enzyme activity in affected persons is 10% of normal or less E. All of the following may cause autoimmune hemolytic anemia Except: A. High altitude C. B. All of the following may cause secondary polycythemia Except: A. 16. Splenic hypofunction is a usual finding in all of the following except A. Endocarditis Malaria Gaucher disease Polycythemia vera Progeria Explanation: (See Chapter 478 in Nelson Textbook of Pediatrics. Sickle cell disease C. Methemoglobin reductase deficiency E. 17th ed. Anabolic steroid therapy Question .) C. Extensive burns Renal disease Liver disease Wilson disease Hypopituitarism Explanation: (See Chapter 457 in Nelson Textbook of Pediatrics.) D. Cold antibodies are primarily of the IgM class and require complement for activity E. C. All of the following may cause splenomegaly Except: A. 17th ed. 17th ed. 12. Hemolytic anemia may be caused by any of the following Except: A. Severe hemolytic anemia E. The prognosis for patients with Evans syndrome (immune thrombocytopenic purpura) is good after the acute episode Explanation: (See Chapter 456 in Nelson Textbook of Pediatrics. Twin-twin hemorrhage B. 14. 15. D. D. Congenital polyspenia Explanation: (See Chapter 479 in Nelson Textbook of Pediatrics.) Question . no underlying cause is found B. The hallmark of the autoimmune hemolytic anemias is a positive result on the direct Coombs test D. Premature infants B. E. 17th ed. Paroxysmal cold hemoglobinuria may account for 30% of immune hemolytic episodes among children Question . B. C. E. 13.) D. Metabolic storage diseases with splenomegaly The Diseases of the Blood Nelson Self Assessments website 17th Edition 4 . All of the following statements are true Except: A. 17th ed.) Question .Question . Plasma volume decrease Explanation: (See Chapter 459 in Nelson Textbook of Pediatrics. In most instances of warm antibody hemolysis. 17.Question . B. Common causes of generalized lymphadenopathy incude all of the following Except: A.) E. Serum sickness The Diseases of the Blood Nelson Self Assessments website 17th Edition 5 . 17th ed. C. Mononucleosis Niemann-Pick disease Leukemia Cat scratch disease Explanation: (See Chapter 482 in Nelson Textbook of Pediatrics. D. ) B. Osteomyelitis Explanation: Bleeding from the nail bed suggests an open fracture (SalterHarris type 1) and places the patient at risk for osteomyelitis. Endocarditis B. (See Chapter 664 in Nelson Textbook Pediatrics. 0 E.) D. 17th ed. The risk of congenital talipes equinovarus (clubfoot) in the offspring of a parent so affected is: A. This child is at risk for: A. 4. (3) mild calf atrophy. 17th ed. 100% B. It may be due to deformational forces in utero hence the association with being firstborn and with hip dysplasias. More common in firstborn D. 1. with bleeding from the nail fold. 50% if the involved parent is female Question . Peripheral neuropathy Question . fibula. (See Chapter 664 in Nelson Textbook of Pediatrics. 20-30% Explanation: The congenital clubfoot is defined as (1) absence of other congenital anomalies. Poor growth of the toe D. A 10-yr-old boy stubs his toe while walking barefoot. 17th ed. and (4) mild hypoplasia of the tibia.) C. the toe should heal quite well without deformity or limitation of growth.) C. Cardiomyopathy Explanation: Idiopathic or neuromuscular disorders are the common causes of cavus feet. Nonoperative treatment Question . in a family with an affected parent it is 20-30%. The toe is tender. Recurrence in siblings is 3%. Metatarsus adductus is associated with all of the following Except: A. (See Chapter 664 in Nelson Textbook of Pediatrics. Spinal cord disorders E. Exostosis Bone and Joint Disorders . In the absence of infection.Bone and Joint Disorders Nelson Self Assessments website 17th Edition Question . 3. The cavus foot is associated with all of the following Except: A. Anemia E. Hindfoot varus position B. and foot bones. (See Chapter 664 in Nelson Textbook of Pediatrics. 2. 3-5% C. (2) variable rigidity of the foot. Bilaterality in 50% C. Hip dysplasia in 10% E. 17th ed.Nelson Self Assessments website 17th Edition 1 . Charcot-Marie-Tooth disease D. Out-toeing Explanation: This condition gives the appearance of intoeing. Slipped capital femoral epiphysis Explanation: External rotation. The most likely diagnosis is: A. Internal femoral torsion is characterized by all of the following Except: A. the condition will resolve. Associated conditions in infants with developmental dysplasia of the hip include all of the following Except: A. 6. 17th ed. A 14-yr-old tall afebrile girl presents with hip pain and an extremely rotated right leg. C. 17th ed. E. Toxic synovitis Bone and Joint Disorders . 7. Thrombophilia C. B. Female gender Explanation: The male to female ratio is approximately 5:1. Preference for sitting in a "W" position among affected children E. A painless limp Question . Associated with ligamentous laxity D. More common in boys Explanation: This disorder is more common in girls with ligamentous laxity who sit on the floor watching television in the "W" position (hips drawn up with legs underneath the buttocks). internal rotation is limited. Legg-Calvé-Perthes disease B. Legg-Calvé-Perthes disease is associated with all of the following Except: A. 17th ed. Breech position Metatarsus adductus Torticollis Ligamentous laxity Muscular dystrophy in males Explanation: DDH is most often seen in females and may be due to in utero position and laxity of ligaments. 17th ed. severe pain. Osgood-Schlatter disease E. (See Chapter 668 in Nelson Textbook of Pediatrics.Question . Diskitis C. Most common cause of intoeing in children older than 2 yr of age B.) C. (See Chapter 665 in Nelson Textbook of Pediatrics. limitation of internal rotation of the leg is classic. and tall stature in an adolescent are important clues.) D.Nelson Self Assessments website 17th Edition 2 . Age at diagnosis of 7 yr E. 8. Limited external rotation to 0-10 degrees Question .) B. Bilateral involvement in 20% D.) Question . (See Chapter 668 in Nelson Textbook of Pediatrics. (See Chapter 668 in Nelson Textbook of Pediatrics. D. 5. On examination. With time and correct sitting. In addition to pain. Sports participation is permitted for patients with all of the following conditions Except: A. Findings on neurologic examination are normal.) E. Blount disease Question . Congenital scoliosis is associated with all of the following Except: A. 11. C. there is full range of motion of the knee without swelling or tenderness. There is decreased hip rotation. Always examine the entire extremity and especially the hip in patients who complain of knee pain but have no demonstrable knee tenderness. 9. reduced lumbar lordosis. and sacral kyphosis. Genitourinary anomalies Congenital heart disease Extradural lipomas Tethered spinal cords Alagille syndrome Explanation: Spinal dysraphism is very common in children with congenital scoliosis and must be investigated with MRI. D. certain dysrhythmias. Spondylolysis C. The most likely diagnosis is: A. Osteogenic sarcoma Ewing sarcoma Osteonecrosis of the femoral head Slipped capital femoral epiphysis Explanation: Be wary of referred knee pain from hip disease. Tethered cord D. C. and bleeding disorders. B. One kidney Bone and Joint Disorders . The buttocks are flattened. and a step-off is felt in the lumbosacral region.Nelson Self Assessments website 17th Edition 3 . 17th ed. Cerebral palsy One functional eye One testis Fever Explanation: An acute febrile illness is a risk for dehydration and hyperthermia. On examination. Relative contraindications to sports participation include acute hepatomegaly or splenomegaly. C. 10.) Question . (See Chapter 668 in Nelson Textbook of Pediatrics. D. 17th ed.) E.Question . Diskitis Question . 12. D. myocarditis. E. (See Chapter 675 in Nelson Pediatrics. 17th ed. (See Chapter 669 in Nelson Textbook of Pediatrics. A 12-yr-old gymnast presents with back pain. B. A 15-yr-old obese girl presents with a history of nontraumatic knee pain. The most likely diagnosis is: A. Spina bifida occulta E. B. Spondylolisthesis with spondylolysis Explanation: This patient had a severe grade of slippage and required spinal fusion to prevent progressive deformity (See Chapter 669 in Nelson 17th ed.) B. the athlete is ready to participate in sports. Prolonged immobilization Explanation: Immobilization may produce further atrophy and weakness. Worsening of pain with prolonged sitting E. physiotherapy is better than steroid injections. 17th ed. "Giving way" and then falling Explanation: Giving way of this magnitude is most often seen with meniscal or ligament tears and not with patellofemoral pain syndromes. B. Dominant negative mechanisms Explanation: Type II collagen is a triple helix of 3 collagen chains produced by the two inherited type II collagen genes. The 5-hop test demonstrates that the athlete can hop as high on the previously injured side as on the well side. Chronic anterior knee pain D. The best approach to management of chronic tennis elbow is: A. Reduce vulnerability to re-injury E. 16. 17th ed. (See Chapter 676 in Nelson Textbook of Pediatrics. Rehabilitation of an ankle injury includes all of the following Except: A. Nonsteroidal anti-inflammatory agents Question . Restoration of peroneal muscle strength D.) E. 17th ed. Imprinting E.) C. Worsening of pain on going up stairs B. Mutations of cartilage mature proteins cause disease by: A. (See Chapter 676 in Nelson Textbook of Pediatrics. Rest Use of wrist splints Steroid injections Physiotherapy Explanation: In the long run. The patellofemoral pain syndrome is characterized by all of the following Except: A.Question . When this happens.) C. (See Chapter 676) B. if one of the three collagen chains is abnormal. 13. Taping the ankle has little value.Nelson Self Assessments website 17th Edition 4 . 14. Mitochondrial inheritance D. C. The five-hop test C. (See Chapter 682 in Nelson Textbook of Pediatrics. Presence of peripatellar tenderness Question . In this autosomal dominant pattern of inheritance. Paternal disomy Bone and Joint Disorders . Autosomal recessive inheritance B. 15. the net result will be a functionally abnormal triple helix: one abnormal chain causes dysfunction of the entire collagen triple helix. D. Use of ankle braces Question . Aortic dilation F. and mitral valve prolapse. 20. Defects in type I collagen B. Apical blebs or pneumothorax Explanation: This is a minor criterion. improvement with decreased bone fractures and pain has been reported with: A. Vertebral compression fractures E. Pamidronate Explanation: The bisphosphates (pamidronate and alendronate) have shown promising results in children with OI. Plain films of the skeleton Explanation: The initial evaluation of a child with a skeletal dysplasia is to define the skeletal abnormalities and the degree of involvement of trunk or distal versus proximal shortening or bowing. Absence of lethal variants Explanation: With OI type II.) C. Fluoride Question . Iliac crest growth plate biopsy E. (See Chapter 682 in Nelson Textbook of Pediatrics. Calcium C. myopia. high-arched palate. Treatment of osteogenesis imperfecta is not curative.Nelson Self Assessments website 17th Edition 5 . which contributes to respiratory failure. Nonetheless. Pectus carinatum requiring surgery C. (See Chapter 689 in Nelson. Which of the following is the most appropriate diagnostic test for this patient? A. There is severe rib involvement. long trunk. (See Chapter 689 in Nelson Pediatrics. mild hypotonia of the lower extremities. Pectus excavatum requiring surgery D.) D. A 20-mo-old female infant presents with disproportionately short limbs. Long bone fractures Question . (See Chapter 690 in Nelson Textbook of Pediatrics. and exaggerated lumbar lordosis. Ectopia lentis E. pectus excavatum of mild to moderate severity.) C. Autosomal dominant inheritance D. the affected child may be stillborn or may die in the first year of life. Additional minor criteria include joint hypermotility. Vitamin D B. MRI of lumbar spine B. 18. 17th ed. 17. Dural ectasia in lumbosacral spine detected by MRI or CT Question . Growth hormone provocative testing Bone and Joint Disorders . 17th ed. Calcitonin E. 17th ed. 19.) B. An examination shows midface hypoplasia. and large head.Question . 17th ed. Osteogenesis imperfecta is characterized by all of the following Except: A. Analysis of DNA for FGFR3 mutations D. Major criteria in the diagnosis of Marfan syndrome include all of the following Except: A. with marked joint laxity. with growth substantially below the 3rd percentile by age 5 yr. Schmid metaphyseal dysplasia Question . A 1-wk-old infant is diagnosed with achondroplasia on the basis of a combination of clinical and x-ray findings. a careful family history is negative for a similar occurrence. normal-stature parents demonstrated deviation from the normal growth curve at age 2 yr. 17th ed. The disorder varies in severity.Question . The disorder is a recessive condition and the parents are carriers (heterozygotes) C.Nelson Self Assessments website 17th Edition 6 . Skeletal radiographs indicate a skeletal dyplasia but do not reveal a specific diagnosis. (See Chapter 682 in Nelson Textbook of Pediatrics. Achondroplasia E. The mutant gene was transmitted but suppressed in previous generations. Which of the following is the most likely potential complication? A. when malabsorption is noted. reaching the threshold for expression only in this infant Explanation: Explanations A-D all are important considerations in counseling parents. However. (See Chapter 682) D. Diastrophic dysplasia C. Physical findings include short. 17th ed. 21. these should not disproportionately affect the skeleton. The family history is negative for a similar occurrence.) Question . 23. An infant dies from respiratory distress shortly after birth. Which of the following is the most likely diagnosis? A. Although you know that these disorders usually have a genetic basis. The differential diagnosis includes other causes of malabsorption. a short neck. 22. The daughter of healthy. deformed extremities. The growth impairment involves both trunk and limbs. One of the parents has germ line mosaicism for the mutation D.) C. and the clinical manifestations are too mild to be recognized in one patient who harbors the mutant gene E. Radiographs show flaring of the metaphyses of tubular bones. Spondyloepiphyseal dysplasia tarda B. and she has a light complexion. Repeated bone fractures Bone and Joint Disorders . The parents are being counseled on potential serious complications to be anticipated during infancy and childhood. and a small thorax. The occurrence in this infant represents a new mutation for normal parents B. Cartilage-hair hypoplasia Explanation: This is a classic picture of a late-appearing skeletal dysplasia. Spinal cord compression Explanation: Spinal compression may occur at the level of the foramen magnum or in the lumbar spine and sites of spinal canal stenosis. (See Chapter 684 in Nelson Textbook of Pediatrics. Which of the following is the least likely explanation for the negative family history? A. Retinal detachment B. She has bowed legs and very short hands and feet. You prepare to provide genetic counseling to the parents. Malignancy E. Precocious osteoarthritis of hips and knees D. Her hair is blond and thin. Anti-inflammatory drugs C. (See Chapter 668.Question . Which of the following is the most likely diagnosis? A.4 in Nelson Textbook of Pediatrics. The most likely diagnosis is: A. 17th ed.) D. Any attempt to touch it is met with a cry. B. C. C.Nelson Self Assessments website 17th Edition 7 . 25. (See Chapter 667.) E. An overweight adolescent boy complains of pain in the medial aspect of his knee. She denies fever or trauma. D. Osteoid osteoma C. best managed by decreased activity of the involved joint. 24.1 in Nelson Textbook of Pediatrics. but she did pull the child by the arm recently when he refused to go into an elevator. Avulsion of the gastrocnemius muscle Question .3 in Nelson Textbook of Pediatrics. The most likely diagnosis is: A.3 in Nelson Textbook of Pediatrics. Legg-Calvé-Perthes disease B. 17th ed. 26. 27. D. Nonaccidental trauma (child abuse) Fracture of the radius Muscle strain of the right pronator Dislocated radial head Explanation: (See Chapter 671. Osteochondritis dissecans E. Antibiotics D. 17th ed. The pain is referred from the hip to the knee. The best treatment for the patient described in Question 24 is: A. B. and he has not had a fever. Osgood-Schlatter disease Explanation: (See Chapter 667. An adolescent girl who is a cheerleader comes to you with a painful bump below her right knee. 17th ed. A 2-yr-old child is brought to you because he refuses to use his right arm. He denies trauma. and the child will not hold objects in his right hand. Excisional biopsy E. Casting for 6-8 wk Question . Osteomyelitis of the tibial tubercle Question .) B. The mother denies trauma. Toxic synovitis Legg-Calvé-Perthes disease Medial collateral ligament (knee) strain Slipped capital femoral epiphysis Explanation: Legg-Calvé-Perthes disease occurs at a younger age than is typical for slipped capital femoral epiphysis. Osteomyelitis Bone and Joint Disorders .) E. Decreased activity of the knee Explanation: The patient described in Question 24 has a classic history of Osgood-Schlatter disease. From the following list. Ecthyma gangrenosum Question .) C. (See Chapter 671.9 in Nelson Textbook of Pediatrics. Clostridium perfringens 2. Alerting the parents to the cause of the problem 5. choose the organism(s) most likely to cause the problem described in Question 29. 1 only 1 and 3 2 and 3 2 and 4 Explanation: Pseudomonas probably came from the sneaker. Radiograph of the arm with thin casting or splinting 2. aureus from the skin. 30. and 5 Question . Foreign body reaction D. Two days later. 1. 1. 17th ed. he limps and complains of pain and swelling in that area.Nelson Self Assessments website 17th Edition 8 . Supination of the forearm is curative. 1. Supination of the forearm 3. Staphylococcus aureus 3. (See Chapter 664. B.9 in Nelson Textbook of Pediatrics. 28. Toxic shock syndrome E.) E. 1 and 4 1 and 3 2 and 3 2 and 4 Explanation: This is a classic history with dislocation of the radial head.3 in Nelson. Staphylococcus epidermidis 4. 17th ed. B.) E. 3. Pseudomonas aeruginosa 5. Osteochondritis Explanation: (See Chapter 664. C. and S. choose the most appropriate measures for management of the child described in Question 27. New sneakers (running shoes) do not have Pseudomonas present until they are worn for some time. A 12-yr-old boy sustains a nail puncture of the right foot through an old sneaker. and counseling parents not to pull small children by the arm is important. Tetanus B. From the following list. D. 17th ed. 3 and 5 Bone and Joint Disorders . D. The most likely diagnosis is: A. Serratia marcescens A. 29. Reporting the case to a child welfare agency A. Antibiotics 4. C.Question . Patellar subluxation B." not as an audible "click.or triple-diapering Explanation: Methods to maintain the unstable newborn hip in the position of flexion and abduction include the Pavlik harness. Surgical closed reduction E. (See Chapter 668. Slipped capital femoral epiphysis E.) D. (See Chapter 664. After the material is cultured and Gram stained. Ciprofloxacin C. the hip usually relocates spontaneously. Scoliosis Question . the patient is started on a combination of intravenous nafcillin and gentamicin.) C.Question . Legg-Calvé-Perthes disease C. (See Chapter 668. Tetanus toxoid E. Observation alone B. Treatment with antibiotics is usually for 7-14 days if debridement is successful.1 in Nelson Pediatrics. although controversial. Double and triple diapers. 33. The Barlow test is used to diagnose: A. 17th ed. Treatment is continued until the hip is clinically stable and ultrasonographic or radiographic measurements of the hip are normal. Piperacillin-tazobactam B. Incision.Nelson Self Assessments website 17th Edition 9 . debridement Explanation: Incision and drainage with debridement of necrotic infected material constitute one of the most important aspects of treatment. Serial spica casts D. Pavlik harness or use of double. This provocative test to dislocate an unstable hip is performed by stabilizing the pelvis with one hand and then flexing and adducting the opposite hip and applying a posterior force. 31. Warm soaks Question .9 ) D. the Frejka splint. drainage. The recommended management of a newborn with an unstable hip examination indicating developmental dysplasia of the hip is: A. If the hip is dislocatable. and a variety of abduction orthoses.1 in Nelson Textbook of Pediatrics. are commonly used in newborns with dislocatable hips for 2-3 wk because initially the splints and harnesses usually do not fit satisfactorily. Open surgical reduction with pelvic or femoral osteotomy Bone and Joint Disorders . 17th ed. The Ortolani test is a maneuver to reduce a recently dislocated hip. the relocation will be felt as a "clunk. The most appropriate first step in management of the boy described in Questions 29 and 30 is: A. it is usually readily felt. Developmental dysplasia of the hip Explanation: The Barlow test is the most important maneuver in examination of the newborn hip to detect developmental dysplasia. if reduction is possible. 32. After release of the posterior force." It is most likely to be positive in infants of age 1-2 mo because adequate time must have passed for the true dislocation to occur. (See Chapter 668.) C. (See Chapter 668. but there appears to be a genetic component B. Avascular necrosis of the capital femoral epiphysis Explanation: The most important and severe complication of developmental dysplasia of the hip is avascular necrosis of the capital femoral epiphysis. The most common cause is idiopathic. All of the following statements regarding scoliosis are true Except: A. but scoliosis is more likely to progress in girls than in boys. whereas in tall. The magnitude of curvature in an affected person is not related to the magnitude of curvature in affected relatives. Hypocalcemia B. Sex hormones. producing cartilaginous compression. minor trauma associated with strenuous physical exercise Question . 17th ed.g. (See Chapter 669.Nelson Self Assessments website 17th Edition 10 .) C.4 in Nelson Textbook of Pediatrics. Bone and Joint Disorders . an overabundance of growth hormone is implicated. 17th ed. extraosseous epiphyseal vessels.. Joint instability D. Slipped capital femoral epiphysis occurs in adolescents who are obese and have delayed skeletal maturation or who are tall and thin and have had a recent growth spurt. The incidence is only slightly greater in girls than in boys. Repeated. Femoral shortening B. and other hormones alter the rate of growth in the capital femoral epiphysis and the rate of skeletal growth. 17th ed. 35. Which of the following is a suspected contributing factor for development of slipped capital femoral epiphysis? A. The most serious complication of developmental dysplasia of the hip is: A.) C. but identical twins are not uniformly affected. In obese adolescents. 34.1 in Nelson. The age at onset is most commonly in adolescence. Rapid alterations of growth hormone and sex hormones Explanation: An endocrine basis of slipped capital femoral epiphysis has been postulated because it is frequently associated with abnormalities of growth. Fat embolism E. Scoliosis is more likely to progress and require treatment in girls than in boys D. Eating disorders (e. bulimia) D. leading to partial or complete infarction.1 in Nelson Textbook of Pediatrics. a low level of sex hormones has been postulated. which can result in occlusion of the intra-articular. Myelokathexis Question . The incidence is much higher in girls than in boys (4:1 ratio) Explanation: Idiopathic scoliosis is the most common form of scoliosis and occurs in healthy. growth hormone. There appears to be a genetic component. This is an iatrogenic complication that results from reduction of the femoral head under pressure. Vitamin C deficiency E.Question . 36. thin patients. neurologically normal children. Daughters of affected mothers are more likely than other children to have scoliosis. 17th ed. and early deafness. (See Chapter 673. Which of the following features defines toddler fractures? A. Deafness D.) Question . C. (See Chapter 697 in Nelson. blue sclerae.Question . minimal soft tissue swelling. 17th ed. 17th ed. (See Chapter 673. Rickets may result from calcium deficiency or phosphate deficiency Bone and Joint Disorders . All patients with rickets have osteomalacia (poor mineralization of trabecular bone). Types of injury requiring immediate attention and orthopedic consultation include: A. E. (See Chapter 689 in Nelson Textbook of Pediatrics.) E. B. refusal to walk. All patients with rickets have osteomalacia C. Rickets is found only in growing children before fusion of the epiphyses E. Fracture with vascular or nerve compromise Open fracture Deep laceration over a joint Grade III (complete) tear of a muscle-tendon unit All of the above Explanation: Blood flow may be obstructed by a dislocated structure.6 in Nelson Textbook of Pediatrics. C.10 in Nelson Textbook of Pediatrics. 40. Clinical features include pain. Any fracture occurring in a toddler Question . Peripheral nerve damage can be repaired after vascular and skeletal stability has been achieved. All of the following statements regarding rickets are true Except: A.Nelson Self Assessments website 17th Edition 11 . D. a slight increase in warmth to palpation over the fracture. 17th ed. D. Rickets results from poor mineralization at the growth plate B.) D. All patients with osteomalacia have rickets Explanation: Mineral deficiency of either calcium or phosphate in growing children before fusion of the epiphyses results in rickets. 37. Infants have shortened bowed extremities and relative macrocephaly. Microcephaly Explanation: The classic triad of findings in osteogenesis imperfecta is composed of fragile bones.) Question . Blue sclerae C. Subluxation of the radial head Salter-Harris type 3 fracture of the distal fibular epiphysis Fracture of the distal radius Spiral fracture of the tibia Explanation: Toddler fractures represent a spiral fracture of the distal third of the tibia. which should be reduced. 38. These fractures occur in children 2-4 yr of age and occasionally up to 6 yr of age. B. Fragile bones B. They are usually the result of simple falls during running or playing. An open fracture should not be immediately reduced because of the risk of further contamination. 39. Principal features of osteogenesis imperfecta include all of the following Except: A. but not all patients with osteomalacia have rickets. and pain with palpation. histoplasmosis. A positive result on an ANA assay (titer 1:80) is associated with all of the following except: Systemic lupus erythematosus Juvenile rheumatoid arthritis Chronic autoimmune hepatitis Salicylate intoxication Explanation: A positive result on an ANA assay is a nonspecific sign of increased lymphocyte activity and is noted in many inflammatory states. phenytoin. 16th ed. is associated with all of the following except: Inflammatory bowel disease Pseudotumor cerebri Explanation: Erythema nodosum in itself is not a disease but is associated with many infections (e. tuberculosis. an erythematous. systemic lupus erythematosus. juvenile rheumatoid arthritis.) Phenytoin . 2. coccidioidomycosis) and inflammatory states (e. group A streptococcal infection.g. nodular.. sulfonamides.RHEUMATOLOGY Question . often pretibial rash. Yersinia infection.. 1. sarcoidosis) and may occur after administration of drugs (e.g. (See Chapter 143 in Nelson Textbook of Pediatrics. oral contraceptives).. Erythema nodosum. inflammatory bowel disease.g.) Sarcoidosis Yersinia Birth control pills Question . 17th ed. (See Chapter 143 in Nelson Textbook of Pediatrics. 5. characterized by small hypopigmented flat scars after a blister formation. Cyclophosphamide Methotrexate Sulfasalazine . Immune complex-mediated diseases reduce total hemolytic complement levels.Question . Low levels of complement may be associated with all of the following except: Nephrotic syndrome Explanation: Nephrotic syndrome is associated with normal levels of complement. 3. Pseudoporphyria is best described as a complication of: Systemic lupus erythematosus Naproxen therapy Explanation: Naproxen. 4. more often than other nonsteroidal anti-inflammatory drugs. Vasculitis Nephritis Serum sickness Systemic lupus erythematosus Question . They are the only drugs needed for at least half of the patients. Which of the following is the preferred agent for initial therapy of most rheumatic diseases? Nonsteroidal anti-inflammatory drugs Explanation: Nonsteroidal anti-inflammatory drugs are the cornerstone of drug treatment of most rheumatic diseases in children. and they provide significant relief for many of the remaining patients. can produce this skin lesion. Oral prednisone therapy Topical steroid therapy Topical chloroquine therapy Question . 17th ed.Azathioprine Question . (See Chapter 145 in Nelson Textbook of Pediatrics. lymphadenopathy.) Bone marrow aspiration .5°C. The most important step in evaluating this patient would be: Erythrocyte sedimentation rate (ESR) Lyme disease titer Chest radiograph Echocardiogram Explanation: An echocardiogram reveals a moderatesized pericardial effusion. and a platelet count of 650. In advising her parents about the probability of persistence of active disease into adulthood. a total white blood cell count of 26. A chest radiograph may reveal cardiomegaly due to pericardial effusion but does not distinguish cardiomegaly from heart failure or effusion. (See Chapter 145 in Nelson Textbook of Pediatrics. a fine but faint macular red-pink rash on the trunk and proximal extremities. An ESR is not diagnostic and occasionally shows low values in serious inflammatory. 7. high spiking fevers. Laboratory studies reveal a hemoglobin of 9. and he has missed school during the last week. or oncologic diseases. you cite which of the following? All symptoms will resolve by age 21 5-10% 50% Explanation: Approximately 45% of children with juvenile rheumatoid arthritis will have active disease that persists into early adulthood.7 g/dL. and a palpable spleen tip. and chills. A 7-yr-old white boy presents with malaise.) 90% 99% Question . A bone marrow aspirate may be informative if leukemia is a consideration. liver edge palpable 4 cm below the right costal margin. with onset of his illness 3 wk previously. 6. chest pain. He has had no ill contacts. A 6-yr-old girl is diagnosed with juvenile rheumatoid arthritis. the most probable cause of this boy's chest pain.000/mm3. 17th ed. a temperature of 40. infectious.000/mm3. Physical examination reveals an acutely ill child with a heart rate of 125/min. Question . hepatosplenomegaly. and lymphadenopathy. The most likely diagnosis for the patient described in Question 8 is: Systemic-onset juvenile rheumatoid arthritis Explanation: Systemic-onset JRA often manifests with prolonged fevers. SLE and rheumatic fever may also produce rheumatologic symptoms and pericardial effusion. asymptomatic pericarditis. but arthritis does not usually develop until later in the course of the illness. 17th ed. 8. anemia of chronic inflammatory disease. a salmon-pink macular rash. Arthralgia or myalgia may be present. 17th ed. 9. The most appropriate next step in approach management would be to: Perform pericardiocentesis Begin inotropic drugs Improve preload with fluids Begin an oral nonsteroidal anti-inflammatory drug Explanation: Administration of an oral anti-inflammatory agent or a corticosteroid is the correct choice for treating the pericardial effusion.) Begin methotrexate Question . (See Chapter 145 in Nelson Textbook of Pediatrics.) Uremia Systemic lupus erythematosus Scleroderma Rheumatic fever . (See Chapter 145 in Nelson Textbook of Pediatrics. On further evaluation. leukocytosis. His pulse normalizes with defervescence. the patient described in Question 7 has no evidence of pericardial tamponade or reduced cardiac function. The most likely diagnosis is: Hypermobility syndrome Dermatomyositis SLE JRA Explanation: The ESR may be normal in patients with active JRA. These are associated with axial arthritis and are nearly universally associated with HLA-B27. She has no rash and very little limitation of range of motion.) Psoriatic arthritis Arthritis with inflammatory bowel disease reactive arthritis secondary to diarrhea Reactive arthritis secondary to genitourinary infection . Nonetheless.Question . A 4-yr-old white girl has had joint swelling in multiple joints for over 6 mo. The enthesitis-related arthritides are characterized by involvement of the axial skeleton and the presence of enthesitis (inflammation at the attachment of tendons to bone). 17th ed. She is slow to move in the morning and moves as if stiff for the first hours of the day. she is a very active child. Her erythrocyte sedimentation rate is 4.) Henoch-Sch nlein purpura Question . Psoriatic arthritis is considered a distinct entity. The C reactive protein may be elevated. as well as the platelet count. These disorders include all of the following except: Rheumatoid factor-negative JRA Explanation: The enthesitis-related arthritides include what has traditionally been called juvenile ankylosing spondylitis. the ESR is not always elevated. 17th ed. 11. (See Chapter 145 in Nelson Textbook of Pediatrics. Thereafter. 10. (See Chapter 146 in Nelson Textbook of Pediatrics. ) Intravenous immunoglobulin . Cryptosporidium parvum. or genitourinary tract infection with Chlamydia trachomatis. and limited forward bending at the waist. Organisms associated with reactive arthritis include: Shigella Chlamydia trachomatis Yersinia enterocolitica Campylobacter jejuni All of the above Explanation: Reactive arthritis may follow enteric infection (with Shigella. 17th ed. 12. 17th ed. except for relief of pain and the functional limitations of arthritis with nonsteroidal antiinflammatory agents. swollen painful joints (right wrist and left ankle). 13. (See Chapter 147 in Nelson Textbook of Pediatrics. Campylobacter jejuni. or Campylobacter jejuni) or genitourinary tract infection with Chlamydia trachomatis. Giardia intestinalis. No specific treatment is necessary for reactive arthritis. Yersinia enterocolitica.Question . his girlfriend was also treated. He was treated 2 wk ago with an antibiotic for a urethral discharge. Yersinia enterocolitica. (See Chapter 147 in Nelson Textbook of Pediatrics. An 18-yr-old boy has a swollen right wrist and left ankle with bilateral pain over both Achilles tendons. The most appropriate therapeutic agent would be: Ceftriaxone Doxycycline Prednisone A nonsteroidal anti-inflammatory agent Explanation: Reactive arthritis may follow enteric infection with non-typhoidal Salmonella. Shigella. Salmonella.) Question . Physical examination reveals tenderness over both Achilles tendons. (See Table 148-2 and Chapter 148 in Nelson Textbook of Pediatrics. cross-reactivity to shared antigenic epitopes between the synovium and the agent.) Thrombocytopenia Pericarditis . Infectious agents associated with arthralgia or arthritis include all of the following except: Rubella vaccine Respiratory syncytial virus Explanation: Agents may produce arthritis by immune complex deposition. All of the following are diagnostic criteria for the diagnosis of SLE except: Malar rash Seizures Raynaud phenomenon Explanation: Diagnosis of SLE requires 4 or more of the 11 diagnostic criteria.) Parvovirus Yersinia Campylobacter Question . or direct infection. 14. 17th ed. (See Chapter 147 in Nelson Textbook of Pediatrics. 15.Question . 17th ed. A 12-yr-old white girl presents with arthralgias of the knees and elbow and swollen hands of 6 months' duration. thrombocytopenia. The best approach to treatment is administration of: Intravenous immune globulin (IVIG) Plasmapheresis Cyclosporine Prednisone Explanation: Prednisone is the treatment of choice for SLE exacerbations.) Question . (See Chapter 148 in Nelson Textbook of Pediatrics. All manifestations need not be present at the same time. Other than swollen joints.) Ibuprofen . photosensitivity. one summer she had severe sunburn. The most likely diagnosis is: JRA ITP Evans syndrome Periarteritis SLE Explanation: The diagnosis of SLE is confirmed by the presence of arthritis. 17th ed. and hematuria. findings on physical examination are normal. 17. 17th ed.Question . Three years earlier. (See Chapter 148 in Nelson Textbook of Pediatrics. hemolytic anemia. Today she has a hematocrit of 25% and a positive result on a Coombs test. mouth ulcers. 16. In addition. and the urinalysis shows multiple red blood cells. She has had intermittent fever and has lost 15 lb. Laboratory testing of the patient described in Question 16 reveals positive results on assays for antibody to double-stranded DNA and antinuclear antibody (ANA) and undetectable complement levels. she was found to have thrombocytopenia and was diagnosed with idiopathic thrombocytopenic purpura (ITP). and 2 yr ago she had mouth sores. Leukopenia.) Subcutaneous nodules False-positive heterophile test Erosive arthritis involving two or more peripheral joints Question . 19. (See Chapter 148. including neurologic disorders such as seizures and psychosis in the absence of other identified causes. The heart block is permanent and often requires cardiac pacing. 17th ed.Question . Which of the following is the most likely etiology? Group B streptococcal infection Neonatal echovirus infection Gestational diabetes Maternal lupus Explanation: Congenital heart block is a prominent feature of neonatal lupus. 18. usually anti-Ro. between the 12th and 16th wk of gestation. malar rash. 17th ed. Which of the following is a component of the diagnostic criteria for systemic lupus erythematosus? Leukocytosis (>15. (See Table 148-2 and Chapter 148 in Nelson Textbook of Pediatrics.000 WBCs/mm3) Seizures Explanation: The diagnosis of SLE is confirmed by the combination of 4 of 11 criteria. A newborn is found to have congenital heart block.) Gray baby syndrome . falsepositive serologic results for syphilis. which results from maternal transfer of IgG autoantibodies. and nonerosive arthritis are also components of the diagnostic criteria.1 in Nelson Textbook of Pediatrics. 17th ed. The most likely diagnosis for the patient described in Question 20 is: Muscular dystrophy Dermatomyositis Explanation: Juvenile dermatomyositis classically affects preadolescent girls with an insidious onset of muscle weakness.) Periarteritis nodosa Systemic lupus erythematosus Myotonic dystrophy . 20. Physical examination reveals a positive Gowers sign and a faint maculopapular rash over the metacarpophalangeal joints. Vasculitic rashes may be present over knuckles. (See Chapter 149 in Nelson Textbook of Pediatrics.500 IU/mL. A 9-yr-old girl reports that she has had difficulty combing her hair and walking up stairs for approximately 1 mo. 17th ed. or the eyelids (which are violet-tinged). The most appropriate laboratory study to order is: Determination of erythrocyte sedimentation rate Measurement of serum creatine kinase level Explanation: The creatine kinase level was 7. the malar area. 21.Question . (See Chapter 149 in Nelson Textbook of Pediatrics.) Rheumatoid factor Motor nerve conduction study Assay for antinuclear antibodies Question . 17th ed. there is a faint erythematous rash over both upper eyelids. 17th ed. (See Chapter 149 in Nelson Textbook of Pediatrics.) Bulbar neuropathy Anterior horn cell disease Botulism Trichinosis . 22. 23.) Complement asay Muscle biopsy Nerve conduction study Question . An ANA assay result was positive but was not considered diagnostic. A 11-yr-old girl has had difficulty in getting out of chairs and combing her hair for 3 mo. The most likely cause of her problems is: Palatopharyngeal muscle weakness Explanation: Striated muscle weakness of the oropharynx produces dysphagia and possible airway compromise. Physical examination reveals tenderness over the quadriceps muscles and 4/5 strength. (See Chapter 149 in Nelson Textbook of Pediatrics.Question . The patient described in Question 22 begins to have difficulty swallowing solids and starts to drool. In addition. The most appropriate next step in the diagnosis is: Serum ANA assay Measurement of serum creatine kinase level Explanation: The creatine kinase level was 2000 IU/mL. approximately 40 times normal. an autoimmune inflammatory disease. 24. The most likely diagnosis is: Contact dermatitis Juvenile rheumatoid arthritis Juvenile dermatomyositis Explanation: The cutaneous findings and proximal muscle weakness are the cardinal features of juvenile dermatomyositis. The most likely diagnosis for the patient described in Questions 22 and 23 is: Polio JRA Juvenile dermatomyositis Explanation: Juvenile dermatomyositis. is characterized by involvement of skeletal muscle and. Physical examination shows periorbital violaceous discoloration. hypertrophic and reddish pink skin over the metacarpal and proximal interphalangeal joints. 17th ed. 17th ed. (See Chapter 149 in Nelson Textbook of Pediatrics. at times.Question .) Mercury ingestion Lyme disease .) Scleroderma Viral myositis Question . Treatment with prednisone is usually effective. the cardiac muscle. (See Chapter 149 in Nelson Textbook of Pediatrics. and proximal muscle weakness. An 8-yr-old boy presents with complaints of fatigue and arthralgias and a rash that is prominent in sun-exposed areas. 25. On exposure to cold. (See Chapter 150 in Nelson Textbook of Pediatrics. The peak age at onset of scleroderma is 30-50 yr. 17th ed. (See Chapter 151 in Nelson Textbook of Pediatrics.) Eyes . She is at increaded risk of developing: Systemic-onset juvenile rheumatoid arthritis Scleroderma Explanation: Raynaud phenomenon may precede extensive skin and internal organ involvement of scleroderma by months or years. 27. Which of the following organ or body systems is least likely to be involved in patients with Behçet disease? Central nervous system Joints Blood vessels Liver Explanation: Hepatic involvement is not characteristic of Behçet disease. and finally erythema of the fingers and toes. then cyanosis. a 14-yr-old girl characteristically develops pallor.Question . 17th ed. children represent fewer than 10% of cases. 26.) Leukemia Histiocytosis Diabetes mellitus Question . The most likely diagnosis is: Scleroderma Sj gren syndrome Explanation: Sj gren syndrome is very uncommon in the pediatric population. 30. self-limited episodes of fever and polyserositis recurring at irregular intervals. She denies a sense of dry mouth. All of the following are features of Sj gren syndrome except: Arthritis Explanation: Arthritis is not a feature of Sj gren syndrome.) Acute HIV infection Mumps Allergic parotitis Question . A 15-yr-old girl complains of burning and itching eyes. 17th ed. 28. A 4-yr-old Middle Eastern boy presents with a history of brief acute episodes of fever and abdominal pain. The most likely diagnosis is: Behçet syndrome Sj gren syndrome Juvenile dermatomyositis Familial Mediterranean fever Explanation: The onset of familial Mediterranean fever is usually before age 5 yr and is characterized by brief.Question . . 17th ed. painless enlargement of the parotid glands. acute. (See Chapter 152 in Nelson Textbook of Pediatrics. 29. (See Chapter 152 in Nelson Textbook of Pediatrics. Physical examination shows bilateral.) Keratoconjunctivitis sicca Parotid gland enlargement Xerostomia Dysphagia Question . Subjective symptoms of xerostomia are less common among younger patients. 32. 31. All of the following diseases are known to be complicated by reactive amyloidosis (AA type) except: Familial Mediterranean fever Juvenile rheumatoid arthritis Henoch-Sch nlein purpura Explanation: Henoch-Sch nlein purpura is not associated with risk for developing amyloidosis.Amyloidosis Question . Which of the following statements regarding familial Mediterranean fever is true? It is inherited as an autosomal dominant disorder The onset of clinical manifestations is usually before age 5 yr Colchicine is the mainstay of treatment Explanation: Colchicine is the mainstay of treatment for familial Mediterranean fever. (See Chapter 153 in Nelson Textbook of Pediatrics.) Inflammatory bowel disease Ankylosing spondylitis . 17th ed. 17th ed.) The disease is common among Native Americans Pulmonary insufficiency is the most important complication and determines the prognosis Question . (See Chapter 154 in Nelson Textbook of Pediatrics. An elevated erythrocyte sedimentation rate. and fatigue for the past 4 wk and also has experienced weight loss. and an elevated angiotensin-converting enzyme level are common. Chest radiograph shows hilar lymphadenopathy. The most likely diagnosis is: Tuberculosis Non-Hodgkin lymphoma Infectious mononucleosis Sarcoidosis Explanation: There are no specific diagnostic tests for sarcoidosis. and enlarged lymph nodes. fever. (See Chapter 155 in Nelson Textbook of Pediatrics. rash and arthritis and characteristic features of noncaseating granuloma on skin biopsy. Which of the following studies should be performed to monitor for irreversible damage from sarcoidosis? Bronchoscopy with transbronchial biopsy Slit-lamp examination of the eyes now and on a yearly basis Explanation: Eye involvement may lead to blindness. A 13-yr-old boy has had chronic cough. hypercalcemia. Chest radiograph shows enlarged hilar lymph nodes. hypercalciuria. 33.) Rheumatoid arthritis Question . Definitive diagnosis requires demonstration of the characteristic noncaseating granulomatous lesions in a biopsy of tissue from an affected area.) Abdominal CT scan now and on yearly basis Serum angiotensin-converting enzyme (ACE) assay now and on a monthly basis Head MRI study now and on a yearly basis . Serum angiotensin-converting enzyme levels correlate with disease activity.Question . A biopsy of the epitrochlear node shows noncaseating granulomas. 34. 17th ed. 17th ed. eosinophilia. mild tachypnea but no crackles or wheezes. hyperproteinemia. most notably the axillary and epitrochlear nodes. (See Chapter 155 in Nelson Textbook of Pediatrics. Physical examination shows slender habitus. A 10-yr-old boy has been diagnosed with sarcoidosis on the basis of clinical features of fever. Question . 35. All of the following statements regarding Kawasaki disease are true except: It occurs in outbreaks Exposure to an affected patient greatly increases the risk of the disease Explanation: Although outbreaks are common, it is uncommon to catch Kawasaki disease from an affected person. (See Chapter 156 in Nelson Textbook of Pediatrics, 17th ed.) Asian children have the highest risk 80% of patients are younger than 5 yr of age It is the leading cause of acquired heart disease in children in the United States Question . 36. Possible complications of Kawasaki disease include all of the following except: Arthritis Splenic rupture Explanation: Splenic rupture and splenomegaly are not associated with Kawasaki disease. Septic meningitis Urethritis Hydrops of the gallbladder Question . 37. The most common cause of death from Kawasaki disease in the early phase of disease is: Myocardial infarction Rupture of an aneurysm Stroke Myocarditis Explanation: In the early stage of disease, before aneurysm of the coronary arteries occurs, severe myocarditis may produce cardiogenic shock. Pericarditis Question . 38. The differential diagnosis of Kawasaki disease includes all of the following except: Dermatomyositis Explanation: Juvenile dermatomyositis is characterized by marked skeletal muscle weakness, which is not characteristic of Kawasaki disease or the other illnesses. (See Chapter 156 in Nelson Textbook of Pediatrics, 17th ed.) Toxic shock syndrome Scarlet fever Stevens-Johnson syndrome Measles Rocky Mountain spotted fever Question . 39. A 2-yr-old girl presents with high, remittent fever of 9 days' duration associated with marked irritability, nonpurulent conjunctival injection, red cracked lips, swollen and erythematous hands and feet, and a maculopapular erythematous rash on the trunk and extremities. Laboratory testing at presentation would most likely show evidence of: Aseptic meningitis Explanation: The platelet count in Kawasaki disease is generally normal initially but rapidly rises by wk 2 or 3 of the illness; counts may exceed 1,000,000/mm3. (See Chapter 156 in Nelson Textbook of Pediatrics, 17th ed.) Thrombocytosis Hyponatremia Lymphocytosis Hemolytic anemia Question . 40. An 11-mo-old boy is treated for acute Kawasaki disease with 2 g/kg of intravenous immune globulin. No cases of measles have reported in his hometown in the last 8 yr. The first measles-mumps-rubella vaccine should be administered to this child at the age of: 12 mo 4 yr 16 mo 3 yr 22 mo Explanation: With the high dose (2 g/kg) of IVIG used for Kawasaki disease, live virus vaccines should be deferred until at least 11 mo following IVIG administration. The antibodies in the IVIG may interfere with the replication of the live virus vaccines, thereby impairing immunogenicity. (See Chapter 156 in Nelson Textbook of Pediatrics, 17th ed.) Question . 41. A 4-yr-old white girl has had a low-grade fever, intermittent crampy abdominal pain with emesis, and swollen knees for 3 days. There is a petechial rash on the lower extremity. The most likely diagnosis is: Meningococcemia Idiopathic thrombocytopenia purpura Henoch-Sch nlein purpura Explanation: Henoch-Sch nlein purpura is a common vasculitis among children and manifests with the classic findings of abdominal pain with or without rectal bleeding, vasculitic rash, arthritis, and nephritis. The platelet count is normal. The gastrointestinal involvement may progress to intussusception. (See Chapter 157 in Nelson Textbook of Pediatrics, 17th ed.) SLE Rocky Mountain spotted fever Question . 42. A 5-yr-old boy presents with the complaint of severe abdominal pain that started 3 days previously. He is unable to eat and has occasional emesis. Physical examination reveals an anxious, acutely ill child with normal vital signs, generalized abdominal tenderness, voluntary guarding of the anterior abdominal muscles, and normal findings on rectal examination. A surgical consultant believes the child has an acute abdomen, possibly appendicitis. Before the child is sent to the operating room, the urinalysis reveals 4+ hematuria and 1+ proteinuria. The most appropriate next step in management would be to: Perform coagulation studies Obtain a complete blood count Perform renal ultrasonography Perform an electrocardiogram Cancel the operation Explanation: You should cancel the operation until the patient is evaluated further and the results of the urinalysis are considered. (See Chapter 157.1 in Nelson Textbook of Pediatrics, 17th ed.) Question . 43. On repeat physical examination, the patient described in Question 42 now has petechiae on the dorsal surfaces of the feet and hands and over the buttocks. His platelet count is 350,000/mm3. The most likely diagnosis is: Kawasaki syndrome Henoch-Sch nlein purpura Explanation: Henoch-Sch nlein purpura is a common form of vasculitis in childhood and manifests with involvement of the skin (nonthrombocytopenic petechiae), joints (arthritis), kidneys (nephritis), and intestine (vasculitis, mucosal hemorrhage, intussusception). (See Chapter 157.1 in Nelson Textbook of Pediatrics, 17th ed.) Rocky Mountain spotted fever Meningococcemia Appendicitis with gram-negative sepsis Question . 44. All of the following are potential complications of HenochSch nlein purpura except: Seizures Coronary aneurysm Explanation: Coronary aneurysms occur most often in the other common childhood vasculitis-Kawasaki disease. (See Chapter 157.1 in Nelson Textbook of Pediatrics, 17th ed.) Pancreatitis Pulmonary hemorrhage Neuropathy Question . 45. The percent of children with Henoch-Sch nlein purpura in whom serious persistent renal disease occurs is: 0% <0.1% Explanation: End-stage renal disease is an uncommon sequela of Henoch-Sch nlein purpura. Nonetheless, 1% of patients do have persistent renal abnormalities. <10% 20% of males 50% of females Question . 46. A 12-yr-old black girl has a 2-yr history of chronic sinusitis. Today she has had an episode of hemoptysis and is experiencing respiratory distress. Her urinalysis reveals hematuria. The most helpful laboratory test is: Sm antibody assay AntiPR3 ANCA assay Explanation: Assay for antineutrophil cytoplasmic antibodies (ANCA) that are directed to protease-3 (PR3) of the neutrophil -granule is the most appropriate test, and results are markedly positive. Measurement of angiotensin-converting enzyme (ACE) ANA assay Erythrocyte sedimentation rate (ESR) Question . 47. The most likely diagnosis for the patient described in Question 46 is: SLE Goodpasture disease Wegener granulomatosis Explanation: This vascular pulmonary renal syndrome is not that rare and fortunately responds to therapy. (See Chapter 157.4 in Nelson Textbook of Pediatrics, 17th ed.) Sarcoidosis Tuberculosis ENDOCRINE Question . 1. A previously healthy 10-mo-old female infant is found unresponsive in her crib. In the emergency department, she is noted to be well developed and well nourished with normal blood pressure and appearance of the genitalia but with increased pigmentation of her skin. Blood glucose level is 30 mg/dL. The most likely diagnosis is: Congenital adrenal hyperplasia due to 21-hydroxylase deficiency Familial glucocorticoid deficiency Explanation: Glucocorticord deficiency results in increased pituitary production of ACTH, which has melanocyte-like stimulatory hormone properties. The combination of normal blood pressure and normal-appearing genitalia in a female patient helps exclude adrenal hyperplasias, which would present much earlier in life (usually in the first month). This familial autosomal recessive disorder does not have salt wasting as a feature and is due in some patients to defects in the ACTH receptor. (See Chapter 569 in Nelson Textbook of Pediatrics, 17th ed.) Cushing syndrome Hyperinsulinemia Congenital adrenal hyperplasia due to 11-hydroxylase deficiency Question . 2. A 4-wk-old female infant is presented to the emergency department with hyponatremia and hyperkalemia. Other than the cardiovascular abnormalities and dehydration, the physical findings are normal. The most informative laboratory examination would be: 17-Hydroxyprogesterone Renin Aldosterone Explanation: Aldosterone is most likely to be deficient in this child. This occurs as a rare autosomal recessive disorder. If she had virilization, one would also consider congenital adrenal hyperplasia. In that case, 17-hyroxyprogesterone assay is the most important diagnostic test. (See Chapter 569 in Nelson Textbook of Pediatrics, 17th ed.) Cortisol DAX-1 Question . 3. An 11-yr-old boy is followed for chronic mucocutaneous candidiasis and hypoparathyroidism. His mother reports increasing fatigue, cutaneous pigmentation, and chronic abdominal pain. The most likely cause is: New-onset diabetes mellitus type 2 Hypothyroidism Hepatitis Adrenal insufficiency Explanation: Polyendocrine autoimmune-induced hypofunction is common in chronic cutaneous mucocutaneous candidiasis. Adrenal involvement is highly suggestive in this case. (See Chapter 569 in Nelson Textbook of Pediatrics, 17th ed.) Celiac disease Question . 4. A 10-yr-old boy is under medical management for adrenal insufficiency. His mother reports he has complained that he is not as good at playing sports as previously and has trouble reading. The first step in the care of this boy is to: Measure very long-chain fatty acids Explanation: This patient probably has a form of adrenal leukodystrophy, which is an X-linked peroxisomal disorder. In the absence of antiadrenal antibodies, this X-linked disorder may be responsible for about 50% of cases of adrenal insufficiency in males. (See Chapter 569 in Nelson Textbook of Pediatrics, 17th ed.) Measure thyroid hormone levels Increase his replacement dose of hydrocortisone Order an MRI study of his brain and pituitary Refer him to an ophthalmologist Question . 5. A 6-yr-old girl underwent removal of a craniopharyngioma 3 mo previously. She is on a thyroid replacement regimen, but her mother reports she is very fatigued. The next step would be to: Increase the dose of her thyroid medication. Schedule a growth hormone stimulation test Measure ACTH and cortisol Explanation: If she already has one endocrine disorder following removal of a pituitary tumor, she is at high risk for another, and in this case adrenal function must be evaluated. If she has adrenal insufficiency from a pituitary hormone deficiency, hyperpigmentation will be absent because ACTH and cortisol levels will both be very low. (See Chapter 569 in Nelson Textbook of Pediatrics, 17th ed.) Obtain an MRI study of her head Measure IGF-1 and IGFBP-3 Question . 6. A 2-mo-old infant is presented with failure to thrive, recurrent emesis, hepatosplenomegaly, and adrenal insufficiency. Adrenal calcification is noted radiologically. The most likely diagnosis is: Adrenal hemorrhage Tuberculosis Neuroblastoma Pheochromocytoma Wolman disease Explanation: This is a classic presentation of this cholesterol storage disease. In the absence of these significant clinical findings, adrenal calcifications may be an incidental finding. But in this case, it is an important clue to the diagnosis. (See Chapter 75 in Nelson Textbook of Pediatrics, 17th ed.) Question . 7. A 2-wk-old male infant is eating poorly, has repeated emesis, and has not regained his birth weight. On physical examination he appears dehydrated and is noted to have increased pigmentation of skin creases and genitalia. In addition to measuring serum electrolytes, assay of which of the following hormones would be most informative? 17-Hydroxyprogesterone Explanation: This is a classic presentation and timing for the presentation of a salt-losing congenital adrenal hyperplasia in a genotypic male. 17-Hydroxyprogesterone assay is the initial diagnostic test. Determination of electrolytes would reveal hyponatremia and hyperkalemia. The patient should be examined to confirm the presence of gonads in the scrotum. If there are not, this patient could be a virilized female with complete fusion of the labial-scrotal folds. (See Chapter 570 in Nelson Textbook of Pediatrics, 17th ed.) Cortisol 17-Hydroxypregnenolone Renin Aldosterone Question . 8. An 8-yr-old girl treated from infancy for adrenal insufficiency is noted to have inguinal gonads. Karyotype is 46,XY. The most likely diagnosis is: True hermaphroditism Testicular feminization DAX-1 mutation Lipoid adrenal hyperplasia Explanation: Adrenal insufficiency rules out testicular feminization and true hermaphrodism, which does not involve the adrenals. This is a classic example of lipoid adrenal hyperplasia. The gonads are probably testes, which will need to be removed in the future. (See Chapter 570 in Nelson Textbook of Pediatrics, 17th ed.) 17-Hydroxylase deficiency Question . 9. A newborn infant is noted to have increased pigmentation of his skin and genitalia, perineal hypospadias with bifid scrotum, and a 2-cm phallus with chordee. Gonads are palpable in the inguinal canal, and no uterus is visualized on ultrasound examination. The most likely diagnosis is: Incomplete testicular feminization 21-Hydroxylase deficiency 3 -HSD deficiency Explanation: This incompletely virilized male (no uterus but partial male genitalia) probably has 3 -HSD deficiency. (See Chapter 570 in Nelson Textbook of Pediatrics, 17th ed.) Lipoid adrenal hyperplasia 11-Hydroxylase deficiency Question . 10. A 5-yr-old boy presents with pubic hair development. He is tall and has increased pigmentation of his genitalia and phallic enlargement. Blood pressure is 130/90 mm Hg. Measurement of which of the following hormones would be most likely to be diagnostic? Testosterone 17-Hydroxyprogesterone 11-Deoxycortisol Explanation: This male has a non-salt-losing form of congenital adrenal hyperplasia due to 11-hydroxylase deficiencies. The metabolic defect produces salt-retaining steroids (deoxycorticosteroids) and is thus associated with hypertension and usually presents later in life than is typical for salt-wasting forms of 21-hydroxylase deficiencies. (See Chapter 570 in Nelson Textbook of Pediatrics, 17th ed.) Aldosterone Deoxycorticosterone Question . 11. A 2-yr-old boy presents with glandular breast development and pubic hair development. His testes are prepubertal. The most appropriate next step in his evaluation is: ACTH stimulation test LHRH stimulation test Ultrasound examination of his abdomen Explanation: These findings suggest increased androgen production (pubic hair) and androgen conversion to estrogen. An adrenal source is a strong possibility and must be excluded before other testing is planned. (See Chapter 556 in Nelson Textbook of Pediatrics, 17th ed.) MRI of brain and pituitary CT of brain and pituitary Question . 12. A 4-yr-old previously normal girl has developed acne and pubic hair. On physical examination she has clitoromegaly and mild increase in her blood pressure. Serum dehydroepiandrosterone sulfate is markedly elevated. The most likely diagnosis is: Malignant adrenal tumor Explanation: This is a dramatic presentation of endogenous androgen production, most likely of an adrenal origin as indicated by the elevation of the adrenal-derived hormone dehydroepiandrosterone sulfate. (See Chapter 556 in Nelson Textbook of Pediatrics, 17th ed.) CAH due to 11-hydroxylase deficiency CAH due to 21-hydroxylase deficiency Malignant ovarian tumor McCune-Albright syndrome 13. (See Chapter 571 in Nelson Textbook of Pediatrics. The next step(s) in his evaluation should include: MRI of his head with and without contrast Explanation: It is very possible that the suppressibility from higher dose dexamethasone represents ACTH production from a central origin. A 6-yr-old girl with extensive café-au-lait lesions and polyostotic fibrous dysplasia is at risk for: Gonadotropin-independent precocious puberty Cushing syndrome Thyroid nodules Pituitary tumor All of the above Explanation: McCune-Albright syndrome is a polyendocrinopathy with excessive hormone production.) Petrosal sinus sampling before and after CRH determination MRI of his abdomen Adrenal vein catheterization All of the above . 17th ed.) Question . 14. A 13-yr-old boy is undergoing evaluation for Cushing syndrome. which requires higher doses of dexamethasone to reduce cortisol levels. Additional features include fibrous dysplasia and cutaneous hyperpigmentation. 17th ed. His cortisol level is unchanged following dexamethasone 30 µg/kg/day in 4 divided doses but decreases to 4 µg/dL following 120 µg/kg/day in 4 divided doses.Question . Precocious puberty that is independent of central gonadotrophic hormones is classic. (See Chapter 556 in Nelson Textbook of Pediatrics. Carney complex includes which of the following components? Large cell calcifying Sertoli cell tumors Cardiac myxomas Primary pigmented adrenocortical disease Autosomal dominant transmission All of the above Explanation: This autosomal dominant disorder is mapped to chromosome 2p16 and may be due to a gain-of-function mutation. A 10-yr-old girl has grown 4 cm and gained 7 kg in the past year.) Question . blue nevi. Her 24-hour urinary free cortisol is increased. 17th ed.Question . and sexual precocity in boys with large cell calcifying Sertoli tumors and melanotic schwannomas. (See Chapter 571 in Nelson Textbook of Pediatrics. which in children is most often due to a primary adrenal disorder. She is diffusely obese with violaceous striae on her trunk and extremities. (See Chapter 571 in Nelson Textbook of Pediatrics.) An ACTH stimulation test . 17th ed. The next step in her evaluation is: MRI of her brain and pituitary MRI of her abdomen Measurement of morning cortisol A 2-step dexamethasone suppression test Explanation: This two-step test will help determine if ACTH is of central origin. cardiac and skin myxomas. The adrenal disorder is a pigmented nodular adrenocortical disease with associated features such as lentigines. 16. 15. This is a classic clinical picture of Cushing disease. 18. 17. .) Liddle syndrome Aldosterone-secreting adenoma Bilateral micronodular adrenocortical hyperplasia Question .Question . The ratio of tetrahydrocortisol to tetrahydrocortisone is increased in her urine. Renin activity is suppressed. 19. Glucocorticoid-remediable aldosterone is caused by: A mutation in the gene for 11-hydroxylation A mutation in the gene for 11 -HSD A mutation in the gene for 17-hydroxylation Formation of a hybrid gene between CYPI IB1 and CYPI IB2 Explanation: This unique mechanism explains the suppressibility of the hyperaldosteronism. 17th ed. The next step in the evaluation is: Dexamethasone suppression test Explanation: Glucocorticoid-suppressible hypertension is a primary adrenal disorder that must be evaluated with the dexamethasone suppression test. The most likely diagnosis is: Glucocorticoid-remediable hyperaldosteronism Apparent mineralocorticoid excess (11 -HSD deficiency) Explanation: The hypokalemic alkalosis in a hypertensive patient without abnormal genitalia or sexual precocity is highly suggestive of a primary disorder of mineralocorticoid excess. A 6-yr-old girl is hypertensive with hypokalemic alkalosis. A 12-yr-old boy is hypertensive with suppressed plasma renin activity and elevated aldosterone. (See Chapter 572 in Nelson Textbook of Pediatrics. with one gene regulating the other. 17th ed. (See Chapter 572 in Nelson Textbook of Pediatrics.) CT study of his abdomen Adrenal vein catheterization ACTH stimulation test Sodium loading test Question . 17th ed. 21. Pheochromocytomas may be associated with: Neurofibromatosis Von Hippel-Lindau disease MEN IIA and MEN IIB Tuberous sclerosis All of the above Explanation: Pheochromocytomas may be isolated tumors. A 10-yr-old girl has hyperaldosteronism that is unresponsive to dexamethasone. The next step is: Adrenal vein catheterization Explanation: Local production of aldosterone can be determined only by selective adrenal vein catheterization.A mutation in the gene for the mineralocorticoid receptor Question . (See Chapter 574 in Nelson Textbook of Pediatrics. 20. (See Chapter 574 in Nelson Textbook of Pediatrics. 17th ed. 22. No adrenal tumor is visualized on CT. 17th ed.) Metyrapone administration Spironolactone (Aldactone) administration Exploratory laparotomy Furosemide (Lasix) administration Question .) . Presenting manifestations of pheochromocytoma may include: Sustained hypertension Abdominal pain Polyuria and polydipsia Weight loss All of the above Explanation: In addition. A small tumor may not be seen on CT. but a good family history as well as physical examination may reveal associated syndromes. (See Chapter 572 in Nelson Textbook of Pediatrics. we have seen some patients present with symptoms initiated only during exercise.) Question . A 10-yr-old boy receiving vincristine for treatment of a malignancy has developed the syndrome of inappropriate antidiuretic hormone secretion (SIADH). 23. (See Chapter 553 in Nelson Textbook of Pediatrics. 17th ed. Vital signs are stable: heart rate 120/min.Question . He appears well hydrated. blood pressure 120/75 mm Hg. His serum osmolality is 310 mOsm/kg and urine osmolality is 100 mOsm/kg. 17th ed. 24. An 11-yr-old boy complains of increasing headaches and recent visual changes affecting his lateral fields of vision. Six hours later he is noted to have a brisk urine output of 4-6 mL/kg/hr. Which of the following is the most likely diagnosis? SIADH Cerebral salt wasting Normal postoperative diuresis Central diabetes insipidus Explanation: Interference with the transport or production of hypothalamically synthesized arginine vasopression may be a temporary or permanent defect following surgery in this area of the brain. (See Chapter 553 in Nelson Textbook of Pediatrics. Other causes of dilutional hyponatremia such as heart failure or cirrhosis are characterized by a reduction in effective renal blood flow. a known complication of this therapy. MRI of the brain reveals a midline mass measuring 2 cm in diameter in the region of the anterior pituitary gland. This is not a problem in SIADH.) Nephrogenic diabetes insipidus Question .) Uric acid Serum sodium . He undergoes transsphenoidal resection of the mass and is transferred to the pediatric intensive care unit for postoperative care. All of the following parameters would be decreased in this patient except: Urine production rate Serum osmolality Intravascular volume Explanation: The pathophysiologic mechanism is excessive free water retention and dilutional hyponatremia. stable weight. 17th ed.You suspect diabetes insipidus.The most appropriate next step in the management of this patient is: Serum and urine electrolytes MRI scan of the brain Renal ultrasound study Quantification of daily fluid input and output Explanation: This is essential to confirm the history but should not be done alone if there are signs of hypovolemia that need immediate treatment. drugs (lithium) and electrolyte disturbances (hypokalemia. She has no history of trauma. and normal findings on the neurologic examination. 17th ed. 25. A random serum sodium is reported to be 148 mmol/L. Physical examination demonstrates a well-appearing child with normal vital signs. followed by parenteral administration of arginine vasopressin (DDAVP) if the deprivation test does not demonstrate effective urine-concentrating ability.) Question . (See Chapter 553 in Nelson Textbook of Pediatrics.Question . 26. recent illness. A 5-yr-old girl with a 6-mo history of excessive urination and thirst is seen for an evaluation.) Water deprivation test . During the review of systems questioning. A response to DDAVP suggests a central origin for the DI. hypercalcemia) are common etiologic factors. If the history is suggestive of diabetes insipidus. or medication use. Current medications include risperidone (Risperdal) and lithium. a water deprivation test is performed. In the acquired form. you note increased thirst and frequency of urination. A 14-yr-old boy with bipolar disorder is seen for a health maintenance visit. The most likely diagnosis in this patient is: SIADH Primary polydipsia Adrenal insufficiency Central diabetes insipidus Nephrogenic diabetes insipidus Explanation: Nephrogenic diabetes insipidus may be congenital or acquired. (See Chapter 552 in Nelson Textbook of Pediatrics. (See Chapter 553 in Nelson Textbook of Pediatrics. chromosomal (XYY. or metabolic (Marfan syndrome. which may manifest with seizures. 28.Question . The hyponatremia often corrects spontaneously. 27. The most likely explanation for this child's seizures is: Hyperthermia Child abuse Hypernatremia Hyponatremia Explanation: Excessive free water intake in children younger than 6 mo can produce hyponatremia. homocysteinuria) disorders. lethargy. but if symptoms are present when the hyponatremia is reported.) Febrile seizure Question . XXY). All of the following are important in the evaluation of tall patients except: Family history IGF-1 Serum homocystine Chromosomal analysis Long-chain fatty acids Explanation: Tall stature may be familial or due to pituitary.) . 17th ed. or hyperthyroidism. (See Chapter 556 in Nelson Textbook of Pediatrics. A CT scan of the brain is reportedly normal. and hypothermia. The family notes having recently diluted the child's formula to provide additional water. hypertonic (3%) saline may be used. A 1-mo-old child is brought to the emergency department during a heat wave after she has had a generalized seizure. obesity. ed. His physical examination is unremarkable except for a Chvostek sign. Serum level of intact PTH is 312 pg/mL (normal 10-60 pg/mL). This is caused by a deletion of chromosome 22q11-12. His serum calcium is 6. The most likely diagnosis is: Primary hyperparathyroidism Pseudohypoparathyroidism type 1B Explanation: Pseudohypoparathyroidism (elevated PTH. In contrast. patients with type 1A have brachydactyly. The seizure is generalized and lasts approximately 8 minutes.) Measurement of serum phosphorus level Question . (See Chapter 565 in Nelson Textbook of Pediatrics. 29.4 mg/dL.1 mg/dL with a serum phosphorus level of 9. cataracts. low calcium) of this type is not associated with phenotypic morphologic features. A 12-yr-old Hispanic boy with a history of a seizure disorder presents to his pediatrician with a complaint of leg cramps. 17th ed. He is of normal stature. (See Chapter 566 in Nelson Textbook of Pediatrics. and calcification of basal ganglia. which is readily detected with FISH techniques. Measurement of serum calcium in the emergency department reveals a level of 6. a combination of manifestations highly suggestive of DiGeorge/velocardiofacial syndrome. The most appropriate next step in diagnosis is: EEG Measurement of intact PTH after the serum calcium is corrected Measurement of intact PTH while the calcium is low followed by a fluorescence in situ hybridization (FISH) assay for DiGeorge syndrome Explanation: This child has hypoparathyroidism and congenital heart disease. A 3. 17th ed. mild mental retardation.5-kg male infant has a seizure at 7 days of age.) Pseudohypoparathyroidism type 1A A calcium-sensing receptor-activating mutation . On examination. 30.Question . the infant appears well but has a grade III/VI systolic murmur.4 mg/dL. 32. His serum estradiol and prolactin levels are normal. Some cases are familial. and the albumin is normal.0 mg/dL. as many as 60% of boys have some degree of pubertal gynecomastia. The most appropriate next step in the treatment of this patient is: Reassurance and explaining the transient nature of the breast findings Explanation: The problem described in the question is transient and compatible with the physiology of puberty in normal males. He complains of occasional episodes of breast tenderness. the phosphorus level is 8.) Mammography Abdominal CT including the pelvis Head CT focusing on the pituitary Karyotype Question . A 12-yr-old girl experiences muscle cramps and tingling of her hands and feet unrelated to exertion. the serum total calcium is 6. 17th ed. When she grabs a door handle to open the door.5 mg/dL. 17th ed. A 14-yr-old boy has unilateral gynecomastia and is Tanner stage 3 in pubertal development. The most important laboratory test is: Serum glucose determination Serum calcium determination Explanation: For the patient described.) Electromyography (EMG) Nerve conduction velocity testing Arterial blood gas determination . which last less than 30 minutes and occur once a month. Indeed. she is unable to release her grasp because her hand is in spasm.Question . 31. (See Chapter 565 in Nelson Textbook of Pediatrics. (See Chapter 556 in Nelson Textbook of Pediatrics. Conditions associated with growth hormone deficiency include all of the following except: Cleft palate Midfacial anomalies Solitary maxillary central incisor Optic nerve hypoplasia VATER syndrome . Breast development and pubic hair have been present for 18 months and are normal. flushed. Her medical history and findings on the complete physical examination are normal.Question . A mother and her 14½-yr-old daughter come to you because the girl has not begun to menstruate. warm skin Question . 33. Tremor Weight loss Tachycardia Smooth. Workup should begiin after age 16. 35. Laboratory evaluation for systemic disease Urinary estriol determination Buccal smear Referral for psychologic counseling Question . Physical findings in Graves disease include all of the following except: Motor hyperactivity Cold intolerance Explanation: Patients with hyperthyroidism have heat intolerance because of their hypermetabolism. 34. Which of the following would be the most appropriate next step in the management of this patient? Reassurance that she probably will begin menstruating within the year Explanation: Reassurance about lack of menarche is appropriate at age 14. A 6-yr-old girl presents with breast enlargement and pubic hair development. 38. as occurs in severe caloric deprivation. The most likely diagnosis is: Hypothyroidism Exposure to exogenous estrogen An estrogen-secreting tumor An adrenal tumor Central precocious puberty . Question . she is asymptomatic. By age 1 yr. the children appear short but are not thin or wasted. Otherwise. Those with multiple pituitary hormone deficiencies may demonstrate a shorter length. 37. Sotos syndrome is characterized by all of the following except: Macrocrania Large size at birth Tall stature Clumsiness Normal intelligence Explanation: Most patients have some degree of mental retardation. Perceptual deficits are also common. especially if short stature is present. Children with growth hormone deficiency are best characterized by all of the following except: Low birthweight Explanation: Most children with isolated GH deficiency are of normal length and weight at birth. suggest pituitary and thus growth hormone deficiency. 36. Her serum LH level is elevated. Question .Explanation: Facial anomalies. Delayed epiphyseal closure Short stature by 1 yr of age Round-shaped head Hypoglycemia Small hands and feet Question . 17th ed.Explanation: Central precocious puberty involves idiopathic release of LH and a normal. Optimal treatment for the girl described in Questions 38 and 39 is: Leuprolide Explanation: Long-acting GnRH analogs in depot form help prevent gonadotropic cell release of their hormones. (See Chapter 556 in Nelson Textbook of Pediatrics. 40. 17th ed. the lesions are seldom malignant and rarely require neurosurgical intervention. albeit early. the risk of such a lesion is low. the risk of such a lesion is low.) Question . In addition. In girls.) No further testing Explanation: C or B. progression of puberty. the risk of identifying a CNS lesion is 25-75%. in boys. In the absence of neurologic or visual signs. the risk of identifying a CNS lesion is 25-75%. 17th ed. (See Chapter 556 in Nelson Textbook of Pediatrics. (See Chapter 556 in Nelson Textbook of Pediatrics. a pituitary or hypothalamic lesion is highly unlikely.) Serum testosterone test Adrenal ultrasound study Question . (See Chapter 556 in Nelson Textbook of Pediatrics.) Growth hormone Progesterone Prednisone Cranial irradiation . in boys. 39. 17th ed. In the absence of neurologic or visual signs. thus abating the early onset of puberty. the lesions are seldom malignant and rarely require neurosurgical intervention. The most appropriate next step in the evaluation of the child in Question 38 is: Serum prolactin level determination Head MRI Explanation: C or B. a pituitary or hypothalamic lesion is highly unlikely. In girls. In addition. Her skin appears mottled. feeding intolerance. A 6-wk-old infant has gained no weight since birth. distended abdomen. thyroid gland growth blocking and cytotoxic antibodies may have a role in the etiology of congenital hypothyroidism. (See Chapter 556 in Nelson Textbook of Pediatrics.) Absence of symptoms at birth Question . constipation. if any.) . and an indirect bilirubin level is measured at 24 mg/dL. 41. and her temperature is 35oC. All of the following are characteristics of congenital hypothyroidism except: Higher incidence in females Incidence of 1:4. 42. However. 17th ed. prolonged sleep.Question .000 Lower incidence in African-Americans Central role of thyroid peroxidase antibodies Explanation: These autoantibodies have a minor role. and a poor cry. (See Chapter 559 in Nelson Textbook of Pediatrics. The most likely diagnosis is: Kernicterus Sepsis Galactosemia Hypothermia Hypothyroidism Explanation: Congenital hypothyroidism may also demonstrate large fontanels. in the etiology of this disorder. 17th ed. Her extremities are cold. and her serum cholesterol is 500 mg/dL. She denies headaches or poor school performance. 17th ed. the serum T4 is the best of these tests. state screening for congenital hypothyroidism has prevented this type of late presentation with the inherent risks of mental retardation. 44. A 14-yr-old girl presents with poor growth and delayed puberty.) Graves disease Congenital hypothyroidism Familial type II hyperlipidemia Pituitary prolactinoma . (See Chapter 559 in Nelson Textbook of Pediatrics. The most likely diagnosis is: Hashimoto disease Explanation: Hypothyroidism that is acquired is insidious in onset and primarily affects growth. The evaluation of the child described in Question 42 is best accomplished by: Serum TSH determination Serum T4 assay Explanation: The serum T4 is markedly depressed. and therefore the TSH is elevated and a thyroid scan shows no uptake or ectopic tissue. Therapy with T4 should be initiated immediately.Question . (See Chapter 559 in Nelson Textbook of Pediatrics. Central hypothalamic pituitary causes have a low TSH. Schoolwork is not as severely affected. 17th ed. Fortunately. Most cases of congenital hypothyroidism are due to dysgenesis of the thyroid gland. Nonetheless. as one would expect. She is physically sluggish and has a small goiter.) Bone age determination Head CT Thyroid scan Question . 43. also known as anti-islet antibodies. . as the serum osmolarity drops below that in the brain. resulting in shift of fluid to the CNS.Question . Type 1 diabetes mellitus is most often associated with: Mumps infection Coxsackievirus infection Antibodies to glutamic acid dehydrogenase Explanation: Anti-GAD antibodies. 47. 45. Common features of Graves disease include all of the following except: A 5:1 male:female ratio Explanation: The female:male sex ratio is actually 5:1. Hyperglycemia during diabetic ketoacidosis may be associated with: Hypocalcemia Hypernatremia Hyponatremia Explanation: Hyponatremia may be due to measurement artifacts of serum glucose levels. 46. are present in at least 90% of children with insulin-dependent diabetes. Cow's milk Mitochondrial DNA deletions Question . Failure of the serum sodium level to rise during therapy places the patient at risk for cerebral edema. A 6-mo to 1-yr delay in diagnosis Emotional disturbances Poor school work Tremors Voracious appetite Exophthalmos Question . Hypokalemia may develop during therapy with insulin.) . 48. placing potassium salts in the intravenous solution given to the patient may reduce this risk. (See Chapter 583 in Nelson Textbook of Pediatrics.Hypomagnesemia Hypocholesterolemia Question . 17th ed. which is usually reversed with improvement in metabolism by insulin and improved tissue perfusion from isotonic fluids. Hyperkalemia in severe diabetic ketoacidosis is due to: Renal failure Hemolysis Hyperglycemia Artifact Acidosis Explanation: Transcellular shifts of hydrogen into the cell with potassium leaving the cell during acidosis produce transient hyperkalemia. In the case described in Question 2.73 m2) is: 35 mL/min/1. which of the following laboratory studies would be the most helpful in determining the cause of this patient's hematuria and proteinuria? Complete blood cell count Serum cholesterol determination Antistreptolysin O antibody level . This clinical presentation is most consistent with: Acute renal failure Acute pyelonephritis Nephrotic syndrome Acute glomerulonephritis Explanation: (See Chapters 500. 100 red blood cells per high-power field. 1+ proteinuria.) Chronic renal failure 2 Question . His urinalysis is remarkable for 3+ hematuria.514 in Nelson Textbook of Pediatrics. 17th ed. 2. serum creatinine 1. progressive facial swelling over the past 3 days. His examination is notable for blood pressure 130/80 mm Hg.0 m2) with chronic renal insufficiency is seen in your clinic and undergoes a 24-hr urine collection for measurement of creatinine clearance.2 g/liter. bibasilar rales. and decreased urine volume over the past day.73 m2 40 mL/min/1.73 m 2 60 mL/min/1. this patient's standard creatinine clearance (mL/min/1. A 10-yr-old girl (body surface area of 1.7 mg/dL.73 m2 70 mL/min/1.Nephrology Question . 3. and red blood cell casts. periorbital edema. 1. 17th ed. The results are as follows: urine creatinine 144 mg/dL. A 3-yr-old boy presents to your office with sudden onset of colacolored urine. Based on these measurements.73 m2 Question . and ankle swelling. His serum electrolytes are normal and the serum albumin is 3. urine volume 700 mL.73 m Explanation: (See Chapter 500 in Nelson Textbook of Pediatrics.) 25 mL/min/1. The most likely mode of inheritance for this child's glomerular disease is: Autosomal dominant with incomplete penetrance X-linked dominant Explanation: (See Chapters 500.514 in Nelson Textbook of Pediatrics. Results of blood chemistry studies are also normal.) Autosomal recessive X-linked recessive Autosomal dominant . and examination findings are normal. serum creatinine of 1.24-hr urine collection for measurement of protein and creatinine clearance Complement C3 and C4 assays Explanation: (See Chapters 500. Which of the following statements is true regarding this child's form of glomerulonephritis? This disease is more common in females The primary treatment is blood pressure control Explanation: (See Chapters 500. This patient's mother also has microscopic hematuria. A 10-yr-old boy is noted to have hematuria and proteinuria on a routine physical examination.514 in Nelson Textbook of th Pediatrics. 4. which reveals mesangial proliferative glomerulonephritis with very bright immunoglobulin A deposits in the mesangium on immunofluorescence. and urinary protein of 2. A 15-yr-old boy with a 12-yr history of microscopic hematuria is noted to have bilateral high-frequency sensorineural hearing loss.) Progressive kidney disease occurs in a majority of children The complement C3 value is usually low Children with this disease rarely present with gross hematuria Question .5 mg/dL.) Question . but analysis of the 24-hr urine specimen reveals 2 g of protein and a normal creatinine clearance. 5. He is without complaints. 17th ed. A renal biopsy is performed. blood pressure of 140/90 mm Hg.514 in Nelson Textbook of Pediatrics.000 mg/24 hr. 17th ed. 17 ed. 17th ed. oliguria.514 in Nelson Textbook of Pediatrics. Her complement C3 is noted to be very low at 15 mg/dL.514 in Nelson Textbook of Pediatrics. The most common clinical presentation for membranous nephropathy in children is: Asymptomatic microscopic hematuria Acute nephritic syndrome Nephrotic syndrome Explanation: (See Chapters 500. 17th ed. When should this patient's complement C3 level be repeated in order to confirm your suspected diagnosis? In 1 week In 2 wk In 3 wk In 4 wk In 8 wk Explanation: (See Chapters 500. Question .) Complete absence of symptoms with normal results on urinalysis Acute renal failure Question . A 5-yr-old girl presents with cola-colored urine. 8. Which of the following statements about membranoproliferative glomerulonephritis in children is true? It occurs most commonly in the first decade of life Hypocomplementemia usually resolves within 2 mo of presentation Alternate-day glucocorticoid therapy may be beneficial in stabilizing the clinical course Explanation: (See Chapters 500. and body edema 2 wk after being treated for group A -hemolytic streptococcal pharyngitis.) Progression to end-stage renal disease is rare It is an uncommon cause of chronic glomerulonephritis .Question .514 in Nelson Textbook of Pediatrics. 6. 7. 17 ed.) Intravenous monthly cyclophosphamide infusions Plasmapheresis Conservative management by continued follow-up evaluation in your clinic before initiation of further treatment Question . 17 ed.514 in Nelson Textbook of th Pediatrics. purpuric rash of the hands and ankles.) Anti-neutrophil cytoplasmic antibody Kidney biopsy Question .514 in Nelson Textbook of th Pediatrics. she is seen to be a tired-appearing adolescent in no acute distress. and chest pain with deep inspiration. A 3-yr-old boy presents to an urgent care clinic with a 3-day history of abdominal pain and difficulty walking. and diffuse periarticular tenderness and swelling of the ankles. bilateral knee pain. The most appropriate initial treatment for the patient described in Question 9 is: Oral chorambucil Oral prednisone Explanation: (See Chapters 500. Cardiac examination findings are normal. she has developed low-grade fevers with temperatures to 100oF. A 14-yr-old girl presents to your clinic with a 1-mo history of fatigue. Which of the following laboratory studies is the most appropriate next step in confirming this patient's diagnosis? Sedimentation rate Rheumatoid factor Antinuclear antibody Explanation: (See Chapters 500.Question . Blood pressure is 130/80 mm Hg. diffuse abdominal tenderness. 9. 11. 10. Abdominal auscultation/palpation reveals no abnormalities. The most likely diagnosis is: Systemic lupus erythematosus Kawasaki's disease . Abnormal findings include blood pressure of 120/80 mm Hg. Breath sounds are diminished over the right lung base. A urinalysis reveals 3+ hematuria and 3+ proteinuria. Over the past week. On examination. 514 in Nelson Textbook of Pediatrics. 17th ed.514 in Nelson Textbook of Pediatrics. 17th ed. On examination.) Stevens-Johnson syndrome Question . Blood pressure is 120/80 mm Hg. 13. the patient is pale and lethargic. The child's mother reports that the child had bloody diarrhea for 5 days that cleared one day prior to presenting to your office.) Question .Juvenile rheumatoid arthritis Henoch-Sch nlein purpura Explanation: (See Chapters 500. The most appropriate next step in diagnosis would be: Urinalysis X-ray examination of the abdomen Urine culture Complete blood cell count Explanation: (See Chapters 500.514 in Nelson Textbook of Pediatrics. 12. All of the following glomerular diseases often manifest with rapidly progressive glomerulonephritis except: Wegener's granulomatosis Systemic lupus erythematosus Membranoproliferative glomerulonephritis Goodpasture syndrome Focal segmental glomerulosclerosis Explanation: (See Chapters 500. She also notes acute onset of cola-colored urine. A 3-yr-old girl presents to your office with acute onset of lethargy and pallor. 17th ed.) Prothrombin time . A stool culture reveals E.0 mg/dL. A full-term male newborn is noted to have gross hematuria of acute onset associated with new bilateral abdominal flank masses at 24 hr of life. and 50 red blood cells per high-power field. pH of 6.514 in Nelson Textbook of Pediatrics.Question . abdominal pain. no protein.) Early institution of dialysis Aggressive nutrition Question .514 in Nelson Textbook of Pediatrics. A 3-yr-old girl develops bloody diarrhea and pallor of acute + onset. coli bacteria Explanation: (See Chapters 500. coli O157:H7. An 8-yr-old girl presents with dysuria. All of the following may be contributing factors in this clinical scenario except: Dehydration Perinatal asphyxia Hypertension Explanation: (See Chapters 500. All of the following are accepted treatments for this patient except: Antihypertensive pharmacotherapy to maintain blood pressure below the 90th percentile for age and height Fluid replacement at rate to cover insensible losses plus urine output Institution of antibiotic treatment against E. 15. 2+ hematuria. A 24-hr urine specimen reveals 6 mg/kg body weight of calcium. total CO2 18 mEq/L.0. A urinalysis reveals specific gravity of 1. 17th ed. 14. K 5.5 mEq/L. 16. Which of the following is an acceptable treatment for this patient's problem? Increased intake of sodium-containing fluids Dietary calcium restriction Single daily dose of hydrochlorothiazide . Cl 90 mg/dL. and creatinine 4.020. 17th ed.) Sepsis Maternal diabetes Question . BUN 100 mg/dL. Laboratory values include Na + 130 mg/dL. and intermittent pink urine. 17th ed.514 in Nelson Textbook of Pediatrics. Which of the following is correct regarding this patient's diagnosis? Hypertension is uncommon This condition is inherited in an autosomal recessive pattern Explanation: (See Chapters 500. 18.) This condition is associated with pancreatic cysts Cerebrovascular hemorrhage is often seen in these patients Respiratory distress is uncommon in the neonatal period Question .514 in Nelson Textbook of Pediatrics. A renal ultrasound study reveals enlarged hyperechoic kidneys bilaterally and an echogenic liver. What is the most appropriate next step in this patient's management? Evaluate the patient for kidney transplantation Check the blood pressure Explanation: (See Chapters 500. 17. 17th ed.514 in Nelson Textbook of Pediatrics.Explanation: (See Chapters 500. 17th ed. His mother is 40 yr of age and demonstrates similar findings on renal ultrasound examination. hepatomegaly.) Obtain a cystogram to evaluate for vesicoureteral reflux Start the patient on an antibiotic for prevention of urinary tract infection Obtain a renal ultrasound study of all siblings . A 15-yr-old boy is noted to have enlarged kidneys with macrocysts seen bilaterally on a renal ultrasound study after developing gross hematuria while playing hockey. A full-term newborn male is noted to have bilateral flank masses. and blood pressure of 120/80 mm Hg.) Vitamin D supplementation Vitamin C supplementation Question . ) Question . the glomerular filtration (GF) increases until kidney growth ceases toward the end of the second decade of life Serum creatinine level does not rise above normal until the GF rate falls by 30-40% Question . A 15-yr-old boy is seen in your office for dysuria and is noted to have 4+ hematuria and >100 red blood cells per high-power field on urinalysis.514 in Nelson Textbook of Pediatrics. 17th ed. All of the following statements are true except: In humans. 21. 20.) Fetal kidney function is not necessary for normal intrauterine homeostasis After birth. .Question . formation of nephrons is complete at birth but functional maturation continues during the first decade of life The plasma filtered through the glomerular capillary walls is cell free but contains all the substances in the plasma Explanation: (See Chapters 500. 17th ed.) Heme-positive urine without RBCs is due to either hemoglobin or myoglobin. All of the following organisms may cause infection leading to these symptoms except: Ureaplasma Chlamydia E.514 in Nelson Textbook of Pediatrics. All of the following statements are true except: Hematuria is defined as the presence of at least 5 red blood cells (RBCs) in the urine Urethrorrhagia refers to urethral bleeding in the presence of urine Explanation: (See Chapters 500. coli Adenovirus Enterovirus Explanation: (See Chapter 500-514 in Nelson Textbook of Pediatrics. 17th ed. 19. False-negative results on Chemstrip testing may be due to a urine preservative such as formalin Screening urinalysis should be performed at well child visits at 5 yr of age Question . 17th ed. 23. 22. All of the following statements are true except: IgA nephropathy commonly manifests with gross hematuria 1-2 days after the onset of an apparent viral upper respiratory tract infection Gross hematuria does not occur in patients with thin glomerular basement membrane disease Explanation: (See Chapters 500.) The acute phase of PSGN usually resolves in 6-8 wk Microscopic hematuria may persist for 1-2 yr following the initial presentation of PSGN The serum C3 level is usually reduced in the acute phase of PSGN The best single antibody titer to document cutaneous streptococcal infection is the deoxyribonuclease (DNase) B antigen .514 in Nelson Textbook of Pediatrics.) Gross hematuria occurs in Alport syndrome The primary treatment of IgA nephropathy is proper blood pressure control The presence of anterior lenticonus is pathognomonic for Alport syndrome Question . 17th ed.514 in Nelson Textbook of Pediatrics. All of the following statements regarding poststreptococcal glomerulonephritis (PSGN) are true except: PSGN is common in children 2 to 5 yr of age Explanation: (See Chapters 500. and acute renal failure Mortality from HUS is less than 10 Question . All of the following statements regarding autosomal recessive and autosomal dominant polycystic kidney disease (ARPKD and ADPKD) are true except: ARPKD typically presents in the 4th or 5th decade of life Explanation: (See Chapters 500.514 in Nelson Textbook of Pediatrics. 17th ed. thrombocytopenia. 17th ed. All of the following statements about hemolytic-uremic syndrome (HUS) are true except: HUS is the most common cause of acute renal failure in young children Verotoxin elaborated by Escherichia coli O157:H7 initiates endothelial cell injury in HUS HUS always presents after an enteritis with diarrhea Explanation: (See Chapters 500. Young infants are particularly susceptible to fluid. 24. 26.514 in Nelson Textbook of Pediatrics.) ADPKD is a systemic disorder affecting many organ systems The treatment of ARPKD and ADPKD is primarily supportive The presentation of ADPKD in older children has a favorable prognosis In about 85% of patients with ADPKD. electrolyte.Question . the trait maps to the PKD1 gene on the short arm of chromosome 16 Question .) The diagnosis of HUS requires microangiopathic hemolytic anemia. or acid-base abnormalities during acute illnesses. All of the following are characteristic of renal physiology in normal young infants except: Renal tubular immaturity Reduced glomerular filtration rate Decreased urinary concentrating ability Decreased nephron number Explanation: (See Chapter 520 in Nelson Textbook of . 25. ) Measure urine electrolytes. potassium 3. 17 ed. left flank pain. dehydrated adolescent. chloride 116 mmol/L. A 4-mo-old boy is noted to have poor growth at a routine well child visit. Physical examination reveals an illappearing. which is the most appropriate next step in the patient's management? Perform renal ultrasonography Perform ammonium chloride loading test Repeat serum electrolyte determination via venipuncture Explanation: (See Chapter 521 in Nelson Textbook of th Pediatrics. Left costovertebral angle tenderness is present.) Diminished responsiveness to antidiuretic hormone (ADH) Question . 27.) Renal Fanconi syndrome Question . Pulse rate is 110/min. 29. Results of laboratory studies include serum sodium 140 mmol/L. and vomiting.5 mmol/L. with calculation of urine anion gap Initiate treatment with oral bicarbonate supplementation Question . 17th ed. temperature is 40°C. and bicarbonate 13 mmol/L. A 15-yr-old-girl is admitted with a 5-day history of worsening fever.7 mmol/L. All of the following should be considered in the differential diagnosis except: Distal renal tubular acidosis Chronic diarrhea Proximal renal tubular acidosis Lactic acidosis Explanation: (See Chapter 521 in Nelson Textbook of Pediatrics. A 3-mo-old patient hospitalized for bronchiolitis is noted on admission laboratory studies to have a serum bicarbonate of 14 mmol/L. 17th ed. chloride 108 mmol/L.Pediatrics. 28. Serum sodium is 131 mmol/L. Of the following. and bicarbonate 15 mmol/L. The most likely diagnosis is: Addison's disease . potassium 6. Laboratory tests reveal a non-anion gap metabolic acidosis. Urinalysis shows a urine pH of 8. but by 9 mo of age.) Small echogenic kidneys bilaterally Enlarged kidneys .3 mg/dL. A patient develops renal Fanconi syndrome after receiving ifosfamide for treatment of Wilms tumor.) Rickets Polyuria Growth retardation Question . 31.Ingestion of high potassium-containing foods Distal renal tubular acidosis Acute pyelonephritis Explanation: (See Chapter 521 in Nelson Textbook of Pediatrics.) Congenital adrenal hyperplasia Question . Renal ultrasonography is most likely to show: Unilateral renal agenesis Polycystic kidneys Nephrocalcinosis Explanation: (See Chapter 521 in Nelson Textbook of Pediatrics. A 9-mo-old male infant is noted to have failure to thrive. 17th ed. with no abnormalities. Serum creatinine is 0. 30.0. A diagnosis of distal renal tubular acidosis is made. height and weight are at less than the 5th percentile. All of the following are features of this condition except: Phosphaturia Metabolic alkalosis Explanation: (See Chapter 521 in Nelson Textbook of Pediatrics. 17th ed. 17th ed.5 mmol/L. his height and weight were at the 25th percentile. At birth. with a serum potassium of 3. Vasopressin is administered. All of the following are potential causes of this patient's laboratory findings except: Pyloric stenosis Surreptitious loop diuretic administration . but no change in urine output or urine osmolarity is seen.000.Question . A 16-yr-old girl has a 3-yr history of bipolar disorder and seizures. Results of laboratory tests include serum sodium 137 mmol/L.) Diabetes mellitus Question . In the last several months she has noted new onset of polyuria and excessive thirst. despite a decrease in oral intake. 17th ed. 32.2 mmol/L. 33. Physical examination reveals a mildly to moderately dehydrated infant. Serum osmolarity is 340 mmol/kg. blood. The patient is given intravenous fluids. Her current medications include lithium and valproic acid. Findings on physical examination are unremarkable. Serum sodium is 170 mmol/L. The genetic defects that cause this congenital condition result in: Inability to respond to aldosterone Inability to produce antidiuretic hormone Overproduction of atrial natriuretic factor Tubular unresponsiveness to antidiuretic hormone Explanation: (See Chapter 522 in Nelson Textbook of Pediatrics. All of the following are potential causes of her recent symptoms except: Psychogenic polydipsia Lithium toxicity Central diabetes insipidus Congential nephrogenic diabetes insipidus Explanation: (See Chapter 522 in Nelson Textbook of th Pediatrics. or leukocytes. potassium 3. chloride 90 mmol/L. with no protein. Urine osmolarity is 240 mmol/kg. 34. A 1-wk-old full-term male infant presents with irritability and low-grade fever. His parents report that his urine output has been very high.) Abnormal regulation of osmoreceptors in the hypothalamus Question . A 1-mo-old infant presents with lethargy. Urinalysis reveals a specific gravity of 1. Physical examination reveals a moderately to severely dehydrated infant. Massive polyuria is noted. and bicarbonate 38 mmol/L. 17 ed. 36. and 15% eosinophils. trace blood. 17th ed.0 mg/dL.Chronic respiratory insufficiency Chronic diarrhea Explanation: (See Chapter 523 in Nelson Textbook of Pediatrics. blood pressure is 110/60 mm Hg. trace leukocytes. Urinalysis demonstrates specific gravity of 1.004. Her pulse is 90/min. 3-5 RBCs per high-power field. and no RBC casts. 5-10 RBCs per high-power field. and no protein. 5-10 WBCs per high-power field. Urinalysis shows specific gravity of 1.010. A 13-yr-old girl develops a sore throat and low-grade fever. small amount of blood. and malaise. Urine output is normal. with 3-5 WBCs per high-power field.) Minimal-change nephrotic syndrome Acute interstitial nephritis Chronic glomerulonephritis . 17th ed. Which of the following is the most likely diagnosis? Acute poststreptococcal glomerulonephritis Chronic interstitial nephritis Explanation: (See Chapter 524 in Nelson Textbook of Pediatrics.000 per mm with 60% neutrophils. and no RBC casts.) Minimal-change nephrotic syndrome Acute tubular necrosis Question . for which she is given oral penicillin.) Bartter syndrome Question . 35.to 3-wk history of nausea. The most likely diagnosis is: Acute poststreptococcal glomerulonephritis Toxic shock syndrome Acute interstitial nephritis Explanation: (See Chapter 524 in Nelson Textbook of Pediatrics. A throat culture is positive for group A streptococcal infection. Seven days later. 17th ed. Serum 3 creatinine is 2. she develops a rash and fever. C3 level is normal. Serum creatinine is 4. 25% lymphocytes. WBC count is 12. A 12-yr-old boy presents with a 1-yr history of worsening polyuria and a 2. fatigue. bicarbonate is 15 mg/dL.4 mg/dL. no protein. ) Salmonella is by far the most common cause of febrile gastroenteritis in early infancy. coli) A parenteral antibiotic (choice based on susceptibility of the Shiga toxin-producing E. and diarrhea that initially is watery but becomes grossly bloody. and/or renal failure Explanation: (See Chapter 510 in Nelson Textbook of Pediatrics. coli) Careful follow-up evaluation for development of thrombocytopenia. TMP/SMX is a poor choice for empirical management because of the frequency of resistant Shigella. has poor sensitivity. A 1-mo-old infant develops bloody diarrhea associated with fever. anemia. coli O157:H7. The most likely agent causing this illness is: Nontyphoidal Salmonella Explanation: (See Chapter 510 in Nelson Textbook of Pediatrics. vomiting. a fecal toxin electroimmunoassay suggests that a Shiga toxin is present. A stool culture on MacConkey sorbitol media suggests E. although the best clinically available tool to confirm shigellosis. 17th ed. as well as fever with temperatures to 105°F. A 3-yr-old boy presents with severe abdominal pain and vomiting of acute onset.Question . and grossly bloody stools. Question . A no-antibiotics approach is inappropriate because waiting for culture results will leave the child ill for many days and because stool culture. After stool cultures are obtained and oral rehydration is begun. . 39. coli O157 infection. 38. A 2-yr-old girl has an acute afebrile diarrheal syndrome characterized by abdominal pain. 17th ed. Loperamide may prolong illness. 37.) Both loperamide and antibiotic therapy appear to increase the risk of HUS in E. Appropriate care includes: Loperamide An oral antibiotic (choice based on susceptibility of the Shiga toxin-producing E. Empirical trimethoprim-sulfamethoxazole No antibiotics pending culture results Question . 17th ed.) Shigellosis is the most likely treatable etiology for this acute dysenteric syndrome. management should include: Loperamide Empirical ceftriaxone administered parenterally Explanation: (See Chapter 510 in Nelson Textbook of Pediatrics. microangiopathic hemolytic anemia with renal failure.) S. Yersinia spp. 40.014. Yersinia is a rare cause. The same process. 42. Any species or serotype of Shigella Question . marina is usually acquired by contact with an iguana. coli. 41. ser. Shigella spp. 17 ed. Rotavirus Question . Urine dipstick testing shows specific gravity 1. Rotavirus rarely (or never) causes bloody diarrhea. Hemolytic-uremic syndrome is typically a complication of infection with E. Campylobacter spp. Shigellosis is a disease that is rare in infancy but common in the 1. California The child has been exposed to raw seafood None of the above?the isolation of this serotype has no special epidemiologic significance Question .to 3-yr-old child. The child lives near a marina The child lives near Marina. 17th ed. coli O157:H7 or other Shiga toxin-producing strain of E. Isolation of S. can also follow infection with: Salmonella typhi or Campylobacter jejuni Shigella fIexneri Shigella sonnei Shigella dysenteriae serotype 1 Explanation: (See Chapter 510 in Nelson Textbook of Pediatrics. Microscopic examination of the urine is . A 14-yr-old healthy girl has a urinalysis as part of her well child visit. marina from an infant suggests: The child is in contact with an iguana Explanation: (See Chapter 510 in Nelson Textbook of th Pediatrics.) Only Shigella dysenteriae serotype 1 commonly produces Shiga toxin and causes HUS. and 2+ proteinuria and is negative for blood.Campylobacter is second in frequency to Salmonella as a bacterial cause of enteritis in infancy. ser.0. pH 6. 43. The differential diagnosis includes all of the following except: Protein-losing enteropathy Acute glomerulonephritis Nephrotic syndrome Hepatic failure Orthostatic proteinuria Explanation: (See Chapters 515-519 in Nelson Textbook of Pediatrics. 44. The most likely cause of the patient's proteinuria is: Transient proteinuria Explanation: (See Chapters 515-519 in Nelson Textbook of Pediatrics.) Measure the urine protein to creatinine ratio Question . The most appropriate next step in diagnosis is to: Collect a 24-hr urine specimen for measurement of protein and creatinine Draw blood for a serum chemistry panel Measure serum complement levels (C3. 45.) Question . Urine dipstick testing of a specimen obtained from a febrile 4yr-old child with acute viral gastroenteritis shows specific gravity 1.0. 17th ed. and no blood cells. 2+ proteinuria. C4) Perform urine dipstick testing on a first morning voided sample Explanation: (See Chapters 515-519 in Nelson Textbook of Pediatrics.) Nephrotic syndrome Orthostatic proteinuria Acute glomerulonephritis Chronic glomerulonephritis Question . 17th ed. A 3-yr-old boy presents with periorbital and pedal edema.unrevealing. The microscopic analysis shows 0-2 .030. pH 5. 17th ed. An asymptomatic 16-yr-old African-American girl with hypertension is found to have 3+ proteinuria by dipstick testing on mid-day and first morning voided urine samples. and a first morning urinalysis reveals specific gravity 1.5. 17th ed. A 12-yr-old girl with a history of vesicoureteral reflux and recurrent pyelonephritis is seen for routine follow-up examination. and 2+ proteinuria. 17th ed. Her blood pressure is 140/90 mm Hg. Which of the following statements is appropriate regarding the patient's proteinuria? The proteinuria is the consequence of reflux nephropathy Explanation: (See Chapters 515-519 in Nelson Textbook of Pediatrics. and no blood.) 24-hr urine collection for measurement of protein and creatinine Serum albumin level Question . 17th ed.) Lupus nephritis Question . 46. The most likely diagnosis is: Postinfectious glomerulonephritis Diabetic nephropathy Minimal-change disease Focal segmental glomerulosclerosis Explanation: (See Chapters 515-519 in Nelson Textbook of Pediatrics.) A 24-hr urine collection would probably show >1000 mg of protein The proteinuria is likely to be transient in nature The major protein in this patient's urine is albumin The primary renal pathology is in the glomerular compartment . Initial evaluation of a child with fixed proteinuria should include all of the following except: Serum creatinine level Complement level (C3) Renal biopsy Explanation: (See Chapters 515-519 in Nelson Textbook of Pediatrics. 47.red blood cells per high-power field.015. pH 6. Urine dipstick testing reveals 4+ proteinuria and is negative for blood. The urinalysis shows 4+ proteinuria and is negative for blood. 17th ed.Question .0 Question .2 mg/dL and serum creatinine is 0. 17th ed. A 2-yr-old boy presents with a 2-wk history of gradually increasing periorbital and pedal edema. A 3-yr-old girl presents to the emergency department with anasarca.5 mg/dL Serum cholesterol 130 mg/Dl Explanation: (See Chapters 515-519 in Nelson Textbook of Pediatrics. Diagnostic studies are likely to show all of the following except: Serum albumin 1. 48. The most likely diagnosis is: Postinfectious glomerulonephritis Minimal-change disease Explanation: (See Chapters 515-519 in Nelson Textbook of Pediatrics.5 mg/dL Complement C3 100 mg/dL Urine protein to creatinine ratio 4.4 mg/dL. Complications that may occur in this patient include all of the following except: Deep vein thrombosis Spontaneous bacterial peritonitis Weight gain Behavioral change Paresthesias Explanation: (See Chapters 515-519 in Nelson Textbook of . Serum albumin is 1.) Serum creatinine 0.) IgA nephropathy Lupus nephritis Focal segmental glomerulosclerosis Question . A 13-yr-old boy with newly diagnosed nephrotic syndrome is seen in an outpatient clinic. 50. 49. All of the following are appropriate treatment recommendations except: Low-sodium diet High-dose prednisone therapy Adherence to schedule for all routine childhood immunizations Explanation: (See Chapters 515-519 in Nelson Textbook of Pediatrics. 17th ed. The most appropriate next step in therapy is to: Discontinue prednisone and monitor closely for relapse Continue high-dose prednisone therapy for a total of 4-6 wk Explanation: (See Chapters 515-519 in Nelson Textbook of Pediatrics. 17th ed. 17th ed.) Hypertension Pleural effusions Fever Question .) Daily monitoring of urinary protein by parents Participation in school and physical activities as tolerated .) Change to alternate-day prednisone dosing for the next 2 mo Add cyclophosphamide therapy Add cyclosporine therapy Question . 52. A 5-yr-old boy presents with new-onset nephrotic syndrome. 51. 53. An 8-yr-old boy with newly diagnosed nephrotic syndrome enters remission 7 days after beginning treatment with high-dose prednisone therapy (30 mg PO bid). Which of the following clinical findings is most likely? Gross hematuria Periorbital edema Explanation: (See Chapters 515-519 in Nelson Textbook of Pediatrics. A 4-yr-old girl is being treated for nephrotic syndrome relapse.Question . serum albumin 2.Question . 24hr urine protein excretion 5.) Elevated maternal serum -fetoprotein Massive proteinuria Question . A 10-yr-old girl presents with edema and gross hematuria.4 mg/dL. 17th ed. 17th ed. A newborn infant develops nephrotic syndrome within the first 2 wk of life. 17th ed. 54.3 mg/dL.3 mg/dL and serum albumin is 1. Which of the following is the most likely cause of this patient's nephrotic syndrome? Congenital toxoplasmosis Abnormality in the nephrin gene Explanation: (See Chapters 515-519 in Nelson Textbook of Pediatrics.0 mg/dL. 56. A newborn infant develops anasarca and poor urine output during the first week of life.) Congenital syphilis Abnormality in the polycystin gene Maternal exposure to ACE inhibitors . The most likely cause of her nephrotic syndrome is: Membranoproliferative glomerulonephritis Explanation: (See Chapters 515-519 in Nelson Textbook of Pediatrics.5 g. and C3 12 mg/dL. 55. Which of the following clinical findings is least likely? Enlarged placenta Prematurity Rapid response to steroid therapy Explanation: (See Chapters 515-519 in Nelson Textbook of Pediatrics. Serum creatinine is 0.) Minimal-change disease IgA nephropathy Membranous nephropathy Focal segmental glomerulosclerosis Question . Her evaluation reveals serum creatinine 1. ) Obstructive uropathy Acute glomerulonephritis Prerenal azotemia Question . 17th ed. 59. A newborn boy is delivered by emergency cesarean section for severe fetal distress and placental abruption. The most likely cause of the infant's acute renal failure is: Renal dysplasia Cortical necrosis Explanation: (See Chapters 515-519 in Nelson Textbook of Pediatrics. she can be expected to regain normal renal function within the next 3 mo She has sustained irreversible renal injury from cortical necrosis Explanation: (See Chapters 515-519 in Nelson Textbook of th Pediatrics.) Avoid nonsteroidal anti-inflammatory agents Question . She remains anuric and dialysisdependent for 3 mo. Follow-up renal ultrasonography reveals a significant decrease in renal size with increased renal echogenicity.Question . An 18-mo-old toddler develops severe hemolytic-uremic syndrome from E. All of the following are appropriate recommendations to prevent nephrotoxic injury except: Substitute ultrasonography or MRI for contrast-enhanced CT scanning Substitute cephalosporin for aminoglycoside therapy Adjust medication doses and intervals according to renal function Administer continuous infusion of renal-dose dopamine Explanation: (See Chapters 515-519 in Nelson Textbook of Pediatrics. 57. Resuscitation is required in the delivery room.) . 58. The infant develops gross hematuria and oliguric acute renal failure with a peak serum creatinine level of 8. coli O157:H7 colitis. The most appropriate statement about this patient's condition is: She probably had pre-existing chronic renal insufficiency With continued dialysis support. 17th ed. 17 ed. A 16-yr-old girl is admitted to the intensive care unit with sepsis following bone marrow transplantation.0 mg/dL at 8 days of age. 5 mg/dL in 2 days. 61. 60. The most likely diagnosis is: Acute tubular necrosis Hemolytic-uremic syndrome Rhabdomyolysis syndrome Prerenal acute renal failure Explanation: (See Chapters 515-519 in Nelson Textbook of Pediatrics. 17th ed. Initial laboratory work reveals BUN of 80 mg/dL and serum creatinine of 2.5 mg/dL. A 3-yr-old girl with congenital heart disease develops acute renal failure during a prolonged stay in an intensive care unit.) Acute glomerulonephritis Question . With rehydration. 17th ed. coli cystitis Explanation: (See Chapters 515-519 in Nelson Textbook of Pediatrics. A 2-year-old boy is admitted to the intensive care unit with severe dehydration from bacterial gastroenteritis. his BUN and creatinine fall to 20 mg/dL and 0. Possible contributory factors include all of the following except: E.) Nephrotoxic antibiotics Hypotensive episodes Contrast agent used for cardiac catheterization Congestive heart failure .Chronic dialysis is the preferred long-term treatment option This is a common clinical course for children with hemolytic-uremic syndrome Question . and decreased urine output. dry mucous membranes.Question .) Acute tubular necrosis Eagle-Barrett syndrome Chronic glomerulonephritis Question . Results of initial laboratory studies include BUN 65 mg/dL.) Hyperkalemia Hyponatremia Hyperphosphatemia Hypoalbuminemia .) Question . potassium 5. Laboratory findings may include all of the following except: Hypercalcemia Explanation: (See Chapters 515-519 in Nelson Textbook of th Pediatrics. The most appropriate next step in treatment is to: Order a renal ultrasound study Administer Kayexalate 1 g/kg per rectum Run intravenous replacement fluid at rate to replace insensible losses plus urine output Give NaHCO3 1 mEq/kg IV to correct acidosis Administer normal saline 20 mL/kg bolus over 30 min Explanation: (See Chapters 515-519 in Nelson Textbook of th Pediatrics. A previously healthy 6-yr-old child presents to the emergency department with a 3-day history of malaise. A 14-yr-old girl develops acute renal failure related to HenochSch nlein purpura glomerulonephritis. A full-term newborn infant boy has poor urine output and demonstrates a rising serum creatinine over the first 4 days of life. Physical examination reveals tachycardia. The most likely cause of his acute renal failure is: Prerenal acute renal failure Obstructive uropathy Explanation: (See Chapters 515-519 in Nelson Textbook of Pediatrics. and sunken eyes. diarrhea.5 mEq/L. Physical examination reveals an enlarged bladder and no congenital anomalies.0 mg/dL. bicarbonate 14 mEq/L. poor oral intake. and serum creatinine 3. 17 ed. 17th ed. 64. 17 ed. 63. 62. Question . 17 ed. A 12-yr-old boy presents with a long-standing history of polyuria and polydipsia. 67. A 5-yr-old boy with Eagle-Barrett syndrome and renal dysplasia has a rising serum creatinine of 4. 66. ESRD in North American children is treated with hemodialysis . All of the following strategies may theoretically help to slow the progression of chronic renal failure except: Careful control of systemic hypertension Reduction of proteinuria using ACE inhibitor therapy Administration of epidermal growth factor Explanation: (See Chapters 515-519 in Nelson Textbook of Pediatrics. 17th ed. 17th ed.) Small. Initial laboratory evaluation reveals BUN of 125 mg/dL and serum creatinine of 8. decreased appetite. progressive fatigue.9 mg/dL and a 24-hr urinary protein excretion of 1800 mg. A 16-yr-old boy with focal segmental glomerulosclerosis has a serum creatinine of 1. 65.5 mg/dL. In addition.7 mg/dL. morning nausea and emesis. he has had no response to a 6-mo course of iron therapy for treatment of anemia. Other expected laboratory features include all of the following except: Elevated parathyroid hormone level Increased anion gap metabolic acidosis Decreased levels of growth hormone Explanation: (See Chapters 515-519 in Nelson Textbook of th Pediatrics. echogenic kidneys on ultrasonography Hypocalcemia Question . The most appropriate statement regarding patient management is: At least a short period of dialysis is necessary before the possibility of renal transplantation is considered Transplantation offers the best opportunity for full rehabilitation for children with ESRD Explanation: (See Chapters 515-519 in Nelson Textbook of Pediatrics.) Normalization of serum calcium/phosphorus balance Treatment of metabolic acidosis with NaHCO3 Question . and impaired growth velocity.) In most cases. weight loss. and pruritus) . decline in school performance. he may follow an unrestricted diet Dialysis or transplantation should be considered only when he becomes symptomatic from ESRD (fatigue. anorexia.With his current degree of renal dysfunction. ) Question . the Potter phenotype (flattened face. receding chin. 17th ed. 17th ed. (See Chapter 530 in Nelson Textbook of Pediatrics.) Renal ultrasonography .Urologic Disorders in Infants and Children Question . 2. broad nose.) Spina bifida Question . Potter phenotype may be due to: Renal agenesis Renal dysplasia Obstructive uropathy Severe amniotic fluid leak None of the above All of the above Explanation: Primarily encountered in renal agenesis or dysplasia. Risk factors for urinary tract infections include all of the following except: Uncircumcised penis Sexual activity Reflux nephropathy Double-ureter systems Chronic use of antibiotics Explanation: Chronic use of antibiotics is not a risk factor for urinary tract infection. 1. 3. The presence of renal parenchymal scarring due to vesicoureteral reflux is best determined by: DMSA scan Explanation: A DMSA radionuclide scan helps to define images that are accurate representations of chronic renal scarring. 17th ed. clubfoot) is also due to severe oligohydramnios. (See Chapter 529 in Nelson Textbook of Pediatrics. low-set ears. All the other choices are significant risks. (See Chapter 530 in Nelson Textbook of Pediatrics. Hydronephrosis and multicystic-dysplastic lesions are the most common renal masses.) Wilms tumor Neuroblastoma Meckel diverticulum Ovarian teratoma Question . resulting in oligohydramnios. resulting in the abnormal physical features (C. 5.000 . The reduced amniotic fluid produces fetal constraint and compression. The most common abdominal mass in a neonate is: Renal dysplasia-hydronephrosis Explanation: Renal masses are the most common lesions in neonates with an abdominal mass. E). 17th ed.) Pulmonary hypoplasia Facial deformation Skeletal dysplasia Question . (See Chapter 529 in Nelson Textbook of Pediatrics. The primary pathology in classic Potter syndrome is best characterized as: Oligohydramnios Renal agenesis Explanation: Renal agenesis results in no fetal urine output. (See Chapter 529 in Nelson Textbook of Pediatrics. D.VCUG CT scan Intravenous pyelography Question . 6. Multicystic dysplastic kidneys are characterized by all of the following except: Usually unilateral Incidence of 1:2. 17th ed. 4. Nephrectomy is the treatment of choice. polycystic kidneys are bilateral. or Ask-Upmark kidney. 17th ed. In contrast. usually beginning at age 10 yr when identified on routine examination. The most appropriate next step in evaluation is to: Measure serum complement Perform a renal ultrasound examination Explanation: Renal ultrasonography demonstrates a small left kidney with a deep groove in the lateral convexity of the kidney. There is no history of recent infection.) Renal infarction Neurofibromatosis . (See Chapter 529 in Nelson Textbook of Pediatrics. 17th ed. 8. The Doppler scan of blood flow in the renal arteries appears normal. A 12-yr-old presents with headaches and a blood pressure of 210/110 mm Hg. inheritance is either autosomal dominant (adult) or autosomal recessive (child). (See Chapter 529 in Nelson Textbook of Pediatrics. 17th ed. Results of urinalysis are unremarkable.) Most common neonatal abdominal mass No function Question . (See Chapter 529 in Nelson Textbook of Pediatrics.Autosomal dominant inheritance Explanation: Multicystic dysplastic kidneys are not inherited. produces severe hypertension. 7. The most likely diagnosis for the patient described in Question 7 is: Polycystic renal disease Multicystic renal disease Segmental hypoplasia Explanation: Segmental hypoplasia.) Perform a voiding cystourethrogram Measure streptococcal titers Perform a renal arteriogram Question . . (See Chapter 530 in Nelson Textbook of Pediatrics. coli and other gram-negative enteric pathogens (e. (See Chapter 530 in Nelson Textbook of Pediatrics.g. (See Chapter 530 in Nelson Textbook of Pediatrics.) . 10. instrumentation. Klebsiella) are the most common pathogens for UTIs at all ages. Other risk factors include use of "bubble bath. Proteus.) Absence of renal scarring Sexual activity in females Question . wearing tight underwear. Risk factors for urinary tract infection include all of the following except: Pinworms Constipation Pregnancy Neurogenic bladder Henoch-Sch nlein purpura Explanation: HSP is not a risk factor for UTI. 17th ed.) Question . All of the following statements concerning the epidemiology of urinary tract infections in children are true except: Average age of affected females is 3 yr Average age of affected males is less than 1 yr Circumcision reduces the risk Bladder reflux increases the risk Staphylococcus saprophyticus is the most common pathogen in male infants Explanation: E." incorrect perineal hygiene practices (wiping from back to front). With fever. reflux. Cystitis is associated with all of the following except: Urgency Adenovirus Fever Explanation: Cystitis is not usually associated with fever. 11. chills. 17th ed. 17th ed.Question . suspect pyelonephritis with or without urosepsis. or rigors. and the potential pathogens listed for Question 9. 9. choose the imaging study or studies needed to evaluate a 9-wk-old infant with pyelonephritis. with 2 in selected cases 4. 5. the role for DMSA is less important). A DMSA scan also shows scarring and is valuable to follow the progression of scarring and the possible need for further intervention. 3. Vesicoureteral reflux is associated with all of the following except: Contralateral kidney of pair with unilateral dysplasia Ureteral duplication Familial inheritance Ureterocele Asymptomatic bacteriuria Explanation: All the rest are important risk factors for reflux. 13. Voiding cystourethrogram 6.Question . Reflux (primary) is uncommon in African-American children. 17th ed. Unfortunately. renal enlargement (pyelonephritis). approximately 35% of siblings of a child with reflux have reflux. and 6 3. Cystoscopy 2 only 1. obstruction.) Question . CT scan 3. 12. 1. A DMSA scan identifies acute pyelonephritis (if pyelonephritis is evident clinically. it misses many renal scars. Intravenous pyelogram 2.) . 50% of the children of a mother with reflux have reflux. From the following list. A VCUG is needed because vesicoureteral reflux is a common cause of UTI in children younger than 6 yr. Of note. 17th ed. (See Chapter 530 in Nelson Textbook of Pediatrics. or renal abscess. DMSA scan 4. Renal ultrasound examination 5. and possibly 3 Explanation: Renal ultrasonography demonstrates renal anomalies. (See Chapter 531 in Nelson Textbook of Pediatrics. In addition. The bladder is smaller than normal and exhibits strong uninhibited contractions. A 10-yr-old boy manifests testicular pain and swelling of acute onset that is not relieved by acetaminophen. she has urge incontinence. UTI. timed voiding and anticholinergic drugs. Treatment is with frequent.) Nephrolithiasis Question . Constipation and UTI may complicate the disorder. The parents of every boy should be questioned about the baby's urinary stream. (See Chapter 532 in Nelson Textbook of Pediatrics. poor stream. The next step in management is to: Apply ice Check for a history of Chlamydia infection Immediately refer to a urologist . 14. 17th ed. The most likely diagnosis is: Renal artery stenosis Renal hypoplasia Urogenic bladder Posterior urethral valves Explanation: Posterior urethral valves may be detected by in utero ultrasound examinations or after birth in children with a big bladder. A 6-yr-old girl has a long history of urinary frequency and urgency. A 7-mo-old white male infant presents with failure to thrive and a BUN of 75 mg/dL. 15.) Wilms tumor Constipation Chronic cystitis Nephrolithiasis Question . 17th ed. She also has nocturnal enuresis. 16. or failure to thrive. The most likely diagnosis is: Unstable bladder Explanation: This is a classic presentation of the pediatric unstable bladder. (See Chapter 535 in Nelson Textbook of Pediatrics.Question . He has a history of a poor urinary stream. Explanation: This child has testicular torsion until proven otherwise. If the torsion is not relieved. (See Chapter 537 in Nelson Textbook of Pediatrics. testicular infarction may result in a necrotic organ in a sexually active male. 17th ed.) Perform laparoscopy Apply a scrotal support device . Gonorrhea and Chlamydia epididymitis must also be considered. Which of the following is useful in the diagnosis of vaginitis in the pediatric patient? Litmus paper Leukocyte esterase test Explanation: The leukocyte esterase test will identify the presence of inflammatory white cells in the discharge. 17th ed. If this is not satisfactory. 3. 17th ed. 1. Clitoromegaly is defined as a clitoral diameter of: 2 mm 3 mm 5 mm 10 mm Explanation: Clitoromegaly is defined by width. and any palpable gonads. the knee-chest position is employed. (See Chapter 540 in Nelson Textbook of Pediatrics. and in an adolescent a clitoris is wider than 10 mm is considered enlarged. 17th ed. (See Chapter 540 in Nelson Textbook of Pediatrics. (See Chapter 541 in Nelson Textbook of Pediatrics.) Question . Which of the following is the ideal patient position for examination of the prepubertal child with a vaginal discharge? Lithotomy Fowler Frog-leg Explanation: The frog-leg position is the least threatening position and provides the best approach to visualize the perineum. acne. It is a good screening test.Gynecologic Problems of Childhood Question . baldness. 2.) Urinalysis Scotch tape test . Other signs of virilization should be looked for such as facial hair.) Prone Question . (See Chapter 541 in Nelson Textbook of Pediatrics. That due to group A -hemolytic streptococci is particularly serious. If it causes dyspareunia. and discharge. 6. Malassezia furfur rash is caused by: Group B streptococci Pityrosporum orbiculare Explanation: Malassezia furfur is the new name for Pityrosporum orbiculare and causes a rash characterized by scaly macules on the trunk. and the specimen most often contains coliform organisms.) Veillonella parvula Propionibacterium . or genital region. (See Chapter 545 in Nelson Textbook of Pediatrics. Gartner's duct cyst is a remnant of: The wolffian duct Explanation: Also called a mesonephric cyst.) The müllerian duct The urogenital sinus A hydatid cyst of Morgagni Question . 17th ed. accompanied by fever. face. intense pain. it may require resection. 5. this remnant is usually asymptomatic. Vulvovaginitis is most often associated with: -Streptococci Enterococci Coliform bacteria Explanation: Nonspecific vulvovaginitis in prepubertal females is often due to enteric flora.Question . 17th ed. 17th ed. (See Chapter 541 in Nelson Textbook of Pediatrics. 4.) Pseudomonas Coagulase-positive staphylococci Question . ) Metformin . 8. 17th ed. The agent most commonly associated with folliculitis is: Staphylococcus aureus Explanation: Infection of the hair follicles is most often due to S.Question . 9. aureus.) Escherichia coli Streptococcus pyogenes Borrelia Question . (See Chapter 541 in Nelson Textbook of Pediatrics. 17th ed. (See Chapter 151 in Nelson Textbook of Pediatrics. (See Chapter 151 in Nelson th Textbook of Pediatrics. infections with Pseudomonas aeruginosa must also be considered. Behçet disease is treated with: Acyclovir Topical estrogen cream Colchicine Explanation: This inflammatory multisystem disorder responds to oral colchicine.) Question . 7. 17 ed. In patients who have recently bathed in hot tubs. Behçet disease is characterized by: Recurrent oral ulcers Recurrent genital ulcers Uveitis All of the above Explanation: Behçet disease may produce painful genital ulceration in association with other systemic manifestations such as uveitis and intestinal ulceration. Survival approaches 80%. (See Chapter 545 in Nelson Textbook of Pediatrics. 11. The most common germ cell tumor of the ovary is: Seminoma Dysgerminoma Explanation: Dysgerminomas are the most common malignant germ cell tumor of the ovary. 12. Dysgerminoma is most often associated with: XY gonadal dysgenesis Explanation: Most dysgerminomas are associated with XY gonadal dysgenesis. they are useful in diagnosis and identification of relapse.) . (See Chapter 545 in Nelson Textbook of th Pediatrics. Because active ovarian cancers may express these embryonic markers. 17 ed.) Endodermal sinus tumor Gonadoblastoma Question . Ovarian malignancies in pediatric patients are evaluated with: -Fetoprotein (AFP) Carcinoembryonic antigen (CEA) CA-125 All of the above Explanation: Various embryonal probes are available in the diagnosis and follow-up of ovarian cancers in children. 10.) RNA-based diagnosis to confirm etiology Bilaterality D Female genotype Question . 17th ed. (See Chapter 545 in Nelson Textbook of Pediatrics. Y-DNA chromosome probes help with this diagnosis. 17th ed. Benign teratomas are more common.Question . ) Cardiovascular Question .) . 14. 15. Endodermal sinus tumor produces a predominance of CA-125 Carcinoembryonic antigen -Fetoprotein Explanation: -Fetoprotein is a useful marker in the management and follow-up of patients with an endodermal sinus tumor. (See Chapter 545 in Nelson Textbook of Pediatrics.) Human chorionic gonadotropin Question . and a mass. (See Chapter 546 in Nelson Textbook of Pediatrics. 17th ed. 13.Question . There is an increased risk for endometriosis. 17th ed. cyclic abdominal pain. Vertical fusion defects can result in: Septate uterus Didelphic uterus Bicornuate uterus Transverse vaginal septum Explanation: Patients with a transverse vaginal septum often present with amenorrhea. The most common system associated with a müllerian anomaly is: Skeletal Gastrointestinal Urinary Explanation: Disorders affecting the embryonic urogenital sinus are common in children with müllerian origin anomalies. (See Chapter 546 in Nelson Textbook of Pediatrics. Spina bifida may also occur with müllerian defects. 17th ed. The most likely diagnosis is: Nonspecific vaginitis Explanation: Nonspecific vaginitis most often occurs in prepubertal girls who wear tight-fitting clothing (leotards) or are exposed to vaginal irritants (soaps) or have poor hygiene. A 7-yr-old girl complains of a brown-green discharge on her underwear. The prolactin level of the young woman described in Question 16 is 1. (See Chapter 543 in Nelson Textbook of Pediatrics. 18. 17. Menarche was at age 12 yr. 16. (See Chapter 543 in Nelson Textbook of Pediatrics. The most useful screening test is: Urine pregnancy test Serum pregnancy test Serum prolactin level Explanation: Patients with spontaneous galactorrhea should be screened by determining their prolactin level. A 17-yr-old Tanner stage 2 girl presents with a history of bilateral spontaneous milky discharge from her breasts for 2 mo. In addition. Hypothyroidism also produces hyperprolactinemia and galactorrhea. Her mother states that for the past 4 mo her daughter has been taking ballet classes and frequently sleeps in her leotards. Larger or persistently symptomatic lesions require surgery.) Abdominal CT study Pelvic ultrasonography Uterine biopsy Mammography Question . 17th ed. The treatment of some small prolactinomas includes bromocriptine (Parlodel). and her periods had been regular until 4 mo before this visit to your office. She has no fever or labial tenderness and denies sexual contact. It is often due to coliform bacteria or group A . 17th ed.Question . The next test in her evaluation should be: Cranial MRI study Explanation: Magnetic resonance imaging (MRI) or computed tomography (CT) of the cranium demonstrates a pituitary prolactinoma.000 times higher than normal.) Serum estrogen level Serum luteinizing hormone level Question . she complains of headache on awakening for the past 2 wk. amoxicillin may be of value in addition to the suggestions in choices A-D.) Topical progesterone . (See Chapter 541 in Nelson Textbook of Pediatrics. If the process is recurrent. (See Chapter 554 in Nelson Textbook of Pediatrics.) Question . 17th ed. 19. Thorough cleansing followed by application of petroleum ointment for 1-2 mo helps prevent recurrences. (See Chapter 541 in Nelson Textbook of Pediatrics. 20. Initial therapy for the girl described in Question 18 should include all of the following except: Instruction in perineal hygiene Sitz baths Use of mild soaps Avoiding tight clothing Metronidazole Explanation: Metronidazole is not indicated for this form of nonspecific vaginitis. 17th ed. 17th ed.) Gardnerella vaginalis vaginitis Gonorrhea Chlamydial vaginitis Candida vaginitis Question .streptococci. The treatment of choice for labial adhesions is: Topical erythromycin Oral erythromycin Oral estrogens Topical estrogens Explanation: Topical estrogen cream each evening for 1 wk is effective in over 90% of cases. 17th ed. Mastodynia is best characterized as: Dependent on breast size Beginning 18 mo after menarche Explanation: Breast pain occurs in cyclic patterns at the time of each menstrual cycle. (See Chapter 545 in Nelson Textbook of Pediatrics. (See Chapter 543 in Nelson Textbook of Pediatrics.) Cervical prolapse Sarcoma botryoides Melanoma .) Noncyclic in nature Unrelated to the menstrual cycle Unresponsive to nonsteroidal anti-inflammatory agents Question . 21. 17th ed.Question . 22. Maternal exposure to DES places the female offspring at increased risk for: Ovarian cancer Clear cell adenocarcinoma of the vagina Explanation: DES is an in utero carcinogen. (See Chapter 585 in Nelson Textbook of Pediatrics.) The risk decreases with higher spine lesions The risk is unknown at birth The risk is less than 25% The risk is related to the cause of the malformation . Which of the following best describes the risk of hydrocephalus in this situation? The risk increases with higher spine lesions Explanation: Hydrocephalus is quite common in children with a myelomeningocele and must be evaluated with a head ultrasound study or CT. 17th ed. A newborn with myelomeningocele is being evaluated. administration of folic acid needs to begin: By 3 months of gestation At the first missed period Before conception Explanation: Folic acid supplementation has been a major public health success in reducing the incidence of neural tube defects in the United States. 1. To be effective for preventing myelomeningocele. (See Chapter 585 in Nelson Textbook of Pediatrics. Supplementation also reduces the recurrence rate of neural tube defects in families with a previously affected child. 17th ed. 2.) By 30 days of gestation At 3 months prior to delivery Question . and the parents ask about the likelihood of the development of hydrocephalus.CNS Question . ) Order a CT scan of the brain Order an MRI study of the brain Order psychometric testing Question . and findings on examination an hour after the seizure are completely normal. 17th ed. The most appropriate management is: Begin therapy with carbamazepine Order an EEG Explanation: An EEG will help define the presence of a seizure focus and if there is a specific seizure type. these are often unpredictable. There is no history of illness or fever. which lasted 10 min. 17th ed. A 15-mo-old girl is evaluated for a 10-min-long generalized seizure associated with a temperature of 40oC. Which of the following factors in the history is most likely to increase the risk of future seizures? APGAR score of 3 at 5 min Family history of epilepsy Explanation: Future nonfebrile seizures can be predicted when there is a family history of seizures. Up to 30% of children with febrile seizures have a recurrence when they become febrile. A normal EEG may help the pediatric neurologist determine if anticonvulsant therapy is needed. A 4-yr-old boy is evaluated for his first generalized tonic-clonic seizure. (See Chapter 586 in Nelson Textbook of Pediatrics.) Clinical evidence of roseola Female gender Presence of 2 café-au-lait spots . 4. 3.Question . (See Chapter 586 in Nelson Textbook of Pediatrics. On examination. with rapid recovery of consciousness once the affected person is in a horizontal position. 5. The chances of severe life-threatening skin reactions such as Stevens-Johnson syndrome are greatest with which of the following anticonvulsant medications? Phenobarbital Gabapentin Carbamazepine Lamotrigine Explanation: In addition.Question . 7. Down syndrome Family history of epilepsy Allergic reactions to phenobarbital Question . It has been determined that sodium valproate is the best anticonvulsant medication for a 12-yr-old boy with epilepsy. thus confusing the assessment. mucous membranes. phenobarbital and carbamazepine are also associated with an acute drug hypersensitivity syndrome involving skin. 6. and the liver. Sodium valproate Question . which of the following factors is most helpful in distinguishing whether this episode was a seizure or syncope? Family history Temperature Blood pressure Level of consciousness Explanation: Syncope usually produces a loss of tone and a fall. Occasionally there may be a few tonic-clonic jerking movements with syncope. It is much safer in 12-yr-old children than those younger than 2 yr. His parents ask about the possibility of fatal liver necrosis if he takes the drug. . Which of the following factors markedly increases the risk of this complication? Male gender Age younger than 2 yr Explanation: Young age is a risk factor for hepatic and perhaps pancreatic complications of valproic acid. he was brought to the emergency department an hour later. A 3-yr-old boy is being evaluated after an episode at home during which he lost consciousness for 5 min. A 5-yr-old child is referred with a 6-wk history of morning headaches. and is an ominous sign. confined to the right optic nerve. 8. Findings on the neurologic. A 6-yr-old child with neurofibromatosis (NF1) is found to have an optic glioma on a routine MRI study.Size of pupils Question . 9. 17th ed. and retinal examinations are normal.) Radiation therapy limited to the right optic nerve . (See Chapter 589 in Nelson Textbook of Pediatrics. often associated with vomiting. must be looked for in all patients with headaches. Papilledema takes time to develop. The visual acuity is 20/20 bilaterally. (See Chapter 588 in Nelson Textbook of Pediatrics. His parents have noted that during this period he has become irritable and moody. In many situations the tumor is slow growing and produces few problems. physical. Which of the following would be most likely to be identified during the physical examination? Marked elevation of blood pressure Tenderness on percussion of frontal sinuses Papilledema Explanation: This child has a brain tumor and increased intracranial pressure. uncorrected. 17th ed.) Loud orbital bruit Significant refractive error Question . Which of the following is the correct management? Surgical removal of the tumor MRI of the optic nerve every 3 mo Chemotherapy Annual examination by a pediatric ophthalmologist Explanation: Patients with NF1 and an optic glioma as in this case can be observed over time. it is a very strong predictor of CP. She has been healthy and has not taken any medications. A 10-yr-old girl is being evaluated for new onset of school problems. 10.Question . 11. Gabapentin Lithium Question . Examination during the afternoon shows that she has rigidity in the leg muscles and dystonic twisting of her feet. A 2-yr-old boy with the spastic diplegia form of cerebral palsy is being evaluated. Normal anatomy . MRI of his brain is most likely to show: Multicystic encephalomalacia Periventricular leukomalacia Explanation: PVL is a common observation in children with CP. L-Dopa Explanation: L-Dopa is the treatment of choice for hereditary progressive dystonia with marked diurnal variation. her walking is much better after a good night's sleep. also called Segawa disease. 12. It is first observed in the neonatal period in both term and preterm infants. This kind of movement disorder often responds dramatically to which of the following medications? Sodium valproate Lorazepam C. obsessive-compulsive behavior. Hydrocephalus Tardive dyskinesia Question . which seems to worsen during the day. If identified in the neonatal period and extensive. and occasional uncontrolled movements of the hands. Further evaluation is most likely to reveal: Brain tumor in the posterior fossa Partial complex epilepsy Evidence of streptococcal infection Explanation: Poststreptococcal obsessive-compulsive disorder is a well-recognized clinical entity. A 5-yr-old girl is evaluated for progressive difficulty in walking. This is one of the dopa-responsive dystonias. 17th ed. (See Chapter 591 in Nelson Textbook of Pediatrics. (See Chapter 592 in Nelson Textbook of Pediatrics.Basal ganglia abnormalities Agenesis of the corpus callosum Question . bone marrow transplantation has had some value in improving the clinical course in some patients with X-linked adrenoleukodystrophy. Which of the following therapies has been found to be effective in long-term follow-up for some patients with the X-linked neurodegenerative disease adrenoleukodystrophy? Vegetable oil as a dietary supplement Hyperbaric oxygen therapy Bone marrow transplantation Explanation: Performed prior to the onset of severe symptoms. the child must be evaluated for hypercoagulability. Although neonatal strokes are not always due to thrombosis.) Total volume exchange blood transfusion Plasmapheresis . 17th ed. A 6-mo-old female infant is evaluated after the mother notes that she is strongly left-handed and delayed in sitting and in using the right hand. MRI of the brain reveals a large porencephalic cyst in the distribution of the left middle cerebral artery. 13. Which of the following information is most likely to contribute to establishing the cause of this disorder? Results of an EEG Chromosome analysis Evaluation for thrombophilic disorders Explanation: The MRI study reveals the sequelae of a neonatal stroke. 14.) History of delivery History of maternal drug ingestion Question . and seizures. Genetic analysis indicates a mutation in a transcription factor called MeCP2 that normally functions to silence transcription of numerous genes. 17th ed. (See Chapter 593 in Nelson Textbook of Pediatrics. An 11-yr-old boy had been fighting and wrestling with another boy at school and sustained several bruises on the face and neck. poor growth and weight gain.) Venous sinus thrombosis Cervical spinal cord injury . 15. In most cases the lesion heals without residual neurologic deficits. microcephaly. hand-wringing movements. A 5-yr-old girl is evaluated for severe mental retardation. 17th ed. (See Chapter 592 in Nelson Textbook of Pediatrics.Question . The most likely cause for his neurologic disorder is: Left-sided brain tumor Focal seizure disorder Carotid artery intimal dissection Explanation: Strokes due to carotid dissection can occur spontaneously or with neck twisting or by flexion-extension of the neck and have been reported following roller coaster rides or visits to the beautician. 16.) Adrenoleukodystrophy Menkes disease Question . This finding indicates that she has which of the following disorders? Juvenile Huntington disease Metachromatic leukodystrophy Rett syndrome Explanation: Rett syndrome is classically described by the features of this case. greater in the arm than in the leg. The next morning he woke up with inability to speak clearly and weakness on the right side of his body. 84% of which are granulocytes. (See Chapter 594 in Nelson Textbook of Pediatrics. The H. lethargy.) Guillain-Barré syndrome Metachromatic leukodystrophy Peripheral neuritis Question .) Group B streptococci . influenzae type b vaccine has almost eliminated this pathogen as a cause of serious bacterial infections in children. (See Chapter 579 in Nelson Textbook of Pediatrics. and absent deep tendon reflexes in the upper extremities associated with grade 3/5 weakness symmetrically in all extremities. 18. Gram stain is negative. A 5-yr-old child has a 6-mo history of increasing difficulty in walking associated with urinary incontinence. The most likely diagnosis is: Myasthenia gravis Spinal cord tumor Explanation: The findings described in this patient are typical of an upper motor lesion. Examination reveals a febrile infant who does not interact with the examiner and cries inconsolably. A 7-mo-old girl is presented to the emergency department with gradual onset of fever. a glucose concentration of 12 mg/dL. A lumbar puncture is performed. The bilateral nature and the other features localize the lesion to the spinal cord. and a protein concentration of 70 mg/dL. Physical examination shows an alert child with increased deep tendon reflexes. 17. 17th ed. and the cerebrospinal fluid contains 1. and irritability. 17th ed.Question .500 white blood cells/mm3. Her immunizations are up to date. The most likely etiologic agent for this infection is: An enterovirus Herpes simplex virus Haemophilus influenzae type b Neisseria meningitides Explanation: Both meningococci and even pneumococci (vaccine covers only some but not all pneumococci) are the most common causes of non-neonatal bacterial meningitis. clonus and bilateral Babinski reflexes in the lower extremities. The greatest risk of meningitis following documented occult bacteremia is associated with infection caused by: Streptococcus pneumoniae Haemophilus influenzae type b Neisseria meningitides Explanation: Meningococci may rarely cause transient bacteremia.) Behavioral disturbance Question . 21. 19. producing an aseptic meningitis. In addition. joint. (See Chapter 362 in Nelson Textbook of Pediatrics. 17th ed. a higher incidence of neuropsychiatric disturbances is noted following H. The most common sequela associated with bacterial meningitis is Mental retardation Chronic seizure disorder Impaired vision Impaired hearing Explanation: Hearing deficits are most common and are potentially attenuated by steroids. The most common cause of viral meningoencephalitis is: An enterovirus Explanation: Enteroviral disease is quite common in the summer and fall. 17th ed. (See Chapter 594 in Nelson Textbook of Pediatrics. or elsewhere.) Salmonella species Question . (See Chapter 594 in Nelson Textbook of Pediatrics.) Herpes simplex virus An arbovirus Mumps virus . but a positive blood culture for this pathogen is usually associated with a localized infection in the meninges. influenzae type b infection. 20. PCR techniques have helped to identify this organism.Question . 17th ed. lung. 17th ed. She has right-sided weakness associated with hyperreflexia and an extensor plantar reflex. Her voice is dysarthric. 17th ed. lethargy. (See Chapter 595 in Nelson Textbook of Pediatrics.) . The child is obtunded and has a temperature of 100. Predisposing factors for brain abscesses in other patients include chronic otic and sinus infections. On examination. 22. The most likely diagnosis is: Multiple sclerosis Metachromatic leukodystrophy Explanation: Metachromatic leukodystrophy is a familial degenerative disease affecting both the central nervous system (CNS) and peripheral nervous system white matter?hence the loss of deep tendon reflexes with CNS symptoms. measured while she is supine. She has horizontal nystagmus and optic atrophy. and vomiting. and blood pressure of 96/70 mm Hg in her right arm. the child is apathetic and uninterested in her surroundings. and computed tomography (CT) of the head shows diffuse symmetric attenuation of the cerebral and cerebellar white matter.A respiratory virus Question . She suddenly has a 10-min focal tonicclonic seizure.2°C). A harsh pansystolic murmur is heard best along the left sternal border. A sibling died at the age of 6 yr with a similar history. The motor nerve conduction velocities show marked slowing. She has a history of perioral cyanosis and dyspnea with exertion beginning in infancy. A 4-yr-old girl has experienced progressive loss of ambulation over a 2-yr period.8°F (38. She is hypotonic. Perioral cyanosis is noted at rest. A 3-yr-old girl has a 2-wk history of fever associated with bifrontal headache. The most likely cause of the hemiparesis is: Moyamoya disease A brain tumor Neurocysticercosis Methemoglobinemia A brain abscess Explanation: This young girl has tetralogy of Fallot and a brain abscess resulting in part from the right-to-left cardiac shunt. and her deep tendon reflexes are absent. Examination of her eye grounds reveals bilateral papilledema. pulse of 118/min. 23. (See Chapter 592 in Nelson Textbook of Pediatrics.) GM2 gangliosidosis (Tay-Sachs disease) Neuronal ceroid lipofuscinosis Acute demyelinating encephalomyelitis Question . not macrocephalus. and resolved spontaneously. Findings on physical examination. 25. storage diseases (lysosomal. The MRI study should include the brain and spinal cord.Question . (See Chapter 585 in Nelson Textbook of Pediatrics. Expansion of the bone marrow (hemolytic anemias). A 12-yr-old girl experiences acute monocular blindness of 2 days' duration. Past medical history reveals that she has had headaches for the past 3 yr that she cannot characterize. leukodystrophies). 17th ed.) An electroencephalogram Peripheral nerve conduction tests A nerve biopsy . 24. excessive cerebrospinal fluid (CSF) (hydrocephalus). 17th ed. Causes of megalocephaly include all of the following except: Thalassemia Chronic subdural effusions Hydrocephalus Canavan disease Congenital CMV infection Explanation: Congenital cytomegalovirus (CMV) infection usually causes microcephalus. The most important diagnostic step is to perform: CT MRI Explanation: MRI is most useful in confirming the diagnosis of a possible demyelinating disease such as multiple sclerosis.to 4-mm plaques compatible with the disease. (See Chapter 592 in Nelson Textbook of Pediatrics. and one episode of paresthesias of the feet. did not occur in immediate proximity to the headache. and familial factors contribute to megalocephaly. intracranial bleeding (subdural). including the funduscopic examination. These episodes were not related in time.) Familial factors Question . MRI demonstrates small 3. are unremarkable other than reduced visual acuity. one brief episode of diplopia. a grand mal seizure. affected patients have poor visual acuity. the visual acuity is usually not reduced in papilledema. 27. 26. Hyperemia of the optic nerve Constricted arterioles of the optic disc Dilated disc veins Indistinct optic nerve border Question . The most likely diagnosis is: Bacterial meningitis Tuberculous meningitis Brain tumor Arteriovenous malformation Explanation: Arteriovenous malformation.Question . The patient had a history of intermittent right-sided headaches without an aura and at times without relief with rest. Papilledema of acute onset is associated with all of the following except: Reduced visual activity Explanation: Acutely. In addition to coma on physical examination. This is in contrast with papillitis noted during optic neuritis. like an aneurysm.) Pott puffy tumor Question . The Gowers sign demonstrates: Poor reflexes Spinal dysraphism Tethered cord Proximal motor weakness . Blood in the subarachnoid space produces nuchal rigidity and may be detected by CT or a carefully performed lumbar puncture. A 12-year-old presents with a history of severe headache. 17th ed. 28. producing hemiplegia or coma. (See Chapter 584 in Nelson Textbook of Pediatrics. may rupture. and sudden collapse with unresponsive flaccid coma. the patient is afebrile and has nuchal rigidity. Hysterical paralysis Question . The etiology is unknown. some patients may talk to those around them during the event. requires the child to stand from a supine position by using the arms to "climb" up the legs to stand erect. 17th ed. especially of the thighs. and the treatment of choice is valproic acid. 30. 17th ed. (See Chapter 586 in Nelson Textbook of Pediatrics. Characteristics of simple partial seizures include all of the following except: Loss of consciousness Explanation: There is never a loss of consciousness in simple partial seizures. hemianopia.) Abnormal EEG findings Possible role of CMV infection Question .) Aphasia . (See Chapter 586 in Nelson Textbook of Pediatrics. Rasmussen encephalitis is characterized by all of the following except: Epilepsia partialis continue Onset before age 10 yr No sequelae Explanation: Sequelae may include hemiplegia. 31. 29. Landau-Kleffner syndrome is characterized by all of the following except: Onset at age 5 yr More common in girls Explanation: The disorder is more common in boys. (See Chapter 586 in Nelson Textbook of Pediatrics. Actually. 17th ed.Explanation: Motor weakness.) Versive seizures Duration of 10-20 sec No postictal state Abnormal EEG findings Question . or aphasia. is defined by six or more café-au-lait macules greater than 5 mm in diameter in prepubertal children or greater than 15 mm in postpubertal children plus at least one of the following except: Axillary or inguinal freckling Lisch nodules of the iris Two or more neurofibromas or one plexiform neurofibroma Ash leaf macule Explanation: The ash leaf-shaped hypopigmented macule is most typical of tuberous sclerosis (being present in over 90% of affected children).) Optic glioma Neurofibromatosis type 1 Tuberous sclerosis Late-onset congenital deafness . Café-au-lait macules may not be present. 32. and is characterized by bilateral acoustic neuromas.Normal hearing Multiple seizure types Question . unsteadiness. A 19-yr-old girl presents with headache. scoliosis) Optic gliomas An affected first-degree relative Question . 17th ed. another autosomal dominant disorder. an autosomal dominant disorder (gene on chromosome 17). 33. has distinctive chromosomal sites. (See Chapter 589 in Nelson Textbook of Pediatrics.) Osseous lesions (sphenoid dysplasia. (See Chapter 589 in Nelson Textbook of Pediatrics. Neurofibromatosis type 1. 17th ed. The most likely diagnosis is: Neurofibromatosis type 2 Explanation: NF2 accounts for 10% of all NF cases. and poor hearing that has worsened over the past 5 yr. Her father's medical history includes some type of brain surgery. and he has been deaf since the age of 35 yr. ) Milkmaid's grip Choreic hand Darting tongue Emotional lability . 17th ed. (See Chapter 590 in Nelson Textbook of Pediatrics. Physical features of Sydenham chorea include all of the following except: Hypertonia Explanation: Hypotonia is the rule.Question . 34. are helpful in detecting impending respiratory failure due to intercostal or phrenic nerve involvement. (See Chapter 607 in Nelson Textbook of Pediatrics. A 12-yr-old girl experienced diarrhea. 2 wk before manifesting progressive weakness and inability to walk. 17th ed.NEUROMUSCULAR Question . Physical examination reveals marked peripheral muscle weakness without atrophy or fasciculations. An upper respiratory tract infection or diarrhea (often due to Campylobacter) often precedes the onset of paralysis. The deep tendon reflexes are absent in her ankles and 1+ at her knees. hypoxia) and should be monitored frequently in any patient with acute progressive muscle weakness. She has intermittent tingling of her fingers and toes.) . Findings on the sensory examination are normal. The most likely diagnosis is: Transverse myelitis Guillain-Barré syndrome Explanation: Guillain-Barré syndrome is an ascending peripheral polyneuropathy that is predominantly motor but may have mild sensory symptoms (paresthesias). 2. such as measurement of negative inspiratory force. 17th ed. The patient described in Question 1 is admitted to the hospital and now has progressive weakness and areflexia of the knees and ankles. An important test to perform is: Determination of urine specific gravity Electrocardiogram (ECG) Serum creatine kinase determination Muscle biopsy Pulmonary function testing Explanation: Pulmonary function tests. (See Chapter 607 in Nelson Textbook of Pediatrics. which lasted for 3 days.) Polio Myasthenia gravis Mononeuritis multiplex Question . 1. Reductions in inspiratory force often precede abnormalities of the arterial blood gases (hypercarbia. Motor involvement is symmetric. and her deep tendon reflexes are 1-2+. A 15-yr-old boy has lost his ability to walk.Qestion . he returned from a camping trip. She has no fasciculations or myalgias. The most likely diagnosis is: Hysterical weakness Muscular dystrophy Spinal muscular atrophy Botulism Myasthenia gravis Explanation: Myasthenia gravis is characterized by progressive muscle weakness that is exacerbated by repetitive muscle use. Symptoms are worse in the evening and are usually less severe on awakening in the morning. Classically. A 10-yr-old girl has had diplopia and ptosis and weakness of her neck flexors for 2 mo. (See Chapter 603 in Nelson Textbook of Pediatrics. One week before these symptoms arose. On removal of the tick (often on the scalp).) Question . the paralysis rapidly resolves. The weakness is greatest in peripheral muscles. the facial and extraocular muscles are involved. (See Chapter 605 in Nelson Textbook of Pediatrics. 3. 4. 17th ed. Cranial nerves all are normal. On physical examination. 17th ed.) . The most likely diagnosis is: Myasthenia gravis Organophosphate poisoning Spinal muscular atrophy Botulism Tick paralysis Explanation: Ticks (wood or dog) may produce a motorsensory neuropathy indistinguishable from Guillain-Barré syndrome. his ankle and knee deep tendon reflexes are noted to be diminished. Question . slow motor development. 5. and hypertrophied calf muscles. 17th ed. (See Chapter 598 in Nelson Textbook of Pediatrics. CK is found in only three organs and may be separated into corresponding isozymes: MM for skeletal muscle. 6. MB for cardiac muscle. All of the following statements regarding creatine kinase (CK) are true except: It is the most useful serum enzyme reflecting damaged muscle fibers The MM isozyme is found primarily in skeletal muscle The MB isozyme is found primarily in cardiac muscle The BB isozyme is found primarily in smooth muscle Explanation: Creatine kinase (CK) is one of several lysosomal enzymes released by damaged or degenerating muscle fibers and is the most useful in laboratory measurement of these enzymes in serum. (See Chapter 600 in Nelson Textbook of Pediatrics. A 4-yr-old has difficulty in climbing stairs. and BB for brain. The most likely diagnosis is: Myasthenia gravis Myotonia congenita Duchenne muscular dystrophy Explanation: Duchenne muscular dystrophy is also called pseudohypertrophic muscular dystrophy. 17th ed.) Hypokalemic periodic paralysis Central core disease Question .) . 8. each transmitted by a different gene and each differing in its clinical course and expression. (See Chapter 599 in Nelson Textbook of Pediatrics.) Diagnosis for some disorders may be confirmed by genetic analyses of lymphocytes Hypotonia is a common feature Question .Question . (See Chapter 600 in Nelson Textbook of Pediatrics. Most of the congenital myopathies are hereditary. the diagnosis may be established using the specific molecular probe on lymphocytes. 17th ed. Characteristic features of muscular dystrophies include which of the following? They are a primary myopathy They have a genetic basis The course is progressive Degeneration and death of muscle fibers occur at some stage of disease All of the above Explanation: The muscular dystrophies are a group of unrelated disorders. All of the following statements regarding congenital neuromuscular disorders are true except: Most are hereditary Most are nonprogressive conditions The definitive diagnosis is best made by electromyography (EMG) Explanation: The definitive diagnosis of congenital neuromuscular disorders is best determined by histopathologic findings in the muscle biopsy sample.) . 17th ed. In a few conditions for which the defective gene has been identified. 7. The muscular dystrophies are distinguished from other neurogenic disorders by the four obligatory criteria listed in the question. some are sporadic. Scoliosis is common. Intellectual impairment occurs in all patients.Question . All of the following statements regarding Duchenne muscular dystrophy are true except: It is the most common hereditary neuromuscular disease It is inherited as an autosomal dominant trait Explanation: Duchenne muscular dystrophy. 17th ed. 17th ed. the most common hereditary neuromuscular disorder. Characteristic clinical manifestations of Duchenne muscular dystrophy include: Cardiomyopathy Intellectual impairment Weakness of respiratory muscles Scoliosis All of the above Explanation: Cardiomyopathy is a constant feature of Duchenne muscular dystrophy.) Symptoms are rarely present at birth or in early infancy The serum creatine kinase is consistently greatly elevated It is more common in males Question . 10. is inherited as an X-linked recessive trait. 9. The gene is on the X chromosome at the Xp21 locus. although only 20-30% have an intelligence quotient (IQ) of less than 70.1 in Nelson Textbook of Pediatrics. although the severity of cardiac involvement does not necessarily correlate with the degree of skeletal muscle weakness. (See Chapter 600. (See Chapter 600 in Nelson Textbook of Pediatrics.) . Hyperparathyroidism causes weakness and reversible muscle wasting. Hyperaldosteronism is accompanied by episodic and reversible muscle weakness. Acute attacks may be prevented by administration of dantrolene sodium before an anesthetic is administered.Question .) Myoglobinuria may result in tubular necrosis and acute renal failure Attacks may be prevented by administration of dantrolene sodium Metabolic acidosis may be severe . All of the following are associated with constant muscle weakness except: Hypothyroidism Hyperparathyroidism Corticosteroids Hyperaldosteronism (Conn syndrome) Explanation: Thyrotoxicosis causes proximal muscle weakness and wasting accompanied by electromyogram (EMG) changes. 12. All of the following statements regarding malignant hyperthermia are true except: It is inherited as an autosomal dominant trait Acute episodes are typically precipitated by intravenous administration of dyes for radiographic studies Explanation: Acute episodes of malignant hyperthermia are precipitated by exposure to general anesthetics and occasionally to local anesthetic drugs. (See Chapter 598 in Nelson Textbook of Pediatrics. consistently produces proximal muscle weakness and hypotonia.2 in Nelson Textbook of Pediatrics. (See Chapter 602.17th ed. Hypothyroidism. whether congenital or acquired. Both natural Cushing disease and exogenous corticosteroid administration may cause proximal muscle weakness. 11.) Question . 17th ed. 17th ed. (See Chapter 603 in Nelson Textbook of Pediatrics. A rare familial form is probably an autosomal recessive trait but is not associated with plasma anti-anti-acetylcholine antibodies.Question . 17th ed. Which of the following is the earliest and most consistent sign of myasthenia gravis? Gowers sign Trendelenburg gait Ptosis and extraocular muscle weakness Explanation: Ptosis and some degree of extraocular muscle weakness are the earliest and most constant signs in myasthenia gravis. Older children may complain of diplopia. usually after either influenza or chickenpox Autoimmune disorder Explanation: Myasthenia gravis is an autoimmune disorder. 14. Which of the following is the etiology in most cases of myasthenia gravis? Inheritance as a recessive trait Inheritance as an X-linked trait Postinfectious. 13.) Respiratory muscle weakness Head lag .1 in Nelson Textbook of Pediatrics.) Idiopathic Question . (See Chapter 603. 1 in Nelson Textbook of Pediatrics. being present in only one third of adolescents. 16. the muscle potentials diminish rapidly in amplitude until the muscle becomes refractory to further stimulation. Which of the following is a common clinical manifestation of myasthenia gravis? Cardiomyopathy Intellectual impairment Weakness of respiratory muscles Explanation: If untreated.) Scoliosis Headache . 15. and electrocardiogram (ECG) findings remain normal. 17th ed.1 in Nelson Textbook of Pediatrics. The best method for diagnosis of myasthenia gravis is: Assay for anti-acetylcholine antibodies Nerve conduction velocity (NCV) studies Electromyogram (EMG) Explanation: Myasthenia gravis is one of the few neuromuscular diseases in which an electromyogram (EMG) is more diagnostic than muscle biopsy. Motor nerve conduction velocity remains normal. (See Chapter 603. 17th ed.) Nerve biopsy Muscle biopsy Question . myasthenia gravis is usually progressive and may become life-threatening because of respiratory muscle weakness. Plasma anti-acetylcholine antibodies should be assayed but are inconsistently found. Cardiomyopathy is not a feature of myasthenia gravis. (See Chapter 603. A decremental response occurs in response to repetitive nerve stimulation.Question . Walking is clumsy.) Guillain-Barré syndrome .Question . has generalized hypotonia and weakness. especially when excited. offspring born to myasthenic mothers have normal muscle strength and are not at increased risk for myasthenia gravis in later life. A 1-day-old newborn. (See Chapter 603.1 in Nelson Textbook of Pediatrics. born to a myasthenic mother. Autonomic crises usually begin after age 3 yr. Which of the following best describes the prognosis? Greatly increased risk of the complete picture of myasthenia gravis Small but increased risk of the complete picture of myasthenia gravis No increased risk of myasthenia gravis Explanation: After the abnormal (maternally derived) antibodies disappear. (See Chapter 606. 18. The most likely diagnosis is: Duchenne muscular dystrophy Fabry disease Chronic organophosphate intoxication Familial dysautonomia (Riley-Day syndrome) Explanation: Familial dysautonomia (Riley-Day syndrome) is usually expressed in infancy as poor sucking and feeding. and evidence of apprehension and irritability. blotchy erythema.1 in Nelson Textbook of Pediatrics. There has been new onset of episodes of cyclic vomiting lasting 24-27 hr. 17th ed. A 4-yr-old child with a history of poor sucking and swallowing as an infant recently experienced excessive sweating and blotchy erythema. with retching and vomiting every 15-20 min as well as profuse sweating.) Some residual weakness until puberty but no increased risk of myasthenia gravis Episodes of diminishing severity of muscle weakness until puberty Question . 17. 17th ed. 20. The most likely diagnosis is: Duchenne muscular dystrophy Fabry disease Familial dysautonomia (Riley-Day syndrome) Bell palsy Explanation: Bell palsy is an acute unilateral facial nerve palsy that is not associated with other cranial neuropathies of brainstem dysfunction. Spontaneous recovery usually occurs in 2-3 weeks. Findings on the remainder of the physical examination are normal. primarily in the motor nerves. Deep tendon reflexes are absent. 17th ed. The onset is typically gradual. 17th ed. A 9-yr-old boy presents with paresis of the upper and lower portions of the face and loss of taste on the right side of the anterior portion of the tongue. especially herpes simplex virus and Lyme disease.Question . with symmetric involvement that begins in the lower extremities and progressively involves the trunk and upper limbs. The most likely diagnosis is: Duchenne muscular dystrophy Fabry disease Familial dysautonomia (Riley-Day syndrome) Bell palsy Explanation: Guillain-Barré syndrome is a postinfectious polyneuropathy that causes demyelination. (See Chapter 607 in Nelson Textbook of Pediatrics. There are no sensory deficits or bowel or bladder dysfunction. the corner of the mouth droops.) Guillain-Barré syndrome Question . On physical examination. A 4-yr-old child presents with symmetric weakness that began in the lower extremities and subsequently progressed over 10-14 days to involve the trunk and upper limbs. Bulbar involvement occurs in about half of cases. and he is unable to close the right eye tightly.) Guillain-Barré syndrome . 19. (See Chapter 608 in Nelson Textbook of Pediatrics. Nerve conduction velocity test results are abnormal. It is a common disorder at all ages and typically develops about 2 weeks after a systemic infection. The cerebrospinal fluid shows protein of 78 mg/dL and 5 white blood cells. 17th ed.) Question . On examination. Delivery at term was uneventful. Pregnancy was uncomplicated.400 IU/L. he has a dolichocephalic head. and undescended testes. A male neonate has respiratory distress. dysphagia. but the mother reports weak fetal movements in the last trimester. The next diagnostic test likely to confirm or refute your clinical diagnosis is: EMG Detailed physical therapy evaluation A repeat of the blood marker assay in the child and also in the mother Muscle biopsy MRI study of calves and thighs Explanation: (See Chapters 598 and 599 in Nelson Textbook of Pediatrics. You suspect Duchenne muscular dystrophy and request assay for a molecular marker in blood for dystrophinopathy. and there is no family history of neurologic disease. 17th ed. A 5-yr-old boy has been falling frequently and cannot climb stairs as well as he did 6-12 months ago. and generalized muscular hypotonia and weakness. but no weakness of cranial or extraocular muscles. Serum creatine kinase is 16. high-arched palate. He is an only child. This clinical picture is least suggestive of: Nemaline rod myopathy X-linked myotubular myopathy Duchenne muscular dystrophy Explanation: (See Chapter 599 in Nelson Textbook of Pediatrics. thin muscle mass in trunk and extremities. 22. he has proximal weakness of the legs and enlargement of the calves. and Apgar scores at 1 and 5 min were 7 and 8.) Myotonic dystrophy. neonatal form Congenital muscular dystrophy with merosin deficiency .Question . but the result is normal. On examination. 21. and weakness of the ankles with inability to dorsiflex her feet. These findings would be compatible with any of the following except: Metachromatic leukodystrophy Hereditary motor-sensory neuropathy type I (CharcotMarie-Tooth disease) Explanation: (See Chapter 604 in Nelson Textbook of Pediatrics. and an MRI study of the brain reveals white matter lesions in the periventricular white matter and cerebellum. 17th ed. deterioration in speech and academic performance in school. An 8-yr-old girl demonstrates progressive ataxia. tingling paresthesias of the feet. 17th ed. 24.) Giant axonal neuropathy Globoid cell leukodystrophy (Krabbe disease) Adrenoleukodystrophy Question .Question . 23. Nerve conduction velocity studies show moderate slowing in motor component of the peroneal nerves.) . The most frequent toxic neuropathy in children in the United States currently is due to: Lead Arsenic Organophosphates used in insecticides Various industrial and agricultural pollutants in drinking water Drugs used in chemotherapy Explanation: (See Chapter 605 in Nelson Textbook of Pediatrics. A 15-yr-old boy complains of slowly progressive difficulty with coordination and intermittent diplopia. and ECG evidence of cardiomyopathy. Studies demonstrate normal serum creatine kinase.Question .) Facioscapulohumeral muscular dystrophy Congenital muscle fiber-type disproportion Metachromatic leukodystrophy Multiple sclerosis Question . a mild lactic acidosis. 17th ed. The muscle biopsy in mitochondrial cytopathies may show all of the following histopathologic features except: Ragged-red fibers with trichrome stain Lymphocytic infliltrates in perivascular and interstitial spaces Explanation: (See Chapter 602 in Nelson Textbook of Pediatrics.) Loss of cytochrome c oxidase (COX) activity in some but not all myofibers Ultrastructural abnormalities in mitochondrial cristae Increased lipid within myofibers . 17th ed. EMG shows nonspecific myopathic features. is: Mitochondrial cytopathy Explanation: (See Chapter 602 in Nelson Textbook of Pediatrics. 26. to be confirmed by other tests. Your provisional diagnosis. 25. He also has mild generalized weakness. or fever. and she has no visceromegaly. No heart murmur is heard. A 13-yr-old girl develops ptosis. 27. easy fatigue. Tendor reflexes are preserved.Question .) Question . The most likely diagnosis is: Systemic lupus erythematosus (SLE) Polymyositis Facioscapulohumeral muscular dystrophy Viral myositis with exanthema Dermatomyositis Explanation: (See Chapter 149 in Nelson Textbook of Pediatrics. and a violaceous rash appears over the malar areas of her face and the extensor surfaces of the finger joints. and she has no myalgias or tenderness of muscles to palpation. a waddling gait. On examination.) Muscle biopsy . No weakness of the extremities or limb girdles is detected. and difficulty swallowing but is afebrile and otherwise in good health. her ptosis is noted to increase with sustained upward gaze for 30 sec. and myalgias of the thighs and shoulders over a period of 6 weeks. 17th ed. 28. 17th ed. An 8-yr-old girl develops progressive proximal weakness. generalized lymphadenopathy. diplopia. The initial diagnostic test most likely to confirm the diagnosis is: Anti-acetylcholine receptor antibodies in blood Computed tomography of the thorax for thymoma Motor nerve conduction velocity studies and EMG Edrophonium (Tensilon) test Explanation: (See Chapter 603 in Nelson Textbook of Pediatrics. B. yet malignancy remains the second leading among children 1-14 yr of age.) E. 17th ed. Prostate cancer E. (See Chapter 483 in Nelson Textbook of Pediatrics. Genetic abnormalities are associated with most pediatric cancers but not with most adult cancers Explanation: Specific genetic conditions are believed to account for <5% of all pediatric malignancies.) B.) D. Death due to childhood cancers results in a much longer loss of potential lifespan than is the case with cancer in adults C. Leukemias Brain and central nervous system tumors Lymphomas Bone tumors Epithelial carcinomas Explanation: In contrast with tumors among adults. Leukemia and central nervous system tumors predominate in children D. In comparison with adult cancers. 3. 1. All of the following statements regarding the epidemiology of childhood cancer are true Except: A. All of the following statements regarding adult and pediatric tumors are true Except: A. E. The incidence of cancer during childhood shows a nadir between 7 and 12 yr of age. Epithelial malignancies (carcinomas) are far more common in adults than in children Question . The Neoplastic Diseases and Tumors Nelson Self Assessments website 17th Edition 1 . Which of the following cancer types is least common among children? A. epithelial carcinomas are uncommon among children. 4. Childhood cancer accounts for approximately 15-20% of all cases of cancer Explanation: Only about 1% of new cases of cancer in the United States occur in children. childhood cancers are infrequent B. Colon cancer Question . Which of the following cancers occurs primarily during childhood? A. 17th ed. 2. Breast cancer B.) Question .Neoplastic Diseases and Tumors Nelson Self Assessments website 17th Edition Question . D. 17th ed. (See Chapter 483 in Nelson Pediatrics. Renal cell cancer C. Wilms tumor Explanation: Wilms tumor occurs most commonly among infants and young children. Malignant neoplasms are the second leading cause of death among children 1-14 yr of age C. (See Chapter 483 in Nelson Pediatrics. Chronic leukemia is more common in adults than children E. C. Distribution of cancer types differs markedly between adults and children D. (See Chapter 483 in Nelson Textbook of Pediatrics. 17th ed. Ewing sarcoma D.) Question . nasopharyngeal carcinoma. (See Chapter 483 in Nelson Textbook of Pediatrics. Which of the following cancers has the highest incidence in young children (<7 yr of age)? A. Osteosarcoma Question . post-transplantation non-Hodgkin lymphoma. which was prescribed 2 wk ago.Question . leiomyosarcomas in immunocompromised persons. D.g. (See Box 485-2 and Chapter 485 in) The Neoplastic Diseases and Tumors Nelson Self Assessments website 17th Edition 2 . 17th ed. (See Chapter 484 in Nelson Textbook of Pediatrics. Infectious mononucleosis (Epstein-Barr virus infection) C. There is mild splenomegaly. 7. Acute lymphocytic leukemia Explanation: Leukemias and lymphomas are the most common malignant neoplasms among young children. The other tumors occur with increasing incidence during childhood and adolescence. the lymph nodes are enlarged. (See Chapter 484 in Nelson Textbook of Pediatrics. 17th ed. On physical examination. C. 8. Non-Hodgkin lymphoma Explanation: Epstein-Barr virus is associated with African (endemic) Burkitt lymphoma. It is slowly worsening despite antibiotic therapy with cephalexin. A 14-yr-old girl presents with bilateral cervical lymphadenopathy that has progressed over the last 4 wk. matted. The most likely diagnosis is: A. Wilms tumor E. Which of the following cancers is most closely associated with EpsteinBarr virus? A. B. 5.) C. Ewing sarcoma Hodgkin disease Testicular cancer Retinoblastoma Explanation: The incidence of retinoblastoma peaks during the first 2 yr of life. E. C. X-linked lymphoproliferative syndrome). non-Hodgkin lymphoma in persons with congenital immunodeficiencies (e. Rhabdomyosarcoma Pelvic osteosarcoma Germ cell tumor Wilms tumor None of the above Explanation: Cryptorchidism is a risk factor for testicular germ cell tumors. Human immunodeficiency virus infection B. Methicillin-resistant Staphylococcus aureus infection E. and some cases of Hodgkin disease. Tuberculosis D. Hepatoblastoma Question . 17th ed. For which of the following malignancies is he at increased risk? A. B.. D.) E. and nontender. 6. Osteosarcoma B. A 2-mo-old boy is found to have cryptorchidism. should prompt further investigation for central nervous system malignancy. (See Chapter 486 in Nelson Textbook of Pediatrics. Metabolic complications of anticancer therapy include: A. Lymph node biopsy and culture Explanation: A lymph node biopsy and culture is the definitive means of diagnosing lymphoma. Urinalysis B. 17th ed. hoarseness. E. 10. (See Box 485-2 and Chapter 485 in Nelson Textbook of Pediatrics.) The Neoplastic Diseases and Tumors Nelson Self Assessments website 17th Edition 3 . and white retinal reflex suggests retinoblastoma. Diffuse enlargement of the pons (brainstem) B. Focal neurologic deficit Explanation: Any focal neurologic deficit in the motor or sensory system. D. Plain x-ray film D. Complete blood count and differential Question . Change in urination or defecation pattern Question . (See Box 485-2 and Chapter 485 in Nelson Textbook of Pediatrics. Change in a mole or wart C.) D. Both A and B Explanation: Diffuse enlargement of the pons suggests central nervous system tumor. the most likely to occur in children is: A. Cough. The definitive diagnostic procedure for the child described in Question 8 is: A. 9. B. Abnormal discharge from a body orifice B. especially a decrease in cranial nerve function. White reflex emanating from the retina C. Hyperuricemia Hyperkalemia Hyperphosphatemia Hypocalcemia All of the above Explanation: Anticancer therapy can result in substantial breakdown of tumor cells (tumor lysis syndrome) with release of large quantities of phosphates and potassium into the circulation. Neither A nor B Question . Hypocalcemia can result in the setting of inadequate renal function. 17th ed.Question . 12. 17th ed. or difficulty swallowing E. Ultrasound study of the neck E. C.) C. Of the following signs of cancer. 17th ed. (See Chapter 485 in Nelson Pediatrics. 11.) D. Highly specific indicators of cancer in children include: A. Most children with ALL have pallor. Exposure to medical radiation is associated with an increased incidence of ALL E. All of the following statements concerning acute lymphocytic leukemia (ALL) are true Except: A. All of the following are likely long-term sequelae of the radiotherapy Except: A. 13. B. and panhypopituitarism. D. scoliosis. C. (See Chapter 486 in Nelson Textbook of Pediatrics. E. and about 1% are derived from mature B cells.) Question . D. neck. Interstitial fibrosis Scoliosis Impaired cognition and intelligence Pituitary dysfunction Explanation: Many sequelae of radiation do not become apparent until the child is fully grown.) B. There is marked variability in response to therapy and in the prognosis. Childhood ALL was the first form of cancer shown to be curable with chemotherapy and irradiation Question . Chromosomal abnormalities are identified in most cases of ALL D. (See Chapter 487 in Nelson Textbook of Pediatrics. leukoencephalopathy. A 12-yr-old girl receives cranial. C. Irradiation can result in infertility. D.) Question . because all involve severe disruption of bone marrow function. 17th ed. BCharacteristic presenting symptoms and signs of childhood leukemia include: A. 50% have petechiae or 4 The Neoplastic Diseases and Tumors Nelson Self Assessments website 17th Edition . B. and spinal irradiation for cancer. All of the following statements concerning childhood lymphocytic and myelogenous leukemias are Except: A. Cardiomyopathy is classically associated with the anthracyclines (doxorubicin and daunomycin) but can also occur with irradiation. 14. Pallor Petechiae Lymphadenopathy Bone or joint pain All of the above Explanation: Most children with leukemia present with symptoms of less than 4 weeks' duration. isolated growth hormone deficiency. B. 17th ed. about 15% are derived from T cells. 16.Question . C. second cancers. impaired cognition and intelligence. Leukemias as a group are the most common childhood cancer Acute lymphocytic leukemia accounts for approximately 75% of cases The incidence is higher in white children than in black children The clinical features at presentation are similar The responses to therapy and prognoses are similar Explanation: The clinical features of the childhood leukemias are similar. 17th ed. Most cases (about 85%) are derived from T-cell progenitors Explanation: About 85% of cases of ALL are derived from progenitors of B cells. Staging of ALL is based on bone marrow biopsy and cerebrospinal fluid examination C. (See Chapter 487 in Nelson. hypothyroidism. pulmonary dysfunction (interstitial fibrosis). 15. E. (See Chapter 487 in Nelson Textbook of Pediatrics. Age greater than 1 yr Age younger than 10 yr Presenting white blood cell count under 100. Findings on physical examination and other laboratory studies are normal.000/mm3 Any chromosomal abnormality Explanation: Chromosomal abnormalities are found in most cases of ALL. Timed intensive induction chemotherapy E. and some have no apparent influence. 17th ed. Acute lymphoblastic leukemia Acute myelogenous leukemia Chronic myelogenous leukemia Preleukemia syndrome None of the above. some unfavorable. D. Which of the following factors indicates an increased risk of relapse for childhood ALL? A.) E. 17th ed. B. some indicate a favorable prognosis. The most likely diagnosis is: A. 60% have lymphadenopathy. No blasts are seen on peripheral blood smear. C.1 in Nelson Textbook of Pediatrics. Liposomal anthracycline The Neoplastic Diseases and Tumors Nelson Self Assessments website 17th Edition 5 . (See Chapter 487. Imatinib Explanation: Imatinib was designed specifically to inhibit BCR-ABL tyrosine kinase.) Question . this is a normal bone marrow biopsy Explanation: ALL is diagnosed by a bone marrow evaluation that demonstrates more than 25% of the bone marrow cells as a homogeneous population of lymphoblasts. A 4-yr-old boy has a bone marrow biopsy that shows 4% blasts. D. E. B.4 in Nelson Textbook of Pediatrics. (See Table 487-1 and Chapter 487. 17th ed. A breakthrough treatment for chronic myelogenous leukemia in patients with the BCR-ABL chromosomal translocation is: A. 18.) Question . C. and about 25% have bone pain and arthralgias caused by leukemic infiltration of the perichondral bone or joint or by leukemic expansion of the marrow cavity. 17. 17th ed. 19.mucous membrane bleeding. 25% have fever. Cis-retinoic acid D. (See Chapter 487 in Nelson Textbook of Pediatrics. Umbilical cord blood transplantation C. Rapid response to therapy Question .) B. Adrenal glands Kidney Lung Heart Central nervous system Explanation: The most important extramedullary sites of relapse of ALL are the central nervous system and the testes. (See Chapter 487.1 in Nelson Textbook of Pediatrics. Almost all neonates with Down syndrome and transient myeloproliferative syndrome eventually develop leukemia Explanation: About 20-30% of neonates with Down syndrome who develop a transient leukemia or myeloproliferative syndrome will develop typical leukemia within the first few years of life. D. 17th ed. classically a translocation involving the q23 band of chromosome 11. D. B. Leukemia occurs more frequently among children with Down syndrome B. Acute lymphoblastic leukemia is the most common type of leukemia that occurs in these children C. The most important extramedullary site of relapse in childhood ALL is: A. Chemotherapy is easier to administer because of decreased toxicity compared with the general population E. Spontaneous resolution Question . Testicular relapse occurs in 1-2% of boys with ALL.) Question . C. Leukemia in infants is commonly associated with: A. 21. 17th ed.) E.6 in Nelson Pediatrics. All of the following statements about leukemia in children with Down syndrome are true Except: A. usually after completion of therapy. Intrathecal therapy is key to prevention of later central nervous system relapse. 20. A better prognosis than in older children Philadelphia chromosome FAB L3 (Burkitt) type A translocation involving chromosome 11 Explanation: More than two thirds of cases of infant leukemia demonstrate rearrangements of the MLL gene. E. B.3 in Nelson Textbook of Pediatrics. (See Chapter 487.Question . 22.) The Neoplastic Diseases and Tumors Nelson Self Assessments website 17th Edition 6 . 17th ed. (See Chapter 487. C. and it is this subset of patients that largely accounts for the very high relapse rate. Acute myeloid leukemia has a better outcome in children with Down syndrome than in children without Down syndrome D. Her liver and spleen are not enlarged. The diagnosis is: A.Question . Acute myelocytic leukemia B. A neonate with Down syndrome experiences transient myeloproliferative syndrome that spontaneously resolves. These children have a 20-30% risk of subsequent acute leukemia within the first few years of life.000/mm3. Acute megakaryocytic leukemia C.) D. The nearest pediatric oncology center requires a 4-hr drive. (See Chapter 487.) D. Erythrocyte sedimentation rate The Neoplastic Diseases and Tumors Nelson Self Assessments website 17th Edition 7 .4 in Nelson Pediatrics. to document mediastinal lymph node involvement. This indicates an increased risk of leukemia Explanation: Neonates and infants with Down syndrome may experience transient myeloproliferative syndrome. weight loss. (See Chapter 487. An older child experiences weight loss and night sweats and has a white blood cell count of 80.22)(q34. Which of the following statements best characterizes the prognosis? A. 24. Chronic myelogenous leukemia Explanation: The Philadelphia chromosome is associated with chronic myelogenous leukemia. This recurs intermittently during early childhood C.3 in Nelson Textbook of Pediatrics. Abdominal CT study C. On physical examination. 17th ed. This invariably develops later into leukemia E. Juvenile chronic myelogenous leukemia E.) D. 25. to determine whether these lymph nodes threaten the patency of the airway. (See Chapter 488 in Nelson Textbook of Pediatrics. which is also known as the Philadelphia chromosome. 17th ed. 17th ed. This almost never recurs B. Chromosomal analysis of bone marrow cells reveals a clonal disorder of stem cells with the specific translocation. Head CT study E. This invariably develops later into malignant histiocytosis Question . Before transfer. Chest radiograph Explanation: Chest radiography is important for two reasons: first. Hodgkin disease Question . t(9. and night sweats for 3 mo. which mimics congenital leukemia. 23. and second.q11). the initial evaluation of the patient should include: A. A 15-yr-old white girl reports that she has had a fever.000/mm3 and a platelet count of 600. she is found to have painless swelling of the left cervical and supraclavicular lymph nodes. Bone marrow aspiration B. 17th ed.Question . Jaw involvement occurs in <20% of U. Erythrocyte sedimentation rate C. A 7-yr-old boy weighing 24 kg presents with painless right supraclavicular lymphadenopathy. t(8:14). cases. C.2 in Nelson Pediatrics. Thoracic CT study Question .2 in Nelson Textbook of Pediatrics. Serum uric acid B. which of the following laboratory studies provides an indirect measure of tumor burden? A. D.S.) D. Small non-cleaved cell non-Hodgkin lymphoma (SNCCL) (Burkitt's type) is characterized by all of the following Except: A. Which of the following indicates a poorer prognosis? A. Weight loss of 2 kg Question . (See Chapter 488 Nelson) E. (See Chapter 488. 27.) E.S. 28. (See Chapter 488. cases. Serum lactic dehydrogenase (LDH) Explanation: Elevation of the level of serum lactate dehydrogenase (>500 U/L) correlates with tumor mass and has proved useful for stratifying therapy intensity. B. 29. D. a lymph node biopsy can confirm suspicion of Hodgkin disease. The chest radiograph of the patient described in Question 25 reveals mediastinal lymphadenopathy. nodular sclerosing type. compared with 70% of younger patients in equatorial Africa. Abdominal CT study Head CT study Bone marrow biopsy Lymph node biopsy Explanation: At this time. the presence of hilar lymphadenopathy with a mediastinal mass larger than one third of the thoracic diameter indicates bulky disease and a poorer prognosis. 17th ed. 26. Chest radiograph reveals a mediastinal mass measuring more than one-third of the thoracic diameter. Thereafter. CT scans and bone marrow biopsy are useful in staging the extent of the lymph node and extranodal involvement.1 in Nelson. Pruritus E.8) translocation B lymphocyte origin Presence of EBV genome Mediastinal lymphadenopathy Explanation: SNCCL presents as an abdominal tumor in 80% of U. Serum ALT (alanine aminotransferase) E. The best diagnostic test at the referral center is: A. C. or t(2. Absolute lymphocyte count The Neoplastic Diseases and Tumors Nelson Self Assessments website 17th Edition 8 .) B. Hilar lymphadenopathy Explanation: For Hodgkin disease.22). Unilateral cervical adenopathy C. Biopsy of the lymph node reveals Hodgkin disease. Intense mediastinal uptake on gallium 67 scan D. B. In children with newly diagnosed non-Hodgkin lymphoma. t(8. (See Chapter 488. 17th ed. Jaw involvement Question . Physical examination also shows hypertension. 17th ed. Which of the following is a favorable prognostic factor in neuroblastoma? A. Chromosome 1p deletion in 80-90% of cells The Neoplastic Diseases and Tumors Nelson Self Assessments website 17th Edition 9 . C. For the boy described in Question 30. B. ANA and high-dose methylprednisolone Question . fixed abdominal mass that causes discomfort.Question . 33.) E. (See Chapter 489 in Nelson Textbook of Pediatrics. Age > 5yr C. Physical examination reveals bilateral papilledema. A 5-yr-old boy presents with progressive headache that is associated with nausea and vomiting. (See Chapter 490 in Nelson Textbook of Pediatrics. Central nervous system lupus Question . either in the adrenal gland or in retroperitoneal sympathetic ganglia. 30. Brain abscess D. 17th ed. (See Chapter 490 in Nelson Pediatrics. whereas other vasoactive substances may produce a secretory diarrhea.) C. The most likely etiology is: A. Catecholamine production may cause hypertension. compared with 2550% for children 1-5 yr of age. 32. Central nervous system tumor Explanation: The classic triad of headache. Cerebrovascular event E. and papilledema is associated with midline and infratentorial tumors. The most likely diagnosis is: A. Hodgkin's disease Ewing sarcoma Nephroblastoma Neuroblastoma Explanation: Most neuroblastomas arise in the abdomen. D. Lumbar puncture. Renal clear cell carcinoma Question . (See Chapter 489 in Nelson Textbook of Pediatrics. Age <1 yr Explanation: Children with neuroblastoma with early-stage disease without amplified MYCN or chromosome 1p deletion can usually be cured with surgery alone. B. Stage 3 or 4 D. the most appropriate management is: A. including measurement of CSF pressure Blood culture and CSF culture and empirical antibiotic therapy Cerebral angiography Emergency MRI scan Explanation: MRI is the preferred neuroimaging test for suspected central nervous system tumors.) B. Children <1 yr of age have a 95% 3-yr survival rate. 17th ed. Pseudotumor cerebri B. nausea and/or vomiting.) E. C. 17th ed. Amplified MYCN E. A 4-yr-old child presents with a hard. 31. D. ) D. It does not resolve but worsens. but the leg appears normal. Lungs are clear to auscultation and the heart sounds are normal. vomiting. Her mother observes her for a week.) C. and pulse 130/min. Blood pressure is 150/95 mm Hg. Blood culture B. Neuroblastoma E. 17th ed. Wilms tumor Explanation: Wilms tumor accounts for most renal neoplasms in children. low-grade fever for 1 wk. 34. but abdominal examination is unrevealing because of his irritability that prevents adequate palpation. weight loss. Renal clear cell carcinoma The Neoplastic Diseases and Tumors Nelson Self Assessments website 17th Edition 10 . Which of the following is the most likely explanation for these clinical findings? A. The most appropriate diagnostic test for this patient is: A. (See Chapter 491 in Nelson Textbook of Pediatrics. There was no decrease in appetite. Hodgkin disease B. Some tumors produce catecholamines that can cause sweating and hypertension. (See Chapter 490 in Nelson Textbook of Pediatrics. When he tries to walk. respiratory rate 24/min. nodular mass that causes abdominal pain. diarrhea. and the child begins to be more uncooperative and to have random. Findings on the MRI study are normal.Question . PTT. darting eye movements. 17th ed. and limp. She is taken to her pediatrician. The most likely tumor is: A. Abdominal CT scan Explanation: Patients with abdominal neuroblastoma present with a firm. 35. 17th ed. and fibrinogen D. but there appear to be bruises below both eyes. (See Chapter 490 in Nelson Pediatrics. Acute hepatic encephalopathy Cysticercosis Behavioral problems Neuroblastoma Explanation: Opsomyoclonus ("dancing eyes and dancing feet") is a paraneoplastic syndrome of autoimmune origin that is associated with neuroblastoma. D. Ears and throat are normal. or fever. He has no history of vomiting or diarrhea or of recent trauma. PT. Acute demyelinating encephalomalacia Question . On physical examination. Plain x-ray film of entire left leg Question . 36. hoping the clumsiness will resolve. he refuses to bear weight on his left leg. She seems to be having trouble with balance but does not appear in pain. C. who also notes these abnormalities and performs an MRI study of the brain.) E. His abdomen appears distended. Synovial fluid analysis of left hip E. A previously healthy 20-mo-old girl is noted by her mother to be more irritable and to be falling more frequently than usual when she walks. B. Ewing sarcoma C. Coagulation screen including platelet count. Weight is decreased 1 kg from his weight at a previous visit at 18 mo of age. he is pale with a temperature of 38°C. A 3-yr-old child presents with an abdominal mass and microscopic hematuria. A previously healthy 2-yr-old boy presents with irritability. Osteosarcoma occurs with the highest incidence in which of the following age groups? A. Orbital cellulitis Question .) E. 39. A 4-yr-old boy presents with urinary obstruction. B.) E. Pseudotumor cerebri Trichinosis Retinoblastoma Rhabdomyosarcoma Explanation: Orbital rhabdomyosarcoma is a common site for rhabdomyosarcoma. 17th ed. The most likely diagnosis is: A. Hydronephrosis Question . (See Chapter 492 in Nelson Textbook of Pediatrics. D. A 9-yr-old previously healthy white girl manifests progressive painless proptosis. The most common soft tissue sarcoma in children is: A. The most likely diagnosis is: A.) E. and decreased visual acuity of the left eye during a 2-mo period. B.Question . B. D. (See Chapter 492 in Nelson Textbook of Pediatrics.) The Neoplastic Diseases and Tumors Nelson Self Assessments website 17th Edition 11 . C. 38. periorbital edema. C. 17th ed. (See Chapter 493 in Nelson Textbook of Pediatrics. There is no history of urinary tract infections or other illnesses. Ultrasound study reveals a pelvic mass. Wilms tumor is an renal tumor. 40. Wilms tumor Neuroblastoma Germ cell tumor Rhabdomyosarcoma Explanation: Rhabdomyosarcoma is the most common soft tissue sarcoma of childhood. which would not be found in the pelvis. (See Chapter 492 in Nelson Textbook of Pediatrics. D. 37. Synovial sarcoma Malignant fibrous histiocytoma Fibrosarcoma Rhabdomyosarcoma Explanation: Rhabdomyosarcoma is the most common soft tissue sarcoma in childhood. Neonates Children <5 yr of age Children 5-10 yr of age Adolescents Explanation: The highest risk period for development of osteosarcoma is during the adolescent growth spurt. 17th ed. Alveolar (soft part) sarcoma Question . 17th ed. B. C. C. which produces local signs as it grows and displaces normal tissues. D. ===================== Figure 493-2 Radiograph of an osteosarcoma of the femur with typical "sunburst" appearance of bone formation Question . or "onion-skinning. 17th ed. Ewing sarcoma B. 41. Benign bone cyst The Neoplastic Diseases and Tumors Nelson Self Assessments website 17th Edition 12 . Histiocytosis X D. Osteosarcoma Explanation: Pain and swelling are the most common presenting symptoms of osteosarcoma and Ewing sarcoma. Ewing sarcoma is associated with periosteal elevation. The most likely diagnosis is: A. Osteosarcoma has a characteristic sunburst pattern on x-ray films. (See Chapter 493 in Nelson Pediatrics. Osteochondroma E. A 15-yr-old boy presents with a palpable swelling of the humerus that is associated with pain that awakens him at night. There is a sunburst pattern on the x-ray film (Figure)." on x-ray.) C. 17th ed. Possible late effects of chemotherapy include all of the following Except: A. 43.Question . C. D. He has also had night pain. B. D. A 15 yr-old female basketball player sustained a knee injury 4 wk previously. A 12-yr-old boy complains of pain and swelling in his right upper arm with onset 2 wk previously. (See Chapter 493 in Nelson Textbook of Pediatrics. Cardiotoxicity Sterility Hearing loss Second malignancy Cataracts Explanation: Cardiotoxicity is associated with doxorubicin hydrochloride (Adriamycin). which is the most appropriate next step in management of this patient? A. An initial knee x-ray was read as normal. Repeat knee x-ray Treatment with a nonsteroidal anti-inflammatory drug Arthroscopy Continuation of physical therapy for another 2 weeks MRI study Explanation: A bone tumor should be suspected in a child or adolescent with persistent bone pain and swelling. The most appropriate next step is: A. with or without systemic symptoms. methotrexate. which is most prominent in the distal femur.) Question . 44. Bone scan Question . C. Repeat examination shows diffuse swelling around the knee. (See Chapter 493 in Nelson Textbook of Pediatrics. An x-ray film of the humerus shows periosteal elevation along the shaft. C. 42. Of the following. 17th ed. He has had fever with temperatures to 101°C for the past 2 days. She has been receiving physical therapy for the past 3 wk with no relief of her pain. (See Chapter 493 in Nelson Textbook of Pediatrics. A 12 yr-old boy has completed multiagent chemotherapy with doxorubicin hydrochloride (Adriamycin).) E. E.) The Neoplastic Diseases and Tumors Nelson Self Assessments website 17th Edition 13 . B. and cisplatin and limb salvage surgery for nonmetastatic osteosarcoma. B. E. 17th ed. Blood culture IV antibiotics Erythrocyte sedimentation rate (ESR) MRI study of humerus Explanation: MRI is the neuroimaging test of choice for bone tumors. D. Question . 47. is present in most cases of the Ewing family of tumors. C. (See Chapter 493 in Nelson Textbook of Pediatrics. Physical examination reveals marked tenderness and warmth.) B. 46. B. The most likely diagnosis is: A. t(11. Osteoid osteoma Aneurysmal bone cyst Osteochondroma Langerhans cell histiocytosis Explanation: Children with Langerhans cell histiocytosis and bone lesions should be evaluated for visceral involvement because treatment of Hand-SchüllerChristian disease and Letterer-Siwe disease is more complex and often requires systemic therapy. C. 17th ed. B. Which of the following benign bone processes may be associated with visceral involvement? A. A 9-yr-old boy develops local pain and swelling of the left lateral aspect of the skull.22). Osteosarcoma C. 17th ed. Osteoid osteoma The Neoplastic Diseases and Tumors Nelson Self Assessments website 17th Edition 14 . (See Chapter 493 in Nelson Textbook of Pediatrics. Plain film shows a well-defined radiolucent skull lesion. (See Chapter 493 in Nelson Textbook of Pediatrics. Eosinophilic granuloma E. 45. D. E. or a variant thereof. D. Fibrous dysplasia Question . 17th ed.) E. Eosinophilic granuloma Letterer-Siwe disease Hand-Schüller-Christian disease Osteomyelitis of the skull Brain abscess Explanation: Eosinophilic granuloma is a monostotic or polyostotic disease with no extraskeletal involvement. Ewing sarcoma Explanation: A specific chromosomal translocation. Which of the following bone neoplasms is associated with a characteristic chromosomal translocation? A. Fibrous dysplasia D. The skeletal survey is otherwise normal.) Question . This differentiates eosinophilic granuloma from other forms of Langerhans cell histiocytosis (Letterer-Siwe or Hand-SchüllerChristian variants). D. with limitation of movement of the right knee. 49. The most likely diagnosis is: A. Many of these tumors are asymptomatic or are detected when the child or the parent notes a bony. C. Most arise in the metaphyses of long bones. sorry Question . A radiograph (Figure) of the femur is shown.) E. A 14-yr-old boy presents with a bony. C. Benign bone cyst The Neoplastic Diseases and Tumors Nelson Self Assessments website 17th Edition 15 . nonpainful mass in the distal femur that has been slowly enlarging. The mass is most likely to represent which of the following tumors? A. There are no associated symptoms. nonpainful mass.. (See Chapter 493 in Nelson Pediatrics. D. B. 17th ed.======================================= . A 5-yr-old boy presents with pain and swelling of the proximal right lower leg over the past 2 weeks. Ewing sarcoma Osteosarcoma Langerhans cell histiocytosis Osteochondroma Explanation: Osteochondroma (exostosis) is one of the most common benign bone tumors in children.. Acute osteomyelitis Chronic osteomyelitis Osteoid osteoma Ewing sarcoma Explanation: Ewing sarcoma has a characteristic radiographic onion-skin appearance. caused by periosteal reaction around the lytic bone lesion. Osteosarcoma Question . (See Chapter 493 in Nelson Textbook of Pediatrics. 17th ed. 48..) E. B. or alternatively a CT scan. Germ cell tumor C. Children 5-10 yr of age D. Adolescents E.000 live births in the United States. On routine office ophthalmoscopic examination of a 14-mo-old girl. 17th ed. 52. A red reflex was reportedly present at birth. 17th ed.) B. you have a hard time seeing the fundus but observe no gross external physical abnormalities. The most appropriate next step in management would be to: Reassure the parents that nothing is wrong Obtain Toxoplasma and cytomegalovirus titers Obtain an MRI scan Refer the patient to an infectious disease specialist Refer the patient to an ophthalmologist for examination performed with the child under general anesthesia Explanation: Difficulty visualizing the fundus raises the suspicion of retinoblastoma and warrants immediate attention and examination. (See Chapter 494 in Nelson Textbook of Pediatrics. D. (See Chapter 494 in Nelson Textbook of Pediatrics. None of the above?the risk of retinoblastoma is similar in all age groups Question .) D. may also identify this ocular tumor. The extraocular muscles appear intact. 50. The highest incidence of retinoblastoma has been noted in which of the following age groups? A. Leiomyosarcoma The Neoplastic Diseases and Tumors Nelson Self Assessments website 17th Edition 16 . (See Chapter 494 in Nelson Textbook of Pediatrics. Wilms tumor B. Question .Question . 51. An infant presents with the finding shown in the Figure. Children 2-5 yr of age C. An MRI scan. E.) ========================================= A. and the remaining physical findings are normal. Rhabdomyosarcoma E. Infants Explanation: Retinoblastoma occurs in 1 of 18. C. B. The cancer most likely to be associated with this finding is: A. Retinoblastoma Explanation: Leukocoria (white pupillary reflex) and strabismus are often the presenting signs of retinoblastoma. 17th ed. Approximately 30% respond dramatically and begin to regress within 1 wk. 56. Kaposi-like form of infantile hemangioma B. 54. C. 17th ed. Maffucci syndrome E. leading to thrombocytopenia and microangiopathic hemolytic anemia. Which of the following is the most common neoplasm involving the liver in children? A. 55. Ollier disease Question . Approximately two thirds of hepatic tumors in children are hepatoblastomas. Possible diagnoses include all of the following Except: A. Wilms tumor E.) D. D.. and the remainder do not respond. 17th ed. Hepatoblastoma Explanation: Primary hepatic tumors are rare in children. D. Albright syndrome C. An infant has a hemangioma that grows rapidly in size. 17th ed. An infant has a hemangioma over the eyelid that grows rapidly in size. Rhabdomyosarcoma Question . accounting for 1% of malignancies in childhood. B. (See Chapter 497 in Nelson. B. Other treatments for hemangiomas that do not respond to corticosteroids include Interferon. Hepatocellular carcinoma B. C. Erythropoietin Heparin Platelet-activating factor Corticosteroids Explanation: For hemangiomas that are life-threatening or that threaten vital functions such as eyesight. Which of the following terms describes this condition? A. laser therapy. (See Chapter 495 in Nelson Textbook of Pediatrics.) E. Kasabach-Merritt syndrome Explanation: Kasabach-Merritt syndrome is trapping of platelet and red blood cells within a hemangioma with activation of the clotting system within the vasculature of the hemangioma. The most reasonable systemic therapy would be administration of: A. 53. Neuroblastoma D. A 10-yr-old boy presents with a painless mass involving the right testicle. Vitamin A The Neoplastic Diseases and Tumors Nelson Self Assessments website 17th Edition 17 . 40% stabilize or show minimal response.) C. Hydrocele Question . obstructing vision. (See Chapter 496 in Nelson Pediatrics. a trial of oral corticosteroids is warranted. Leukemia Rhabdomyosarcoma Germ cell tumor Testicular torsion Explanation: Testicular (spermatic cord) torsion produces acute pain and swelling of the scrotum. and surgery.) E.Question . 17th ed. (See Chapter 497 in Nelson Pediatrics. Leukemia E. The most likely diagnosis is: A. B. 59.) E. Corticosteroids Question . C. (See Chapter 499 in Nelson Textbook of Pediatrics. A 1-yr-old girl has a history of fever.) B. 58. Corticosteroids The Neoplastic Diseases and Tumors Nelson Self Assessments website 17th Edition 18 . (See Chapter 499 in Nelson Textbook of Pediatrics. and there were no other symptoms. which of the following would be the recommended treatment? A. A skull radiograph (Figure) shows multiple lesions. treatment should be minimal and directed at arresting the progression of a lesion (e. generalized lymphadenopathy. and pulmonary infiltrates. 17th ed. 57. Langerhans cell histiocytosis Explanation: Bone lesions with Langerhans cell histiocytosis may be single or numerous and are seen most commonly in the skull..g. Regarding the radiographic finding in Question 57. Curettage of the bone lesions IVIG InterferonCombination chemotherapy Explanation: Multisystem Langerhans cell histiocytosis should be treated with systemic multiagent chemotherapy. if the same finding were seen in a 4-yr-old girl with localized swelling on her skull. (See Chapter 499 in Nelson Textbook of Pediatrics. D. 17th ed. Therefore. Osteochondritis C. 17th ed. Ewing sarcoma Question .) D. The recommended treatment for the child described in Question 57 is: A. severe skin eruption. a bone lesion) that could result in permanent damage before it resolves spontaneously. Combination chemotherapy E.Question . Osteosarcoma B. Curettage or low-dose radiation therapy may accomplish this goal. IVIG C. anemia. InterferonD. Curettage of the bone lesions Explanation: Curettage of the bone lesion. (See Chapter 619 in Nelson Textbook of Pediatrics. Amblyopia Explanation: Amblyopia is a vision loss that is due not to a specific organic lesion but rather to deprivation or disuse of the retina. in its classic form. 17th ed. Oxygen does not directly cause cataracts. Rubella (congenital) Galactosemia Galactokinase deficiency Neonatal hypoglycemia Hypocalcemia Lowe syndrome Hyperoxygenation Explanation: Hyperoxia in preterm neonates usually causes retinopathy of prematurity.) H. B. Steroid therapy I. 3. Epilepsy Congenital blindness Neuroblastoma Dysmetria Spasmus nutans Explanation: The acquired triad of nystagmus. head nodding. 17th ed. which occasionally is associated with cataracts. is self-limited and benign. Delayed removal of a congenital and complete unilateral cataract may lead to: A. Uveitis D. E. C. F. Strabismus E. 17th ed. Even after delayed removal of a cataract or correction of strabismus.Nelson Self Assessments website 17th Edition 1 . E. Glaucoma C. D. and torticollis. (See Chapter 614 in Nelson Textbook of Pediatrics. head nodding. This child is most likely to have: A. and torticollis. Nonetheless. Nyctalopia Question . D. (See Chapter 619 in Nelson Textbook of Pediatrics. Findings on a cranial MRI scan are normal. Child abuse Question .Disorders of the Eye Nelson Self Assessments website 17th Edition Question . Cataracts are noted in all of the following Except: A.) Disorders of the Eye . children with brain tumors may have signs resembling components of spasmus nutans. 1. G. An 18-mo-old manifests pendular nystagmus. C. the previously unused retina "tunes out" the image. 2. B.) B. Discontinue the oral contraceptives and re-evaluate in 7-14 days B. also known as "cat eye") instead of the normal red pupillary reflex. Perform lumbar puncture for determination of cerebrospinal fluid pressure Question .Question . Neuroimaging should be performed. Rhabdomyosarcoma Explanation: Rhabdomyosarcoma of the face. 6.Nelson Self Assessments website 17th Edition 2 . Red reflex Question . Aniridia E. 7. Retinoblastoma B. The most appropriate action is: A. Perform computed tomography (CT) or magnetic resonance imaging (MRI) study of the head Explanation: Papilledema constitutes a neurologic emergency. Discontinue the oral contraceptives and administer promethazine (Phenergan) as needed for nausea and vomiting C. Review of systems reveals history of increased tearing and apparent sensitivity to light. Coloboma E. A 7-yr-old girl experiences fullness of the right upper eyelid and downward displacement of the eye over a 2-mo period. (See Chapter 624 in Nelson Textbook of Pediatrics. This patient may have pseudotumor cerebri. The cornea appears cloudy.) E. 17th ed. including the cornea. Glaucoma Explanation: Symptoms of infantile glaucoma (glaucoma that begins in the first 3 years of life) include the classic triad of epiphora (tearing). The most likely diagnosis is: A. and vomiting. but the initial sign in the majority of cases is leukocoria (white pupillary reflex. a lumbar puncture for determination of cerebrospinal fluid pressure should be performed. Coloboma Disorders of the Eye . Physical examination reveals papilledema. nausea. and if no intracranial masses are found. Leukocoria Explanation: The clinical manifestations of retinoblastoma vary. (See Chapter 623 in Nelson) C. Which of the following is the most likely diagnosis? A. Chorioretinitis D. A 17-yr-old girl taking oral contraceptives presents with headache. The right eye also appears to be proptotic. 5. (See Chapter 621) D. (See Chapter 622) E. Heterophoria B. which are usually attributed to corneal irritation. 4. orbit. The most common presenting sign of retinoblastoma is: A. Chalazion D. and blepharospasm (eyelid squeezing). Hypothyroidism Question . Right superior oblique palsy C. An increase in intraocular pressure leads to expansion of the globe. Hypopyon C. Discontinue the oral contraceptives and administer oral corticosteroids D. photophobia (sensitivity to light). Myasthenia gravis B. and sinus often presents early because of the space-occupying and displacement effects of tumor growth. A 1-yr-old child presents with increased size of the cornea. Procedures used to treat glaucoma in children include surgery to establish a more normal anterior chamber angle (goniotomy and trabeculotomy). 10. (See Chapter 623 in Nelson Pediatrics. A 12-yr-old boy has a small corneal abrasion detected by Wood lamp examination. and a topical anesthetic as necessary for pain E. to create a route for aqueous fluid to exit the eye (trabeculectomy and seton surgery). Furthermore. B. 17th ed. C. Which of the following is the most appropriate initial treatment for the child described in Question 9? A. Topical antibiotic ointment Explanation: Corneal abrasions are treated with frequent applications of a topical antibiotic ointment until the epithelium is completely healed. Tears are not usually produced with crying until a few months of age. or to reduce aqueous fluid production (cyclocryotherapy and photocyclocoagulation). 17th ed.Nelson Self Assessments website 17th Edition 3 . 8. a semipressure patch. Topical antibiotic ointment. Which of the following is the most appropriate treatment? A. All of the following statements concerning the newborn eye are correct Except: A. A topical cycloplegic agent (cyclopentolate hydrochloride 1%) can relieve the pain from ciliary spasm in patients with large abrasions. an improperly applied patch may itself abrade the cornea. 9. several operations are required to lower and maintain the intraocular pressure adequately. The cornea is smaller than that of an adult Tears are often not present with crying until 1-3 mo of age Ocular alignment may not be perfect in the first several months of life Most infants are myopic (nearsighted) Explanation: Most infants are hyperopic (farsighted). the treatment of infantile glaucoma is primarily surgical. C.Question . Topical antibiotic ointment and a semipressure patch C. and a topical anesthetic as necessary for pain Question . Topical antibiotic ointment. The cornea is smaller in infants and grows to reach adult size near the age of 2 yr. In many children.) B. Iris color may change in the first several months of life Disorders of the Eye . Medications and surgery Question . B. Topical antibiotic ointment. D. a semipressure patch. Topical glaucoma medications Oral glaucoma medications Corticosteroids Surgery Explanation: Unlike glaucoma in adults. The use of a semi-pressure patch does not improve healing time or decrease pain. and a topical cycloplegic agent D. D. Long-term medications may be necessary as well.) E. It is common for children to have strabismus in the first few months of life. (See Chapter 609 in Nelson) E. (See Chapter 625 in Nelson Pediatrics. Topical anesthetics should not be given at home because they retard epithelial healing and inhibit the natural blinking reflex. in which medications are the first line of therapy. a topical cycloplegic agent. However. Children with myopia will frequently squint to see better. Which of the following statements concerning abnormalities of refraction is correct? A. the term anisometropia does not indicate a difference in vision by itself. What is the most likely reason for her poor vision? A. (See Chapter 612 in Nelson Textbook of Pediatrics. Anisometropia occurs when there is a difference in refractive error between the two eyes. (See Chapter 611 in Nelson Textbook Pediatrics. these problems are far less common in this age group. Although the need for glasses in only one eye or a cataract in one eye could present in the same way.Nelson Self Assessments website 17th Edition 4 . Ocular motility Question . 12. D. Pupil assessment C. Anisometropia occurs when one eye sees better than the other E. Astigmatism may be caused by an irregular shape of the cornea Explanation: Astigmatism is commonly caused by an irregular corneal shape. 11. A 4-yr-old girl presents with a large esotropia and poor vision in one eye. Children with hyperopia will often squint to see better B.) C. External examination E. Myopia can be cured with the use of bifocal lenses D. Preferential looking test Explanation: Preferential looking tests are done by experienced examiners and are not part of a routine eye examination. C.) Disorders of the Eye . Need for glasses Malingering Cataract Suppression Amblyopia Explanation: This child probably has amblyopia secondary to her strabismus. 17th ed. B. It may lead to amblyopia and decrease vision in one eye. Visual acuity B. (See Chapter 610 in Nelson) D. Both eyes are normal on examination. Children with hyperopia will accommodate to see better. 13. E.Question . Myopia is always a benign condition Question . Which of the following tests is not part of a routine eye examination? A. High levels of myopia may lead to retinal detachment. 17th ed. Scientific studies have not proved that bifocals can cure myopia. An appropriate study to be performed in the future is: A. On further examination. Amblyopia can occur if the ptosis causes a unilateral astigmatism. Many cases of nasolacrimal duct obstruction will resolve within the first year of life. (See Chapter 616) Disorders of the Eye . Amblyopia cannot occur once the lid is surgically elevated D. (See Chapter 613 in Nelson Textbook of Pediatrics. The most likely diagnosis is: A. the pupils look large and nonreactive. Amblyopia will occur only if the lid covers the pupil C. Surgery does not need to be performed early if the vision is developing normally. Lumbar puncture E. Probing of the nasolacrimal duct if the symptoms do not resolve with time E. Renal ultrasound study Explanation: In aniridia. 17. Because children so affected are at risk for Wilms tumor. Which of the following statements concerning congenital ptosis is true? A. Surgery should be performed shortly after birth in all cases B. Cocaine test Question . 17th ed. Her external examination is otherwise normal. 17th ed. Amblyopia can continue to occur even after successful ptosis surgery. 15. Congenital esotropia Question . Topical antibiotics for infections if they occur C. There is no family history of similar ocular findings. Nasolacrimal massage B. It can occur even if the pupil is not completely occluded.) C. Visual evoked potentials C.Nelson Self Assessments website 17th Edition 5 . 16. (See Chapter 615 in Nelson Textbook of Pediatrics. (See Chapter 614 in Nelson Textbook Pediatrics. Intermittent exotropia D. renal ultrasound study is indicated. Congenital esotropia does not develop at this age. CT scan of the head and orbit B. A newborn is presented with what appear to be large pupils that do not constrict to light. he is found to be excessively farsighted for his age. Frontalis suspension surgery is used for mild cases of ptosis Question . 14. Duane syndrome E. A 2-yr-old boy presents with pronounced crossing of his left eye of 2 months' duration. A 3-mo-old girl is presented with tearing and mucous discharge from her left eye that has been present since near birth. Congenital superior oblique palsy B. 17th ed. Ptosis may occur in association with other ocular or systemic disorders Explanation: Ptosis is often associated with other systemic disorders.) E. Accommodative esotropia Explanation: Accommodative esotropia causes a crossing of the eyes secondary to a high degree of farsightedness.) D. Frontalis suspension is used for cases of severe ptosis. Cleansing of the lids D. All of the above Explanation: All of the above are correct.Question . Appropriate forms of treatment would include: A. The age of the child at presentation E. The health status of the posterior segment of the eye Question .Nelson Self Assessments website 17th Edition 6 . Ophthalmia neonatorum Explanation: Ophthalmia neonatorum occurs shortly after birth.Question . The compliance with patching for amblyopia C. Early diagnosis. The length of time the cataract was present D. (See Chapter 617 in Nelson Pediatrics. 19. age at time of cataract development. (See Chapter 619 in Nelson Textbook of Pediatrics. Vernal conjunctivitis Question .) B. not the cornea. 18. Sympathetic ophthalmia is unilateral and occurs in the uninjured eye following a significant ocular injury. 21. Epibulbar dermoid Question . 17th ed. Congenital cataract Explanation: A cataract is opacity of the lens. The most likely diagnosis is: A. Juvenile rheumatoid arthritis Explanation: Uveitis in patients with JRA commonly presents without pain or redness. A young girl is found to have bilateral iritis on routine examination. Toxoplasmosis Disorders of the Eye . Whether a contact lens or an intraocular lens implant is used Explanation: The mode of visual rehabilitation is not the most important factor in the outcome of pediatric cataract surgery. (See Chapter 618 in Nelson Textbook of Pediatrics.) C. The most likely diagnosis is: A. A nasolacrimal duct obstruction does not usually cause significant conjunctival redness this early. A 2-day-old infant is presented with bilateral conjunctival redness and purulent discharge. 17th ed. and success with occlusion therapy are the most important factors. Blepharitis and vernal conjunctivitis do not generally occur at this age.) C. Herpes simplex may cause an anterior uveitis but is usually unilateral and associated with pain. 17th ed. Sympathetic ophthalmia B. Peters anomaly E. Infantile glaucoma D. Blepharitis E. On further questioning. All of the following disorders may present as a congenital corneal opacity Except: A. Congenital glaucoma D. 20. Congenital glaucoma is often associated with tearing but not with a purulent discharge. All of the following are important factors in the visual prognosis following the removal of the cataract Except: A. A child presents with a dense unilateral cataract. Nasolacrimal duct obstruction B. it is found that she has been limping for the last several weeks. Sclerocornea B. Examination of the other eye for smaller tumors should always be performed. 22. Cataract C. On ophthalmic examination. genetic counseling is indicated. On examination. (See Chapter 621 in Nelson Textbook of Pediatrics. D. Genetic counseling is indicated for this family Removal of the eye may be required A CT scan may demonstrate calcium within the lesion A biopsy should be performed as soon as possible Explanation: This patient is most likely to have a retinoblastoma. and it is not associated with photophobia. Nasolacrimal tear duct obstruction C. and light sensitivity. Herpes simplex D. Infantile glaucoma Explanation: The classic findings in infantile glaucoma are enlarged corneas. C. Kawasaki disease is not associated with arthritis and usually causes conjunctivitis. Treatment of a large unilateral tumor may require enucleation. Uveitis Disorders of the Eye . His anterior segment is normal.) E. and his corneas appear large and cloudy. tearing.causes a posterior uveitis and is usually unilateral. Toxoplasmosis E. Biopsy of the suspected tumor is contraindicated. it does not cause a change in the size or clarity of the cornea. Patients with unilateral optic nerve abnormalities will demonstrate an afferent pupillary defect. Appropriate work-up may include a CT scan to demonstrate calcium within the eye. Kawasaki disease Question . The most likely diagnosis is: A. 17th ed. and his vision cannot be improved with glasses. Examination of the other eye should be performed with the patient under anesthesia Question . All of the following statements regarding this patient are correct Except: A. he is tearing.Nelson Self Assessments website 17th Edition 7 . as it may lead to spread of the tumor cells. A 20-mo-old girl is noted by her parents to have a unilateral white pupil. (See Chapter 620 ) C. Although a tear duct obstruction also causes tearing. (See Chapter 623 in Nelson) B. Funduscopic examination reveals a very large white mass that fills most of the eye. Retinal detachment Question . A 3-mo-old boy is brought in by his parents because he appears to be very light sensitive. Peters anomaly D. Because it can be a hereditary tumor. The most likely diagnosis is: A. (See Chapter 622 in Nelson) D. he is noted to have an afferent pupillary defect on the affected side. Optic nerve hypoplasia Explanation: Optic nerve hypoplasia may be associated with growth hormone deficiency and poor vision. Glaucoma E. A 4-yr-old boy who is being followed by his primary care doctor for short stature is found to have poor vision in one eye. 23. Strabismus B. 24. cloudy corneas. Infantile cataracts E. B. (See Chapter 625 in Nelson Textbook of Pediatrics.Nelson Self Assessments website 17th Edition 8 . and sinuses Ophthalmologic consultation Administration of a broad-spectrum antibiotic Surgical drainage of any abscess Determination of TSH and TRH blood levels Explanation: Although thyroid disease may manifest with unilateral proptosis.Question . Which of the following statements regarding children with traumatic hyphemas is most correct? A. Patients with a history of a hyphema are at risk for the development of glaucoma later in life. Blood within the anterior chamber can lead to corneal blood staining and/or glaucoma.) Question . Appropriate next steps in the diagnosis and treatment of this child could include all of the following Except: A. Traumatic hyphemas can be a significant cause for vision loss. Once the blood leaves the anterior chamber. 25. C. (See Chapter 624 in Nelson Pediatrics. E. Children with sickle cell disease are at increased risk for loss of vision Explanation: Children with sickle cell disease are at increased risk for vision loss following a traumatic hyphema. 17th ed. there is no longer any risk from the injury causing the hyphema E. it is not associated with fever and does not usually develop this rapidly. CT scan of the head. there is little risk of further complications C. 26. D. B. The other answers are appropriate steps in the management of a child with a preseptal or orbital cellulitis. 17th ed. If secondary bleeding does not occur by the second day. The blood itself produces no ill effects Disorders of the Eye . orbit. A 2-yr-old child presents with a 1-day history of unilateral proptosis and fever. Rebleeding is most common 3 to 5 days after the initial injury.) D. This is generally a benign condition B. thus. (See Chapter 630) B. High-dose oral amoxicillin Explanation: Although respiratory syncytial virus (RSV) may cause otitis media.Nelson Self Assessments website 17th Edition 1 . Diagnostic tympanocentesis is performed. the more likely it is that these pathogens will be found. No initial antibiotic treatment.The Ear Nelson Self Assessments website 17th Edition Question . On pneumatic otoscopy. He has not complained of ear pain. 3. Physical examination is unremarkable except for a bulging. translucent. whereas all children who tug at their ears don't have otitis media. and retracted. Which of the following treatment regimens is the most appropriate to institute? A. right acute otitis media Bilateral otitis media with effusion Normal left ear. Oral cefixime D. watchful waiting E. Oral azithromycin Question . coli and group B streptococci are common causative agents in neonatal otitis media. A 10-day-old infant develops irritability and rectal temperature of 38oC on the day of planned discharge from the NICU after an uneventful recovery from mild respiratory distress syndrome. The earlier in the first month of life plus the more complicated the neonatal course. retracted. A previously healthy 8-mo-old infant develops bronchiolitis. his left eardrum is pink. RSV infection should be treated as for a bacterial otitis media. (See Chapter 630 in Nelson Textbook of Pediatrics. Which of the following is the likeliest ear-related diagnosis? A. D. (See Chapter 630 in Nelson Textbook of Pediatrics. 17th ed. opaque. Not all children with otitis media complain of ear pain. On the fourth day of illness she is noted to have bulging. 2. right acute otitis media The Ear . C. Cytomegalovirus Question . opaque left eardrum.) E. Intramuscular ceftriaxone C. answer D would be appropriate. and immobile. 1. his right eardrum is reddish. it may be a co-pathogen with the typical bacterial causes of otitis media. 17th ed. Which of the following organisms is the one most likely to be found? A. right otitis media with effusion Explanation: The right ear has classic features on physical examination for an otitis media with effusion. Chlamydia trachomatis Group A streptococci Haemophilus influenzae type b Escherichia coli Explanation: E. D. Left otitis media with effusion.) E.5oC. B. with 3+/4+ mobility. B. white eardrums bilaterally. A 2½-yr-old boy presents with an upper respiratory tract infection and axillary temperature of 38. Bilateral acute otitis media Normal left ear. In some European countries. A lumbar puncture reveals normal cerebrospinal fluid. opaque. C. Treatment with -lactamase competitors or -lactamase-resistant penicillins is not effective. Production of -lactamase Production of streptolysin toxin Production of nitric oxide Alteration in penicillin-binding proteins of the cell wall Explanation: Antimicrobial resistance among pneumococci to penicillin is due to alterations of penicillin. Otitis media is equally prevalent in poor and in well-to-do children Explanation: Medically underserved children have a higher risk of otitis media than that documented for more affluent children. and retracted.Nelson Self Assessments website 17th Edition 2 . D. A 12-mo-old infant in day care develops severe bilateral acute otitis media. A 10-day course of amoxicillin A 10-day course of amoxicillin-clavulanate Short-course treatment with azithromycin Observation without treatment for at least 2 more mo Explanation: The finding of a retracted opaque immobile eardrum after successful treatment of otitis media is not uncommon and warrants close observation. Which of the following bacterial mechanisms is responsible for the organism's resistance? A. Otitis media tends to run in families Question . (See Chapter 630 in Nelson Textbook of Pediatrics.) E. 17th ed. On pneumatic otoscopy. 6. both his eardrums appear amber. A 2-yr-old boy is seen for his routine check-up. and both are immobile. Otherwise his ENT examination is unremarkable. Her infection continues despite administration of a succession of antibiotics. 5. C. The reason for this is not well established but could be related to frequent use of day care.) E. C. opaque. The peak prevalence of otitis media is in the first 2 years of life D. 17th ed. and other factors. the presence of smoking.) C. B. B. but his mother reports that he is not hearing as well as usual. Otitis media is more prevalent among boys than among girls B. Breast feeding provides protection against otitis media E. Referral for consideration of myringotomy and tube insertion The Ear . Alteration of the nucleus Question . D.Question . (See Chapter 630 in Nelson Textbook of Pediatrics. Which of the following treatment regimens is the most appropriate to institute at this time? A. He seems generally well. 4.binding proteins. All of the following statements regarding otitis media are correct Except: A. Tympanocentesis reveals the presence of penicillin-resistant Streptococcus pneumoniae. 4 weeks after an episode of bilateral acute otitis media that resolved uneventfully with antibiotic treatment. (See Chapter 630 in Nelson Textbook of Pediatrics. 17th ed. In most cases the abnormalities resolve spontaneously. His cognitive and speech development will be unaffected but his language development will be impaired The Ear . Explanation: Adenoidectomy is one important approach to management of a child with recurrent otitis media after failure of tympanostomy tubes. the effusion has cleared. Both his cognitive and his speech and language development will be impaired C. but his speech development will be impaired E. (See Chapter 630 in Nelson Textbook of Pediatrics. She breathes easily through her nose and does not snore. At present her eardrums are normal in appearance except for some atrophic scarring. The tubes were extruded at age 2 yr. 17th ed. Adenoidectomy has shown efficacy only during the first 2 yr of life C. 8.) D. 17th ed. Adenoidectomy has shown greater efficacy in boys than in girls E. The child's cognitive development will be unaffected but his speech and language development will be impaired B. 1 yr after onset. 7. She remained well for 6 months but then experienced several episodes of tube otorrhea. An otherwise healthy 12-mo-old boy develops bilateral otitis media with effusion in September that persists for 3 mo.) B. Which of the following statements will best help the parents decide about surgery? A. His cognitive and language development will be unaffected. The parents again decline surgery. The parents do not accept a recommendation for myringotomy and insertion of tympanostomy tubes. and a repeat audiogram the following April shows the same results as previously. This is true despite the absence of obstruction or significant enlargement of adenoidal tissue (for age). Adenoidectomy has failed to show efficacy in preventing recurrent otitis media Question . Her internist father is inquiring about the advisability of adenoidectomy. Adenoidectomy has shown efficacy only in children with enlarged adenoids D. which of the following outcomes should the parents anticipate as most likely at the time the child enters school? A. which was recommended on the golf course by an ENT colleague. Adenoidectomy has shown greater efficacy in preventing recurrent otitis media in children who have previously received tympanostomy tubes than in those who have not. A 4-yr-old girl has had repeated bouts of acute otitis media since early infancy. (See Chapter 630 in Nelson Textbook of Pediatrics. The effusion persists.Question . The effusion is still present at a checkup in July. Without further intervention. At age 10 mo she underwent bilateral myringotomy with tube insertion. An audiogram performed in December shows a pure-tone average threshold of 30 dB. Neither his cognitive nor his speech and language development will be impaired Explanation: This is an important observation that has been confirmed by many studies. and since then she has averaged 6 episodes of recurrent acute otitis media each year.Nelson Self Assessments website 17th Edition 3 . but in September. Always associated with ossicular abnormalities D. benign paroxysmal vertigo. All of the above Explanation: Down syndrome may be associated with conductive or sensorineural hearing loss. 9. Frequently associated with otitis media Usually related to motion sickness Most likely to be related to Meniere disease An uncommon symptom in children Explanation: Vertigo. tympanic membrane. The most common causes of otorrhea may be otitis externa and perforation of the tympanic membrane in children with otitis media. cholesteatoma. congenital ear defects. C. Otorrhea (purulent ear drainage) may be associated with all of the following Except: A. neuronitis. C. (See Chapter 627 in Nelson Textbook of Pediatrics. Narrow ear canals B. 17th ed.Nelson Self Assessments website 17th Edition 4 . Always associated with a hearing loss Question . A permanent condition related to nerve damage C. Ménière disease. 17th ed. Down syndrome (trisomy 21) is commonly associated with: A. Conductive hearing loss is: A. should not be confused with dizziness. 11. 10. Acute otitis media Question . Any pathologic condition of the pinna. external ear canal. Chronic ear fluid D. trauma. (See Chapter 626 in Nelson Textbook of Pediatrics. Common in children with chronic ear fluid Explanation: Otitis media is the most common form of acquired conductive hearing loss. 17th ed.) E. a sense of motion.) The Ear .) E. Vertigo in children is: A.) B. Vertigo may be associated with nystagmus and may be due to labyrinthitis. An uncommon complication of otitis media E. Speech delay E. 17th ed. Conductive hearing loss C. or ossicles can produce congenital or acquired conductive hearing loss. Presence of a tube in the ear Perforated eardrum Cholesteatoma Lyme disease Explanation: Lyme disease may cause cranial neuropathies (most often cranial nerve VII) but should not cause otorrhea. (See Chapter 626 in Nelson Textbook of Pediatrics. D. Unlikely when an intact tympanic membrane is present Question . B. or CNS diseases. 12. (See Chapter 627 in Nelson Textbook of Pediatrics. B.Question . D. Most likely to be inherited in autosomal dominant fashion B.Question . C. It facilitates excellent assessment of the mobility of the tympanic membrane and any pain associated with the applied pressure. 16. Caused by a genetic abnormality in less than 20% of cases D.) D. Most commonly caused by bacterial meningitis E. B. Pneumatic otoscopy is: A. Mutations of the connexin gene and other genetic syndromes (long Q-T syndrome). (See Chapter 627 in Nelson Pediatrics. 17th ed.) E. C. Used only to assist in diagnosis of infection B. especially if started before age 6 mo.) E. Used only if cochlear implantation cannot be performed The Ear . (See Chapter 627 in Nelson Pediatrics. An important office tool for diagnosis of ear fluid and negative middle ear pressure Explanation: This is a valuable method that all pediatricians should learn during residency. Whether all infants should be screened in the first week of life or before nursery discharge remains controversial. Usually placed when the child starts kindergarten Difficult for parents to deal with and require skilled nursing care Not well tolerated and require surgical placement over the mastoid bone Placed as early as age 2-3 mo when the deficit is diagnosed early Explanation: These are very well tolerated and helpful. 17th ed. 17th ed. 15. are responsible for many genetic causes of hearing loss. Hearing screening in infants is: A. 14. D. hearing aids are: A. Not recommended for acute otitis media Question . Reserved for the operating room to assist in ear tube placement C. Unlikely to detect severe or profound hearing loss Question . Most likely to be inherited as an autosomal recessive disorder when genetically transmitted Explanation: In about 40% of the cases. sensorineural hearing loss is due to an autosomal recessive disorder.Nelson Self Assessments website 17th Edition 5 . D. Performed by an audiologist to assess hearing E. An elective procedure for diagnosing the presence of middle ear fluid Performed only in healthy infants without risk factors for hearing loss Performed in all hospitals in the first week of life Most commonly performed using otoacoustic emissions (OAE) and/or auditory brainstem evoked response (ABR) Explanation: These are the most accurate and reliable tests. B.) C. (See Chapter 627 in Nelson Textbook of Pediatrics. 13. 17th ed. (See Chapter 630 in Nelson Textbook of Pediatrics. Sensorineural hearing loss in the United States is: A. In moderate to severe SNHL. Usually related to chronic ear infections Question . (See Chapter 627 in Nelson Textbook of Pediatrics. Facial cellulitis D. C. Brain abscess Question . Pneumococcal meningitis Explanation: Pneumococcal meningitis may be a complication of cochlear implants. 18.Question . Sinusitis C. 19. 17th ed.) Question . All of these indicate the need for referral for audiologic assessment Explanation: All of these indicate the need for audiologic assessment. C. (See Chapter 627 in Nelson Textbook of Pediatrics. 17th ed. B. No differentiated babbling or vocal imitation at age 12 mo No use of single words at age 18 mo Single-word vocabulary of 10 words or fewer at age 24 mo Less than a 100-word vocabulary. 17. Cochlear implants may be associated with: A. It can occur without swimming. 17th ed. B.) The Ear . All patients should be immunized with the currently available vaccines. D. E. D. or no evidence of two-word combinations. 20. at age 30 mo E. C.Nelson Self Assessments website 17th Edition 6 . Most speech sounds missed at normal conversational levels Language retardation Unvoiced consonant sounds missed Inattention All of the above Explanation: Unvoiced consonant sounds are missed even with slight hearing loss. Otitis externa is: A. B.) B. (See Chapter 629 in Nelson Textbook of Pediatrics. 17th ed. A hearing deficit of moderate loss is associated with an average sound threshold of 30-50 dB in combination with: A. Septic jugular vein thrombosis E. Commonly related to swimming Most commonly caused by Pseudomonas aeruginosa Best treated with topical antibiotic drops Treated with wick placement and drops if severe canal swelling is present All of the above Explanation: This is the classic swimmer's ear and is due to wetness with tissue maceration and secondary infection. E.) Question . (See Chapter 627 in Nelson Textbook of Pediatrics. All of the following indicate the need for referral for audiologic assessment Except: A. D. These are common. 17th ed. Exploration by probing C. 21. with an incidence of approximately 8 cases in 10. (See Chapter 628 in Nelson Textbook of Pediatrics. A 4-yr-old child who is new to your clinic has a small pitlike depression anterior to the helix and above the tragus. Computed tomography (CT) or magnetic resonance imaging (MRI) to evaluate for possible branchial cleft cyst D. Referral for surgical excision E. Observation only Explanation: A pitlike depression just anterior to the helix and above the tragus may represent a cyst or an epidermis-lined fistulous tract. Which of the following is the recommended initial management? A.Nelson Self Assessments website 17th Edition 7 . There are no symptoms.000 children but do not require surgical removal unless there is recurrent infection.Question . Referral for chromosome analysis The Ear .) B. 2. All of the following statements about ethics are true except: The age at which a competent patient may legally exercise voluntary informed consent for medical care varies from state to state The decision for a DNAR order does not imply a decision to withhold other aspects of providing medical treatment The decision to withhold life-sustaining medical treatment implies an intent or choice to hasten a child's death Explanation: (See Chapter 2 in Nelson Textbook of Pediatrics.9/1000 live births in 2000 Postnatal infant mortality in the United States was 2. For which age group are unintentional injuries not the leading cause of death? 1-4 yr 5-9 yr Under 1 yr Explanation: (See Chapter 1 in Nelson Textbook of Pediatrics. 17th edition.) Medically beneficial treatment can be withheld from a severely disabled infant who is permanently unconscious Teenagers may not be able to support themselves but still be competent to consent to health care .The Field of Pediatrics Question . over 25% of children under 18 yr were living with one parent Infant mortality in the United States was 6. 1.530 for a family of four persons) Question . 17th edition. 18% of all children in the United States lived in families with income below the poverty level ([dollar]16. 17th edition. Which of the following statements is false? Children (0-18 yr) make up about 50% of the population Explanation: (See Chapter 1 in Nelson Textbook of Pediatrics. 3.) 10-14 yr 15-19 yr Question .) In 1999.3/1000 live births in 2000 In 1998. 2 mo. What is an optimal schedule for health supervision visits of a child during the first year of life? 1 mo. 5. 12 mo 1 wk. 1 mo. 6 mo. 2 mo. 12 mo 1 wk. Which of the following statements is false? The World Health Organization (WHO) has a program called Integrated Management of Childhood Illness (IMCI). 4 mo.000 children are adopted into American families from abroad each year. 4. Explanation: (See Chapter 4 in Nelson Textbook of Pediatrics. 3 mo. 6 mo. 12 mo Prenatal. Question . 12 mo Explanation: (See Chapter 5 in Nelson Textbook of Pediatrics. 6 mo. 12 mo 1 mo. 1 mo.) Question . Which of the following statements is true? Children are at the forefront of societal and cultural change related to health and well-being Ethnicity encompasses culture Explanation: (See Chapter 3 in Nelson Textbook of Pediatrics.Question . 1 wk. 1 mo. Economic conditions and family resources drive health care decision-making.) About one-third of children younger than 15 yr in developing countries either have or are at risk for impaired learning. 9 mo. 17th edition. 3 mo. which focuses on a single medical condition providing all levels of care. 6 mo. 17th edition. 9 mo. 17th edition. 6. Most disasters occur in developing countries and are due to natural events such as hurricanes.) Variability within any cultural or ethnic group often exceeds that between groups The use of complementary and alternative medicine occurs in educated and uneducated affluent groups Children are active shapers of their cultural environment rather than passive recipients of socialization . About 18. 6 mo. Her growth and development are normal. which is the most likely explanation for this behavior? Diminishing breast milk production Increased growth velocity Separation anxiety Explanation: (See Chapter 5 in Nelson Textbook of Pediatrics. Which of the following statements is false? Ninety percent of children in the early 21st century are born into the developing world The mortality rate of children under 5 yr worldwide is 80-90/1000 Between 1980 and 2000 the percentage of children immunized in developing countries stayed constant Explanation: (See Chapter 4 in Nelson Textbook of Pediatrics. A 9-mo-old girl has repeated night awakenings after going to sleep and has been unable to return to sleep without breastfeeding.) Slapping his hand . Part A Of the following. prior to which she slept the entire night without waking or requiring middle-of-the-night feedings. Of the following. He attempts to run away from her but she quickly grabs his hand before he darts out in front of a car. This behavior has been present for the past month. She is exasperated and scared by his actions.Question . 17th edition. 7. 8. She breastfeeds 6-7 times/day and eats a variety of solid foods.) Feeding schedule: trained night feeder Nightmares Question .) Approximately one third of children under 15 yr in developing countries either have or are at risk for impaired learning The Integrated Management of Childhood Illness (IMCI) program uses any point of contact as an opportunity to assess the overall health of the child and family in developing countries Question . 17th edition. 17th edition. A 2-yr-old boy refuses to hold his mother's hand when walking across the parking lot of a shopping mall. 9. which technique is most effective in addressing the behavior? Verbal reprimand Spanking Scolding Time-out Explanation: (See Chapter 5 in Nelson Textbook of Pediatrics. 17 edition.) Be done at a point in time removed from the incident Question . The sensitivity of carefully performed observation.) . Part B To be effective. 17th edition. 17th edition.Question . 11.) 100% 50% Question . history. Most observation data that pediatricians gather during an acute illness should focus on assessing which of the following? Eye behavior Gross motor ability Fine motor ability Work of breathing Response to stimuli th Explanation: (See Chapter 6 in Nelson Textbook of Pediatrics. and physical examination in the child with an acute illness is: 70% 60% 90% Explanation: (See Chapter 6 in Nelson Textbook of Pediatrics. time-outs should: Be accompanied by verbal reprimands Be accompanied by punishment Last at least 5-10 min Last 1 min per year of age Explanation: (See Chapter 5 in Nelson Textbook of Pediatrics. 10. ) Pulmonary auscultation Range and ease of extremity motion Cardiac auscultation Question . Paradoxic irritability is a finding from observation that is helpful in diagnosing: Pneumonia Urinary tract infection Septic arthritis Meningitis Explanation: (See Chapter 6 in Nelson Textbook of Pediatrics. The white blood cell count and differential is most helpful as a screening test in which of the following entities? Meningitis Bacteremia Explanation: (See Chapter 6 in Nelson Textbook of Pediatrics.) Bacteremia Question . 17th edition. which of the following components of the clinical assessment is best done on the examination table? Eye examination Abdominal examination Explanation: (See Chapter 6 in Nelson Textbook of Pediatrics. 12.Question . 13. 14.) Pneumonia Urinary tract infection Septic arthritis . In the acutely ill child younger than 36 mo. 17th edition. 17th edition. which is not considered an appropriate option to either withdraw or withhold? Artificial hydration and nutrition Adequate sedation and analgesia Explanation: (See Chapter 2 in Nelson Textbook of Pediatrics. which is the most important factor for a pediatrician to consider when evaluating a parent's decision about the health care for a 7-yr-old child? The expressed wishes of the child What a reasonable parent would do in that situation The pediatric clinician's independent professional obligation to act in a child's best interest Explanation: (See Chapter 2 in Nelson Textbook of Pediatrics. A parent must make decisions about a child's health care based on a physician's recommendations. a competent adult can make a decision to accept or reject a physician's recommendation regardless of the impact on his or her health. At times. 17th edition. In seeking a child's assent.Question . given the medical recommendation that there are no further treatments that offer any hope of extending the child's life. The range of acceptable decisions that a parent can make. Of the following. Of the following treatments. which of the following is not recommended? Help a child understand his or her condition Provide guidance to the child on what is best for him or her Explanation: (See Chapter 2 in Nelson Textbook of Pediatrics. 17. Generally. 15.) Antibiotics Mechanical ventilation Inotropic medications . 17th edition. 16. 17th edition. however. is restricted. the assent of a child to a proposed medical intervention is necessary to assure appropriate treatment.) Tell a child what he or she can expect Assess a child's understanding and whether a child feels pressured to assent Solicit a child's willingness to participate Question .) The ability of the parent to pay for the recommended medical care The opinions of other pediatric clinicians Question . A parent has made a decision not to pursue further treatment for a child's condition. Of the following. 19. and such testing will thus not be available in the future . except under specific circumstances. and families Case consulting Responding to parental requests for ethical advice Triage of scarce hospital resources Explanation: (See Chapter 2 in Nelson Textbook of Pediatrics. 18. As many adults choose not to be tested for late-onset disorders.) Question . which is the best reason for genetic testing of late-onset disorders in childhood? The parents need to make decisions concerning long-term savings for educational expenses Scarce family resources require triaging educational support to that child without the genetic condition Such testing will result in interventions that have been shown to reduce morbidity and mortality when initiated in childhood Explanation: (See Chapter 2 in Nelson Textbook of Pediatrics.) The parents want to seek suitable marriage partners and need to know the child's carrier state The family health insurance is set to expire. 17th edition. patients. 17th edition. Many health care institutions have established ethics committees made up of a diverse membership. which is generally not a function of the ethics committee? Drafting a review of institutional policy Educating health care professionals. Of the following. Genetic testing of childhood and adolescents for late-onset disorders is generally inappropriate. we cannot assume that a child would want or would benefit from similar testing.Question . ) 4 mo . for 7.5/2 Age (years) × 2 + 8 Explanation: The formula in choice A is used for 7. 1. C.Growth and Development.g.to 12-yr-olds in pounds.) Age (months) + 9/2 (Age [years] × 5 + 17) (Age [years] × 7 + 5) Question . and E.. This peak is typically at age: 2 wk 4 wk 6 mo 6 wk Explanation: Crying may or may not be in response to obvious stimuli (e. 17th ed. (See Chapter 15 in Nelson Textbook of Pediatrics. D. Behavioral states in the newborn period include: Quiet and active sleep Drowsy and alert states Fussy and crying states All of the above Explanation: Newborn infants have six characteristic organizational behavioral states.) None of the above Question . need for a diaper change). 2. 17th ed. A normal infant may cry for up to 3 hr/day during the developmental peak time of this behavior. (See Chapter 9 in Nelson Textbook of Pediatrics. (See Chapter 10 in Nelson Textbook of Pediatrics. 17th ed. 3.to 12-yr-old children. The best formula to approximate average weight (kg) for a 4-year-old is: Age (years) × 7 . for 1. Set 1 Question .to 6-yr-olds in pounds. for 3.to 12-mo-old infants. ) 60 min for each feeding Question . Object permanence appears at approximately 8 mo of age. 5. which is noted at 8 to 9 mo. out of mind" is the characteristic response of a 2-mo-old. The response to a ball dropped in front of the child is: Staring momentarily at the spot the ball was dropped from Explanation: "Out of sight. This is also called object constancy. The best feeding protocol for a temperamentally irregular infant is: A fixed schedule One based on the parents' schedule Every 1-2 hr One based on demand Explanation: Demand feedings prevent periods of hunger and episodes of being fed while not being hungry for a child with an irregular rhythm.) Eyes lowering as the ball descends Crying when the ball hits the ground Smiling at the game of hide-and-seek None of the above Question . 17th ed. Object permanence is not present at age 2 mo. (See Chapter 10 in Nelson Textbook of Pediatrics. along with increasing mobility. 17th ed. (See Chapter 11 in Nelson Textbook of Pediatrics. enables an infant to explore the environment. (See Chapter 10 in Nelson Textbook of Pediatrics.) 10-12 mo . 4.Question . The ability to manipulate small objects with the pincer grasp is usually noted at age: 0-2 mo 3-5 mo 6-7 mo 8-9 mo Explanation: The pincer grasp. 6. 17th ed. 7. 17th ed.Question . (See Chapter 10 in Nelson Textbook of Pediatrics. After age 4 years. The probable age of a developmentally normal child who is just able to sit without support. 17th ed. 17th ed. a spontaneous change in handedness should lead to the suspicion of a central nervous system lesion.g. Transitional objects include: Training underwear Shoes without laces Cups with a special drinking spout Blankets and teddy bears Explanation: Transitional objects help toddlers (18-24 mo) cope with separation (e. can transfer objects from hand to hand. at daycare).) 11 mo Question ..) None of the above Question . (See Chapter 11 in Nelson Textbook of Pediatrics. 9. at nighttime for sleep. Handedness is usually determined by age: 2-4 mo 6-12 mo 15-18 mo 20-24 mo 36-48 mo Explanation: Handedness should not be attempted to be modified because this leads to frustration. 8.) . and speaks in a monosyllabic babble is: 3 mo 4 mo 9 mo 6 mo Explanation: Each pediatrician should learn key developmental milestones such as these. (See Chapter 12 in Nelson Textbook of Pediatrics. with baby sitter. A mother brings her 6½-mo-old circumcised boy to you for a "sick" visit. 10. 11. 12. 17th ed. Whether this is related to separation anxiety or something else (teething?) is not clear (edentulous babies wake up. when applied to the child's height. The child's history is otherwise normal. (See Chapter 10 in Nelson Textbook of Pediatrics. The best approach for parents to help a preschool child overcome monster fears is to: Rationalize that monsters do not exist Read books that do not have monsters in them Have the pediatrician explain that monsters are make-believe Use "great power" like monster spray to keep monsters away Explanation: It is impossible to rationalize away a preschool child's fear of monsters. and the child appeared well. includes all of the following except: Genetic endowment Personal eating habits Access to food . The biopsychosocial model of development. including a DTP immunization. D is wrong because DTP reactions occur 4-36 hr after the shot. You saw the child 2 wk previously for health maintenance.) Reassure the mother that the behavior will pass because it is a reaction to the DTP shot Question .Question . especially when she leaves him. Choice B would be unnecessary because increased food intake does not relieve night fussiness. and physical examination reveals no problems. The most appropriate approach to management is to: Perform urinalysis and obtain a complete blood count to rule out urinary tract infection Request that the mother feed the infant more Reassure the mother that the behavior is normal and will pass in time Explanation: Waking up at night (if in fact the baby had already slept through part of the night) at 6 to 8 mo is common behavior. 17th ed. not 2 wk. The mother's complaint is that the baby is waking up every night and is fussy during the day. Choice A would be highly unlikely. too). because 6½-mo-old circumcised boys who have grown normally rarely contract urinary tract infections.) None of the above Question . (See Chapter 12 in Nelson Textbook of Pediatrics. both means of infant feeding are equally effective. attention span. (See Chapter 7 in Nelson Textbook of Pediatrics.) Biology influences temperament It is a pattern of the child's responses It is relatively resistant to parents' attempts to modify It helps parents understand the child's behavior without guilt Question . All of the following statements regarding a child's temperament are true except: Temperament is absolute and stable throughout the life span Explanation: Temperament is only moderately stable over time." "she is never hungry at the same time. quality of mood. (See Chapter 7 in Nelson Textbook of Pediatrics. 17th ed." This child is best described as: Autistic Having a specific temperament Explanation: The seven characteristics of temperament have a wide range of responsiveness.) Question . breast milk has specific biologic advantages. rhythmicity. adaptability. Nonetheless. in less advantaged environments. The parents of a 3-yr-old girl report that "she ran before she walked. distractibility. intensity of reaction. approach and withdrawal. (See Chapter 7 in Nelson Textbook of Pediatrics. these characteristics are activity level.Parents' beliefs Differences between breast milk and formula Explanation: In general. 17th ed. 14." and "she goes from toy to toy. and a mellow 2-yr-old does not always equate to a mellow 22-yr-old.) Having attention deficit hyperactivity disorder Having developmental pervasive disorder Being deaf . threshold of responsiveness. 17th ed. all other socioeconomic factors held constant. and persistence. The biopsychosocial model helps combine the nature versus nurture aspects of previous theories of development. 13. The six behavioral states of the neonate include all of the following except: Quiet sleep Active sleep Drowsiness Alertness Colic Explanation: Colic is not a neonatal state and does not even occur during the neonatal period. 15.) It represents a seizure It is due to apnea Question . 16. (See Chapter 9 in Nelson Textbook of Pediatrics. 17.) Fussiness Crying . 17th ed. (See Chapter 9 in Nelson Textbook of Pediatrics. 17th ed. Which of the following statements regarding this lack of activity is true? It is suggestive of sepsis It is suggestive of sedation It is normal Explanation: This normal sleep after a 40-min period of social interaction is a great time to continue the bonding process.) 15-24 inches 24-30 inches 30-36 inches Question . 17th ed. A newborn infant spends about 40 min with the mother but then falls asleep and does not respond to the mother's voice. The visual acuity of a newborn permits recognition of an object held at a distance of: 1-2 inches 8-12 inches Explanation: The near-sighted neonate has a fixed focal length of 8-12 inches. The newborn also has a visual preference for faces. (See Chapter 9 in Nelson Textbook of Pediatrics.Question . the infant should gain 30 g/day Preterm infants take longer to regain birthweight The high fat content of colostrum enhances weight gain in the first week of life Explanation: Colostrum has a high protein content. All of the following statements regarding growth in the first month of life are true except: Weight may decrease 10% in the first week Weight should equal or exceed birthweight by 2 wk Once gaining weight.) . 20. (See Chapter 10 in Nelson Textbook of Pediatrics. The best formula for approximating average weight in kilograms for a 9-mo-old is Age (mo) + 9/2 Explanation: B is best for 9-mo-olds in pounds. 18. 17th ed. Mature milk has a higher fat and lower protein content than those of colostrum. (See Chapter 10 in Nelson Textbook of Pediatrics.Question . (See Chapter 10 in Nelson Textbook of Pediatrics.5 .6 Question .5 + 30 Age (yr) × 6 + 77 Explanation: A is in inches. 19. 17th ed.) Age (yr) × 7 + 5 Age (yr) × 5 + 7 Age (yr) × 2. The best formula for approximating average height in centimeters for a 4-yr-old is Age (yr) × 2. 17th ed.) Age (mo) + 11 Age (yr) × 2 + 8 Age (yr) × 5 + 17 Age (yr) × 7 + 5 Question . walks alone. 22.) Peaking at 6 wk Peaking at a total of 3 hr/day Crying in response to obvious stimuli Crying when no stimulus is obvious Question . 21. (See Chapter 10 in Nelson Textbook of Pediatrics. exploration of small objects. 17th ed. speaks one real word." and bangs two cubes is: 7-8 mo Explanation: These milestones correspond respectively to control of exploration. and comparison of objects. 17th ed. 17th ed. exploration labeling. response to tone.) 10-12 mo 12-15 mo 3-4 mo 15-18 mo . The probable age of a child who rolls back to front. The probable age of a child who scribbles. Crying in the first 2 mo of life is characterized by all of the following except: Teething Explanation: Teething does not occur until approximately 5-6 mo.Question . (See Chapter 10 in Nelson Textbook of Pediatrics. (See Chapter 10 in Nelson Textbook of Pediatrics.) 16 mo 20 mo 24 mo Question . and pretends to drink from a cup is: 8 mo 13 mo Explanation: These milestones correspond respectively to visual-motor coordination. has a thumb-finger grasp. self-inhibits to "no. and symbolic thought. 23. (See Chapter 10 in Nelson Textbook of Pediatrics.) Appearing autonomous Eating finger foods Turning away from the spoon Holding a spoon Question .) Accelerating to a rate of 20 g/day Demonstrating no change compared with rate between 0 and 2 mo Question . 24. and social development. Feeding between 6 and 12 mo of age is characterized by all of the following except: Being willing to be fed by a stranger Explanation: Indeed. (See Chapter 10 in Nelson Textbook of Pediatrics. and dresses and undresses is: 15 mo 24 mo 30 mo 18 mo 60 mo Explanation: This is normal. 17th ed. 17th ed. The probable age of a child who skips. 25. names four colors.) . Growth between 3 and 4 mo of age is best characterized as: Accelerating to a rate of 45 g/day Slowing to a rate of 10 g/day Slowing to a rate of 20 g/day Explanation: This is a normal response that does not cause the child to cross growth percentiles in a growth chart. appropriate motor. 17th ed. stranger anxiety may start to set in at this time. (See Chapter 11 in Nelson Textbook of Pediatrics.Question . 26. language. Early walking suggests: Preoccupation with objects Advanced social development Advanced language development High-activity type Explanation: More active types tend to walk early. 17th ed. (See Chapter 12 in Nelson Textbook of Pediatrics. 29. language increases.Question . such that language is dependent on the environment and verbal interactions with adults. as a rule. 17th ed.) Delayed language may signify deafness Delayed language may signify mental retardation . The other choices are incorrect.) Independent of the environment Independent of the number of questions asked the child by adults Based on the ABCs Question . (See Chapter 11 in Nelson Textbook of Pediatrics. (See Chapter 12 in Nelson Textbook of Pediatrics. Between 2 and 5 yr of age. the number of words in a sentence is: Based on knowledge of numbers Equal to the age of the child in years Explanation: A 2-yr-old has two-word sentences.) Spasticity Question . 17th ed. 27. 28. All of the following statements regarding language development are true except: Deaf children may create their own language The basics for language may be "hard-wired" in the brain Language has no role in behavior regulation Explanation: Language plays a critical part in the regulation of behavior as children internalize speech. (See Chapter 13 in Nelson Textbook of Pediatrics. 3.5 cm 5 kg. (See Chapter 13 in Nelson Textbook of Pediatrics. 17th ed. 30. 5 cm 1.5 kg. 6 cm Explanation: 3. 5 cm Question .Question . Word-finding difficulties may result in all of the following except: Difficulty in expressing feelings Difficulty in verbal self-defense Frustration Success in English class Explanation: Language-based classes produce difficulties for children with word-finding difficulties. 17th ed.) Physical acting out t .) 6 kg. 10 cm 10 kg. 31.5 kg. Growth during the years between 6 and 12 yr is characterized by annual weight and height increments of: 3.5 in) per year are the average increments in this time period.5 kg (7 lb) and 6 cm (2. with absence of empathy.) Sedation Question . with onset usually before 30 mo of age. Neuroleptic malignant syndrome (malignant hyperthermia) and tardive dyskinesia are rarer complications. At daycare. teeth grinding. The most likely diagnosis is: Attention deficit hyperactivity disorder Dysthymic syndrome Deaf-mutism Autism Explanation: Autism is a disease of unknown cause and is more common in males. sedation.) Cerebral palsy Question . 17th edition. 17th edition. (See Chapter 21 in Nelson Textbook of Pediatrics. he has not made any friends. Neuroleptic antipsychotic agents produce all of the following unwanted side effects except: Bradykinesia Hyperthermia Tardive dyskinesia Inappropriate secretion of ADH Explanation: Common complications of neuroleptic antipsychotic agents include extrapyramidal symptoms (Parkinson-like syndrome). Head banging. (See Chapter 27 in Nelson Textbook of Pediatrics. 3. 17th edition. 2. hair twirling. It is characterized by the symptoms noted in this patient. A 4-yr-old boy is noted to have stereotypic body movements and poor verbal and nonverbal communication. 1.Psychologic Disorders. Set 1 Question . rocking. (See Chapter 27 in Nelson Textbook of Pediatrics. thumb sucking. and nail biting all are: Habit disorders that probably relieve tension Explanation: The belief that choice B is correct leads to repeated attempts to "break" children (a distressing term and concept). and anticholinergic symptoms. probably in effect reinforcing the behavior.) Behavior problems that are easy to cure in children Evidence of insecurity in the majority of children and poor parenting by their parents Tics . is a rare condition in children. but research has shown that drugs that increase dopaminergic action precipitate or worsen both tics and Tourette syndrome. The syndrome can be fairly well managed with haloperidol.) Anger within the family . 17th edition.Question .m. Neither the use of antipsychotic medications nor overeating after 7:00 p. It is more common in first-degree relatives of patients with Tourette syndrome than in the general population and affects boys three to four times more frequently than girls. a dopaminergic antagonist. The incidence of night terrors is said to be between 1% and 4% and is greater in boys. Many environmental precipitants have been noted to serve as emotional stress sources. tachypnea. Sleepwalking may occur during night terrors and may put a child at risk for injury. There is a familial pattern in the development of the symptoms. The cause is uncertain. has ever been shown to be associated with night terrors. Which of the following statements about Gilles de la Tourette syndrome is true? It is characterized by tics and coprolalia Explanation: Gilles de la Tourette syndrome. It is characterized by multiple tics. compulsive barking. They usually begin in the preschool years. tachycardia). (See Chapter 20. sweating. Night terrors are associated with: REM sleep Overeating after 7:00 p. A child having night terrors is confused and disorientated and shows signs of intense autonomic activity (labored breathing. 5. Anecdotal reports in the literature suggest that the serotonin reuptake inhibitors are also efficacious in its treatment. which has a lifetime prevalence rate of 0. and febrile illnesses may serve as precipitating factors. 17th edition. (See Chapter 21 in Nelson Textbook of Pediatrics.m.5 per 1000 persons. The use of antipsychotic medication Inception in preschool years and occasional somnambulism Explanation: Night terrors most commonly occur during stage IV. 4.5 in Nelson Textbook of Pediatrics. and shouting obscene words (coprolalia). dilated pupils.) It is characterized by tics and encopresis It is treated with haloperidol and methylphenidate It is a common disorder of childhood It affects girls more often than boys Question . deep sleep. armed robbery. Among preadolescents. The most effective results have been obtained with parent training management. Little is known about the antecedents of each of these subtypes or the outcome of patients suffering from them. jumping from heights is the . 6. Conduct disorder in childhood and adolescence is associated with all of the following except: Antisocial behavior Criminality in the father Physical abuse Marital discord within the home Mental retardation Explanation: Conduct disorder is a distinct clinical entity manifested by several different antisocial behaviors: stealing. property destruction. Many different approaches have been used in the treatment of children and adolescents with aggressive behavior.Question . a parent. and repeated attempts to run away from home. antisocial behavior. 17th edition. 7. intermittent antisocial behaviors. truancy. fire setting. use of weapons while fighting. a sibling. destructive behavior. lying. physical abuse within the home. Firearms serve as the major method of death in adolescent suicide. physical cruelty to others. rape. in which parents are trained directly to promote prosocial behaviors within the home and to place reasonable limits on unwanted. and marital discord within the home. cruelty to animals.) Question . substance abuse. and delinquency. Many argue that conduct disorder is not a unitary illness but instead comprises three different syndromes characterized primarily by aggression. and a family history of depression and suicide are all closely related to both suicide attempts and completed suicides in both children and adolescents. or another first-degree relative has previously shown overt suicidal behavior. In more than one third of suicides. and delinquency. Risk factors associated with the development of conduct disorders include antisocial behavior within family members. Fifteen to 40% of completed suicides are preceded by other suicide attempts. (See Chapter 25 in Nelson Textbook of Pediatrics. easy access to firearms. Completed suicides in childhood and adolescence may be associated with all of the following except: Previous suicide attempts Alcohol or drug abuse A history of depression and suicide within the family Easy access to firearms Perfectionism within the classroom Explanation: Previous suicide attempts. criminality in the father. Her handwriting is legible. but in her own writing she makes multiple errors and mistakes in punctuation and capitalization. 17th edition. (See Chapter 29 in Nelson Textbook of Pediatrics. 9. Many children or parents do not admit to this problem. An 11-yr-old child has excellent ideas in a class discussion. but what she records on paper is primitive and unsophisticated. Her teacher has noticed that this girl has trouble following directions. She is skilled in art and so far has performed well in arithmetic. This girl most likely is having problems with: Expressive language Graphomotor production Ideation Attention Simultaneous retrieval memory Explanation: Simultaneous retrieval memory defects are depicted by the history of the child described in the question. Although perfectionism in the classroom has been shown to be associated with specific types of anxiety.) A psychiatric assessment to rule out depression A neurologic examination Question .) Question . 8. She can spell well in isolation and understands rules of punctuation and capitalization. Which of the following diagnostic procedures is most likely to yield useful findings in this child? An attention deficit questionnaire An intelligence test A language evaluation Explanation: The question offers a classic example of a language problem. A child in the third grade has problems with spelling and reading. particularly in a child with partial understanding.most common method.) . (See Chapter 29 in Nelson Textbook of Pediatrics. 17th edition. no correlation has been shown between perfectionism and suicidal ideation or suicidal behavior. 17th edition. (See Chapter 24 in Nelson Textbook of Pediatrics. but writing is painfully slow. She appears very quiet and confused in class. Her mind seems to wander whenever the teacher tells a story or explains something complicated. (See Chapter 17 in Nelson Textbook of Pediatrics.Question . Manifestations reappear when the involved parent returns to the hospital. (See Chapter 19 in Nelson Textbook of Pediatrics. as part of psychosomatic illness. 12. 17th edition. a patient's symptoms disappear if the parent goes home. one should suspect: Immaturity Embarrassment Negativism Fearfulness Explanation: Children may be frightened and react to fear with a personal manner of withdrawal or poor cooperation. Somatoform disorders. (See Chapter 35 in Nelson Textbook of Pediatrics. 17th edition.) Hypochondriasis Pain disorders Question .) Oppositionism . include all of the following except: Conversion reaction Asthma Explanation: Asthma may be exacerbated by psychologic factors and is therefore a psychophysiologic disorder. 11.) An attentive parent caregiver who never goes home An unworried parent caregiver Poor response to therapy Doctor shopping Question . Münchausen syndrome by proxy is characterized by all of the following except: Recurrent illness that cannot be explained Statements from experienced pediatricians that they have never seen such a case Symptoms that disappear with the parent present Explanation: In the Münchausen by proxy syndrome. 17th edition. 10. When a 7-yr-old child fails to cooperate with care in the hospital. 15. 16. ashamed. 17th edition. (See Chapter 18 in Nelson Textbook of Pediatrics. 17th edition.) Question .) Mistrust Negativism Oppositionism Aggression Question . aggression.Question . and learning problems." They may be angry. anxiety. 17th edition. Psychosocial problems may manifest as disturbances in: Feelings Body function Behavior Performance All of the above Explanation: Disturbances may manifest as depression.) Head trauma Mental retardation Epilepsy Prematurity Encephalitis . If a parent does not appear readily reassured by the diagnosis or treatment plan. (See Chapter 27 in Nelson Textbook of Pediatrics. one should suspect: Hidden anxiety Explanation: Parents often hold back questions that are highly charged or that may appear "stupid. (See Chapter 17 in Nelson Textbook of Pediatrics. Psychiatric disorders are more common than in the general population of children in all of the following except: Smart students Explanation: An important aspect of these conditions is the capacity of the parents to adjust and cope. 13. or uncomfortable in asking these questions. ) Being more common in males . Conversion reactions are best characterized by: Sudden onset Traceability to a precipitating event Involvement of special senses Pseudoseizures Abrupt end All of the above Explanation: The loss or alteration of function without a demonstrable organic cause defines a conversion reaction. 17th edition. 17th edition. 18.) Wetting 2 times per week for any 3 mo in a year Not being dry at 3 yr of age Not being dry at 4 yr of age Wetting at 5 yr of age on two occasions Question . Nocturnal enuresis is described by all of the following except: Being primary or secondary Having a strong genetic component Occurring at all stages of sleep When primary. (See Chapter 19 in Nelson Textbook of Pediatrics. 19. (See Chapter 20 in Nelson Textbook of Pediatrics. 17th edition. a type of somatoform disorder. being associated with emotional disorders Explanation: There is no higher rate of emotional disturbances in agematched enuretic and nonenuretic children. it may be defined as wetting that produces clinically significant distress for the child. Enuresis is defined as: Wetting 2 times per week for 3 consecutive mo Explanation: Alternatively.) Question . (See Chapter 20 in Nelson Textbook of Pediatrics.Question . 17. Treatment of enuresis should include all of the following except: Enlisting the cooperation of the child Having the child void before retiring Using alarms Having the child launder the soiled sheets Waking the child repeatedly Explanation: Repeated waking may be beneficial for a few children but creates added stress and hostility.Question . (See Chapter 20 in Nelson Textbook of Pediatrics.) Question .) Tangier disease Narcolepsy . The child has also had problems staying awake in preschool and has had behavioral problems. Physical examination of the child reveals large. 17th edition. 21. 17th edition. 20. pink. The most likely diagnosis of the patient in Question 21 is: Tonsillitis Peritonsillar abscess Obstructive sleep apnea syndrome Explanation: Obstructive sleep apnea syndrome (OSAS) commonly presents with airway obstruction during sleep with resultant sleep disturbances at night and daytime sleepiness. (See Chapter 20 in Nelson Textbook of Pediatrics. A 5-yr-old is noted by the parents to snore at night. nonexudative tonsils. 22.) Ambulatory apnea monitoring Telemetry Arterial blood gas analysis Question . The most appropriate next step is: Laryngoscopy Polysomnography Explanation: Polysomnography reveals episodes of apnea and hypoxia. (See Chapter 20 in Nelson Textbook of Pediatrics. The father also snores. 17th edition. 17th edition.) Thumb sucking . however. (See Chapter 20 in Nelson Textbook of Pediatrics. (See Chapter 21 in Nelson Textbook of Pediatrics. it is probably not a true habit in that it is not regarded as a tensionrelieving activity.) Tracheostomy Parapharyngeal muscle surgery Theophylline Bilevel positive airway pressure Question . 23. The appropriate therapy for severe obstructive sleep apnea syndrome is: Adenotonsillectomy Explanation: Removal of the hypertrophied tissue relieves the symptoms of OSAS. 24.) Question . 17th edition. All of the following are considered habit disorders except: Tics Bruxism Trichotillomania Stuttering Explanation: Stuttering is often discussed with habit disorders. Risk factors for obstructive sleep apnea syndrome include all of the following except: Retroposition of the mandible Small triangular chin Long oval face Long or soft palate All of the above Explanation: All of the answers are risk factors as well as the more obvious such as Pierre Robin syndrome and Prader-Willi syndrome.Question . 25. (See Chapter 20 in Nelson Textbook of Pediatrics. 17th edition. 26. (See Chapter 22 in Nelson Textbook of Pediatrics. A third-grade student refuses to go back to school after the winter break. 17th edition. The most likely diagnosis is: Stranger anxiety School anxiety Stranger reaction Separation anxiety disorder Explanation: Parents often consciously or unconsciously encourage this intense fear of separation because of a fear that something will happen to the caregiver. and muscle pain. 17th edition.) Question . 28. bellyache.) Disappearance during sleep Possible occurrence following encephalitis Normal EEG Question . but you notice the child is very clingy to the mother. She now needs her mother to go to sleep with her and complains of headache. (See Chapter 21 in Nelson Textbook of Pediatrics. transient amnesia after the tic Explanation: This would be more compatible with a minor seizure.Question .) Narcolepsy . Findings on physical examination are totally normal. (See Chapter 21 in Nelson Textbook of Pediatrics. 17th edition. Tics are characterized by all of the following except: Difficulty in controlling behavior Occurrence of brief. 27. All of the following are true of Tourette syndrome except: It is made worse by drugs that increase dopaminergic action Some patients have pediatric autoimmune neuropsychiatric disorder Some patients have oppositional defiant disorder Lyme disease may sometimes mimic Tourette syndrome Management with haloperidol or pimozide is often unsuccessful Explanation: Both drugs provide a fair degree of relief. ) Question . 17th edition. and poor concentration.) Delay of onset until after adolescence Absence of hallucinations No risk of depression in adulthood None of the above . (See Chapter 22 in Nelson Textbook of Pediatrics. 17th edition. (See Chapter 23 in Nelson Textbook of Pediatrics. (See Chapter 23 in Nelson Textbook of Pediatrics. Major depression in children may be characterized by: A strong genetic component Explanation: Twin studies show a strong genetic component to major depression. 17th edition. Major depression is characterized by: Weight loss Weight gain Insomnia Hypersomnia Dysphoria All of the above Explanation: Major depression may also include manifestations of fatigue.) Excessive checking of locks Question . agitation or retardation. 29.Question . Obsessive-compulsive disorder may be associated with all of the following except: Overconcern with body wastes Prior group A streptococcal infection Excessive fears A need for sameness Increased metabolic activity in the corpus callosum Explanation: Positron emission tomography may reveal excessive metabolic activity in the frontal lobes and basal ganglia. excessive guilt. feelings of worthlessness. loss of interest or pleasure (dysphoria). 30. 31. 34. 32. 17th edition. 33. All of the following statements about suicide are true except: 15-40% of completed suicides are preceded by attempts There are 4-5 attempts for each suicide Access to guns increases the risk of suicide Alcohol use is unrelated to suicide Explanation: Use of alcohol and other drugs is associated epidemiologically with suicide.) .Question . 17th edition.) Benzodiazepines None of the above Question . Important questions after an attempted suicide include all of the following except: Is the patient less depressed? Is the patient physiologically stable? Does the patient still want to die? Are precipitating events still active? Does the patient have a future view or orientation? Have the shame and guilt been moderated? All of the above Explanation: These are all helpful in discharge planning and in assessing recurrence risks. The treatment of choice for childhood-onset major depression is: Monoamine oxidase inhibitors Tricyclic antidepressants Serotonin reuptake inhibitors Explanation: SSRIs have proven efficacy and have a reasonable safety profile. (See Chapter 24 in Nelson Textbook of Pediatrics.) Depression is related to suicide Question . (See Chapter 23 in Nelson Textbook of Pediatrics. (See Chapter 24 in Nelson Textbook of Pediatrics. 17th edition. 17th edition.) They should return the stolen item It may be learned from parents It may be impulsive . The parent should do all of the following except: Yell and punish him Explanation: Caregiver anger reinforces the child's behavior and teaches a vicious cycle of oppositional behavior. lying should be approached calmly with an understanding of the fears of the child.Question . 37. (See Chapter 25 in Nelson Textbook of Pediatrics. 17th edition. His lying is most likely to represent: Pathologic behavior. (See Chapter 25 in Nelson Textbook of Pediatrics. avoidance of punishment.) Acknowledge the child's frustration Quietly explain that his response is not acceptable Give him time and space to recover Nonemotionally place him on a time out Question . A 2-yr-old doesn't get his way in a crowded toy store. which needs punishment Displacement Reaction formation Avoidance of an unpleasant punishment None of the above Explanation: Lying is common at every developmental age and has reasons at each level of development. He starts to cry and hit and roll on the floor. he lies.) Question . It may include fantasy. Pathologic repeated chronic lying often occurs in combination with other antisocial behavior. 36. 35. and fear. honesty must be reinforced. When asked what he has done. 17th edition. Nonetheless. A 4-yr-old is seen hitting his sister. (See Chapter 25 in Nelson Textbook of Pediatrics. At any time. All of the following statements regarding minor stealing are true except: All (almost) children steal at least once It must be overemphasized to avoid a second episode Explanation: Overemphasis may create an excitement to repeat the offense. anxiety disorder in 20-25%. It is characterized by impaired verbal and nonverbal communication. ritualistic behavior. 38. 17th edition.Question . 39. 17th edition. (See Chapter 27 in Nelson Textbook of Pediatrics. and rocking.) Preoccupation with body parts Prevalence of 3/1000-4/1000 population Association with fragile X Visual scanning of the fingers Question . mood disorders in 15-20%. repetitive. (See Chapter 27 in Nelson Textbook of Pediatrics. tantrum-like rages. and reciprocal social interaction. (See Chapter 29 in Nelson Textbook of Pediatrics. All of the following statements regarding attention deficit hyperactivity disorder are true except: It is more common in males than in females Tic disorders may coexist with ADHD Patients dislike or avoid sustained mental efforts Oppositional defiant disorders never coexist with ADHD Explanation: Co-morbidity includes oppositional defiant disorder in 50%. He attends daycare but spends most of his time in solitary play. A 26-mo-old boy has a history of poor speech development.) Substance abuse occurs in affected adolescents . The most likely diagnosis is: Encephalitis Latent slow virus infection Rasmussen disease Autism Explanation: Autism begins before 30 mo of age and has a 4:1 male-tofemale ratio. conduct disorders in 30-50%. 17th edition. and learning disorders in 10-25%. 40. imaginative activity. Additional features of the disease affecting the patient described in Question 38 include all of the following except: Empathy Explanation: Lack of social relations and absent empathy are typical of children with autism.) Prader-Willi syndrome Question . 17th edition.) Most adolescents do not experience more stress during this developmental stage than at others Some "symptomatic" actions of children may be part of normal development Chronic stresses are more difficult for a child to manage than a single acute stressful episode Infants tend to react to stressful situations with impaired physiologic functions Question . 17th edition. (See Chapter 29 in Nelson Textbook of Pediatrics. 41. there may be educational problems in 25-50% of affected children . Which of the following statements is false? The clinical interview is a tool for gathering information. 17th edition.Question .) Question .) Even when head injury is judged to be mild. Possible complications of stimulant drug therapy for attention deficit hyperactivity disorder include all of the following except: Jitteriness Difficulty sleeping Abdominal pain Tics Increased appetite Explanation: Anorexia is common. not for enhancing behavior Explanation: (See Chapter 17 in Nelson Textbook of Pediatrics. Which of the following statements is false? Head injury is relatively common in pediatric populations Serious brain injuries early in life may be more generally incapacitating than those that occur later There is an increased risk of a psychiatric disorder after brain injury Psychosis is often seen as a result of brain injury in childhood Explanation: (See Chapter 18 in Nelson Textbook of Pediatrics. this affects growth. 42. 43. ) Pseudoseizures are the most common conversion symptom 10-15% of children have ongoing somatic symptoms Psychoactive medications may be useful in treating anxiety or depression in childhood Question . 17th edition. but in factitious disorders the unconscious need to be cared for motivates the falsification of symptoms Conversion disorder is a type of somatoform disorder that usually presents in early childhood Explanation: (See Chapter 19 in Nelson Textbook of Pediatrics.Question . 46. 45. Each of the following disorders is often co-morbid with Tourette syndrome except: Borderline personality disorder Explanation: (See Chapter 21 in Nelson Textbook of Pediatrics. 17th edition.) Question . 44. Which of the following choices is not a helpful treatment for enuresis? Desmopressin acetate Imipramine Maintaining a star chart Bell-and-pad apparatus Punishment Explanation: (See Chapter 20 in Nelson Textbook of Pediatrics. Which of the following statements is false? In somatoform disorders symptoms are associated with unconscious conflict. 17th edition.) Obsessive-compulsive disorder Attention deficit hyperactivity disorder Oppositional defiant disorder Tic disorder . A 7-yr-old boy is referred to you by the school for problems with hyperactivity. you increase the dose to the recommended maximum dose. and stomach aches. She has treated the child for 9 mo and is concerned that the child is quite "jumpy" at school. she states that his grades have fallen because he cannot pay attention in class. After performing an interview you diagnose OCD and begin treatment with sertraline. disruptive behavior.) . She tolerates the medication without side effects but continues to be impaired by the obsessions and washing rituals. His mother reports few problems at home except that he complains of stomach aches when he wakes up in the morning. Which of the following statements concerning the use of medications for the treatment of post-traumatic stress disorder (PTSD) in children is true? Clonidine and guanfacine are helpful for sleep disturbance and persistent arousal Explanation: (See Chapter 22 in Nelson Textbook of Pediatrics. A 9-yr-old boy who witnessed the murder of his mother by his father is referred to you by the school psychologist. 49. They report that he "acts up" in class and have requested that the parents pick him up from school early on several occasions. 17th edition.) Obtain permission from school officials for the child to be homeschooled for the rest of the semester Question . His mother appears depressed and "tears up" while sharing that her husband was recently diagnosed with cancer. He does not appear fidgety during the evaluation. 17th edition. In addition. 48. He reports difficulty falling asleep at night. The child presents to your clinic with his mother. The next step in treatment involves: Increasing the sertraline Adding fluvoxamine to the sertraline Referring the girl for cognitive-behavioral therapy Explanation: (See Chapter 22 in Nelson Textbook of Pediatrics. 47. A 12-yr-old girl presents with a history of repetitive handwashing related to contamination obsessions.) Adding risperidone to the sertraline Obtaining an ASO titer Question . After an initial response. 17th edition. You diagnose the child to be suffering from separation anxiety disorder and recommend the following: A trial of an SSRI for the child A trial of dexedrine or methylphenidate Tell the mother she needs to be strong for the rest of the family Referral of the family for therapy Explanation: (See Chapter 22 in Nelson Textbook of Pediatrics.Question . ) 35-50% 60-70% Question . All of the following statements about adolescent suicide are true except: It is the leading cause of death in adolescence Mood problems and violence are major risk factors Substance abuse is a major risk factor The suicide rate is higher in younger adolescents than in older adolescents Explanation: (See Chapter 24 in Nelson Textbook of Pediatrics.Stimulants are the treatment of choice for inattention in PTSD SSRI-medications are not helpful in the treatment of co-morbid depression. and withdrawal. 17th edition. anxiety. 51. 52. 17th edition. parents should be advised to give the child time and space to recover by turning away briefly . so medications will be of little help Question . Which of the following statements is false? Breath holding is not unusual during the first year of life In response to tantrums.) Previous suicide attempts are a major risk factor Question . 50. The lifetime prevalence for depression starting in adolescence is: 2-4% 6-8% 15-20% Explanation: (See Chapter 23 in Nelson Textbook of Pediatrics. Alprazolam and lorazepam are recommended treatments for anxiety and sleep problems associated with PTSD Too much time has passed after the trauma. targeted toward speech and language development.Lying is more common in children with high self-esteem Explanation: (See Chapter 25 in Nelson Textbook of Pediatrics. 17th edition. 54. 53. Which of the following statements about homosexuality in adolescence is true? The prevalence is less than 1% It is a risk factor for suicide. intensive behavioral therapy.) It is designated as a psychologic disorder by the American Psychiatric Association Behavioral treatments for homosexuality have been shown to be effective in changing orientation It is associated with various psychologic disorders in adulthood Question . 17th edition. intelligence measured by conventional psychological testing usually falls in the functionally retarded range Genetic factors play a significant role in autism Early. is successful in improving language and social function of autistic children . especially in males Explanation: (See Chapter 26 in Nelson Textbook of Pediatrics.) Ritualistic behavior is common with autism In autism.) Truancy and run-away behavior are never developmentally appropriate Children exposed to aggressive models on television display more aggressive behavior than that noted in children not so exposed Question . Which of the following statements is false? Autism is typically diagnosed after 36 mo Explanation: (See Chapter 27 in Nelson Textbook of Pediatrics. 17th edition. 17th edition.) Children with special ordering weaknesses may be late in discriminating between right and left Dyspraxia in general relates to difficulty in developing an ideomotor plan and activating coordinated motor actions to complete a task Question .1 in Nelson Textbook of Pediatrics. All of the following statements are true except: Chromosome abnormalities can lead to unique patterns of language defects Children who can register and consolidate facts and procedures in memory may have great difficulty accessing or retrieving these items when they are needed Children with special ordering weaknesses do not have problems with letter and word recognition Explanation: (See Chapter 29. 17th edition.Question .2 in Nelson Textbook of Pediatrics. 56.) Question . Which of the following medications is not used as a mood stabilizer? Depakote Lithium carbonate Carbamazepine Topiramate Fluoxetine Explanation: (See Chapter 28 in Nelson Textbook of Pediatrics. and these scales are sufficient to make the diagnosis Explanation: (See Chapter 29.) . 57. 17th edition. behavioral. All of the following statements are true except: ADHD frequently occurs with other emotional. language. and learning disorders ADHD is 3 to 4 times more common in males than in females Several standard behavior rating scales do well in discriminating between children with ADHD and controls. 55. The child's motor milestones were all normal. but he is unable to finish his tests. Her mother reports that her father. 59.Depression and anxiety disorder may present many of the same symptoms as those of ADHD Psychosocial interventions. 58. Of the following. explicit instruction in phonics Multisensory training Referral to ophthalmology Question . The most important element in making a diagnosis of dyslexia in this child is: Verbal IQ tested at a standard score of 94 Poor balance on tandem gait testing Many errors on a continuous performance test History of aggressive behavior with classmates Word reading test at a standard score of 84 Explanation: (See Chapter 29.3 in Nelson Textbook of Pediatrics. he seems to understand the concepts. and medication are effective in treating various components of ADHD Question .3 in Nelson Textbook of Pediatrics.) . An 8-yr-old girl is referred for evaluation of learning problems in school. 17th edition. an independent plumbing contractor. A 20-yr-old college student is referred from the university health service because of very poor grades during the previous semester. the most therapeutic recommendation is: Intensive phonologic awareness training Provision of extra time on tests Explanation: (See Chapter 29. 17th edition. had similar problems. behavior management training. His admission folder indicates that he was diagnosed as having a reading problem in fourth grade and received special help in grades 5-8. but she did not say her first words until 18 months.) Systematic. His class participation is good. He was allowed to take his SATs untimed. ) Parent-child problems in the home . Parents of a 12-yr-old girl are concerned because at a parent-teacher conference they were told that their daughter does not seem to be keeping up with her schoolwork in language arts. 17th edition. She reads the necessary school assignments but does not read for pleasure. she seems to be able to read words accurately but reads very slowly. She had difficulty in early grades with reading but then seemed to catch up. although her abilities in mathematics place her at the top of the class. Currently. 60. Which of the following is the most likely explanation for this history? Poor motivation Inability to decode words Low intelligence Poor reading influence Explanation: (See Chapter 29.3 in Nelson Textbook of Pediatrics.Question . 1.) Question . 17th ed. A chest film reveals two posterior rib fractures. He becomes apneic and unresponsive after a generalized seizure. 17th ed.Social Issues Question . 3. 17th ed. 2. produces rib fractures. after which he is noticed to have a bulging fontanel and bilateral retinal hemorrhages. (See Chapter 33 in Nelson Textbook of Pediatrics. He requires 10 min of cardiopulmonary resuscitation. (See Chapter 35 in Nelson Textbook of Pediatrics. A young child's response to the death of a parent often is characterized by: Depression and weight loss Denial and magical wishing Explanation: Many young children continue in their daily activities and use denial and magical wishful thoughts for reunion and reappearance. The parents state that he was perfectly well in the car on the way to the hospital and that they only brought him to the emergency room because of constipation. a computed tomography scan of the head reveals diffuse cerebral edema and hemorrhage. CPR in young children does not usually produce retinal hemorrhages and rarely. (See Chapter 33 in Nelson Textbook of Pediatrics.) Anger and crying Wishes of death for himself or herself None of the above Question . The effect that statements such as "stop it or you'll give me a headache" have on young children is to: Teach a child to behave Give children a pattern of headaches Create guilt and unrealistic fault Explanation: Statements such as "stop it or you'll give me a headache" may cause a child to suffer significant and unrealistic guilt. The most likely diagnosis is: CPR-induced retinal hemorrhages and rib fractures Hemorrhagic shock and encephalopathy Hemophilia Status epilepticus Child abuse-shaken baby syndrome Explanation: In the classic shaken baby syndrome. especially if the parent leaves for some time or is hospitalized. if ever.) . A 6-mo-old boy is brought to the emergency room and is afebrile but responds poorly to tactile and auditory stimuli. 4. All of the following statements regarding foster care are true except: Chronic medical illness is present in 35% of children 60% of preschool children in foster care experience developmental delay 42% are white children A majority of foster children receive EPSDT services Explanation: Early prevention screening and developmental testing programs are underutilized in the foster care system. All of the following statements about adoption are true except: 42% are stepparent or relative adoptions Most adopted children are from foreign countries Explanation: Five percent of adoptions are from overseas.Provide parents with a way to cope Prepare children for separation Question . (See Chapter 30 in Nelson Textbook of Pediatrics. 17th ed. (See Chapter 32 in Nelson Textbook of Pediatrics.) 1 million children are adopted in the United States each year 15% are adopted through foster care 2-4% of American families have adopted children Question . (See Chapter 31 in Nelson Textbook of Pediatrics.) Scores on standardized tests . 6.) Children frequently stay at more than one foster care home Question . 5. 17th ed. 17th ed. High-quality child care can influence all of the following except: Child cognition Future academic achievement Social development Sibling rivalry Explanation: Sibling rivalry and aggression may not be affected as much as the cognitive issues. 8. All of the following are age-related behavioral responses to experiencing violence except: Infants-poor sleep Adolescents-short-fuse responses Toddlers-excessive appetite Explanation: All the rest are true. as well as poor appetite. After divorce. A 3-yr-old boy is missing from his mother's house approximately 1 mo after a divorce.) Question . 17th ed. and poor school performance with hyperactivity in older children. The most likely explanation is: Sleepwalking Drug reaction Running away from his mother Searching for his father Explanation: It is not unusual for the young child to keep asking for the missing parent.Question . or to go outside to look for the parent. most children fantasize about the possibility that their parents will remarry. 7. 9. children may demonstrate all of the following except: A feeling of being overburdened by residence in two homes Withdrawal Indifference at times of reunions Academic deterioration Expectations that the parents will never get back together Explanation: Indeed.) Anxiety reaction Question . decreased exploration of the environment in toddlers. 17th ed. (See Chapter 34 in Nelson Textbook of Pediatrics. (See Chapter 33 in Nelson Textbook of Pediatrics.) Toddlers-clingy behavior School age-post-traumatic stress syndrome . (See Chapter 33 in Nelson Textbook of Pediatrics. to wait at the door or window. 17th ed. the baby appeared fine. 17th ed. the grandmother noted that the baby cried when she changed the diaper and that the leg was swollen. An x-ray film reveals a spiral fracture of the child's femur. nor do they readily know or admit to the identity of the person who committed the trauma. Of note. 17thed. A 2-mo-old is admitted with a fracture of the right femur. 12. abuse has been reported in all communities and from all socioeconomic levels. but that the father of the baby is well. Factors that may be associated with an increased risk of child abuse include: Poverty Military base residence Spouse abuse Unplanned pregnancy All of the above Explanation: Each is a risk factor. Thereafter. Munchausen syndrome by proxy is characterized by all of the following except: 10% mortality Multiple hospitalizations Induced manifestations by caregiver Ready admission of abuse by parents Explanation: Parents often deny their involvement in inducing symptoms in their children and will rapidly change doctors if it is discussed with them. 11. 17th ed. a bruise was noted over the sternum that was also said to have occurred during the fall 3 days ago.) Use of medications or toxins Question . The mother states that the baby fell off a low couch onto a plush carpeted floor and did not cry. (See Chapter 35 in Nelson Textbook of Pediatrics. Three days later. (See Chapter 35 in Nelson Textbook of Pediatrics. The mother states that she bleeds easily.) Question . (See Chapter 35 in Nelson Textbook of Pediatrics. 10. In the emergency department.Question . relatives do not know the nature or the cause of the injury. Features of this case suggestive of abuse include all of the following except: Multiple sites of injury Implausible explanation for injury Grandmother's deep concern Explanation: Often.) Injury incompatible with the nature of the fall Delay in seeking medical attention . 13. 17th ed. Pedophiles are best described as: Being female Never having repeated experiences Seeking opportunities to be in contact with children Explanation: Pedophiles often seek out positions and opportunities to be around children. Certain children may be particularly vulnerable. (See Chapter 35 in Nelson Textbook of Pediatrics.) Showing pornography to a child Use of a child to create pornography Incest Question . The nature of rib fractures is also different in abuse. Cardiopulmonary resuscitation in a child with head trauma from abuse commonly results in all of the following except: Recovery of a pulse Retinal hemorrhages and broken ribs Explanation: Retinal hemorrhages and broken ribs rarely follow CPR.) Recovery of respirations Normal sinus rhythm Recovery from cyanosis Question . (See Chapter 35 in Nelson Textbook of Pediatrics.Question .) Preferring females Being highly violent . 14. 17th ed. (See Chapter 35 in Nelson Textbook of Pediatrics. 15. Sexual abuse includes all of the following except: Exposing sexual anatomy Touching genitals by two preadolescents Explanation: This is sexual play and is usually normal exploratory behavior if there is no force or coercion and the children are not more than 4 years apart in age. such as those with mental and physical handicaps. 17th ed. (See Chapter 35 in Nelson Textbook of Pediatrics.Question . The hemoglobin and platelet counts were normal. Often. 18.) Question . as was the serum estradiol level. Pelvic ultrasound results were normal. 17.) Families should be encouraged to speak freely and repeatedly about adoption with the adopted child. Tanner stage 1. the perpetrator has some medical background. 17 ed.) Interrogate the father Isolate the parents and child Determine von Willebrand factor levels Measure fibronectin in the vagina Question . Findings on pelvic examination conducted under anesthesia also were normal. All of the following statements regarding adoption are true except: Federal law requires that children in foster care who cannot be safely returned to their families within a reasonable period of time be placed with adoptive families Agencies in the United States that arrange international adoptions have no legal obligation to obtain accurate and complete health histories on children whom families are considering adopting th Explanation: (See Chapter 30 in Nelson Textbook of Pediatrics. The next step in the examination is to: Determine the blood type of the blood on the underwear Explanation: By DNA typing. beginning in toddler years and continuing through adolescence . Previous examinations revealed a normal 4-yr-old girl. A 4-yr-old girl is admitted to the hospital for her third evaluation for vaginal bleeding. The mother noted bright red blood on the child's underwear. the blood turned out to be the mother's. as were the bleeding time and coagulation studies. 16. (See Chapter 35 in Nelson Textbook of Pediatrics. The mother has diabetes and employs home glucose monitoring and would purposely put blood on her daughter's underwear. 17th ed. The most likely diagnosis for the child described in Question 16 is: Precocious puberty Sexual abuse Vaginitis Coagulopathy Munchausen syndrome by proxy Explanation: Munchausen by proxy brings attention to the child and caregiver. with normal external genitalia. 17th ed. 17th ed. 21.) Middle-class children are not protected from the effects of poor-quality child care Licensure of child-care providers signifies that minimal health. and sanitary practices are being followed Accreditation of a child-care provider suggests that a program is of sufficient quality to promote children's development Question . 20.Most adopted children adjust well and lead healthy. All of the following statements are true except: Recurrent separations tend to accustom children to separations and make them less wary and guarded about reestablishing the relationship with an absent parent Explanation: (See Chapter 33 in Nelson Textbook of Pediatrics. All of the following statements regarding child care are true except: Nearly one half of employed mothers with 3. 17th ed. recognize .) A minority of children in foster care have a history of abuse or neglect An increasing proportion of children entering foster care are adolescents Children in foster care have low utilization rates for all types of care A minority of foster care children have behavioral and adjustment problems Question .and 4-yr-old children use center care as their primary supplemental care High-quality child care does not influence the cognitive and social development of disadvantaged children th Explanation: (See Chapter 32 in Nelson Textbook of Pediatrics. 19. 17 ed. Which of the following statements regarding foster care is true? A permanency plan must be made for a child in foster care no later than 12 mo from the child's entry into care Explanation: (See Chapter 31 in Nelson Textbook of Pediatrics. safety. productive lives Many children in foster care waiting to be adopted have "special needs" Question .) Most bereaved families remain socially connected and expect that life will return to some new sense of normalcy School-aged children think more concretely than younger children. 23. as a first line of treatment. 17th ed. 17th ed. and physical problems Occasional wife battering is estimated to occur in 16% of all families The most ubiquitous source of exposure to violence for children in the United States is television Question .) Question . social. academic.the permanence of death. 24. which is essential in learning in . or cluster of behaviors identifies the child or family in need of help with bereavement after the death of a family member Medication. and begin to understand biologic processes of the human body No specific grief sign. rarely proves useful in normal or uncomplicated grief reactions Question .) Fear may thwart their exploration of the world. All of the following statements regarding children and violence are true except: The source of first exposure to violence for children is often television Explanation: (See Chapter 34 in Nelson Textbook of Pediatrics.) The violence children experience and witness has a profound impact on health and development High levels of witnessing violence place children at risk for psychological. The violence children witness affects their development in all of the following ways except: It influences how they view the world and their place in it Children become more capable in motor function from the constant threat of violence Explanation: (See Chapter 34 in Nelson Textbook of Pediatrics. The source of first exposure to violence for children is often: Community violence War violence School violence Domestic violence Explanation: (See Chapter 34 in Nelson Textbook of Pediatrics. symptom. 22. 17th ed. Do a complete examination including .childhood Higher exposure to violence correlates with poorer performance in school. examine and do a skeletal survey of the sibling Perform a skeletal series. refer the child to an orthopedist for casting. 17th ed. Speak to the child with the mother out of the room. ask what happened. and report the spiral fracture to appropriate authorities as suspected physical abuse after screening the mother for risk factors and informing the mother of your plan Explanation: (See Chapter 35 in Nelson Textbook of Pediatrics. examined the child's genitalia because the child complained of dysuria. with no need to report suspected child abuse Because this fracture is probably due to osteogenesis imperfecta. Ask if anything has happened to various body parts. Begin with general questions. She found the genitalia to be red and open to the "size of a dime." She asked the girl if the father had done anything to her and the child remained silent. The child returned from a visit with the father yesterday. If the child answers in the affirmative. It is likely that the mother has prompted the child. The mother.) Tell the mother you suspect physical abuse and interrogate her until you get a history. A 6-mo-old child is brought to your office with the chief complaint of leg swelling and decreased leg movement of 3 days' duration. who has primary custody of the child. An x-ray film reveals a new (less than 7 days old) spiral fracture of the femur. make a report to appropriate authorities. The child has been reluctant to visit the father. send the child for genetics evaluation and connective tissue biopsy for a definitive diagnosis Question . Ask the father to come to your office so that you can interview him. refer the child to an orthopedist for casting. The parents of a 4-yr-old girl are in the process of a divorce. If the child answers in the affirmative. 25. Report what the mother has told you to appropriate authorities. The mother. Determine knowledge of body parts and private parts. She was unaware of any trauma to the child but said that her 2-yr-old "plays rough" with the 6-mo-old. The mother brings the child to you because she is concerned that the child has been abused. examine and do a skeletal survey of the sibling. you conclude that the finding is in keeping with the fracture. 26. Which of the following should be your next action? Refer the child to an orthopedist for casting Perform a skeletal series. Which of the following constitutes the most appropriate next step in management? With the mother in the room for support. and lower self-esteem Question . said that the child awoke 3 days ago with the swelling and guarding. when the mother tells you that the child fell from the crib to a carpeted floor and sustained the injury. symptoms of anxiety and depression. interview the child by asking if the father has ever done anything to her genitalia. She said that she delayed coming to see you because she had no transportation. a single parent. . If results are positive. a nurse. Have the mother bring you a sample of the vomitus. 17th ed. If the condition persists despite treatment. make a report to the appropriate authorities. Indicate that you suspect she is lying about the symptoms. Request a psychiatric consultation for the mother. 17th ed. Findings on laboratory studies including an upper GI series and on swallowing studies have been normal. Put the child on a restrictive diet and observe for symptoms. hospitalize the child and obtain surgical consultation. report as suspected Munchausen syndrome by proxy to proper authorities.inspection of the anus and genitalia.) Question . If the child gives any history suggestive of abuse or has findings suggestive of trauma to the anus or hymen that are unexplained. The mother had similar symptoms as a child. Explanation: (See Chapter 35 in Nelson Textbook of Pediatrics. has contacted to diagnose and treat these persistent symptoms. A 6-mo-old child has a 2-mo history of daily projectile vomiting. Test it for the presence of ipecac. Consult an allergist and request a work-up for food allergy. and nephrology. You are the fourth physician that the mother. Persist in your evaluation until a diagnosis is made. Confront the mother with lack of medical findings for the condition. A 3-yr-old sister has been treated for gastroesophageal reflux. Report your suspicions to proper authorities with a request that the child remain in the home under weekly supervision by a public health nurse. The most appropriate next step in management is: Conduct a complete and detailed work-up for cyclic vomiting with referrals for consultation to gastroenterology. Inspection of hospital records indicates that no vomiting took place during either hospital admission.) Place the child on treatment for gastroesophageal reflux in light of the high rate of false-negative results on laboratory testing for this condition. neurology. 27. Perform toxicology studies. She has been hospitalized on two occasions. Explanation: (See Chapter 35 in Nelson Textbook of Pediatrics. (See Chapter 37 in Nelson Textbook of Pediatrics. other. (See Chapter 37.g. 17th ed. chorioretinitis. 2.) Isolation Psychologic or behavioral problems . Frequent problems of children with common chronic illnesses include all of the following except: Unpredictability Pain Expense Multiple providers Failure to graduate high school Explanation: Many children with common chronic diseases of childhood (e. cystic fibrosis. herpes simplex) infection. sickle cell anemia) attend high school and graduate. In addition. 1..Children with Special Health Needs.) Fetal alcohol syndrome Galactosemia Hyperammonemia Question . Set 1 Question . arthritis. microcephaly. and a history of a neonatal petechial rash is most likely to have: A chromosomal syndrome TORCH infection Explanation: This pattern of abnormalities is most compatible with a congenital TORCH (toxoplasmosis. asthma. diabetes. cytomegalovirus. intrauterine and postnatal growth retardation may be evident. 17th ed. rubella. A mentally retarded child with microphthalmia.2 in Nelson Textbook of Pediatrics. seizures. 2 in Nelson Textbook of Pediatrics. The difficulty of long-term prognostication aside. (See Chapter 37 in Nelson Textbook of Pediatrics. A preschooler with Down syndrome is seen for a routine health supervision visit. 17th ed.) Hypothyroidism Conductive hearing loss . 4. is just beginning to master simple reading skills. she says she wants to see her. and has one close friend.) Question . She cannot be moved from where she is receiving intensive care. The youngster is in a mainstreamed educational program.) Tell her she would not want to see the baby Tell her that it is too late for her to see the baby Question . Which of the following conditions is least likely to be found in this child? Atlantoaxial instability Neurogenic bladder Explanation: Neurogenic bladder is not typically encountered in Down syndrome. Most parents benefit from seeing their child with anomalies and often identify aspects of beauty or normalcy with the help of a nurse or physician. possible life goals for this child include: Holding a regular job Getting married Having children All of the above Explanation: This child demonstrates important educational and social milestones that are partially predictive of future achievements with respect to these life goals. The parents of a 10-yr-old girl with mental retardation are seeking information on what to expect for her future. but Hirschsprung disease may occur. 17th ed. 5. 3.Question . (See Chapter 37. 17th ed.2 in Nelson Textbook of Pediatrics. A knowledgeable clinician will pay particular attention to screening for problems that are known to occur with increased frequency in children with this condition. An infant with multiple grotesque congenital anomalies dies on the third day of life. (See Chapter 37. Her mother has not had an opportunity to see her before death owing to postpartum complications. When informed of the baby's death. Which of the following is the most appropriate response to her request? Tell her that she is too sick to see the baby Tell her that she will be able to see the baby later Take the baby to her bedside Explanation: Parents who are not able to see their deformed neonate may greatly exaggerate the perceived severity of any anomaly and may have excessive feelings of guilt. including test for fragile X Audiologic evaluation Cranial CT scans Explanation: Cranial computed tomography is not indicated until other evaluations are completed unless a patient has macrocephaly. abnormal neurologic findings. 8.) Formal speech and language evaluation Question . Findings on physical examination are essentially unremarkable except for mild hypotonia.) . (See Chapter 37. microcephaly. The general approach to management of the child with psychosocial failure to thrive includes all of the following except: Keeping meal time brief Offering solid foods before liquids Forcing the child to eat Explanation: Forced feeding exacerbates abnormal psychosocial tension between a child and his or her caregiver. Appropriate initial laboratory studies include all of the following except: Karyotype. Use corrected age (subtract weeks premature) until age 1-2 yr 2. 17th ed. (See Chapter 36 in Nelson Textbook of Pediatrics. Determining whether growth percentiles are crossed is the other solution.2 in Nelson Textbook of Pediatrics. choose the most appropriate way(s) to assess growth in premature infants to diagnose failure to thrive.) Minimizing environmental distractions Minimizing the intake of water and juice Question . or significant dysmorphology. 17th ed. The "real" gestational age of a 24-wk premature infant who is now 20 wk old is 44 wk (1 month corrected age may also be valuable). 6. A 3-yr-old boy with a limited vocabulary is referred for formal psychometric testing and is found to have an IQ of 60. Add additional weight as if the child had been born at term 4.Question . Do not use head circumference until age 24 mo 1 only 1 and 2 Explanation: Corrected age rather than chronologic age is one solution. 17th ed. Determine whether two major growth percentiles are crossed 3. 7. 1. Determine the weight to length ratio 5. From the following list. (See Chapter 36 in Nelson Textbook of Pediatrics. but B is No.) Gastrointestinal reflux Cystic fibrosis Inborn errors of metabolism Question .3 and 4 3 and 5 1 and 5 Question . malabsorption is not a major cause of failure to thrive. 15. 16. 17th ed. Major causes of failure to thrive include all of the following except: Formula feeding Explanation: In infants who are fed formula in sufficient amounts. 17th ed. Snoring and mouth breathing as a cause of failure to thrive suggest: Streptococcal pharyngitis Mononucleosis Obstructive sleep apnea Explanation: Adenoid hypertrophy or poor oropharyngeal motility (possibly related to cerebral palsy) can cause failure to thrive from obstructive sleep apnea. 17th ed. (See Chapter 36 in Nelson Textbook of Pediatrics. 1.) Failure to provide sufficient calories Failure to ingest sufficient calories Failure to retain sufficient calories Malabsorption Question . 14. (See Chapter 36 in Nelson Textbook of Pediatrics.) Anterior meningocele Cerebral palsy . The leading cause of failure to thrive in infants between 0 and 3 mo of age is: TORCH infection Psychosocial pathology Explanation: All must be considered. (See Chapter 36 in Nelson Textbook of Pediatrics. Mononucleosis is rare in infants. ) Prevention of secondary psychosocial handicaps Treatment in the context of the family . (See Chapter 36 in Nelson Textbook of Pediatrics. parotitis. and hepatosplenomegaly are classic features of AIDS in infants.) Psychosocial failure to thrive Question . 35% have chronic physical conditions Question . 18. there is also bilateral parotitis.Question . Lymphadenopathy. (See Chapter 37 in Nelson Textbook of Pediatrics. All of the following are true regarding children with chronic illness except: Children with disabilities rarely survive to adulthood Explanation: Indeed. 17th ed. The most likely diagnosis is: Mononucleosis Familial histiocytosis X-linked combined immunodeficiency AIDS Explanation: Babies with AIDS usually present with failure to thrive and recurrent infections.) 6-7% of children has some limitation of activity 1-2% of children meet the definition of severe disability Of disabled children. On physical examination the patient has generalized lymphadenopathy and hepatosplenomegaly. 17th ed. 19. (See Chapter 37 in Nelson Textbook of Pediatrics. 40% have learning and developmental disorders Of disabled children. 17. The past medical history includes severe thrush and Candida diaper rash and recurrent otitis media with perforation. most survive to be adults. A disease should not define a child. 17th ed. An 8-mo-old presents with failure to thrive. Principles of care for children with chronic diseases include all of the following except: Early detection Amelioration of functional consequences Designation of children with asthma as "asthmatics" Explanation: Labeling depersonalizes the child and marks the child for life. 22.) Education in the least restrictive manner Support of the state programs providing special education Question . (See Chapter 37 in Nelson Textbook of Pediatrics. 17th ed. A sixth-grade child with chronic arthritis typically views the cause of chronic illness as due to: Germ theory Explanation: At this age. 20. Mental retardation is best classified by: A system designating the degree as mild. it does help with compliance in taking medicine. (See Chapter 37 in Nelson Textbook of Pediatrics. profound The designators imbecile. 17th ed. 17 ed. 21.Question . severe. retarded. moderate. pervasive) Explanation: This functional approach places the child's needs for support into the context of the environment. limited. germ theory is an easily comprehended but unfortunately not always accurate view of chronic illness. Nonetheless. functional IQ percentiles Support needs (intermittent. The Individuals with Disabilities Education Act includes all of the following except: Support of state programs providing early intervention Providing cash assistance for children with disabilities Explanation: Supplemental security income programs provide cash support for people of all ages with disabilities. (See Chapter 37 in Nelson th Textbook of Pediatrics.) None of the above . extensive.) Punishment for bad behavior Physiologic mechanisms Not following rules Failure to take medicines Question . (See Chapter 37 in Nelson Textbook of Pediatrics. Common identifiable causes of mental retardation include all of the following except: Trisomy 21 Hypothyroidism Fetal alcohol syndrome Fragile X syndrome Cystic fibrosis Explanation: Cystic fibrosis itself does not cause mental retardation. 17th ed. hypothyroid-induced retardation is preventable by early screening and rapid therapy. 23. 25. All of the following statements about mental retardation are true except: About 3% of the population have an IQ less than 2 standard deviations below the mean Severe retardation is inversely related to socioeconomic status Explanation: Mild retardation is related to low socioeconomic status. AD are common. (See Chapter 37 in Nelson Textbook of Pediatrics. 17th ed. 24.) Question . 17th ed.Question . (See Chapter 37 in Nelson Textbook of Pediatrics. whereas more severe retardation is equally distributed in all such groups.) Up to 5% of children with mental retardation are profoundly affected The reported incidence increases at entry to school Language development is a first clue to mild retardation Question . Adaptive skill areas to assess in children with mental retardation include all of the following except: Communication Sports Explanation: Sports per se are not an adaptive skill but could be categorized into leisure activities.) Self-care Home living Social skills Community use . 17th ed. It is relatively common. which have resolved. Thanks to active immunization programs. the next step in evaluation should be: Plasma ammonia assay Blood lead level determination EEG Chromosome analysis Explanation: Fragile X syndrome manifests in males as mental retardation. 28. 26. 17th ed. 27. At birth. In male patients with mental retardation without an obvious etiology. congenital rubella is rare. The most likely diagnosis is: Congenital HIV infection Congenital rubella Explanation: Congenital rubella was once a common cause of mental retardation. large ears. (See Chapter 37 in Nelson Textbook of Pediatrics.Question .) It is a functional state . and large testes. It is important not to reinforce the immature child's belief that death is like sleep because the child will be fearful of sleeping lest he or she may die. the baby had petechiae and hepatosplenomegaly.17th ed. A 3-yr-old girl presents with microcephaly and mental retardation. (See Chapter 37 in Nelson Textbook of Pediatrics. A 4-yr-old's concept of death may include all of the following except: It may be reversible It is like sleep Dead people still eat and breathe They understand causality Explanation: Causality is a late developmental state of understanding.) Congenital parvovirus infection Isoimmune neonatal thrombocytopenia Subacute sclerosing panencephalitis Question . Her mother had a flu-like illness during the second month of pregnancy. (See Chapter 38 in Nelson Textbook of Pediatrics.) Cranial CT Question . (See Chapter 38 in Nelson Textbook of Pediatrics. Giving an estimate of how long a child with a life-threatening condition will survive will: Be inaccurate Explanation: Population-based statistics are poor for predicting the time of death of an individual child. 17th ed. 30. 17th ed. (See Chapter 38 in Nelson Textbook of Pediatrics. 31.Question . (See Chapter 38 in Nelson Textbook of Pediatrics. 29. Perpetuating the myth of "everything is going to be all right" with a dying child will: Help reassure the child Hide "bad things" from the child Enhance an awareness of eventual death Alleviate fears Prevent exploration of fears Explanation: Most chronically ill children have a sense of impending death and need to articulate this with parents or caregivers who can discuss fears and provide some reassurance (at least for comfort).) Transcendentalism Existentialism Question . The fear of dying in young children is best described as fear of: The afterlife The unknown Separation Explanation: Separation from loved ones is the dominant and often only concern of young children.) Help accept the diagnosis Avoid unnecessary expenses Provide time to remove the child from school Enhance communication between siblings and the child .) Question . 17th ed. All of the following statements are true except: In the United States the most common presentation of failure to thrive is poor growth detected in the ambulatory setting The laboratory evaluation of children with failure to thrive is usually helpful Explanation: (See Chapter 36 in Nelson Textbook of Pediatrics. All of the following statements are true except: The BSID II is used to test children 1 mo-3½ yr of age The BSID II is used to identify infants with severe mental retardation The WPPSI-R is used to test children with mental ages of 3-7 yr The Stanford-Binet Intelligence Scale measures verbal abilities. 33. quantitative reasoning.5% of the population can be expected to have MR About 0. 32. physical examination. a biologic cause can be identified in over ¾ of cases Statistically. 17th ed. All of the following statements about mental retardation (MR) are true except: Mild MR is defined as an IQ of 80 or below Explanation: (See Chapter 37 in Nelson Textbook of Pediatrics. abstract thinking.Question . 2. 34.5% of the population have severe MR The brains of 10-20% of individuals with severe MR are normal by standard neuropathologic studies Question . 17th ed. and adaptive functioning Explanation: (See Chapter 37 in Nelson Textbook of Pediatrics.) In children with severe MR.) The WISC III is used for children who function above a mental age of 6 yr . and observation of the parent-child interaction usually suggest the diagnosis of failure to thrive Failure to thrive in the first year of life regardless of cause is particularly ominous Approximately one third of children with psychosocial failure to thrive are developmentally delayed and have social and emotional problems Question . 17th ed.) The history. 17th ed. The most likely diagnosis is: Dementia Hypothyroidism Adjustment disorder Explanation: (See Chapter 37 in Nelson Textbook of Pediatrics. 37. the most common is: Inborn errors of metabolism Developmental brain anomalies Perinatal causes Chromosomal abnormalities Explanation: (See Chapter 37 in Nelson Textbook of Pediatrics. 17th ed.) Depression Atlantoaxial subluxation Question . All of the following statements are true except: Surveys indicate that 15-18% of children and adolescents have some form of chronic condition About 6-7% of all children and adolescents have some limitation of activity due to a chronic condition At least 90% of children with severe long-term illnesses survive to young adulthood About 2-4% of children with severely long-term illnesses account for at least 35% of child health care expenditures Children face a relatively small number of common chronic conditions and a few rare diseases Explanation: (See Chapter 37 in Nelson Textbook of Pediatrics. 17th ed.Question . 36. Of the following causes of severe mental retardation. 35.) Postnatal causes . He is less lively and more irritable. John has been spending more time in his room and less time with the family. John is 12 yr old and has Down syndrome.) Question . He has recently transitioned from an elementary school that used an inclusion model to a middle school program that continues to focus on reading and math. Findings on the motor examination are normal. The most common reason for people with mental retardation to be placed out of the home is: Single-parent family Greater degree of mental retardation Greater intensity of medical needs Lack of an appropriate school. Further evaluation should include: Chromosome analysis Assessment of language skills Explanation: (See Chapter 37 in Nelson Textbook of Pediatrics. necessitating residential placement Behavioral disturbance Explanation: (See Chapter 37 in Nelson Textbook of Pediatrics.) Question . In addition to the deficits in cognition and onset before age 18 yr. 17th ed.) Stereotypic behavior Self-stimulatory behavior Play Moor behavior Question . Jane is an 18-mo-old girl who presents because of delayed walking. 17th ed. the diagnosis of mental retardation requires deficits or impairments in: Adaptive behavior Explanation: (See Chapter 37 in Nelson Textbook of Pediatrics. 39. diffuse hypotonia. 17th ed.) EMG and nerve conduction studies MRI Audiologic evaluation . 40. 38. save for mild.Question . Question . 17th ed.) Children's fear of death is centered on the concrete fear of being separated from parents and other loved ones In some families and cultures. All of the following statements are true except: Physicians should ask parents of the patient about their economic circumstances The medical problems of children of migrant farm workers are similar to those of children of homeless families By age 20-30 yr. 17th ed. truth telling and autonomy are much less valued than family integrity Many dying children do not have their pain successfully treated Question .) Most states authorize treatment of homeless youth Recurrent otitis media is an especially frequent problem among Native American children . 42. All of the following statements are true except: Provision of respite services is especially important to families caring for children with complex chronic conditions over prolonged periods of time Good end-of-life care cannot be effectively carried out in a hospital setting Explanation: (See Chapter 38 in Nelson Textbook of Pediatrics. few children who were at special risk have made moderate success of their lives Explanation: (See Chapter 39 in Nelson Textbook of Pediatrics. 41. Fruits. parathyroid hormone level is elevated. or milk products and is thus deficient in vitamin B12. (See Chapter 44 in Nelson Textbook of Pediatrics. and cow's milk are poor sources of iron. yellow vegetables. the iron is more bioavailable than the iron in cereals.) Rosary . meat. 17th ed. and 44 in Nelson Textbook of Pediatrics.) 2% low-fat cow's milk Question . 17th ed. (See Chapters 41.Nutrition Question . 2. (See Chapters 40 and 41 in Nelson Textbook of Pediatrics. Even though osteoid of the legs is uncalcified. The best source of iron for 1-mo-old infants is: Iron-fortified cereals Yellow vegetables Fruits Breast milk Explanation: Although breast milk contains relatively less iron by weight. which results in low serum phosphate levels. 42. The breast-fed infant of a mother who is a strict vegan may experience deficiency of which of the following vitamins if the mother is not receiving supplements of the vitamin? K B6 B12 Explanation: A strict vegan diet contains no eggs. 1.) Folate Biotin Question . Low serum phosphate levels result in abnormal osteoblastic activity. A 4-mo-old child with vitamin D deficiency rickets would be expected to show all of the following except: Craniotabes Bowleg Explanation: In rickets. 3. 17th ed. bowing does not occur until weight is borne on the legs. which may result in craniotabes and a rachitic rosary with enlargement at the costochondral junctions. The DRI also considers the EAR.and gender-specific to meet the needs of 50% of persons Explanation: The EAR is one form of nutrient evaluation. (See Chapter 40 in Nelson Textbook of Pediatrics. 6. All of the following are advantages of breast-feeding except: Reduced incidence of allergy Reduced incidence of otitis media Reduced incidence of colic Increased psychologic comfort Vitamin K content Explanation: Vitamin K must be given (intramuscularly at birth) to all . 5. The estimated average requirement (EAR) of a nutrient is best defined as: A dietary reference index The recommended dietary allowance Age.) The range encompassing the lower and upper limits of a nutrient A daily average calculated once each year Question .) Intakes that will prevent deferrable nutrient deficiencies Question . and tolerable upper level (UL). The AI (adequate intake) in infants is estimated from: Bomb calorimetry The EAR The RDA The intake of nutrients from human milk Explanation: The AI is one component of the DRI (dietary reference intakes). 17th ed. 17th ed. 4. The RDA (answer B) is the daily dietary intake sufficient to meet 97-98% of individual nutrient needs in age and gender groups.Low serum phosphate levels High alkaline phosphatase levels Question . RDA. (See Chapter 40 in Nelson Textbook of Pediatrics. ) Question . All of the following observations regarding infant feeding are true except: Self-feeding with a spoon at age 12 mo should be encouraged Infants will select a balanced diet Consistent rejection of one food group should suggest food allergy Infants should be put to bed with a bottle of milk Explanation: Baby bottle caries syndrome is a significant problem because the milk bathes the teeth. Atypical features of infant colic include all of the following except: Fever Onset in the first week of life Onset at age 6 mo Sudden onset Crying mainly in the early morning Explanation: Colic usually recurs in the early evening. Breast-fed infants whose diet is not supplemented with vitamin K are at risk for bleeding. setting up multiple caries. (See Chapter 41 in Nelson Textbook of Pediatrics. 7.infants. 17th ed. 8.) The likes and dislikes of infant tastes should be respected . 17th ed. (See Chapter 41 in Nelson Textbook of Pediatrics. Problems associated with breast-feeding include all of the following except: Less than optimal nutrients for infants <1000 g Vitamin K content Transmission of live viruses Hyperbilirubinemia Contraindication in erythroblastosis fetalis Explanation: Antibodies in mother's milk are inactivated in the infant's intestines and do not contribute to intravascular hemolysis. (See Chapter 41 in Nelson Textbook of Pediatrics. (See Chapter 41 in Nelson Textbook of Pediatrics. 17th ed.) Utility for preterm infants weighing less than 2000 g Question . 17th ed Question . 9. (See Chapter 42 in Nelson Textbook of Pediatrics. Complications of obesity in childhood include all of the following except: Angina Explanation: Although hyperlipidemia secondary to obesity may be .) Single-gene disorders are rare causes of obesity Obesity may be associated with insulin resistance The highest prevalence of obesity in the United States is in the Northeast Menarche may be earlier in obese girls Question . All of the following statements about obesity in children are true except: Obese children eat more junk food Explanation: Obese children do not necessarily eat more food or more junk food than leaner children. All of the following are clinical manifestations of kwashiorkor except: The presence of edema Rash in sun-exposed areas Explanation: The rash of kwashiorkor is in areas of irritation. Sunexposed dermatitis is typical of pellagra.Question .) Hypoalbuminemia Hypoglycemia Potassium deficiency Low serum amylase levels Question . 17th ed. All of the following are laboratory manifestations of kwashiorkor except: Persistent ketonuria Explanation: Ketonuria is present early but does not persist into the later stages. 11. 17th ed. (See Chapter 43 in Nelson Textbook of Pediatrics. 12. (See Chapter 42 in Nelson Textbook of Pediatrics.) Hypochromotrichia Muscle weakness An increased susceptibility to infection Question . 10. 13. 17th ed. 15. coronary artery disease does not manifest in children. 17th ed. (See Chapter 44 in Nelson Textbook of Pediatrics.) Blount disease Slipped capital femoral epiphysis Sleep apnea Glucose intolerance Question . (See Chapter 44 in Nelson Textbook of Pediatrics.present. 17th ed. Clinical features of vitamin E deficiency include all of the following except: Cerebellar ataxia Muscle weakness Peripheral neuropathy Hemolysis Hepatosplenomegaly Explanation: A-C often occur in patients with malabsorption. D has been reported in premature infants.) Craniotabes Enlargement of the costochondral junctions Thickening of the ankles and wrists Large anterior fontanel Bowleg Question . 14.) . 17th ed. Physical features of vitamin D deficiency rickets include all of the following except: Bitot spots Explanation: Bitot spots are seen in vitamin A deficiency (dry plaques on the bulbar conjunctiva). (See Chapter 43 in Nelson Textbook of Pediatrics. The pediatric patient with obesity typically presents with: Tall stature Explanation: (See Chapter 43 in Nelson Textbook of Pediatrics. The group that has exhibited the most striking increase in the prevalence of obesity in the United States is: Caucasian males Caucasian females African-American and Hispanic males African-American and Hispanic females Explanation: (See Chapter 43 in Nelson Textbook of Pediatrics. 17. 17thed. 16. 18. The risk of developing obesity in adulthood correlates negatively with: High birthweight Number of obese parents Months of breast-feeding Explanation: (See Chapter 43 in Nelson Textbook of Pediatrics.Question . 17thed.) Delayed bone age Negative family history of obesity Delayed puberty Elevated serum cholesterol Question .) Asian-American males and females Question .) Hours of television viewing Weight during adolescence . 17thed. ) . 17th ed.) Question . (See Chapter 45. Diabetes insipidus may be due to all of the following except: Pituitary adenoma Renal epithelial ADH reception defect Hypokalemia Hypercalcemia Adrenal deficiency Explanation: Adrenal deficiency may cause renal salt wasting and usually does not affect free water excretion.3 in Nelson Textbook of Pediatrics. (See Chapter 45.Pathophysiology of Body Fluids and Fluid Therapy Question . and a temperature of 41°C. emesis. 2. Serum electrolyte measurements reveal a sodium level of 185 mEq/L. and the urine specific gravity is 1001. The most likely diagnosis is: Adrenal insufficiency Salt poisoning Hypernatremic dehydration Malignant hyperthermia Nephrogenic diabetes insipidus Explanation: Nephrogenic diabetes insipidus is a sex-linked recessive disorder due to deficient binding of ADH to the renal tubular cell. A 1-mo-old boy presents with severe failure to thrive. 17th ed. Exogenous administration of ADH is therefore ineffective. 1.3 in Nelson Textbook of Pediatrics. 17th ed. (See Chapters 45. 3. Hyperkalemia may be associated with all of the following except: Succinylcholine use Burns Trauma Chemotherapy Metabolic alkalosis Explanation: Metabolic alkalosis produces hypokalemia.) Hypoglycemia Hypernatremia Hypokalemia Question .8 in Nelson Textbook of Pediatrics. Because it is the end of the month.4 and 45. (See Chapter 45. A normal anion gap acidosis is most likely to be due to: Diabetes mellitus Renal tubular acidosis Explanation: Renal tubular acidosis with renal bicarbonate loss and diarrhea-induced stool losses of bicarbonate are the common causes of a normal anion gap acidosis.) Nephrotic syndrome Uremia Shock . 5.17th ed. (See Chapter 46 in Nelson Textbook of Pediatrics.) Digitalis toxicity Uremia Question . 17th ed.Question . 4. she has begun to dilute the remaining formula with water as there is not enough to last until she receives her next allotment of formula next week. The diet history reveals that the mother is a participant in the WIC program. A well-grown 6-mo-old presents with a tonic-clonic seizure lasting 30 min. The child is found to be hypothermic and remains lethargic. The most likely diagnosis is: Hypocalcemia Hyponatremia Explanation: Hyponatremia due to feeding diluted formula or excessive amounts of sodium-free fluids (especially water) is relatively and unfortunately common among poor families who run out of formula.8 in Nelson Textbook of Pediatrics. Other methods shift potassium from the extracellular to the intracellular space. and if idiogenic osmoles remain in neurons during rehydration. A serious complication of the treatment of hypernatremic dehydration is: Cerebral thrombosis Cerebral edema Explanation: Cerebral edema occurs if free water is given in excessive amounts.) Hyperchloremia Hypoglycemia None of the above Question . Tachycardia reflects the intravascular volume loss.Question . Cerebral thrombosis may occur before therapy is started and may be associated with inherited hypercoagulable conditions. (See Chapter 45. The best method to reduce the potassium level during hyperkalemia. normal blood pressure. The percent dehydration for this patient is: 0-3% 3-5% 6-9% Explanation: Dehydration of 6-9% represents moderate dehydration and early shock. A 10-mo-old infant presents with vomiting and diarrhea. deep respirations. and irritability. 17th ed. a potassium-binding resin. 17th ed. is: Sodium bicarbonate infusion Glucose and insulin infusion Calcium infusion Albuterol aerosol Kayexalate enema Explanation: Kayexalate. if the serum sodium falls more than 10 mEq/L/day. dry mucous membranes. and dialysis are the only methods to remove potassium from the body. 17th ed.) 10-12% 12-15% Question . tachycardia.3 and 47 in Nelson Textbook of Pediatrics. 7. a capillary refill time of 2 sec. 8.) . 6. (See Chapters 45. and deep respirations represent the pulmonary response to metabolic acidosis.4 in Nelson Textbook of Pediatrics. by reducing the body burden of potassium. (See Chapters 47 and 48 in Nelson Textbook of Pediatrics. but such a repeat in this case would confirm the finding. 2. 11.) Nicotine -Adrenergic drugs . 1. Sweat 2. (See Chapter 45. repeat laboratory tests.3 in Nelson Textbook of Pediatrics. Obligate water for urinary solute excretion 1 and 3 1. although laboratory error is possible. Respiratory water loss 5. Fecal loss 3. 9. 17th ed.2 in Nelson Textbook of Pediatrics. 17th ed. and 3 3 only 3 and 4 Explanation: Insensible water losses usually occur independently of total body water homeostasis. (See Chapter 45.17th ed. The finding of marked metabolic alkalosis with acidic urine indicates: Marked sodium depletion Marked potassium depletion Explanation: Marked potassium depletion is more likely. When in doubt. 10. choose the route(s) by which insensible water loss may occur. Evaporative loss from skin 4.4 in Nelson Textbook of Pediatrics.) 2 and 5 Question . Which of the following drugs or agents may inhibit antidiuretic hormone release? Demerol Barbiturates Alcohol Explanation: All the others may stimulate ADH release.Question . From the following list.) Hyperventilation Diabetes mellitus Laboratory error Question . (See Chapter 45. 17th ed.Question .3 in Nelson Textbook of Pediatrics. 17th ed. Manifestations of hyperkalemia include all of the following except: Paresthesias Weakness Paralysis Wide QRS complex Tetany Explanation: A-D are noted in hyperkalemia. Lengthening of the P-R interval and QRS complex occurs later.) . The first ECG change is peak T waves. Infants'. 14. Another common cause is feeding excess amounts of pure water in general. 17th ed. 12.) Furosemide (Lasix) therapy Vegan diets Question . (See Chapter 45. (See Chapter 45. The most common cause of nutritional hyponatremia is: Salt substitutes Low-salt diets The WIC syndrome Explanation: The WIC syndrome in families receiving formula from the Women's. and Children's nutritional supplementation program occurs when families run out of formula toward the end of the month and begin to dilute the formula with water.4 in Nelson Textbook of Pediatrics. (See Chapter 45. 13. Hypernatremia may be induced by all of the following except: Hyperglycemia Explanation: Indeed. hyperglycemia may produce pseudohyponatremia.3 in Nelson Textbook of Pediatrics.) Adipsia Insufficient breast-feeding Gastroenteritis Nephrogenic diabetes insipidus Question . Serum magnesium determination 4. Anion gap measurement 1 and 3 2 only 1. Arterial blood gas analysis 3. After resuscitation and stabilization. A preterm infant born to a mother with severe preeclampsia is noted to be hypotonic and apneic in the delivery room. magnesium sulfate.8 in Nelson Textbook of Pediatrics. 3. The mother's treatment included hydralazine. 17. An increased anion gap occurs in all of the following except: Diabetic ketoacidosis Renal tubular acidosis Explanation: RTA produces a non-anionic gap acidosis due to loss of bicarbonate by the kidney and not due to net gain of new acid in the circulation. she remains hypotonic with decreased deep tendon reflexes in the arms and knees. 2. Serum calcium determination 2. CBC 5.) Salicylate poisoning Methylmalonicacidemia Ethylene glycol poisoning Question .Question .4 in Nelson Textbook of Pediatrics. 16. (See Chapter 45. A CBC would be of some use if anemia or infection were suspected. 17th ed. 2. (See Chapter 45.6 in Nelson Textbook of Pediatrics.) Captopril overdose Question . 17th ed. and 3 Explanation: 1. Potential causes of hyperkalemia include all of the following except: Succinylcholine Digitalis toxicity Acute renal failure Albuterol overdose Explanation: -Agonists often cause hypokalemia.5 and 45. 15. and 5 .) 2 and 4 2. 17th ed. (See Chapters 45. and 3 would be helpful in the management of these immediate problems. and indomethacin. The laboratory evaluation of this patient should include: 1. 5 and 45. (See Chapters 45. (See Chapter 45. 1. 19. From the following list.5 mg/dL.) Hemolysis Rhabdomyolysis Paresthesias Confusion . most patients do quite well. Begin chelation therapy 5. 17th ed. The serum magnesium level for the patient in Question 17 is 6. select the most appropriate next step(s) in treatment.) 3 only 4 only 1 and 5 2 and 4 Question . and 3 are correct. Add calcium to the intravenous solution 4.Question . and 3 Explanation: 1. 18. Infuse normal saline 3. Possible consequences of hypophosphatemia include all of the following except: Hypocalcemia Explanation: Indeed. Administer KCl 1. hypercalcemia is more common as a compensatory mechanism to release phosphate from bone. Continue mechanical ventilation 2. although with supportive care and time. 2.6 in Nelson Textbook of Pediatrics. 17th ed. 2.7 in Nelson Textbook of Pediatrics. and congenital heart block. (See Chapter 637 in Nelson Textbook of Pediatrics. 2. with a past medical history significant only for hyperbilirubinemia treated with phototherapy. but only 1 in 10 has both ECG changes and a rash. B. Electrocardiogram Question . neutropenia. 17th ed. and history of UV light exposure. NLE babies can have ECG abnormalities. 1. Maternal antibodies cross the placenta (IgG) and produce autoimmune phenomena in the baby. B. morphology of rash. Neonatal lupus should be considered diagnostically based on the age at presentation. E would also be correct. erythematous eruption on the forehead. She was born 8 wk premature with an otherwise benign medical history. the next best step diagnostically would be serum assay for autoantibodies.Nelson Self Assessments website 17th Edition 1 . (See Chapter 640 in Nelson Textbook of Pediatrics. E. A 10-day-old infant develops an annular. D.) E. which should be diagnostic. His mother has intermittent episodes of joint pain. All manifestations except heart block resolve with time. although her parents wonder if she may be developing asthma. scaly. C. Laser surgery Skin biopsy Reassurance that hemangiomas will go away on their own Inhaler prescription ENT evaluation Explanation: A beard area hemangioma may herald a subglottic lesion that could be life-threatening and may be the source of the perceived asthma symptoms. The most appropriate next step in patient management is: A. thrombocytopenia. C. 17th ed. Results of a KOH examination are negative. cutaneous vasculitis. Liver function tests Fungal culture Skin biopsy Autoimmune work-up Explanation: This vasculitic-type rash is compatible with neonatal lupus.The Skin Nelson Self Assessments website 17th Edition Question . with symmetric extension onto the chin and anterior neck. These include hemolytic anemia. D. The most appropriate next step in diagnosis is: A. He was born at term after an uncomplicated delivery. Therefore. because a rash is present.) The Skin . A 2-mo-old girl has a rapidly growing hemangioma involving the perioral region bilaterally. (See Chapter 639 in Nelson Textbook of Pediatrics. Reassurance that the lesion will resolve spontaneously in time B. Referral to a plastic surgeon for excision D. Anomalous detention D.) C. In younger infants. The most appropriate next step in management is: A. this entity is included in the differential diagnosis of fever of unknown origin. Waardenburg syndrome D. Hypertrichosis Explanation: Patients usually have hypotrichosis. (See Chapter 643 in Nelson Pediatrics. 4. Counseling of the parents regarding melanoma risk E. 17th ed. Tuberous sclerosis Explanation: These hypopigmented macules resemble ash leaves and are best identified with a Wood lamp. Neurofibromatosis type 1 B. (See Chapter 637 in Nelson Textbook of Pediatrics. The child's mother states that the lesion has been present since birth and seems to be asymptomatic. A 6-mo-old infant is presented for evaluation of a tuft of thick black hair located on the sacrum. but as infants grow older. Hypohidrotic ectodermal dysplasia is associated with all of the following Except: A. 17th ed. McCune-Albright syndrome E. 17th ed. On examination he is alert.Question . MRI of the spinal cord Explanation: Lumbar or sacral hair tufts or hemangiomas may be associated with spinal dysraphisms. His skin examination is significant for confetti-type white spots on his right lower extremity his parents state is a "birthmark. A previously healthy 1-yr-old boy presents to the emergency department with seizures. MRI is indicated. these children often have many episodes of high fever.Nelson Self Assessments website 17th Edition 2 . Partial or complete absence of sweat glands B. and afebrile with normal vital signs. 5. such as a tethered cord. and has had no ingestion history and is taking no new medications. an ultrasound study is a useful examination. Vitiligo Question . therefore.) C. Mutations of ectodysplasin The Skin . He has had no recent illnesses or high fevers. Because of the inability to sweat. 3.) C. X-linked recessive inheritance E." The most likely diagnosis is: A. interactive. Watchful waiting Question . and the child is otherwise healthy. Capillary hemangiomas Question . Yellow-orange-appearing plaque B. Herpes simplex Explanation: Both HSV herpes labialis and HSV herpes genitalis have been implicated in the majority of cases of recurrent EM. Coarctation of the aorta E. Port-wine stain E. (See Chapter 644 in Nelson Textbook of Pediatrics. Tufted angioma Explanation: This syndrome. Facial hemangiomas C. 9. (See Chapter 640 in Nelson Textbook of Pediatrics. Risk of benign tumors D. Mononucleosis The Skin . They should be resected before adulthood. Strokes are a major source of morbidity.) C. Kasabach-Merritt syndrome is seen most often with: A. Ocular anomalies F. 6.) D. microangiopathic hemolytic anemia.) B. (See Chapter 640 in Nelson Textbook of Pediatrics. Risk of malignancy E. 8. Verrucous changes during adolescence Question . 17th ed. Human herpesvirus 6 B. Aneurysms and stroke D. Nevus flammeus C. 17th ed. Sternal anomalies Question . associated with thrombocytopenia. Nevus sebaceus (Jadassohn sebaceous nevus) is associated with all of the following Except: A. may develop into benign or malignant tumors in adulthood. 17th ed. PHACES syndrome is associated with all of the following Except: A. and consumption coagulopathy. Phalangeal dysplasia Explanation: PHACES is associated with posterior fossa defects such as Dandy-Walker cysts or cerebellar hypoplasia.Question .) D. 17th ed.Nelson Self Assessments website 17th Edition 3 . is due to a tufted angioma or a kaposiform hemangioendothelioma. Hemangiomas B. Psychosis E. 7. usually seen on the head or neck of infants. Anxiety C. (See Chapter 641 in Nelson Textbook of Pediatrics. Recurrent erythema multiforme is associated with: A. Cardiac failure Explanation: These initially benign-appearing plaques. Stevens-Johnson syndrome C. Topical gentamicin The Skin . 12. Ultraviolet light D. Crotamiton lotion B. Hypersensitivity reactions to anticonvulsant drugs occur 1-3 mo after initiation of therapy and probably represent a cell-mediated immunologic reaction to the drug or a drug metabolite. 10. D. Bullae and exfoliation Corneal ulceration Polyarthritis Osteomyelitis Explanation: Secondary local bacterial infections of the skin or sepsis may occur. Prolonged oral therapy is needed to penetrate the growing nail. 13. Serum sickness E. Toxic epidermal necrolysis Question . and elevated values on liver function tests. The treatment of choice for scabies is: A.) E.) C. Myocarditis F. Onychomycosis is associated with all of the following Except: A. making it resistant to fungal infections. eosinophilic leukocytosis. 17th ed. Hepatitis Question . (See Chapter 644 in Nelson Pediatrics. Antibiotics are indicated for proven infections or if there is a high index of suspicion for infection. B. mentagrophytes superficial infection A good response to treatment with oral terbinafine A good response to topical ketoconazole Explanation: Topical therapy is of little value in fungal infections of the nail. 17th ed. A good response to oral itraconazole Question . lymphadenopathy. She now has a diffuse erythematous macular rash. A 15-yr-old girl who had new-onset seizures was started on carbamazepine 4 wk ago. (See Chapter 656 in Nelson Textbook of Pediatrics. However. The most likely cause of these abnormalities is: A. Erythema multiforme B. (See Chapter 658 in Nelson Pediatrics. B. C. (See Chapter 644) D. 17th ed. Hypersensitivity syndrome Explanation: The rash may look like any of the other choices (more often choices B and E). Treatment for patients with Stevens-Johnson syndrome is similar to that for patients with extensive burns. hepatitis and lymphadenopathy are uncommon findings with choice B or E. Permethrin 5% cream Explanation: One percent lindane cream (or lotion) is also recommended but is potentially neurotoxic. Salicylate baths E. T. 11. Complications of Stevens-Johnson syndrome include all of the following Except: A. Steroids may increase the risk of infection. C. D. rubrum invasive infection T.Question . fever.Nelson Self Assessments website 17th Edition 4 .) E. (See Chapter 658 in Nelson Textbook of Pediatrics. Microphthalmos F. Giant congenital pigmented nevi are associated with all of the following Except: A. (See Chapter 642 in Nelson Textbook of Pediatrics. C. Application of 10% permethrin ointment C. Application of 0. Infectious causes F. An incidence of less than 1 case in 20. A good response to prednisone Explanation: There is no agreed-on indication for systemic steroids in StevensJohnson syndrome. Erythematous linear streaks and vesicles C.) B.) B.5% malathion Explanation: In addition. Involvement of two mucous membranes C. Lethality in females Explanation: Incontinentia pigmenti is X-linked dominant and is lethal in males during fetal life. Esophageal stricture D. Stevens-Johnson syndrome (erythema multiforme major) is associated with all of the following Except: A.000 births Leptomeningeal involvement Malignant melanoma Hydrocephalus Male predominance Explanation: Giant pigmented nevi are difficult-to-manage lesions that affect males and females with equal frequency. Hypodontia E. 17th ed. Alopecia D. 16. Brushing and combing D. (See Chapter 644 in Nelson Textbook of Pediatrics. The treatment of choice for pediculosis capitis is: A. D. 17th ed. Hydrogen peroxide E. Incontinentia pigmenti is associated with all of the following Except: A. 17th ed. 17th ed.) Question . Drug-related causes The Skin . Corneal scarring E. E. Topical steroids are indicated for ocular involvement to prevent scarring. 14. B. Permethrin plus oral trimethoprim-sulfamethoxazole is a promising therapy for hard-to-treat cases. 17. (See Chapter 641 in Nelson Textbook of Pediatrics.Question . 15. Selenium-based shampoo Question . Seizures Question . Malathion is not indicated in infants or neonates. 1% permethrin cream is also a treatment of choice.) B.Nelson Self Assessments website 17th Edition 5 . Question . which is prominent when the neonate is in a cool environment. self-limited. This therapy is targeted at the lesion and avoids thermal injury to the surrounding normal tissue. Harlequin color change E.) E. 18. but new lesions may erupt during the first few days as the rash waxes and wanes. C. excision. Acropustulosis D. yellow-white. A 2-day-old well-appearing full-term white neonate experiences multiple firm. Erythema toxicum Question . 17th ed. (See Chapter 640 in Nelson Textbook of Pediatrics. 19. preterm infants are affected less commonly. Peak incidence is on the second day of life.to 2-mm papules or pustules with a surrounding erythematous flare. The lesions are firm. grafting. 1.Nelson Self Assessments website 17th Edition 6 . Herpes simplex virus infection Question . The correct diagnosis is: A.) D. 17th ed. cryosurgery. B. Palms and soles are usually spared.to 2-mm papules or pustules with a surrounding erythematous flare on the trunk. Mongolian spots B. Wright stain of the lesions shows numerous eosinophils. yellow-white. 1. (See Chapter 637 in Nelson Textbook of Pediatrics. evanescent eruption that occurs in approximately 50% of full-term infants. Interferon- The Skin . 20. 17th ed. Alternative therapies include masking with cosmetics. Erythema toxicum Explanation: Erythema toxicum is a benign. Eosinophilic pustular folliculitis E. Salmon patch C. red and/or blue cutaneous pattern over most of the body. The most likely diagnosis is: A. (See Chapter 637 in Nelson Pedia.) B. A well-appearing newborn has a lacy. Tattooing Excision and skin grafting Cryosurgery Laser therapy Explanation: The most effective treatment for port-wine stains is the flashlamp-pumped-pulsed dye laser. D. The recommended therapy for port-wine nevus is: A. and tattooing. reticulated. Cutis marmorata Explanation: Cutis marmorata is an accentuated physiologic vasomotor response that disappears with increasing age during the first year of life. Pustular melanosis C. Lesions may be sparse or numerous and clustered in several sites or widely dispersed over much of the body surface. Which of the following is the most likely diagnosis? A. precocious puberty.Nelson Self Assessments website 17th Edition 7 . 17th ed. In both keloids and hypertrophic scars. X-ray shows polyostotic fibrous dysplasia of bone. The essential defect is a quantitative deficiency of collagen. 22. Pseudoxanthoma elasticum E. Anetoderma E. burns. often asymmetric café-au-lait spots with irregular borders are characteristic of McCune-Albright syndrome. (See Chapter 649 in Nelson Textbook of Pediatrics. Keloid Explanation: Keloids are usually induced by trauma and commonly follow ear piercing.) C. The skin snaps back into place when pulled. Tuberous sclerosis D. new collagen forms over a much longer period than in wounds that heal normally. firm. A black teenager presents with a sharply demarcated. An 8-yr-old girl presents with large café-au-lait spots with irregular borders and precocious puberty. This disorder also includes polyostotic fibrous dysplasia of bone. leading to pathologic fractures. Unlike keloids. but skin hyperelasticity. Affected children appear normal at birth. and joint hypermobility develop over time. Which of the following is the most likely diagnosis? A. McCune-Albright syndrome Explanation: Large. Anetoderma D. (See Chapter 642 in Nelson Textbook of Pediatrics. dense. (See Chapter 649 in Nelson Textbook of Pediatrics. Granuloma annulare B. Neurofibromatosis B.) C. 23. 17th ed. and surgical procedures. 21. and numerous hyperfunctional endocrinopathies. especially blacks. Elastosis perforans serpingosa The Skin . The most likely diagnosis is: A. Cutis laxa B. Certain individuals. hypertrophic scars remain confined to the site of injury and gradually involute over time. Maffucci syndrome E. rubbery growth on the face at the site of a previous smaller laceration that was incurred long ago. scalds. Mastocytosis Question . 17th ed.) C. Ehlers-Danlos syndrome Explanation: Ehlers-Danlos syndrome is a group of genetically heterogeneous connective tissue disorders. Normal child Question . seem predisposed to keloid formation. Ehlers-Danlos syndrome has been classified into 10 clinical forms.Question . A young child has unusual skin hyperelasticity and joint hypermobility of gradual onset. Necrobiosis lipoidica D. fragility of the skin and blood vessels. (See Chapter 659 in Nelson Textbook of Pediatrics.) E. acral. B. 1. There is often alopecia and chronic diarrhea. Papules of molluscum contagiosum may occur anywhere on the body. and perineal areas and on the cheeks. Which of the following is the most serious adverse effect associated with use of isotretinoin? A. depression. Verruca vulgaris (common warts) Verruca plana (flat warts) Condylomata acuminata Molluscum contagiosum Explanation: The lesions of molluscum contagiosum are discrete. A child presents with multiple. dome-shaped. myalgias. axillae. There is mild alopecia and a history of chronic diarrhea. Keloid Question . 17th ed. Initial signs and symptoms occur during infancy and consist of a rash in the perioral. Hepatitis B.Question . and elbows. Acrodermatitis enteropathica Explanation: Acrodermatitis enteropathica is caused by an inability to absorb sufficient zinc from the diet. Carcinogenicity Question . 25. They typically have a central umbilication from which a plug of cheesy material can be expressed.to 3-mm dome-shaped smooth papules on the face with central umbilication. (See Chapter 661 in Nelson) D. 24. Most patients experience cheilitis. The most likely diagnosis is: A. temporary thinning of the hair. Pellagra The Skin . (See Chapter 657 in Nelson Textbook of Pediatrics. pearly. formation of pyogenic granulomas. eyelids. and blepharoconjunctivitis. discrete. smooth papules that may range in size from 1 to 5 mm. secondarily infected dermatitis. paronychia. D. and scalp folliculitis. and perineal areas and on the cheeks. Increased serum triglyceride and cholesterol levels are also common. It is teratogenic and is contraindicated for use during pregnancy. Scurvy E. A 2-yr-old child presents with symmetric psoriasiform skin lesions distributed in the perioral. skincolored. Pseudotumor cerebri D. xerosis. Less common adverse effects include arthralgias. knees. Teratogenicity Explanation: Isotretinoin has many adverse effects. 17th ed. knees. Juvenile xanthogranuloma C. Some patients with cystic fibrosis present with a similar rash.) E. pregnancy should be avoided for 1 mo after discontinuation of therapy. Cardiomyopathy C.Nelson Self Assessments website 17th Edition 8 . and colonization of the skin with Staphylococcus aureus leading to impetigo. but the face. acral. increased susceptibility to sunburn. Treatment of acrodermatitis enteropathica is with oral zinc compounds. and elbows. 26. skin-colored. Psoriasis B. The most likely diagnosis is: A. C. neck. periodic epistaxis. and thighs are areas of predilection. Biotinidase deficiency should be in the differential diagnosis.
Copyright © 2024 DOKUMEN.SITE Inc.